Вы находитесь на странице: 1из 263

General Knowledge Today

Current Affairs: October 2016 [Datewise Compendium]

Published Date: November 8, 2016


Published by: GKTODAY.IN
GKToday 2016 | All Rights Reserved
Suitable for Bank Probationary, Bank Clerical, IBPS Recruitment, NDA, CDS,
LIC/GIC, Railways, SSC, UGC and state / UPSC preliminary Examination.

Current Aairs: October 2016 [Date-wise Compendium]

Contents
Month Highlights: October, 2016 .................................................................................................................... 3
October 1, 2016 .................................................................................................................................................. 12
Infosys unveils Skava Commerce, a modular e-commerce platform ............................................. 12
EU ministers approve ratication of Paris climate Agreement ......................................................... 12
OPEC agrees on modest oil production curbs .................................................................................... 13
About Organization of the Petroleum Exporting Countries (OPEC) ............................................. 13
PM Narendra Modi inaugurates India Sanitation conference in New Delhi ................................ 14
About India Sanitation conference (INDOSAN) ................................................................................ 14
Sports Sector Gets the Infrastructure Status ....................................................................................... 14
Pakistan announces postponement of 19th SAARC Summit in Islamabad ................................... 15
About the South Asian Association for Regional Cooperation (SAARC) ..................................... 15
Tamil Nadu tops list of endemic owering plants: BSI ..................................................................... 16
Music Director Uttam Singh named for Maharashtras Lata Mangeshkar Award ..................... 17
About Uttam Singh ................................................................................................................................... 17
About Lata Mangeshkar Award ............................................................................................................. 17
30 September: The International Translation Day ............................................................................ 17
National Mission on Bioeconomy launched in Shillong, Meghalaya .............................................. 18
Rajasthan becomes rst state to adopt LED street lights under all ULBs ...................................... 18
October 2, 3 , 2016 ........................................................................................................................................... 19
Murga wins rst prize at Swachh Bharat Short Film Festival ......................................................... 19
Indian-American Vinaya Manchaiah named Future Leader of Audiology .................................... 19
About Vinaya Manchaiah ......................................................................................................................... 19
Black money worth Rs. 65,250 crore declared under Income Disclosure Scheme, 2016 ....... 20
Delhi Police launches Mobile App for senior citizens ....................................................................... 20
2 October: International Day of Non-Violence .................................................................................. 20
About International day of Non-Violence ............................................................................................. 21
BRICS adopts New Delhi Declaration on Education ......................................................................... 21
Abu Dhabi Crown Prince to be Chief Guest on 2017 Republic Day ............................................ 22
New species of pika named Ochotona sikimaria discovered in Sikkim ........................................ 22
India raties Paris Climate Agreement ................................................................................................. 23
3 October: World Habitat Day ............................................................................................................... 23
Indian-origin Kiara Nirghin wins 2016 Google Science Fair prize ................................................ 24
October 4, 2016 ................................................................................................................................................ 24
Kersti Kaljulaid elected as rst female President of Estonia ............................................................. 24
About Kersti Kaljulaid .............................................................................................................................. 25
Elephant researcher Prajna Chowta conferred with knighthood by France ................................ 25
About Prajna Chowta ............................................................................................................................... 25
Reliance, Dassault Aviation ink deal to set up joint venture ............................................................ 25
Jharkhand becomes rst state to implement DBT in Kerosene ..................................................... 26
2016 GKToday | All Rights Reserved | www.gktoday.in

Current Aairs: October 2016 [Date-wise Compendium]


About Direct Benet Transfer in Kerosene (DBTK) scheme ......................................................... 26
Urban areas of Gujarat, Andhra Pradesh declared open defecation free ..................................... 26
About Swachh Bharat Mission ............................................................................................................... 27
Japans Yoshinori Ohsumi wins 2016 Nobel Prize in Medicine ...................................................... 27
About Nobel Prize in Physiology or Medicine .................................................................................... 28
SBI becomes rst domestic bank to open branch in Yangon, Myanmar ..................................... 28
MS Sahoo takes charge as Chairman of Insolvency and Bankruptcy Board of India ............... 29
About Insolvency and Bankruptcy Board of India (IBBI) ................................................................ 29
October 5, 2016 ................................................................................................................................................ 30
Nayanjot Lahiri wins 2016 John F Richards Prize ............................................................................. 30
About Nayanjot Lahiri .............................................................................................................................. 30
About John F. Richards Prize ................................................................................................................. 30
British trio David Thouless, F Duncan M Haldane and Michael Kosterlitz win 2016 Nobel Prize
in Physics ............................................................................................................................................. 30
RBI cuts repo rate by 25 bps in 4th Bi-monthly Monetary Policy Statement .............................. 31
Aadhaar card must for LPG subsidy after November 2016 ............................................................ 32
About Aadhaar ........................................................................................................................................... 32
Union Government launches Indian Bridge Management System ................................................. 32
India, Singapore sign three agreements ............................................................................................... 33
Former ISRO Chariman UR Rao becomes rst Indian to be inducted into IAF Hall of Fame
................................................................................................................................................................ 34
About UR Rao ............................................................................................................................................ 34
October 6, 2016 ................................................................................................................................................ 34
Union Cabinet approves MoU between India and AARDO ............................................................ 35
About African Asian Rural Development Organization (AARDO) ................................................ 35
Union Cabinet approves MoU between India and European Union on water cooperation
................................................................................................................................................................ 35
Sushil Chandra appointed as Chairman of CBDT ............................................................................. 36
About Central Board of Direct Taxes (CBDT) .................................................................................. 36
CCEA approves OVLs proposal to buy 11% stake in JSC Vankorneft .......................................... 36
Indias communication satellite GSAT-18 successfully launched from French Guiana ............. 37
Jean-Pierre Sauvage, J Fraser Stoddart and Bernard L Feringa win 2016 Nobel Prize in
Chemistry ............................................................................................................................................ 38
Cabinet approves amendments to HIV and AIDS (Prevention and Control) Bill, 2014 ........... 39
October 7, 2016 ............................................................................................................................................... 40
All ASI protected historical monuments declared polythene-free zones ...................................... 40
RBI Panel moots easing bank branch norms ...................................................................................... 40
Sikkim CM Pawan Chamling honoured with Sustainable Development Leadership Award
................................................................................................................................................................ 41
India seeks greater Pharma market access in Japan .......................................................................... 41
President Pranab Mukherjee inaugurates World Sustainable Development Summit ................. 42
2016 GKToday | All Rights Reserved | www.gktoday.in

Current Aairs: October 2016 [Date-wise Compendium]


India to host 2016 Asian Ministerial Conference for Disaster Risk Reduction ........................... 42
About Sendai Framework for Disaster Risk Reduction 2015-30 ................................................... 43
Renowned Manipuri theatre legend Heisnam Kanhailal passes away ........................................... 44
About Heisnam Kanhailal ....................................................................................................................... 44
Maharashtra Government allocates land to LIGO India Project .................................................... 44
Payment Banks need RBIs Prior product approval ........................................................................... 45
India slams hold on declaring Masood Azhar global terrorist ........................................................ 46
About United Nations Security Council 1267 Committee .............................................................. 46
Union Government approves Rs.114 crore worth projects under HRIDAY Scheme ................ 47
About Heritage Infrastructure Development and Augmentation Yojana (HRIDAY) ................ 47
2016 Nobel Peace Prize awarded to Colombian President Juan Manuel Santos ....................... 47
October 8, 2016 ............................................................................................................................................... 48
EC bars political parties from using public space for propagating symbol .................................. 48
Election Commission partners with Facebook to encourage voter registration ......................... 49
Maharashtra becomes 17th state to join UDAY Scheme ................................................................. 49
About Ujwal DISCOM Assurance Yojna (UDAY) ............................................................................ 50
SC stays commercial release of GM mustard ..................................................................................... 50
New termite species named Chiraharitae discovered ........................................................................ 51
India-Pakistan border to be sealed completely by 2018 ................................................................... 52
Adoption of GST tax will boost Indias medium-term growth: IMF ............................................. 52
BRICS Contingent Reserve Arrangement declared operational ...................................................... 53
Indias rst Medipark will be set up in Tamil Nadu .......................................................................... 54
Union Government set up Public Debt Management Cell ............................................................... 55
Indian Pharma companies rank 19th in Biopharmaceutical Competitiveness & Investment (BCI)
Survey .................................................................................................................................................. 56
Indias rst international arbitration centre inaugurated in Mumbai ............................................. 57
October 9-10, 2016 .......................................................................................................................................... 57
October 9: World Post Day .................................................................................................................... 57
About Universal Postal Union (UPU) .................................................................................................... 58
October 10: World Mental Health Day ................................................................................................ 58
Union Government extends anti-dumping duty on certain Chinese products ............................ 58
Union Government to invite bids soon for 36 inland waterways .................................................. 59
IAE to tie up with India for its global LED programme .................................................................. 60
About Unnat Jyoti by Affordable LEDs for All (UJALA) scheme .................................................. 60
Hubble Space Telescope detects Great Balls of Fire ......................................................................... 61
About Hubble Space Telescope (HST) ................................................................................................ 61
Janani Suraksha Yojana pays dividends: Study .................................................................................... 61
About Janani Suraksha Yojana (JSY) .................................................................................................... 62
SC widens ambit of Domestic Violence Act, 2005 ........................................................................... 62
Indian researcher produces stable Perovskite Nanocrystal solar cells ........................................... 63
Oliver Hart and Bengt Holmstrom win 2016 Nobel Prize in Economics .................................... 64
2016 GKToday | All Rights Reserved | www.gktoday.in

Current Aairs: October 2016 [Date-wise Compendium]


About Nobel Memorial Prize in Economic Sciences ........................................................................ 64
October 11, 2016 ............................................................................................................................................... 64
October 11: International Day of the Girl Child ................................................................................. 64
4th BRICS Science, Technology and Innovation Ministerial Meeting held at Jaipur ................. 65
Himansh, Indias remote, high-altitude station opens in Himalayas .............................................. 65
Russia, Turkey agree to intensify military, intelligence contacts .................................................... 66
Union Government to invest Rs 15000 crore to increase Panipat oil renery capacity ........... 67
Union Government to commemorate birth centenary of Nanaji Deshmukh ............................. 68
Who is Nanaji Deshmukh? ..................................................................................................................... 68
October 12, 2016 .............................................................................................................................................. 68
Abdelilah Bekirane elected as Prime minister of Morocco .............................................................. 68
Jitu Rai wins Champion of Champions pistol award of ISSF .......................................................... 69
Harikatha: traditional art of story-telling in South India .................................................................. 69
Telangana Government creates 21 new districts ............................................................................... 70
Union Government to adopt three-pronged approach to spur internal trade ............................. 71
China raps US over Asia interventions .................................................................................................. 71
China-US Issue ........................................................................................................................................... 71
India-Indonesia bilateral maritime exercise commences at Belawan, Indonesia ......................... 72
NASA develops electroactive bandage to speed up wound healing .............................................. 73
CERC appointed committee suggests overhaul in transmission planning .................................... 73
October 13, 2016 .............................................................................................................................................. 74
Finance Ministry constitutes committee to consolidate regulation of pension products .......... 74
About Pension Fund Regulatory and Development Authority (PFRDA) ..................................... 75
Union Cabinet approves MoU between India and Hungary on cooperation in eld of water
management ....................................................................................................................................... 75
First World Tsunami Awareness Day to be celebrated at AMCDRR 2016 ................................. 75
About World Tsunami Awareness Day ............................................................................................... 76
Union Cabinet approves establishment of IIM in Jammu ................................................................ 76
Four PSBs may struggle to pay AT1 bond coupons ......................................................................... 77
October 13: International Day for Disaster Reduction ..................................................................... 78
Draft National Water Framework Bill, 2016 suggests basin-level management ........................ 79
IISER researchers use human hair to produce cheaper cathodes for solar cells ........................ 80
CCEA approves revision of ethanol price for supply to Public Sector OMCs ............................ 80
October 14, 2016 ............................................................................................................................................... 81
Union Cabinet approves MoU between EXIM bank and NDB ....................................................... 81
Antonio Guterres elected as new Secretary-General of UN ........................................................... 82
About Secretary-General of United Nations ....................................................................................... 83
India receives rst Iranian oil parcel for emergency reserves ......................................................... 83
Bob Dylan wins 2016 Nobel Prize in Literature ................................................................................ 84
About Bob Dylan ...................................................................................................................................... 84
About Nobel Prize in Literature ............................................................................................................ 84
2016 GKToday | All Rights Reserved | www.gktoday.in

Current Aairs: October 2016 [Date-wise Compendium]


India ranked 97th in 2016 Global Hunger Index .............................................................................. 85
22 MPs from smaller parties, independents in Rajya Sabha recognised as a consolidated bloc
............................................................................................................................................................... 86
About Rajya Sabha ................................................................................................................................... 87
India to eliminate use of HFC-23 by 2030 ......................................................................................... 87
India, Russia ink agreement to set up 25 Agro Irradiation Centres .............................................. 88
October 15, 2016 .............................................................................................................................................. 89
Union Government to use post ofces to distribute pulses ............................................................. 89
Women of India Festival 2016 of organic products inaugurated in New Delhi ......................... 90
Ajay Kumar Bhalla appointed as DG of Directorate General of Foreign Trade ........................ 90
About Directorate General of Foreign Trade (DGFT) ..................................................................... 91
Bangladesh, China agree to elevate their ties to strategic partnership ........................................... 91
15 October: International Day of Rural Women ................................................................................ 92
National Seismic Programme launched in Odisha ............................................................................. 93
IISc researchers develop self-powered UV photodetector for self-charging energy storage
devices ................................................................................................................................................. 93
2nd Rashtriya Sanskriti Mahotsav begins in New Delhi .................................................................. 94
Brazil wins inaugural BRICS U-17 Football Tournament ................................................................ 95
About BRICS U-17 Football Tournament ........................................................................................... 95
Union Government launches satellite-based illegal Mining Detection System ............................ 95
India, Russia sign 16 agreements across diverse sectors .................................................................. 97
October 16, 2016 .............................................................................................................................................. 97
Rosneft-led consortium buys Indias Essar Oil for $13 billion ......................................................... 97
About Essar Group ................................................................................................................................... 98
Russia, India ink defence deal on S-400 Air Defence Systems ...................................................... 98
Saraswati River existed: KS Valdiya committee .................................................................................. 99
16 October: World Food Day ............................................................................................................... 100
About Food and Agriculture Organisation (FAO) ........................................................................... 101
8th BRICS Summit concludes in Goa ................................................................................................. 101
17 October: International Day for the Eradication of Poverty ....................................................... 103
Japanese Encephalitis claims 54 lives in Odishas Malkangiri district .......................................... 103
BRICS to set up credit rating agency ................................................................................................. 104
Historic Global Greenhouse gas emission Agreement signed in Kigali ...................................... 105
China successfully launches its longest ever manned mission to space ...................................... 106
National Summit on Fortication of Food inaugurated in New Delhi ........................................ 107
BIMSTEC countries express commitment to combat terrorism in all its forms ....................... 108
About BIMSTEC ..................................................................................................................................... 108
October 17, 18, 2016 ..................................................................................................................................... 109
Indian wrestlers win four medals at 2016 TAFISA World Games .............................................. 109
About Association for International Sport for All (TAFISA) ........................................................ 109
Eminent writer C Radhakrishnan wins Mathrubhumi Literary Award 2016 ............................ 109
2016 GKToday | All Rights Reserved | www.gktoday.in

Current Aairs: October 2016 [Date-wise Compendium]


About C Radhakrishnan ......................................................................................................................... 110
About Mathrubhumi Literary Award .................................................................................................. 110
Amandeep Singh Gill appointed as Indias Ambassador to UN Conference on Disarmament
............................................................................................................................................................... 110
Ashok Leyland rolls out rst India-made electric bus ....................................................................... 111
India completes nuclear triad by commissioning INS Arihant ........................................................ 111
PM Narendra Modi inaugurates three mega hydro projects in Himachal Pradesh ................... 113
PM launches National SC/ST hub and Zero Defect Zero Effect scheme ............................... 114
October 19, 2016 ............................................................................................................................................. 115
Delhis air quality very poor: SAFAR ................................................................................................... 115
Saina Nehwal appointed member of IOCs Athletes Commission ............................................... 115
About Saina Nehwal ................................................................................................................................ 115
Government funds Ro Pax Ferry Service in Gulf of Cambay under Sagarmala Programme
............................................................................................................................................................... 116
Union Government proposes 4 GST slabs ........................................................................................ 117
NPL Scientists produce electricity from water without using energy .......................................... 118
Auto manufacturers to provide emission and noise pollution details for every vehicle ............ 119
Indian Navy commissions INS Tihayu ................................................................................................ 119
About INS Tahiyu .................................................................................................................................... 119
SC asks whether mass religious appeals for electoral gains can be categorised corrupt practice
.............................................................................................................................................................. 120
Milk from Tasmanian devils could ght superbugs: Study ............................................................ 120
About Tasmanian devil ........................................................................................................................... 121
October 20, 2016 ............................................................................................................................................. 121
World Bank pledges $2 billion aid to Bangladesh to tackle climate change ............................... 121
About World Bank .................................................................................................................................. 122
Sir David Cox honoured with rst International Prize in Statistics .............................................. 122
About Sir David Cox ............................................................................................................................... 122
About International Prize in Statistics ................................................................................................. 123
India, Algeria mull fertilizer Joint Venture .......................................................................................... 123
EC hosts rst ever International Conference on Voter Education ................................................ 123
India, China conduct second Joint tactical Exercise in Ladakh ...................................................... 124
India, Myanmar signed 3 MoUs including Cooperation in eld of Power Sector .................... 125
India signs pact with Russia to lease second nuclear submarine ................................................... 125
IUNC to declare Kashmirs Red Stag as Critically Endangered .................................................... 126
About Kashmiri Red Stag ...................................................................................................................... 126
About International Union for Conservation of Nature (IUCN) .................................................. 127
Okha-Kanalus, Porbandar-Wansjaliya sections of Gujarat become Green Train Corridors
.............................................................................................................................................................. 127
No relook into allocation of Krishna water: Tribunal ..................................................................... 128
October 21, 2016 ............................................................................................................................................. 129
2016 GKToday | All Rights Reserved | www.gktoday.in

Current Aairs: October 2016 [Date-wise Compendium]


SBI declares Silchar as its 5th zone in North East ........................................................................... 129
Maharashtra Government launches Kaushalya Setu skill-development programme ............... 129
Railway ministry inks MoU for Smart Railway Stations ................................................................. 130
Triple talaq a highly misused custom: NCW ..................................................................................... 130
About National Commission for Women (NCW) ............................................................................ 131
Pakistan warns India against breaching Indus Water Treaty .......................................................... 131
Protein-enriched rice variety developed by IGKV Chattisgarh ...................................................... 133
IIL develops CYSVAX, worlds rst vaccine to ght against tapeworms in pig ........................ 133
Union Government inks loan pact with World Bank for Eastern Freight Corridor ................. 134
SC freezes BCCIs transactions with state cricket bodies ............................................................... 135
October 22, 2016 ............................................................................................................................................ 136
BCCI approves use of DRS in India-England Test series ............................................................... 136
Smooth-coated otter sighted for rst time in Krishna sanctuary ................................................. 137
About smooth-coated Otter .................................................................................................................. 137
ICGS Aryaman and Atulya Commissioned into Indian Coast Guard .......................................... 137
About Indian Coast Guard (ICG) ........................................................................................................ 138
Union Government launches UDAN Scheme for Regional Air Connectivity ............................ 138
Global Conference to strengthen Arbitration & Enforcement held in New Delhi .................... 140
NABARD sanctions Rs. 19,702 crore loan to National Water Development Agency ............. 140
About National Water Development Agency (NWDA) .................................................................. 141
India wins 2016 Kabaddi World Cup .................................................................................................. 141
About Kabaddi World Cup .................................................................................................................... 141
Arena By TransStadia ............................................................................................................................. 141
October 23-24, 2016 ..................................................................................................................................... 142
Western Zonal Council Meeting held in Mumbai ............................................................................ 142
ITBP celebrates its 55th raising day .................................................................................................... 143
About Indo-Tibetan Border Police (ITBP) ........................................................................................ 143
October 24: United Nations Day ......................................................................................................... 143
India falls short in female literacy compared to its neighbours: Study ........................................ 144
World Bank must aid countries to manage shift away from coal ................................................ 145
Government to provide higher subsidy for airlines connecting two regional airports ............ 145
October 25, 2016 ............................................................................................................................................ 146
PM Narendra Modi launches Urja Ganga in Varanasi .................................................................... 146
ISRO starts landing tests for Chandrayaan-2 mission ..................................................................... 147
About Chandrayaan-2 mission ............................................................................................................. 148
CO2 level reaches record high in 2016: WMO ................................................................................ 148
About World Meteorological Organization (WMO) ....................................................................... 149
ECI amends election rules to allow e-transfer of postal ballot ...................................................... 149
Cyrus Mistry removed as Chairman of Tata Group ....................................................................... 150
About Cyrus Pallonji Mistry .................................................................................................................. 150
PM Narendra Modi inaugurates rst National Tribal Carnival ..................................................... 150
2016 GKToday | All Rights Reserved | www.gktoday.in

Current Aairs: October 2016 [Date-wise Compendium]


Chinas Space Lab Tiangong-2 launches Micro-Satellite Banxing-2 ............................................ 151
About Banxing-2 ...................................................................................................................................... 151
October 26, 2016 ............................................................................................................................................. 151
Uranus may have two more moons: Study ........................................................................................ 151
About Uranus ........................................................................................................................................... 152
Uttarakhand High court bans sale of whitener ................................................................................. 153
Union Government forms panel to resolve telecom penalty row ................................................. 153
Paul Beatty wins 2016 Man Booker Prize ......................................................................................... 154
About Paul Beatty .................................................................................................................................... 154
About Man Booker Prize ....................................................................................................................... 154
India-Sri Lanka joint military exercise Mitra Shakti 2016 begins ................................................. 155
India ranks 87th on the WEFs Global Gender Gap Report 2016 ............................................... 155
India ranks 130th in 2017 Ease of Doing Business index ............................................................... 157
Government launches International Household Survey Network Toolkit ................................. 158
Life expectancy highest in Jammu and Kashmir: RGI Data ........................................................... 159
October 27, 2016 ............................................................................................................................................ 159
IPS ofcer Karnal Singh appointed as Director of Enforcement Directorate ............................ 159
One-man judicial Committee on OROP submits report to Defence Minister ........................... 160
Union Government launches Biotech-KISAN and Cattle Genomics to empower farmers .... 160
India, New Zealand ink three agreements including avoidance of double taxation .................. 161
Union Government constitutes Munialappa Committee to monitor bird u situation ............ 162
Union Cabinet approves agreement between India and Estonia on the transfer of sentenced
persons ............................................................................................................................................... 163
Union Cabinet gives approval to IISER, Berhampur in Odisha ..................................................... 164
RBI allows startups to raise $3 million via ECBs annually ............................................................. 164
October 28, 2016 ............................................................................................................................................ 165
Andhra Pradesh and Singapore inks FinTech Cooperation agreement ...................................... 165
Two-thirds of wild animals may go extinct by 2020 ..................................................................... 166
2 Yazidi women Nadia Murad Basee and Lamiya Aji Bashar win 2016 Sakharov Human Right
Prize ................................................................................................................................................... 166
About Sakharov Prize ............................................................................................................................ 167
28 October: National Ayurveda Day .................................................................................................. 167
Union Cabinet approves establishment of National Academic Depository ................................ 168
Union Cabinet approves new Agreement on Trade, Commerce and Transit between India and
Bhutan ............................................................................................................................................... 168
Union Government gives in-principle approval for strategic sale in some PSUs ...................... 169
Microsoft launches Indias rst Cybersecurity Engagement Center in Delhi ............................. 170
Antarticas Ross Sea declared as worlds largest Marine Protected Area ................................... 170
India, Russia agree to double the range of BrahMos Missile ......................................................... 171
October 29, 2016 ............................................................................................................................................ 173
Union Government recognizes Boxing Federation of India as National Sports Federation for
2016 GKToday | All Rights Reserved | www.gktoday.in

Current Aairs: October 2016 [Date-wise Compendium]


Boxing ................................................................................................................................................
Sunil Bharti Mittal appointed as Chairman of GSMA ....................................................................
About Sunil Bharti Mittal ......................................................................................................................
About the GSM Association .................................................................................................................
DoPT launches Employees Online Application ................................................................................
Himachal Pradesh declared Indias second Open Defecation Free State ....................................
Union Government reconstitutes Inter-State Council .....................................................................
October 30-31, 2016 ......................................................................................................................................
EU, Canada sign CETA ..........................................................................................................................
First meeting of NCTF held in New Delhi ........................................................................................
October 31: World Cities Day ..............................................................................................................
October 31: Rashtriya Ekta Diwas .......................................................................................................
About Sardar Vallabhbhai Patel ...........................................................................................................
Compendium of Multiple Choice Questions .............................................................................................

173
174
174
174
174
175
175
176
176
177
177
178
178
178

2016 GKToday | All Rights Reserved | www.gktoday.in

10

Current Aairs: October 2016 [Date-wise Compendium]

Month Highlights: October, 2016


Persons in news Appointments / Elections

Kersti Kaljulaid

The parliament of Estonia has elected Kersti Kaljulaid (46) as the President of the Baltic
country. With this, she became Estonias first female president.

MS Sahoo

The Union Finance Ministry has constituted four-member Insolvency and Bankruptcy Board
of India (IBBI) with Financial markets expert MS Sahoo as its Chairman.

Sushil Chandra

The Union Government has appointed senior Indian Revenue Service (IRS)
officer Sushil Chandra as the Chairman of the Central Board of Direct Taxes (CBDT).

Antonio Guterres

The United Nations General Assembly (UNGA) has formally elected Antonio Guterres (67) as
the new Secretary-General of the United Nations.

Ajay Kumar Bhalla

Senior IAS officer Ajay Kumar Bhalla has been appointed as new Director General of
Directorate General of Foreign Trade (DGFT).

Abdelilah Bekirane

Abdelilah Bekirane was re-appointed as the Prime Minister of Morocco for second
term. Benkirane has been serving as Prime Minister since November 2011.

Amandeep Singh Gill

Senior IFS officer Amandeep Singh Gill was appointed as the Indias Ambassador to the UN
Conference on Disarmament, Geneva.

Karnal Singh

The Union Government has appointed Senior IPS officer Karnal Singh as the Director of
Enforcement Directorate (ED).

Sunil Bharti Mittal

The Founder and Chairman of Bharti Enterprises Sunil Bharti Mittal was appointed as
Chairman of GSM Association (GSMA).
Government Policy and Schemes

Sports Sector Gets the Infrastructure Status

The Union Finance Ministry has decided that sports infrastructure will be included under
the harmonised master list of infrastructure sub-sectors.

National Mission on Bioeconomy

The National Mission on Bioeconomy has launched in Shillong, Meghalaya by the Institute of
Bio-resources and Sustainable Development (IBSD).

Rajasthan

Rajasthan became the first state in India to adopt Central Governments Street Lighting
2016 GKToday | All Rights Reserved | www.gktoday.in

Current Aairs: October 2016 [Date-wise Compendium]


National Programme (SLNP) in all its urban local bodies (ULB).

Jharkhand

Jharkhand has become the first state in the country to implement Direct Benefit Transfer
(DBT) in Kerosene Scheme.

Indian Bridge Management System (IBMS)

The Union Ministry of Road Transport and Highways has launched the Indian Bridge
Management System (IBMS), a major step towards ensuring safety of bridges in the country.

Public Debt Management Cell (PDMC)

The Union Finance Ministry has set up a Public Debt Management Cell (PDMC)
with an objective of deepening bond markets in the country.

National Seismic Programme

The Union Government has launched National Seismic Programme (NSP) to trace
hydrocarbon resources like oil and natural gas in Mahanadi basin.

Mining Surveillance System (MSS)

Union Government has launched Mining Surveillance System (MSS), a pan-India


surveillance network to check illegal mining using latest satellite technology.

SC/ST hub and Zero Defect Zero Effect scheme

Prime Minister Narendra Modi launched the National SC/ST hub and the Zero Defect, Zero
Effect (ZED) scheme for Micro, Small and Medium Enterprises (MSMEs) at Ludhiana in
Punjab.

Kaushalya Setu

Maharashtra government has launched Kaushalya Setu, a skill development programme for
students.

UDAN Scheme

The Union Government has launched the regional air connectivity scheme UDAN
(Ude Desh ka Aam naagrik) which seeks to get more people to fly in the smaller towns.

Urja Ganga

Prime Minister Narendra Modi has laid the foundation stone of Urja Ganga, the highly
ambitious gas pipeline project in Varanasi, Uttar Pradesh.

International Household Survey Network (IHSN) Toolkit

The Union Minister for Statistics and Programme Implementation,


D.V. Sadananda Gowda has launched the International Household Survey Network (IHSN)
Toolkit.

L Narasimha Reddy Committee

The one-man judicial committee on One Rank One Pension (OROP) set up by the Union
Government, has submitted its report to Defence Minister Manohar Parrikar.

Munialappa Committee

2016 GKToday | All Rights Reserved | www.gktoday.in

Current Aairs: October 2016 [Date-wise Compendium]


The Union Agricultural Ministry has constituted a High-Level Committee to monitor the
bird flu situation and help state governments take steps to contain the disease.

Biotech-KISAN and Cattle Genomics Schemes

The Union Ministry of Science and Technology has launched two new schemes BiotechKISAN and Cattle Genomics in order to apply science to boost rural economy.

National Academic Depository

The Union Cabinet approved establishment and operationalisation of a National Academic


Depository (NAD). Under it all academic degrees, certificates and awards in the country will
be made digitally available for verification on single spot.

Himachal Pradesh

The State of Himachal Pradesh was officially declared Open Defecation Free (ODF). Every
individual household in the state has functional toilet under the Swachh Bharat Mission.
Science, Technology & Environment

Tamil Nadu
Pika

According to the recent publication released by the Botanical Survey of India (BSI), Tamil
Nadu accounts for the highest number flowering plants in the country.
Scientists have discovered a new species of Pika, a mammal belonging to the rabbit and hare
family in the Himalayas in Sikkim.

India ratifies Paris Climate Agreement

India has ratified the Paris Agreement on climate change on the 147th birth anniversary of
Mahatma Gandhi. India was the 62nd country to ratify the agreement. These 62 countries
including India are responsible for almost 52% of Green House Gases (GHG) emissions.

Communication satellite GSAT-18

Indias latest communication satellite GSAT-18 was successfully launched from the spaceport
of Kourou in French Guiana. The satellite was launched on board of heavy duty Ariane-5
VA-231 rocket of Arianespace.

LIGO India Project

Maharashtra Government has allocated 40.68 hectare land to Department of Atomic Energy
(DAE) to build LIGO (Laser Interferometer Gravitational Wave Observatory) India Project
at Dudhala village in Hingoli district.

Chiraharitae: new termite species

A new termite species Glyptotermes Chiraharitae was discovered at Kakkayam in the Malabar
Wildlife Sanctuary, Kerala. It has been named Chiraharitae after the tropical evergreen
forests of the Western Ghats, where it was spotted.

Great Balls of Fire (GBF)

NASAs Hubble Space Telescope (HST) has detected Great Balls of Fire (GBF), a mysterious
2016 GKToday | All Rights Reserved | www.gktoday.in

Current Aairs: October 2016 [Date-wise Compendium]


super-hot blobs of gas seen near a dying star.

Perovskite Nanocrystal solar cells

For the first time, Indian researcher has successfully produced a stable, high-efficiency, allinorganic perovskite nanocrystal solar cells.

Himansh

A high altitude glaciological research station in Himalaya called Himansh (meaning a slice of
ice) began functioning above 13,500 ft (4000 m) in a remote region in Spiti, Himachal
Pradesh.

High-tech electroactive bandage

The NASA (National Aeronautics and Space Administration) has developed a new hightech electroactive bandage that uses electricity to significantly promote healing of injured
wounds.

Kigali Amendment

A historic global climate deal was reached in Kigali, Rwanda at the Twenty-Eighth Meeting
of the Parties to the Montreal Protocol on Substances that Deplete the Ozone Layer
(MOP28). The so called Kigali Amendment which amends the 1987 Montreal Protocol aims
to phase out Hydrofluorocarbons (HFCs), a family of potent greenhouse gases by the late
2040s.

China launches its longest ever manned mission to space

China has successfully launched longest-ever manned mission by taking two astronauts
Jing Haipeng (50) and Chen Dong (37) into the orbit.

Kashmiri Red Stag

The International Union for Conservation of Nature (IUCN) is going to declare the Kashmiri
Red Stag (also known as Hangul) as a Critically Endangered species.

Smooth-coated otter

Smooth-coated otter was sighted for the first time in the mangrove forest near the Krishna
wildlife sanctuary (KWS) in Krishna district, Andhra Pradesh.

Banxing-2 micro-satellite

Chinas experimental space lab Tiangong-2 orbiting the Earth with two astronauts on board
has successfully launched a micro-satellite Banxing-2.

Antarticas Ross Sea

Antarticas Ross Sea was declared as worlds largest Marine Protected Area (MPA) to protect
the Earths most pristine marine ecosystem.
Defence Related

CORPAT

The 28th India and Indonesia Coordinated Patrol (CORPAT) and Second Bilateral Maritime
Exercise have commenced at Belawan, Indonesia in the Andaman Sea.
2016 GKToday | All Rights Reserved | www.gktoday.in

Current Aairs: October 2016 [Date-wise Compendium]

S-400 Triumf air defence systems

India has signed a defence deal worth nearly Rs 39,000 crore with Russia to purchase five state
of art S-400 Triumf air defence systems.

India completes nuclear triad

India has completed its nuclear triad by inducting the first indigenously built strategic nuclear
submarine INS Arihant into service.

INS Tihayu

Indian Navy has commissioned INS Tihayu, a Car Nicobar Class water jet fast attack craft
(WJFAC).

Sino India Cooperation 2016

India and China have conducted second joint military exercise Sino India Cooperation 2016
in Ladakh region along the Line of Actual Control (LAC) in Jammu and Kashmir.

ICGSs Aryaman and Atulya

The Indian Coast Guard Ships (ICGSs) Aryaman and Atulya were commissioned into
Indian Coast Guard in Kochi, Kerala.

Mitra Shakti 2016

The fourth edition of the India-Sri Lanka Joint Military Exercise MITRA SHAKTI 2016
began at Sinha Regimental Centre in Ambepussa, Sri Lanka.
Events & Observances

India Sanitation conference (INDOSAN)

Prime Minister Narendra Modi inaugurated India Sanitation conference (INDOSAN) in New
Delhi ahead of second anniversary of Swachh Bharat Mission.

30 September: The International Translation Day

The International Translation Day (ITD) is observed every year on September 30 to promote
the translation profession in different countries.

2 October: International Day of Non-Violence

The International Day of Non-violence is being celebrated on 2 October throughout the


world on the birth anniversary of Mahatma Gandhi.

3 October: World Habitat Day

The World Habitat Day (WHD) is observed every year on the first Monday of October
throughout the world. This year it was observed on 3 October 2016.

World Sustainable Development Summit (WSDS)

President Pranab Mukherjee inaugurated the first edition of World Sustainable Development
Summit (WSDS) at Vigyan Bhawan in New Delhi.

Asian Ministerial Conference for Disaster Risk Reduction

India is hosting the 2016 Asian Ministerial Conference for Disaster Risk Reduction
(AMCDRR) from November 3 to 5, 2016 in New Delhi. This will be second time India
hosting AMCDRR.
2016 GKToday | All Rights Reserved | www.gktoday.in

Current Aairs: October 2016 [Date-wise Compendium]

October 10: World Mental Health Day

The World Mental Health Day (WMHD) is observed every year on 10 October to raise
awareness about mental health issues around the world and mobilizing efforts in support of
mental health.

October 9: World Post Day

The World Post Day is observed each year on October 9 to spread awareness about the postal
services and their role in the everyday lives of people and businesses.

October 11: International Day of the Girl Child

The International Day of the Girl Child (IDGC) is observed every year across the world on 11
October to recognize girls rights and the unique challenges girls face around the world.

October 13: International Day for Disaster Reduction

The International Day for Disaster Reduction (IDDR) is observed annually on 13 October
across the world encourage citizens and governments to take part in building more disaster
resilient communities and nations.

15 October: International Day of Rural Women

The International Day of Rural Women is observed annually across the world on 15 October
to recognize the critical role and contribution of rural women.

17 October: International Day for the Eradication of Poverty

The International Day for the Eradication of Poverty is observed globally on 17 October for
eradication of poverty throughout the world.

16 October: World Food Day

The World Food Day is observed globally on 16 October to mark foundation of Food and
Agriculture Organisation (FAO) of the United Nations in 1945.

International Conference on Voter Education

The Election Commission of India hosted the first ever International Conference on Voter
Education in New Delhi.

October 24: United Nations Day

The United Nations Day is being observed globally on October 24. The day marks the
anniversary of the entry into force of UN Charter (the founding document of UNO) in 1945.

First National Tribal Carnival

Prime Minister Narendra Modi inaugurated the first National Tribal Carnival in New Delhi
to promote a sense of inclusiveness among the tribals.

28 October: National Ayurveda Day

The first National Ayurveda Day was observed on 28 October on the occasion
of Dhanwantari Jayanti. The theme for this year is Ayurveda for Prevention and Control of
Diabetes.
Finance, Business and Economy
2016 GKToday | All Rights Reserved | www.gktoday.in

Current Aairs: October 2016 [Date-wise Compendium]

Indias first Medipark

Indias first Medipark (Medical Park) will be set up in Tamil Nadu at Chengalpet near
Chennai, Tamil Nadu. It will boast of the first medical devices manufacturing park in the
country.

Indias first International Arbitration Centre

The first International Arbitration Centre of India was inaugurated in Mumbai, Maharashtra
to provide an arbitration platform for Indian business houses to negotiate commercial
disputes.

India ranks 97th in 2016 Global Hunger Index

India has been ranked low 97th among the 118 countries surveyed in 2016 Global Hunger
Index (GHI). In 2016 GHI, India has scored low 28.5 on a 0-100 point scale of the index.

Agro irradiation centres

India and Russia have signed an agreement for cooperation in the development of a network
of 25 integrated infrastructure Agro irradiation centres.

Eastern Dedicated Freight Corridor (EDFC)-III project

The Union Government has signed $650 million loan agreement with the World Bank for
construction of the Eastern Dedicated Freight Corridor (EDFC)-III project.

Cyrus Pallonji Mistry

The board of Indias largest conglomerate Tata Sons Ltd has removed
Cyrus Pallonji Mistry (48) as Chairman. However he will remain a director of the individual
companies.

India ranks 130th in 2017 Ease of Doing Business index

India has been placed at 130th position among the 190 countries in the recently released
World Banks ease of doing business index for the year 2017.

India ranks 87th on the WEFs Global Gender Gap Report 2016

India has been ranked 87th out of 144 countries on the World Economic Forums (WEF)
Global Gender Gap Report 2016. India has climbed 21 spots from 108th position in 2015.
Awards and Honours

Uttam Singh

The noted music director and violinist Uttam Singh (68) has been named for
2016 Lata Mangeshkar Award for Lifetime Achievement by the Maharashtra government.

Vinaya Manchaiah
Murga

Indian-American associate professor from US, Vinaya Manchaiah has been named as the
Future Leader of Audiology.
Murga has won the first prize at the Swachh Bharat Short Film Festival (SBSFF). The short
film was directed by young filmmaker Katyayan Shivpuri from Maharashtra.
2016 GKToday | All Rights Reserved | www.gktoday.in

Current Aairs: October 2016 [Date-wise Compendium]

Kiara Nirghin

Indian-origin South African Kiara Nirghin (16) has won a $50,000 scholarship at the annual
Google Science Fair held in United States.

Prajna Chowta

Film-maker and elephant researcher Prajna Chowta (46) has been appointed Chevalier
de lOrdre National du Mrite (Knight in the National Order of Merit) by the French
government.

Yoshinori Ohsumi

Yoshinori Ohsumi (71) of Japan has won the 2016 Nobel Prize for physiology or Medicine for
his pioneering work on autophagy.

Nayanjot Lahiri

History writer Nayanjot Lahiri has won the prestigious 2016 John F. Richards Prize for her
book Ashoka in Ancient India.

David Thouless, F Duncan M Haldane and Michael Kosterlitz

British trio of physicists David Thouless, F Duncan M Haldane and Michael Kosterlitz have
won the 2016 Nobel Prize in Physics.

Ramachandra Rao

Former ISRO Chairman and space scientist Prof Udupi Ramachandra Rao became the first
Indian to be inducted in the Hall of Fame of International Astronautical Federation (IAF).

Jean-Pierre Sauvage, J Fraser Stoddart and Bernard L Feringa

Trio of Jean-Pierre Sauvage, J Fraser Stoddart and Bernard L Feringa have won the
prestigious 2016 Nobel Prize in Chemistry for their individual efforts in developing molecular
machines.

Pawan Chamling

Sikkim Chief Minister Pawan Chamling was honoured with prestigious Sustainable
Development Leadership Award of the The Energy and Resources Institute (TERI).

Juan Manuel Santos

The 2016 Nobel Peace Prize has been awarded to Colombian President Juan Manuel Santos
for his resolute efforts to bring the more than 50-year-long civil war of Colombia to an end.

Oliver Hart and Bengt Holmstrom

Two economists Oliver Hart and Bengt Holmstrom have won the 2016 Nobel Memorial Prize
in Economic Sciences for their contributions to contract theory which tells how contracts
help people deal with conflicting interests.

Bob Dylan

American singer-songwriter, artist and writer Bob Dylan (75) has won the 2016 Nobel Prize
in Literature. He is the first American to win the prize since novelist Toni Morrison in 1993.

Sir David Cox

2016 GKToday | All Rights Reserved | www.gktoday.in

10

Current Aairs: October 2016 [Date-wise Compendium]


Noted British statistician Sir David Cox (92) was awarded inaugural recipient of the
International Prize in Statistics.

C Radhakrishnan

Eminent writer C Radhakrishnan was selected for the prestigious Mathrubhumi Literary
Award for the year 2016.

Paul Beatty

Paul Beatty (54) has won the 2016 Man Booker Prize. With this he becomes the first US
author to win this prestigious literary award.

Nadia Murad Basee and Lamiya Aji Bashar

Two Yazidi women Nadia Murad Basee and Lamiya Aji Bashar have been selected for
European Unions prestigious Sakharov Prize for Human Rights for year 2016.
Sports

Jitu Rai
Brazil

Ace Indian Shooter Jitu Rai has won the 2016 Champion of Champions pistol Award of the
International Shooting Sport Federation (ISSF) for pistol shooting.
Brazil has won the inaugural BRICS U-17 Football Tournament 2016 held at Goa on the
occasion of the 8th BRICS hosted in India.

Saina Nehwal

Ace Indian shuttler Saina Nehwal has been appointed as a member of the International
Olympic Committees (IOC) Athletes It is rare honour for an Indian sportsperson.

2016 Kabaddi World Cup

India has won the 2016 Kabaddi World Cup by defeating Iran by 38-29 score in the final
match played at the Arena By TransStadia in Ahmedabad,Gujarat.
Miscellaneous Terms and Facts

SAAR Summit

Pakistan has postponed the 19th South Asian Association for Regional Cooperation (SAARC)
Summit after India and four other countries announced to boycott the Summit.

BRICS New Delhi Declaration on Education

The BRICS nations have adopted New Delhi Declaration on Education to ensure inclusive
and equitable quality education and promote life-long learning opportunities for all.

HIV and AIDS (Prevention and Control) Bill, 2014

The Union Cabinet has given its approval to introduce amendments to the HIV and AIDS
(Prevention and Control) Bill, 2014.

BRICS Contingent Reserve Arrangement

The Contingent Reserve Arrangement (CRA) of BRICS nations viz. Brazil, Russia, India,
China and South Africa was declared operational.
2016 GKToday | All Rights Reserved | www.gktoday.in

11

Current Aairs: October 2016 [Date-wise Compendium]

Heisnam Kanhailal

Renowned Manipuri theatre legend Heisnam Kanhailal passed away. He was actively been
involved in threatre for the last 40 years in different roles of a playwright, director and also an
actor.

October 1, 2016
Infosys unveils Skava Commerce, a modular e-commerce platform
October 1, 2016

Indias second largest software services major Infosys has launched Skava Commerce a new
standard for modern, mobile-first and modular e-commerce platform for retailers.
The platform has been developed by Skava, a Silicon Valley-based e-commerce start-up that Infosys
had acquired in June 2015 for 120 million dollars.
Key Facts
The Skava Commerce platform can help brick-and-mortar retailers to quickly launch new
offerings, improve conversion rates of traffic coming from digital channels.
It can integrate with existing technologies, while providing a future-ready architecture that
will enable next generation shopping experiences.
It leverages Artificial Intelligence (AI) and machine learning, natural language processing and
virtual reality (VR).
It provides suite of e-commerce micro-services that can be used stand alone or in conjunction
with others and can be implemented without downtime.
Skava Commerces modern architecture scales as traffic increases in order to provide a
consistent experience across all channels.

EU ministers approve ratification of Paris climate Agreement


October 1, 2016

The European Union (EU) ministers have approved the ratification of Paris Agreement (on Climate
Change). This decision brings the Paris Agreement closer to entering into force.
Once approved by the European Parliament, the EU deposit its ratification instrument before
national ratification processes which are completed in each member state. As of now, only 5 EU
nations have ratified the agreement at the national level.
Comment
The 28 EU member nations together account for close to 12% of global greenhouse gas
(GHG) emissions.
The addition of EU will take the cumulative emissions of ratifying parties of the agreement
well beyond the 55% minimum required for the treaty to enter into force.
The Paris Agreement has been already crossed the first condition of 55 countries ratifying the
2016 GKToday | All Rights Reserved | www.gktoday.in

12

Current Aairs: October 2016 [Date-wise Compendium]


treaty to enable its entry into force.
As of now, 61 countries, accounting for almost 48% of global GHG emissions have ratified
the deal.
India, which accounts for 4.1% of global GHG emission also has agreed to ratify it on October
2, 2016.
Together with the EU, total global GHG emissions covered by ratifying parties will make the
treaty ready to enter into force.

OPEC agrees on modest oil production curbs


October 1, 2016

The Organisation of Petroleum Exporting Countries (OPEC) reached an agreement to cut oil
production for the first time since 2008 after an informal meeting in Algiers, Algeria.
It was decided that OPEC would reduce output to a range of 32.5 to 33.0 million barrels per day
(bpd) from its current output at 33.24 million bpd.
However, how much each country will or reduce its output will be decided at the OPECs next formal
meeting scheduled in November 2016. In this meeting, special invitation will be sent to Russia (nonOPEC member) to join cuts in production.
Impact on India
India, being the 3rd largest importer of crude oil imports 85% of total oil and 95% of natural
gas from OPEC nations.
In recent time due to cheaper oil prices in international market due to overproduction and
non-coordination among OPEC countries Indian economy had immensely benefited.
However, this decision may result in spike in oil prices which can have major implications for
the Indias current account deficit and overall economy in general.
In recent times, lower oil prices kept the Indian economy on the shining path and managed to
keep inflation under control making it fastest growing economy in G20 countries.

About Organization of the Petroleum Exporting Countries (OPEC)


The OPEC is an intergovernmental organization of 14 oil-exporting developing nations that
coordinates and unifies the petroleum policies of its member countries.
Established: 1960 in Baghdad, Iraq by the first five members.
Headquarters: Vienna, Austria.
OPEC Members: Algeria, Angola, Libya, Nigeria and Gabon (from Africa); Indonesia, Iran,
Iraq, Saudi Arabia (the de facto leader) Kuwait, Qatar, United Arab Emirates (from Asia);
Ecuador and Venezuela (from Latin America).
As of 2015, these 14 OPEC member countries accounted for an estimated 43% of global oil
production and 73% of the worlds oil reserves.
2016 GKToday | All Rights Reserved | www.gktoday.in

13

Current Aairs: October 2016 [Date-wise Compendium]


Two-thirds of OPECs oil production and reserves are in its six Middle Eastern (west Asian)
countries that surround the oil-rich Persian Gulf.`

PM Narendra Modi inaugurates India Sanitation conference in New Delhi


October 1, 2016

Prime Minister Narendra Modi inaugurated India Sanitation conference (INDOSAN) in New Delhi
ahead of second anniversary of Swachh Bharat Mission.
PM Modi in his inaugural speech highlighted need of behavioural change to achieve the dream of
Swachch Bharat and also need to ensure a dirt free India like Mahatma Gandhis Satyagraha which
was instrumental for the end of colonialism.
Cleanliness awards
The Prime Minister also gave away cleanliness awards to Sindhudurg District (Maharashtra), Mandi
district (Himachal Pradesh), Pune, Chandigarh and Mysore Municipal Corporations, Gangtok city
(Sikkim), National Cadet Corps (NCC), Surat railway station, PGIMER Chandigarh, heritage site
Rani ki vav in Gujarat and Kendriya Vidyalaya FRI Dehradun.

About India Sanitation conference (INDOSAN)


INDOSAN is envisaged as an annual national event, that seeks to brings together all
stakeholders working in sanitation.
Thus, it will be one platform for a shared vision for Government, NGOs, academicians,
researchers, partner agencies and corporates.
It will create an opportunity for collective understanding, collective vision of the key
elements of the Swachh Bharat Mission programme as sanitation is seen as a citizen
movement with involvement of all sectors of the society.
INDOSAN will discuss on all these approaches, learnings, emerging innovations for the
benefit of all.
Besides, each state will share their approaches most suitable to them to reach the goal of
making India Open Defecation Free (ODF) by the year 2019 which marks 150th Birth
Anniversary of Father of the Nation Mahatma Gandhi.

Sports Sector Gets the Infrastructure Status


October 1, 2016

The Union Finance Ministry has decided that sports infrastructure will be included under the
harmonised master list of infrastructure sub-sectors.
Earlier, the Ministry of Youth Affairs & Sports Affairs had moved the proposal for inclusion of
sports in the list to address the issue of deficit of sports infrastructure in the country.
Key Facts
It will include sports stadia and infrastructure for academies for training and research in
2016 GKToday | All Rights Reserved | www.gktoday.in

14

Current Aairs: October 2016 [Date-wise Compendium]


sports and sports-related activities in the infrastructure sub-sectors.
With this, sports sector becomes eligible for obtaining long term financial support from
banks and other financial institutions on same principle available to other infrastructure
projects.
It will also encourage private investment in a public good which has socio-economic
externalities in a country with young population.
It will also bolster investment in sports infrastructure sector which will contribute to the
economy and help in promotion of health and fitness of the people.
It will also provide opportunities for employment in the new and existing sectors. Besides,
investment from private sector will widen the platform from where the country can become a
sporting power in future.

Pakistan announces postponement of 19th SAARC Summit in Islamabad


October 1, 2016

Pakistan has postponed the 19th South Asian Association for Regional Cooperation (SAARC)
Summit after India and four other countries announced to boycott the Summit.
The summit was schedule to be held on November 9th and 10th, 2016 in Islamabad, capital city of
Pakistan.
Besides, it was also announced that new set of dates for holding the summit at Islamabad will be
announced soon by Nepal, the Chair of SAARC.
Comment
Pakistans decision is a boost for Indias diplomatic efforts to corner Pakistan over the terror attack at
Ur. It also comes a day after Pakistan suffered humiliating Indian cross-border raids (surgical strikes)
on terrorist launch pads in Pakistan-occupied Kashmir (PoK).
Background
Earlier India and three other countries Afghanistan, Bangladesh and Bhutan in a synchronized effort,
had informed the SAARC Secretariat that their leaders will not attend the summit citing concerns
over cross-border terrorism in the region. Later, Sri Lanka also expressed its unwillingness to attend
the Summit. The move by the five countries means the Summit cannot go ahead as the SAARC
charter mandates that all decisions of the regional bloc must be made unanimously. The absence of
even one member-state leads to the automatic postponement or cancellation of a Summit.

About the South Asian Association for Regional Cooperation (SAARC)


SAARC is regional intergovernmental organization and geopolitical union in South Asia. It
promotes development of economical and regional integration.
As of 2015, SAARC member countries compromise of 3% of the worlds area, 21% of the
worlds population and 9.12% of the global economy.
2016 GKToday | All Rights Reserved | www.gktoday.in

15

Current Aairs: October 2016 [Date-wise Compendium]


Member Countries: Bangladesh, Bhutan, India, Maldives, Nepal, Pakistan, Sri Lanka and
Afghanistan (joined in 2007).
Nine observer states: Australia, China, European Union, Japan, Iran, Mauritius, Myanmar,
South Korea, and United States.
Formation History: The idea for the SAARC was proposed by Ziaur Rahman, the then
President of Bangladesh on May 2, 1980.
The seven founding countries had met for first time in April 1981 and then for second time
in 1985 creating SAARC Charter.
The first SAARC summit was held in Dhaka (Bangladesh) in December 1985. The SAARC
Secretariat was established in Kathmandu (Nepal) on 16 January 1987.

Tamil Nadu tops list of endemic flowering plants: BSI


October 1, 2016

According to the recent publication released by the Botanical Survey of India (BSI), Tamil Nadu
accounts for the highest number flowering plants in the country.
The publication titled Endemic Vascular Plants of India also revealed that almost one of every four
species of flowering plants found in India are endemic to the country.
Key Facts
Of the total 18,259 flowering plants reported in the country, 4,303 (over 23%) are found only
in India or they are endemic to the country.
Of these, Tamil Nadu accounts for the highest number of species with 410, followed by
Kerala (357 species) and Maharashtra (278 species).
In case of the geographical distribution of endemic plants, Western Ghats tops the list with
about 2,116 species, followed by the Eastern Himalayas with 466 species. These two regions
are among the biodiversity hot spots of the country.
At least 37 species of Black plum Syzyguim (Jamun), 274 species of orchids and 10 varieties of
Musa (banana) are found only in the country.
Four different varieties of roses, 2 herbs and 2 climbers and 12 species of jasmines are
exclusively found in India.
There are 45 species of black pepper family, 19 species of ginger and 13 different kinds of
large cardamom endemic to India.
There are also 40 species of bamboos (Bambusoideae) endemic to India.
Some of the plants species are restricted to only certain areas of the country like an
insectivorous plant Nepenthes khasiana which is only found in the Khasi hills of Meghalaya.

2016 GKToday | All Rights Reserved | www.gktoday.in

16

Current Aairs: October 2016 [Date-wise Compendium]

Music Director Uttam Singh named for Maharashtras Lata Mangeshkar Award
October 1, 2016

The noted music director and violinist Uttam Singh (68) has been named for 2016 Lata Mangeshkar
Award for Lifetime Achievement by the Maharashtra government.
He has been selected for the prestigious award for his contributions in the field of music. He will be
presented this award later this year.

About Uttam Singh


He had stared his music at tender age. He had acquired his basic training from his father who
was a sitar player.
He later learnt to play violin and table. At the age of 15 he played violin for Mohammad Safi,
assistant of legendary Bollywood music composer Naushad.
Since then he had played for top music directors of the era, including Naushad, C.
Ramchandra, Roshan, S.D. Burman and later became the chief violinist for R.D. Burman.
Later he had joined hands with Jagdish Khanna. The Uttam-Jagdish duo became one of the
top music team and had worked for over 65 Bollywood films.
He also worked for various Tamil films with Ilaiyaraja as music arranger and in 1999 he had
composed music for the Malayalam film Prem Poojari.
Awards and Honours: International Indian Film Academy Award, Filmfare Award and
Screen Award for best music director, Filmfare Award, Screen Award and Zee Cine Award.

About Lata Mangeshkar Award


The coveted award was instituted by the Maharashtra government in honour of Bharat Ratna
Lata Mangeshkar in 1992.
It is awarded annually to a doyen of music. It carries a citation, a trophy and Rs 500,000 in
cash.

30 September: The International Translation Day


October 1, 2016

The International Translation Day (ITD) is observed every year on September 30 to promote the
translation profession in different countries.
Observance of the day seeks to show solidarity of worldwide translation community in an effort to
promote translation profession among all people across the world.
2016 theme: Translation and Interpreting: Connecting worlds.
To mark this day series of dedicated events, seminars and symposiums were organised across the
world.
Background
The International Translation Day (ITD) was established by the International Federation of
2016 GKToday | All Rights Reserved | www.gktoday.in

17

Current Aairs: October 2016 [Date-wise Compendium]


Translators (FIT) in 1953.
It is observed annually on 30 September on the feast of St Jerome, the Bible translator who is
considered as the patron saint of translators.
In 1991, FIT officially recognised ITD to show solidarity towards translation community in
an effort to promote the translation profession worldwide.

National Mission on Bioeconomy launched in Shillong, Meghalaya


October 1, 2016

The National Mission on Bioeconomy has launched in Shillong, Meghalaya by the Institute of Bioresources and Sustainable Development (IBSD).
The mission is unique to south-east Asia and India has become one of the few countries to tap bioresources.
Key Facts
The purpose of the mission is to boost rural economy by utilizing bio-resources. Besides, it
can be optimally utilized can create a large number of jobs at village level.
It also focuses on sustainable utilization of renewable biological resources for food, bioenergy and bio-based products through knowledge-based approaches.
It has potential to generate new solutions for the planets major challenges in the field of
energy, food, health, water, climate change and deliver social, economic and environmental
benefits.
Bioeconomy is a new concept and few countries like US, Canada, European Union (EU) and
Australia have started initiatives in this field. India is the fast-growing bioeconomy at US 35 billion
dollars in 2015, which can even rise to USD 100 billion in future.
The Institute of Bio-resources and Sustainable Development (IBSD): It functions under the
Department of Biotechnology, Union Ministry of Science and Technology.

Rajasthan becomes first state to adopt LED street lights under all ULBs
October 1, 2016

Rajasthan has become the first state in India to adopt Central Governments Street Lighting National
Programme (SLNP) in all its urban local bodies (ULB).
Under this programme, around 5 lakh conventional street lights were replaced with LED street lights
across the state.
Key Facts
The project was funded by Energy Efficiency Services Limited (EESL), a Public Energy
Services Company under the administration of the Union Ministry of Power.
Through implementation of this project, 152 million units of energy savings has been
achieved so far have been and cost saving of Rs. 60 lakh daily to some municipalities.
2016 GKToday | All Rights Reserved | www.gktoday.in

18

Current Aairs: October 2016 [Date-wise Compendium]


The energy saving has resulted in reduction of 225 tonnes of CO2 per day.
The SLNP was launched in January 2015. Under the programme around 3.5 crore conventional
street lights will be replaced with smart and energy efficient LED street lights by March 2019.

October 2, 3 , 2016
Murga wins first prize at Swachh Bharat Short Film Festival
October 3, 2016

Murga has won the first prize at the Swachh Bharat Short Film Festival (SBSFF). The short film was
directed by young filmmaker Katyayan Shivpuri from Maharashtra.
The Union Information and Broadcasting Minister M Venkaiah Naidu bestowed award upon
Katyayan with a certificate and cash prize of 10 lakhs rupees.
Key Facts
The SBSFF was organised and curated by National Film Development Corporation (NFDC)
on behalf of the Union Ministry of Information and Broadcasting.
The festival was the cinematic presentation of short films of not more than 3 minutes
duration, dedicated to the Swachh Bharat Mission initiative of Central Government.
It had received 4,346 entries in more than 20 languages from across the country, including
Punjab, Jammu and Kashmir, Uttar Pradesh, Telangana, Kerala and Tamil Nadu.

Indian-American Vinaya Manchaiah named Future Leader of Audiology


October 3, 2016

Indian-American associate professor from US Vinaya Manchaiah has been named as the Future
Leader of Audiology.
In this regard, he was inducted into the 2016 class of the Jerger Future Leaders of Audiology by the
American Academy of Audiology.
He is among the one of only a dozen individuals selected from across the US for this prestigious
honour.

About Vinaya Manchaiah


Manchaiah is associate professor at the Lamar University in Texas. He hails from South India,
Manchaiah and holds a PhD in disability research from Linkoping University, Sweden.
He is the co-founder of non-profit NGO Audiology India, for which he served as president
from 2011 to 2015. The organisation seeks to foster hearing and ear health care in India.
Audiology: It is a branch of science that studies hearing, balance and related disorders.
American Academy of Audiology: It is the worlds largest professional organisation of
audiologists. The members look to provide hearing care services through research, education, and
increased public awareness of hearing and balance disorders.
2016 GKToday | All Rights Reserved | www.gktoday.in

19

Current Aairs: October 2016 [Date-wise Compendium]

Black money worth Rs. 65,250 crore declared under Income Disclosure Scheme, 2016
October 3, 2016

The Central Board of Direct Taxes (CBDT) has received total disclosure of Rs. 65,250 crore black
money in the form of cash and other assets under the Income Disclosure Scheme, 2016.
The scheme launched in June 2016 has provided a one-time opportunity to those who had not paid
full taxes in the past. It had provided four-month window for declaring undisclosed income or black
money and ended on 30 September 2016.
Income Declaration Scheme
The scheme had provided a one-time opportunity to black money holders who had not paid
full taxes in the past to come clean by declaring their domestic undisclosed income and assets.
Declaration were made in respect (i) Any undisclosed domestic income. (ii) Any investment
in asset representing undisclosed income, upto financial year 2015-2016.
The undisclosed income or black money and assets declared was liable to total 45% tax of the
undisclosed income.
Benefits of Declaration: (i) No wealth tax on asset declared. (ii) No enquiry under Income Tax
Act or Wealth Tax Act. (iii) Immunity from the Benami Transaction (Prohibition) Act, 1988.
The scheme gave an opportunity to the defaulter to come clean by paying more than normal
tax and did not reward the dishonest tax defaulters.

Delhi Police launches Mobile App for senior citizens


October 3, 2016

The Delhi Police has launched a mobile application Delhi Police Senior Citizen for senior citizens
to provide immediate assistance to them in distress.
The facility was launched by Delhi Lieutenant Governor Najeeb Jung at a function in New Delhi on
the occasion of International Day of Older Persons (observed on October 1).
Key Features of app
The application has an SOS button integrated with a dedicated existing helpline for Senior
Citizen 1291.
The app has been designed with features that allow users to press the SOS button in case of
an emergency. The call will reach a dedicated station.
If the user is unwell, an ambulance will be sent. In case for any other reason, a beat constable
will be asked to contact the user.
In case, the user does not respond to operators calls, a beat constable will swiftly visit the
house to verify the reason of distress.

2 October: International Day of Non-Violence


October 3, 2016

2016 GKToday | All Rights Reserved | www.gktoday.in

20

Current Aairs: October 2016 [Date-wise Compendium]


The International Day of Non-violence is being celebrated on 2 October throughout the world on
the birth anniversary of Mahatma Gandhi.
Observance of this day seeks to promote the principles of non-violence through education and public
awareness and is observed by all countries of the UN.
In India, this day is celebrated as Gandhi Jayanti to mark birth anniversary of Mahatma Gandhi who
was leader of Indian independence movement and also pioneer of philosophy of non-violence.

About International day of Non-Violence


The International Day of Non-violence was established by the UN General Assembly
(UNGA) by passing the resolution in 2007 in an effort to spread the message of non-violence.
The resolution reaffirms the universal relevance of Mahatma Gandhis principle of nonviolence and desire to secure a culture of peace, tolerance, understanding and non-violence.

BRICS adopts New Delhi Declaration on Education


October 3, 2016

The BRICS nations have adopted the New Delhi Declaration on Education to ensure inclusive and
equitable quality education and promote life-long learning opportunities for all.
The declaration was adopted at fourth Meeting of BRICS Education Ministers Conference held in
New Delhi. It was attended by Ministers of Education and assigned representatives of BRICS nations
viz. Brazil, China, India, Russia and South Africa.
Key Features
Reiterates commitment to Sustainable Development Goal (SDG4) which aims to ensure
inclusive and equitable quality education and the Education 2030 Framework for Action.
Initiate actions to formulate country-specific targets within the broader scope of the SDG4.
Integrate the SDG4 related targets with sub-national and the national level education sector
development plans and programmes
Build capacity at the subnational and national levels for measuring and monitoring progress
towards SDG4 and corresponding targets using the indicators adopted by the UNGA.
Share the best practices available in BRICS nations on collaboration in education, innovation
and research through the BRICS Network University (BNU).
Organize annual conference of BNU in country of current BRICS Chair. Encourage
participation of more universities in BRICS University League to collaborative research and
facilitate student mobility.
Strengthening coordination within the BRICS TVET Working Group to develop national
reports and share experience related to workforce demands and supply of skilled personnel.
Information and communication technologies (ICTs) will be used for improving access to
education, teacher development, enhancing quality of teaching-learning process etc.
2016 GKToday | All Rights Reserved | www.gktoday.in

21

Current Aairs: October 2016 [Date-wise Compendium]


Developing a nodal institution within each country and also institutional network to share
ICT policies, e-resources, including e-Libraries and Open Educational Resources.

Abu Dhabi Crown Prince to be Chief Guest on 2017 Republic Day


October 3, 2016

The crown prince of Abu Dhabi, Sheikh Mohammed bin Zayed Al Nahyan will be the chief guest for
the 68th Republic Day celebrations on 26 January 2017.
In this regard, announcement was made the by Union Ministry of External Affairs (MEA)
Spokesman Vikas Swarup in New Delhi after the crown prince accepted invitation.
Earlier Chief guests of Republic Day parades: 2016- Francois Hollande (French President), 2015Barack Obama (US President), 2014- Shinzo Abe (Japanese Prime Minister), 2013- Jigme Khesar
Namgyel Wangchuck (King of Bhutan), 2012- Yingluck Shinawatra (Then Prime Minister of
Thailand), 2011- Susilo Bambang Yudhoyono (Then President of Indonesia).
It should be noted France holds distinction of being the guest of honour for the maximum five times
of Indian Republic Day parade followed by Bhutan (four times); Mauritius and Russia (three times
each).
India-UAE Relations
India-UAE Relations deeply rooted in history and strategic cooperation which is driven by
mutual aspirations of both countries.
UAE is lynch-pin of the economic, defence and strategic strands of the Indias coordinated
strategy to outreach west Asia.
UAE is Indias third largest trading partner after China and the United States. Bilateral trade
between UAE and India is around 60 billion dollars.
Besides, there are also more than 2.6 million Indians living in the UAE and their annual
remittance is estimated to be around 14 billion dollars.

New species of pika named Ochotona sikimaria discovered in Sikkim


October 3, 2016

Scientists have discovered a new species of Pika, a mammal belonging to the rabbit and hare family
in the Himalayas in Sikkim.
It has been identified as Ochotona sikimaria. It is quite distinctive from all other pika species based on
the fecal pellets and tissue samples.
Ochotona sikimaria Pika species looks similar to the Moupin Pika, genetically it is completely
different. Their specific differences are not visible in physical observation. But they are genetically
very different.
Key Facts
Pikas are members of the rabbit family and look like tailless rats.
2016 GKToday | All Rights Reserved | www.gktoday.in

22

Current Aairs: October 2016 [Date-wise Compendium]


They live in the mountains or in temperate regions.
They are a keystone species and ecosystem engineers.
They do not hibernate unlike other mammalian species inhabiting such cold climates.
The common name pika is basically used for any member of the Ochotonidae family.
What are keystone species?
Keystone species determine the ability of a large number of other species to survive in the ecosystem.
When they become extinct, it has cascading effect on extinction of other species. Example of
keystone species is bees as pollination of fruits and flowers is dependent on them.

India ratifies Paris Climate Agreement


October 3, 2016

India has ratified the Paris Agreement on climate change on the 147th birth anniversary of Mahatma
Gandhi.
Indias Permanent Representative to UN, Syed Akbaruddin handed over the Instrument of
Ratification signed by President Pranab Mukherjee to the United Nations in New York.
With this, India became 62nd country to ratify the agreement. These 62 countries including India
are responsible for almost 52% of Green House Gases (GHG) emissions.
The Paris Agreement on climate change will enter into force one month after 55 countries that
account for 55% of global GHG emissions ratify the agreement.
Background
The Paris Agreement was adopted by more than 190 nations at the 21st Conference of Parties
of UNFCCC (UN Framework Convention on Climate Change) held in Paris in December
2015.
It seeks to accelerate and intensify the actions and investment needed for a sustainable low
carbon future.
The agreement caps limit global temperature rise (global warming) to well below 2 degrees
Celsius. It also seeks to funnel trillions of dollars to poor countries facing climate catastrophe.
Way Forward
India being worlds third largest emitter of GHGs, accounting for 4.1% of the total global emission
was seeking more time earlier to complete its national processes as it feared that any hasty decision
may impact its developmental projects. Indias Intended Nationally Determined Contributions (INDC)
pledge estimated a cost 2.5 trillion dollars for its climate-action plan. India will ask developed nations
to provide 100 billion dollars per year in climate finance for developing nations.

3 October: World Habitat Day


October 3, 2016

The World Habitat Day (WHD) is observed every year on the first Monday of October throughout
2016 GKToday | All Rights Reserved | www.gktoday.in

23

Current Aairs: October 2016 [Date-wise Compendium]


the world. This year it was observed on 3 October 2016.
2016 Theme: Housing at the Centre. It aims to raise awareness about need for affordable housing for
all in urban areas, cities and towns.
Focus of the theme
Access to adequate housing is a global challenge due growing fast with urbanization.
Around one quarter of the worlds urban population continues to live in informal settlements
and slums.
An increasing number of urban dwellers, especially the poor and vulnerable groups are living
in precarious conditions.
Background
The United Nations General Assembly had every year on the first Monday of October as World
Habitat Day by passing resolution 40/202 in 1986. The observance of the day seeks to remind the
world of its collective responsibility for the habitat of future generations.

Indian-origin Kiara Nirghin wins 2016 Google Science Fair prize


October 3, 2016

Indian-origin South African Kiara Nirghin (16) has won a $50,000 scholarship at the annual Google
Science Fair held in United States.
He has won this prestigious scholarship for developing a cheaper super-absorbent material using
orange peel that helps soil retain water. She is Grade 11 student at private school St Martins.
Kiara Nirghins project
She had submitted her project titled No More Thirsty Crops that aimed at alleviating the
severe drought plaguing South Africa.
Her solution to the problem of drought is simple and uses the peels from orange and avocado
fruits, which were normally discarded.
She had developed super-absorbent polymers (SAPs), which absorb and carry about 300 times
its weight in liquid relative to their own mass. These SAPs are biodegradable and not costly
and does not contain harmful chemicals.
SAPs developed by her can retain large amounts of water, keep soil moist and also improve
crop growth without regular water supplements.
Google Science Fair: It is an annual programme of the search engine giant Google for any budding
scientists between the ages of 13 to 18 who are invited something to solve the worlds biggest
challenges using science and technology.

October 4, 2016
Kersti Kaljulaid elected as first female President of Estonia
October 4, 2016

2016 GKToday | All Rights Reserved | www.gktoday.in

24

Current Aairs: October 2016 [Date-wise Compendium]


The parliament of Estonia has elected Kersti Kaljulaid (46) as the President of the Baltic country.
With this, she became Estonias first female president.
She had won vote 81-0, with 20 members absent in the parliamentary voting. She will be Estonias
fifth President and is due to take office on 10 October 2016. She is also the youngest person to get
elected as President of Estonia.

About Kersti Kaljulaid


Kersti Kaljulaid is trained biologist specialising in genetics and also holds an MBA from
University of Tartu.
In the late 1990s she had worked as investment banker at Hansapank that was later taken
over by Swedbank.
In 1999, she had joined the office of then Estonian Prime Minister Mart Laar as an economic
policy adviser.
Since May 2011, she was serving as the representative of Estonia in the European Court of
Auditors (ECA).

Elephant researcher Prajna Chowta conferred with knighthood by France


October 4, 2016

Film-maker and elephant researcher Prajna Chowta (46) has been appointed Chevalier de lOrdre
National du Mrite (Knight in the National Order of Merit) by the French government.
She has been conferred with the Frances one of the highest civilian recognitions in recognition of a
life devoted to caring for wild Asian elephants.

About Prajna Chowta


She is the founder of Aane Mane Foundation, which has been researching and conserving
wild Asian elephants for the past 16 years.
As elephant researcher, she was instrumental in developing one of the first GPS collars and
an online monitoring system for elephants in India.
She has authored Elephant Code Book on captive elephant management as well as the French
book Enfant dElephant (Elephants Child) in 2014.
With this Prajna Chowta joins few other Indian women to be appointed a Knight by French
Government. Earlier in 2016, businesswoman Kiran Mazumdar-Shaw and actor Kamal Hassan were
recognised under the French governments Legion of Honour award.

Reliance, Dassault Aviation ink deal to set up joint venture


October 4, 2016

Anil Ambani led Reliance Group and French aerospace giant Dassault Aviation have signed deal to
set up a joint venture called Dassault Reliance Aerospace.
The Dassault Reliance Aerospace will execute offsets for the recently concluded India-France deal for
2016 GKToday | All Rights Reserved | www.gktoday.in

25

Current Aairs: October 2016 [Date-wise Compendium]


36 Rafale fighter jets.
It will be the largest offset contract to be executed by any defence company in India. The contract
value is estimated up to Rs. 30,000 crore.
Key Facts
The proposed strategic partnership between Dassault and Reliance will also focus on
promoting research and development (R&D) projects under the IDDM program.
IDDM stands for Indigenously Designed, Developed and Manufactured. It is programme on
the lines with the new provision introduced in Defence Procurement Procedure (DPP) 2016.
The JV will focus on aero structures, engines and electronics and intends to infrastructure
and supply chain in India for the aerospace sector.
Background
In September 2016, India and France had signed an Inter-Governmental Agreement (IGA) for 36
Rafale multi-role jets in fly-away condition worth 7.87 billion (about Rs. 59,000 crore). The deal
has a 50% offset clause to be executed by Dassault Aviation and its partners in India. Through the
offsets, government seeks to bring in much needed expertise to build the domestic defence and
aerospace industry.

Jharkhand becomes first state to implement DBT in Kerosene


October 4, 2016

Jharkhand has become the first state in the country to implement Direct Benefit Transfer (DBT) in
Kerosene Scheme.
The scheme is being implemented in four identified districts of the state from 1st of October 2017.
These districts are Chatra, Hazaribagh, Khunti and Jamtara.

About Direct Benefit Transfer in Kerosene (DBTK) scheme


Under the DBTK scheme, PDS kerosene will be sold at non-subsidised price and the subsidy
will be directly transferred to consumers directly into their bank accounts.
This initiative was launched by Union Petroleum and Natural Gas Ministry on the lines of a
similar DBT programme for LPG subsidy.
It aims at rationalising subsidy based on the approach to cut subsidy leakages but not the
subsidy.
It will give priority to eliminate subsidized Kerosene from supply chain for better targeting of
beneficiaries.
It also seeks to eliminate pilferage and black-marketing and cut down adulteration of the
Kerosene, a cheap cooking and lighting fuel with diesel.

Urban areas of Gujarat, Andhra Pradesh declared open defecation free


October 4, 2016

2016 GKToday | All Rights Reserved | www.gktoday.in

26

Current Aairs: October 2016 [Date-wise Compendium]


The Union Urban Development Ministry has declared urban areas of Gujarat, Andhra Pradesh open
defecation free (ODF). These are the first states to become open defecation free in urban areas.
It was announced on the occasion of second anniversary of the Swachh Bharat Mission and 147th
birth anniversary of Mahatma Gandhi observed on October 2, 2016.
Key Facts
In all, 180 and 110 cities and towns in Gujarat and Andhra Pradesh respectively were declared
ODF by their chief ministers.
So far, total of 405 out of the 4,041 cities and towns across India have claimed to have become
open defecation free.
The Union Government has set ambitious target of making 334 more cities open defecation
free by March 2017.
In rural India, Himachal Pradesh, Sikkim and Kerala have achieved more than 90% of toilet
construction in households.
But Bihar, Odisha and Jammu and Kashmir are still trailing with fewer than 30% of
households with toilets in rural areas.
What is Open defecation?
Open defecation means people defecate in open spaces such as fields, bushes, forests, open
bodies of water rather than using the toilet.
Hazards of Open Defecation: It is important factor in causing various diseases like intestinal
worm infections, diarrhoea, polio, hepatitis etc.
Prevention: Creating awareness about adverse impact of open defecation on health,
changing behavioural habit of people, building of community toilets etc.

About Swachh Bharat Mission


The Swachh Bharat Mission is the pet project of NDA Government led by Prime Minister
Narendra Modi.
It was launched on 2 October 2014 to make India clean and open defecation free by 2019 to
mark Mahatma Gandhis 150th birth anniversary.
The current cleanliness drive aims to end the wide-spread practice of open defecation, build
more toilets and improve waste management.

Japans Yoshinori Ohsumi wins 2016 Nobel Prize in Medicine


October 4, 2016

Yoshinori Ohsumi (71) of Japan has won the 2016 Nobel Prize for physiology or Medicine for his
pioneering work on autophagy.
With this he becomes the 23rd Japanese national to win a Nobel prize and overall the sixth Japanese
medicine Nobel laureate.
2016 GKToday | All Rights Reserved | www.gktoday.in

27

Current Aairs: October 2016 [Date-wise Compendium]


Mr. Ohsumi had received a PhD from the University of Tokyo in 1974. Currently, he is a professor
at the Tokyo Institute of Technology (TIT).
What is Autophagy?
Autophagy is a process whereby cells eat themselves. It is a fundamental process in cell
physiology dealing with how the body breaks down and recycles cellular components.
It is essential for the orderly recycling of damaged cell parts and its better understanding has
major implications for health and disease, including cancer.
It was first observed by Belgian scientist Christian de Duve who had won Nobel Medicine
Prize in 1974 for it.
Christian de Duve had coined the term autophagy, which comes from the Greek meaning
self-eating.
Yoshinori Ohsumis Research in Autophagy
Ohsumis discoveries in Autophagy have led to a new paradigm in the understanding of how
the cell recycles its content.
In his research, Mr. Ohsumi had used bakers yeast to identify genes essential for autophagy.
He explained the mechanisms for autophagy in yeast and showed that similar sophisticated
machinery is used in human cells.
Significance: Ohsumis research had located the genes that regulate this self-eating process
and also related that errors in these genes can cause disease.
His findings have opened new path to understand importance of autophagy in many
physiological processes, such as how body adapts to starvation or responds to infection.
It has helped to establish links to Parkinsons disease, type 2 diabetes and other disorders that
appear in the elderly.

About Nobel Prize in Physiology or Medicine


The Nobel award for medicine is given to persons whose discoveries have significantly
enhanced the understanding of life or the practice of medicine.
The winners are chosen by the Nobel Assembly at the Karolinska Institute and are always
announced before the Nobel Prize for other categories.
The prestigious award carries prize money of 8 million Swedish kroner or 1.1 million dollars.
It is one of five Nobel Prizes established by Swedish chemist Alfred Nobel, the inventor of
dynamite in 1895.

SBI becomes first domestic bank to open branch in Yangon, Myanmar


October 4, 2016

Indias largest lender State Bank of India (SBI) has announced the opening up its foreign branch in
Yangon, the capital city of Myanmar. With this, SBI became the first domestic bank of India to open
2016 GKToday | All Rights Reserved | www.gktoday.in

28

Current Aairs: October 2016 [Date-wise Compendium]


a branch in Myanmar.
The Yangon branch is the 54th foreign branch of the SBI. This branch further extends the global
presence of Indias largest lender in 37 countries through 198 offices.
The Yangon branch was started after the Myanmarese central bank allowed SBI to open a branch
with the primary objective of extending wholesale banking services to foreign corporates.
Background
India has been a major trading partner of Myanmar for centuries. Since the signing of India and
Myanmar trade agreement in 1970, bilateral trade between two neighbours has been growing
steadily and rose from US 328 million dollars in 1997-98 to US 2.052 billion dollars in 2015-16.
Earlier SBI was also associated with Myanmar since 1861, when the erstwhile Bank of Bengal (BoB)
operated its branch in the then Rangoon. Later the operations of the Rangoon branch of SBI were
taken over by the Peoples Bank of Burma in February 1963 as part of bank nationalisation.

MS Sahoo takes charge as Chairman of Insolvency and Bankruptcy Board of India


October 4, 2016

The Union Finance Ministry constituted four-member Insolvency and Bankruptcy Board of India
(IBBI) with Financial markets expert MS Sahoo as its Chairman.
MS Sahoo was administered the oath of the office by Union Finance and Corporate Affairs Minister
Arun Jaitley in New Delhi. He will have tenure of five years i.e. till 2021.
Prior to this appointment, he was serving as a member of anti-trust regulator Competition
Commission of India (CCI). Earlier he also had served as member at capital markets regulator
Securities and Exchange Board of India (SEBI). He also had served as Chairman of the governmentappointed committees on depository receipts, domestic and overseas capital markets, and external
commercial borrowing (ECBs).
Other members of IBBI: Ajay Tyagi (Additional Secretary, Union Finance Ministry) Amardeep
Singh Bhatia (Joint Secretary, Union Ministry of Corporate Affairs), GS Yadav (Joint Secretary,
Department of Legal Affairs) and Unnikrishnan (Legal Advisor of the Reserve Bank of India).

About Insolvency and Bankruptcy Board of India (IBBI)


IBBI has been tasked to regulate functioning of insolvency professionals, insolvency
professional agencies and information utilities under Insolvency and Bankruptcy Code 2016.
The Code was notified by the Union Government in May 2016 replacing existing bankruptcy
laws.
It seeks to consolidate and amend laws relating to reorganisation as well as insolvency
resolution of corporate persons, individuals and partnership firms in a time-bound manner.
Under this new law, employees, creditors and shareholders will have powers to initiate
winding up process at first sign of financial stress such as serious default in repayment of
2016 GKToday | All Rights Reserved | www.gktoday.in

29

Current Aairs: October 2016 [Date-wise Compendium]


bank loan.

October 5, 2016
Nayanjot Lahiri wins 2016 John F Richards Prize
October 5, 2016

History writer Nayanjot Lahiri has won the prestigious 2016 John F. Richards Prize for her book
Ashoka in Ancient India.
The prestigious prize will be bestowed upon her at the American Historical Association (AHAs)
131st Annual Meeting to be held in Denver, Colorado in January 2017.
Her book Ashoka in Ancient India has been critically acclaimed for its riveting account of an emperor
Ashoka. She had deftly adjudicated between textual, archaeological, and geographical evidences to
offer a dazzling interpretation of Ashoka and the ancient world.

About Nayanjot Lahiri


Nayanjot Lahiri is an eminent historian and archaeologist of ancient India.
Currently, she is teaching at the newly created Ashoka University.
She has several research papers and publications to her credit.
Her research interests mostly include Indian archaeology, heritage studies, archeological
theory and ancient India.
For her work in archaeology, she was bestowed with the 2013 Infosys Prize in the
humanities.

About John F. Richards Prize


The John F. Richards Prize is awarded annually by the American Historical Association
(AHA).
The award has been named after John F. Richards (November 1938 to August 2007) who was
eminent historian of South Asia and in particular of the Mughal Empire.
It recognizes the most distinguished work of scholarship or book on South Asian history
published in English.

British trio David Thouless, F Duncan M Haldane and Michael Kosterlitz win 2016 Nobel
Prize in Physics
October 5, 2016

British trio of physicists David Thouless, F Duncan M Haldane and Michael Kosterlitz have won the
2016 Nobel Prize in Physics. They will share the 8 million Swedish kronor prize.
The Royal Swedish Academy of Sciences has selected them for their individual researches on
theoretical discoveries of topological phase transitions and topological phases of matter.
Key Facts
Their research work centres on Topology, a branch of mathematics involving step-wise
2016 GKToday | All Rights Reserved | www.gktoday.in

30

Current Aairs: October 2016 [Date-wise Compendium]


changes like making a series of holes in an object.
For example when matter goes from solid to liquid to gas different phases are obvious, but
materials can also undergo topological step changes which affect their electrical properties.
Such changes can be seen in a superconductor, which at low temperatures conducts electricity
without resistance.
These trio physicists had worked in the field of condensed matter physics and have
discovered totally unexpected behaviours of solid materials.
Based on their individual discoveries they came up with a mathematical framework in the
field of topology to explain these weird properties.
The discoveries have paved the way for designing new materials with all sorts of novel
properties that have significant potential revolutionize advances in electronics and future
quantum computers.
About Awardees
David J Thouless: He is Emeritus Professor at the University of Washington, Seattle, US.
Duncan M. Haldane: He is the Eugene Higgins Professor of Physics at Princeton University,
US.
J Michael Kosterlitz: He is the Harrison E. Farnsworth Professor of Physics at Brown
University, US.

RBI cuts repo rate by 25 bps in 4th Bi-monthly Monetary Policy Statement
October 5, 2016

The Reserve Bank of India (RBI) in its fourth bi-monthly monetary policy review for year 2016-17
has cut the repo rate by 25 basis points to 6.25%.
This monetary policy decision was taken by the newly constituted Monetary Policy Committee
(MPC). This was also Urjit Patels maiden monetary policy announcement as RBI Governor.
All the six members of MPC unanimously decided to cut key policy rate with the aim of achieving a
midterm inflation target of 4% within a band of plus or minus 2%. With this, RBI moved away from
tradition of RBI governor having the final say on monetary policy decisions.
Policy Rates
Repo rate under the liquidity adjustment facility (LAF): Reduced by 0.25 basis points to
6.25 percent.
Reverse repo rate under the LAF: It was adjusted to 5.75 per cent.
Marginal standing facility (MSF) and Bank Rate: It was adjusted to 6.75 per cent.
Reserve Ratios Cash Reserve Ratio (CRR) of scheduled banks: Unchanged at 4.0 per
cent of net demand and time liability (NDTL).
2016 GKToday | All Rights Reserved | www.gktoday.in

31

Current Aairs: October 2016 [Date-wise Compendium]


Statutory Liquidity Ratio (SLR):20.75 per cent.

Aadhaar card must for LPG subsidy after November 2016


October 5, 2016

The Union Government has made Aadhaar card mandatory for availing cooking gas (LPG) subsidies
after November 2016.
In this regard, the Union Petroleum Ministry has asked Oil marketing companies (OMCs) to
facilitate enrollments of Aadhaar of the consumers to avail the subsidy.
Currently, the government gives 12 LPG cylinders at subsidised rates per household in a year. The
subsidy on every LPG cylinder is transferred directly into bank accounts of individuals who then buy
the cooking fuel at market rates.
Why there is need of Compulsory Aadhar?
The LPG subsidy is an expenditure incurred from Consolidated Fund of India. The use of Aadhaar as
identifier for delivery of government services/subsidies/ benefits will simplifies the delivery
processes of government. It will also bring in transparency and efficiency in the delivery processes by
plugging leakages and enable beneficiaries to get their entitlements from Government directly to
them without any middlemen.

About Aadhaar
Aadhaar is a 12-digit unique number which acts as a proof of identity and address anywhere in the
country. It is considered as the worlds largest national identification project. Its objective is to collect
the biometric and demographic data of residents, store them in a centralised database. The unique
number is issued by the Unique Identification Authority of India (UIDAI).

Union Government launches Indian Bridge Management System


October 5, 2016

The Union Ministry of Road Transport and Highways has launched the Indian Bridge Management
System (IBMS).
It was launched by Union Minister of Road Transport and Highways Nitin Gadkari in New Delhi as
a major step towards ensuring safety of bridges in the country.
Key Facts
IBMS is being developed to create an inventory of all bridges in country and rate structural
condition of the bridges.
The rating will help for their timely repair and carrying rehabilitation work based on the
criticality of the structure.
IBMS is the largest platform in the world owned by a single owner, with database that could
exceed one lakh fifty thousand bridge structures.
How it will work?
2016 GKToday | All Rights Reserved | www.gktoday.in

32

Current Aairs: October 2016 [Date-wise Compendium]


Each bridge will be assigned a unique identification number or National Identity Number
(NIN) based on the state, RTO zone or located on National Highway, State Highway.
Bridge Location Number (BLN) will be also assigned to each bridge based on the precise
location of the bridge in terms of latitude-longitude is collected through GPS.
Thereafter, Bridge Classification Number (BCN) will be assigned to the bridge structure
based on the different engineering characteristics.
Each bridge will be assigned a Structural Rating Number (SRN) based on allocated numbers
which will be used for structural rating of bridge structures on a scale of 0 to 9.
The rating will take in consideration of each component of the bridge structure like integral
and non-integral deck, substructure, superstructure, structural evaluation, bank and channel,
deck geometry, waterway efficiency, vertical clearance, etc.
In addition Socio-Economic Bridge Rating Number will be also assigned to the bridges. It will
decide importance of bridge in relation to its contribution to daily socio-economic activity of
the area in its vicinity.
Based on this inventory of various numbers, IBMS will analyse data and identify bridges that
need attention.
Further physical inspection will be carried out to improve the operational availability of the
structure, enhance its life and prioritize repair and rehabilitation work.

India, Singapore sign three agreements


October 5, 2016

India and Singapore have signed three agreements (MoUs) in the fields of skill development and IPR
to boost bilateral relations between them.
These agreements were signed in New Delhi during Singapore Prime Minister Lee Hsien Loongs
official state visit to India.
Signed MoUs are
MoU in the field of Industrial Property Cooperation: It was signed between Department
of Industrial Policy & Promotion (DIPP) and Intellectual Property Office (IPO), Singapore
MoU on collaboration in the field of Technical and Vocational Education and
Training: It was signed between National Skill Development Corporation (NSDC) and
ITEES Singapore.
MoU on collaboration in the field of Technical and Vocational Education and
Training: It was signed between Government of Assam and ITEES Singapore.
Besides, delegation-level talks headed by Prime Minister Narendra Modi and Singapore Premier Lee
Hsien Loong were also held in New Delhi.
2016 GKToday | All Rights Reserved | www.gktoday.in

33

Current Aairs: October 2016 [Date-wise Compendium]


Key Highlights of delegation-level talks
India and Singapore decided to strengthen cooperation in countering threats of rising
terrorism.
India seeks to enhance cooperation with Singapore in key sectors including trade and
investment.
Highlighted importance of Defence and security cooperation as the key pillar of strategic
partnership between India and Singapore.
Raised concern over rising tide of terrorism, especially cross-border terrorism and the rise of
radicalisation as a grave challenges to our security.
India agreed to expedite the second review of Comprehensive Economic Cooperation
Agreement (CECA).

Former ISRO Chariman UR Rao becomes first Indian to be inducted into IAF Hall of Fame
October 5, 2016

Former ISRO Chairman and space scientist Prof Udupi Ramachandra Rao became the first Indian to
be inducted in the Hall of Fame of International Astronautical Federation (IAF).
He was inducted during the closing ceremony of the 67th IAF Congress 2016 held at Guadalajara,
Mexico for his outstanding contribution to the progress of astronautics.

About UR Rao
He had served as Chairman of Indian Space Research Organisation (ISRO) from 1984 to
1994.
Under his guidance, ISRO had designed more than 20 satellites including Indias first satellite
Aryabhatta.
Under his leadership, India had initiated the development of the geostationary launch vehicle
GSLV and the development of cryogenic technology in 1991.
He was also responsible for successful launch of INSAT satellites during his stint at ISRO.
He also had served as the first Chairman of Antrix Corporation, the commercial arm of the
ISRO. He has published more than 260 scientific and technical papers in various journals.
He had played pivotal role in promoting the use of space technology for broadcasting,
meteorology, education, disaster warning and remote sensing.
Awards and Honours: Padma Bhushan (1976). He was inducted into the Satellite Hall of
Fame, Washington in March 2013 by the Society of Satellite Professionals International.
IAF Hall of Fame: It consists of a permanent gallery of these personalities, including biographical
information, citation and a picture in a special part of the IAF web presence.

October 6, 2016
2016 GKToday | All Rights Reserved | www.gktoday.in

34

Current Aairs: October 2016 [Date-wise Compendium]

Union Cabinet approves MoU between India and AARDO


October 6, 2016

The Union Cabinet gave its approval to sign Memorandum of Understanding (MoU) between India
and African Asian Rural Development Organization (AARDO) for capacity building programmes in
the field of rural development.
The MoU is being signed under which capacity building programmes for AARDO member countries
for the triennium 2015 2017.
Key Facts
The capacity building programmes will be organized every year during the triennium at
various Institutions of Excellence in India.
These Indian institutions will include National Institute of Rural Development & Panchayati
Raj (NIRD & PR), Indian Institute of Management (IIM), institutions governed by the Indian
Council of Agricultural Research (ICAR) and others.
Under these programmes, the Post Graduate Diploma in Rural Development Management
Course at NIRD & PR will be for one year.

About African Asian Rural Development Organization (AARDO)


The AARDO is an autonomous inter-Governmental organization with its headquarters in
New Delhi. It was established in 1962.
Its main objective is to promote cooperation among countries of African-Asian Region in
direction of eradicating hunger, thirst, poverty, illiteracy and disease in region.
AARDO currently has 31 countries under its fold from the African-Asian Region.
India is one of the founder Members of AARDO and is the largest contributor in terms of
membership contribution.
India also has provided funds for building for AARDO Secretariat in Delhi and also
considerable financial assistance for maintenance of the building.
India also has provided 70 fully paid training scholarships for Human Resource Development
Programme for AARDO under the Indian Technical & Economic Cooperation Programme
(ITEC) to the Organization.

Union Cabinet approves MoU between India and European Union on water cooperation
October 6, 2016

The Union Cabinet has given its approval for the signing Memorandum of Understanding (MoU)
between India and European Union in the field of water resources.
The MoU envisages strengthening technological, scientific and management capabilities of India and
EU in field of water management on basis of reciprocity, equality and mutual benefit.
Key Facts
2016 GKToday | All Rights Reserved | www.gktoday.in

35

Current Aairs: October 2016 [Date-wise Compendium]


The MoU aims to identify key environmental issues and approaches to sustainable
development in the field of water management between EU and India.
Under it, exchange of experiences and cooperation will take place between India and EU to
strengthen and further develop cooperation in the field of water management.
It provides technical exchange on water issues including on integrated water resource
management plans within river basins and through study visits.
It envisions sustainable management of water resources in India for tackling challenges posed
by water management in the context of a changing climate, growing population, and
competing water demands.
Under the MoU, a Joint Working Group will be formed to monitor the activities to be
carried out for its fulfillment.
Background
The Union Ministry of Water Resources, River Development and Ganga Rejuvenation has
been envisaging bilateral cooperation with other countries in the field of water management
and resources development.
India has decided to benefit from experience and expertise from EU keeping in view of their
success in distribution of water resources, water pricing and water use efficiency
The EU member states have adopted water pricing policies to provide adequate incentives for
users to use water resources efficiently thereby contributing to environmental objectives.

Sushil Chandra appointed as Chairman of CBDT


October 6, 2016

The Union Government has appointed senior Indian Revenue Service (IRS) officer Sushil Chandra
as the Chairman of the Central Board of Direct Taxes (CBDT).
Decision in this regard was taken by the Appointments Committee of the Cabinet (ACC) chaired by
Prime Minister Narendra Modi in New Delhi.
Sushil Chandra is IRS officer of 1980 batch. Prior to this appointment he was serving as member of
CBDT. He will succeed Rani Singh Nair, who will retire on October 31, 2016.

About Central Board of Direct Taxes (CBDT)


CBDT is nodal policy-making body of the Income Tax (IT) department under the Union Finance
Ministry. It is a statutory authority established under The Central Board of Revenue Act, 1963. It is
also the supreme body in India for framing policies related to direct taxes. The composition of CBDT
includes Chairman and six members.

CCEA approves OVLs proposal to buy 11% stake in JSC Vankorneft


October 6, 2016

The Cabinet Committee on Economic Affairs (CCEA) has given approval ONGC Videsh Limited
2016 GKToday | All Rights Reserved | www.gktoday.in

36

Current Aairs: October 2016 [Date-wise Compendium]


(OVL) proposal to buy 11% stake in JSC Vankorneft.
OVL will buy this stake in JSC Vankorneft from Rosneft Oil Company (Rosneft), National Oil
Company of Russia which operates Vankor oil fields.
OVL will pay US 930 million dollars for acquiring 11% stake in Vankorneft. This acquisition is also
in line with ONGCs stated objective of adding high quality international assets to Indias Exploration
and Production (E&P) portfolio for augmenting Indias energy security.
Benefits
Provide OVL 3.2 Million Metric Ton of Oil Equivalent (MMTOE) by 2017.
Provide opportunity to Indian Oil and Gas companies especially PSUs to acquire new
technologies from Rosneft.
Background
Earlier in May 2016, OVL had completed the formalities on acquisition of 15% stake in
Vankorneft at a cost of US 1.284 billion dollars which gave OVL 4.11 MMTOE.
Besides, consortium of Indian oil companies comprising of Indian Oil Corporation Limited
(IOCL), Oil India Limited (OIL) and Bharat PetroResources Limited (BPRL) had acquired
23.9% stake in Vankorneft at a cost of US 2020.35 million dollars which gave them 6.56
MMTOE.

Indias communication satellite GSAT-18 successfully launched from French Guiana


October 6, 2016

Indias latest communication satellite GSAT-18 was successfully launched from the spaceport of
Kourou in French Guiana.
The satellite was launched on board of heavy duty Ariane-5 VA-231 rocket of Arianespace. It was
successfully injected into a Geosynchronous Transfer Orbit (GTO).
GSAT-18 was the 20th satellite from ISRO to be launched by the European space agency and overall
280th mission of the Arianespace rocket launcher family.
Key Features of GSAT-18
GSAT-18 is indigenously built by the Indian Space Research Organisation (ISRO). It weighs
around 3,404 kgs at lift-off. It has a mission life of about 15 years.
It carries 48 communication transponders including Ku-band beacon for accurately pointing
ground antennas towards the satellite.
These transponders will provide services in Upper Extended C-band, Normal C-band and
Ku-bands of the frequency spectrum.
GSAT-18 will provide services like television, telecommunication, VSAT and digital satellite
news gathering.
The satellite will play important role in strengthening ISROs current fleet of 14 operational
2016 GKToday | All Rights Reserved | www.gktoday.in

37

Current Aairs: October 2016 [Date-wise Compendium]


telecommunication satellites.
It will also enable continuity of vital communication services by replacing ageing satellites of
ISRO. Master Control Facility (MCF) of ISRO at Hassan, Karnataka will control the satellite.

Jean-Pierre Sauvage, J Fraser Stoddart and Bernard L Feringa win 2016 Nobel Prize in
Chemistry
October 6, 2016

Trio of Jean-Pierre Sauvage, J Fraser Stoddart and Bernard L Feringa have won the prestigious 2016
Nobel Prize in Chemistry.
Royal Swedish Academy of Sciences has chosen them for this award for their individual efforts in
developing molecular machines. These three laureates will share the 8 million Swedish kronor
(around $933,000) prize equally.
What are molecular machines?
Molecular machines or nanomachines are the worlds smallest machines.
Their working is inspired by proteins that naturally act as biological machines within cells.
Molecular machines are discrete number of synthetic molecular components fused together.
They produce quasi-mechanical movements in response to specific external stimuli such as
light or temperature change.
Molecular machines can be put to work as tiny motors, pistons ratchets or wheels to produce
mechanical motion and can move objects many time their size.
Future Potential Applications: Molecular machines can be developed to function as
artificial muscles to power tiny robots or even prosthetic limbs in case of Bionics.
They may lead to developments like new sensors, materials and energy storage systems.
They can be used to deliver drugs within the human body directly to target a specific area of
tissue to medicate or cancerous cells.
They can be used to design of a molecular computer which could be placed inside the body to
detect disease even before any symptoms are exhibited.
Contributions of
Jean-Pierre Sauvage (France): He had taken first step towards a molecular machine in
1983, after he successfully linkied together two ring-shaped molecules to form a chain.
J Fraser Stoddart (Britain): In 1991, he threaded a molecular ring onto a thin molecular
axle and successfully demonstrated that the ring was able to move along the axle.
Bernard L Feringa (Netherlands): He is the first person to develop a molecular motor. In
1999 successfully designed molecular rotor blade to spin continually in the same direction. He
also had designed nanocar using molecular motors.
2016 GKToday | All Rights Reserved | www.gktoday.in

38

Current Aairs: October 2016 [Date-wise Compendium]

Cabinet approves amendments to HIV and AIDS (Prevention and Control) Bill, 2014
October 6, 2016

The Union Cabinet has given its approval to introduce amendments to the HIV and AIDS
(Prevention and Control) Bill, 2014.
The Bill seeks to safeguard the rights of people living with HIV and affected by HIV. It seeks to
prevent social stigma and discrimination against people living with HIV (PLHIV).
It also seeks to strengthen legal accountability and establish formal mechanisms for inquiring into
complaints and redressing grievances to probe discrimination complaints against those who
discriminate against PLHIV.
Key Features of Bill
Prevent and control the spread of HIV and AIDS. It prohibits discrimination against persons
with HIV and AIDS.
No person will be compelled to disclose his HIV status except with his informed consent, and
if required by a court order
Establishments keeping records of information of PLHIV must adopt data protection
measures.
Obligations on establishments to safeguard rights of persons living with HIV arid create
mechanisms for redressing complaints.
Lists various grounds on which discrimination against HIV-positive persons and those living
with them is prohibited.
These include the denial, discontinuation, termination or unfair treatment with regard to
employment, educational establishments, health care services, renting property etc.
Prohibits, requirement for HIV testing as a pre-requisite for obtaining employment or
accessing health care or education.
Prohibits any individual from publishing information or advocating feelings of hatred against
HIV positive persons and PLHIV.
Background
There are approximately 21 lakh persons estimated to be living with HIV in India. The prevalence of
HIV is decreasing over the last decade but percentage of PLHIV receiving Anti-Retroviral therapy
(ART) treatment are merely 28.82% against global percentage of 41%.
Comment
The Bill would provide essential support to National AIDS Control Programme in arresting new
spread of HIV infections and thereby achieving the target of Ending the epidemic by 2030 to meet
goal Sustainable Development Goals.
2016 GKToday | All Rights Reserved | www.gktoday.in

39

Current Aairs: October 2016 [Date-wise Compendium]

October 7, 2016
All ASI protected historical monuments declared polythene-free zones
October 7, 2016

All Archaeological Survey of India (ASI) protected historical monuments and archaeological sites
have declared polythene-free zones.
Union Ministry of Culture has issued an advisory to all State Governments/UTs to support ASI in
keeping monuments polythene free up to 300 meters from protected boundaries.
Rani ki Vav (Gujarat), a 900-year-old World Heritage Site was declared as the cleanest iconic place
(among top 25 Adarsh Monuments) in the country by the Union Ministry of Culture.It was
presented with the Cleanest Iconic Place Award at the Indian Sanitation Conference (INDOSAN)
2016.
ASI had ranked these 25 Adarsh Monuments on the basis of Cleanliness parameters such as
amenities like toilets, Polythene Free Zone, green lawns, disabilities access, signage for awareness,
drinking water and provision for garbage bins etc.
Rani ki Vav (the Queens Stepwell): It was built in the 11th century AD as a memorial to a king
by Queen Udaymat. It is located in Patan district of Gujarat. It was added as part of the UNESCOs
World Heritage List in 2014.

RBI Panel moots easing bank branch norms


October 7, 2016

The Reserve Bank of India (RBI) Internal Working Group (IWG) on Rationalisation of Branch
Authorisation Policy has proposed easing bank branch norms. It was chaired by Lily Vadera.
It has proposed to relax norms that a bank branch has to follow, like a building, number of
employees etc to facilitate financial inclusion.
Key Recommendations
Bank branches including those manned by business correspondents providing minimum 4
hours of service for 5 days a week, should be allowed to be treated as a full-service branch.
Any other fixed point unit of the bank not complying with minimum working period should
be considered a part-time banking outlet
A part-time banking outlet can be opened in any centre. It will be counted in for computing
requirement of having 25% branches in rural areas.
Redefined the un-banked rural centre (URC) as a rural (tier V and VI) centre that does not
have a core banking solution-enabled banking outlet.
Comment
The acceptance of recommendations will significantly help to reduce costs for a bank while for
opening branches in un-banked rural centres. It will further help to enhance financial inclusion by
2016 GKToday | All Rights Reserved | www.gktoday.in

40

Current Aairs: October 2016 [Date-wise Compendium]


taking idea of bank as an outlet that delivers basic banking services. Banks, even without traditional
branches, can use technology to offer services in areas that so far had no access.

Sikkim CM Pawan Chamling honoured with Sustainable Development Leadership Award


October 7, 2016

Sikkim Chief Minister Pawan Chamling was honoured with prestigious Sustainable Development
Leadership Award of the The Energy and Resources Institute (TERI).
He was presented with the award by President Pranab Mukherjee during World Sustainable
Development Summit in New Delhi.
The award bestowed upon him in recognition of his vision and leadership in environment and
sustainable development that made Sikkim Indias first into a fully organic state.
Background
Sikkim is only state in India that has attained the official status of fully organic state in
January 2016 announced by Prime Minister Narendra Modi.
Around 75,000 hectares of land in Sikkim has been converted into certified organic farms
following the guidelines as prescribed by National Programme for Organic Production.
Sikkim contributes around 80,000 tonnes of organic production out of total 1.24 million
tonnes of recorded production in India.
For more details: Sikkim, Indias First Organic State

India seeks greater Pharma market access in Japan


October 7, 2016

India is seeking greater market access for the domestic pharmaceuticals sector in the Japanese market
as Indias share in the Japanese drug market continued to be below par.
It was announced by Union Commerce Minister Nirmala Sitharaman in seminar on promoting
Indias exports to Japan under India-Japan CEPA (Comprehensive Economic Partnership
Agreement) in New Delhi.
Why India seeks greater Pharma market access in Japan?
Indias strength in pharma sector is well established. Share of India in the Japanese drug
market continued to be low. It is limited mostly to active pharmaceutical ingredients (API).
Japanese pharmaceutical market offers a huge untapped potential for Indian pharma industry.
This is coupled with the decision of Japanese Government towards attaining an 80% share of
generic medicines by 2018.
This will provide an opportunity for the generic drug industry of India. Thus, it will be winwin situation for both countries as there is demand for generic medicines in Japan and India
has capability to meet this demand.
Background
2016 GKToday | All Rights Reserved | www.gktoday.in

41

Current Aairs: October 2016 [Date-wise Compendium]


Japan is one of the closest and most reliable trading partners of India.
Both countries bilateral ties were elevated to status of special strategic and global partnership
during Prime Minister Narendra Modi visit to Japan in 2014.
The CEPA between both countries is one of the most comprehensive agreements concluded
by India and it had come into force in August, 2011.
The bilateral trade between India and Japan pre-CEPA in 2010 was $10.4 billion and
currently in 2016 stands at $14.5 billion.
However, Indias trade deficit with Japan increasing from $3.1 billion before the CEPA was
inked in 2011 to $5.2 billion thereafter.

President Pranab Mukherjee inaugurates World Sustainable Development Summit


October 7, 2016

President Pranab Mukherjee inaugurated the first edition of World Sustainable Development
Summit (WSDS) at Vigyan Bhawan in New Delhi.
The summit was organised by The Energy and Resources Institute (TERI) with the theme Beyond
2015: People, Planet and Progress.
Key Facts
WSDS has replaced TERIs earlier called Delhi Sustainable Development Summit (DSDS).
The first DSDS was organised in 2005.
It underscored need for businesses and private sector to take lead in poverty reduction and to
ensure rapid and sustained adoption of Sustainable Development Goals (SDGs).
It had brought together Nobel laureates, decision-makers political leaders from around the
world to deliberate on issues related to sustainable development.
The aim of the summit is to provide various stakeholders a single platform in order to
provide long-term solutions for the benefit of the global community.
2016 WSDS also hosted events like World CEO Sustainability Summit, Thematic Tracks,
Media Colloquium, Youth Leaders and Greenovation Exhibition.
For more information: About TERI

India to host 2016 Asian Ministerial Conference for Disaster Risk Reduction
October 7, 2016

India is hosting the 2016 Asian Ministerial Conference for Disaster Risk Reduction (AMCDRR)
from November 3 to 5, 2016 in New Delhi. This will be second time India hosting AMCDRR.
The conference will be hosted by Union Government in collaboration with the United Nations
Office for Disaster Risk Reduction (UNISDR).
Key Facts
It will be first AMCDRR after advent of Sendai Framework for Disaster Risk Reduction
2016 GKToday | All Rights Reserved | www.gktoday.in

42

Current Aairs: October 2016 [Date-wise Compendium]


(SFDRRR) which was adopted at 3rd UN World Conference in Sendai, Japan in March,
2015.
India by hosting AMCDRR re-affirms its commitment to the cause of Disaster Risk
Reduction. It will also set the direction of Sendai Framework implementation in the region.
The aim of the conference is to transform the commitments of governments and stakeholders
during the Sendai Conference into national and local action.
AMCDRR 2016 will focus on consultation, collaboration and partnership with governments
and stakeholders to mainstream Disaster Risk Reduction (DRR) in the region.
It will adopt the Asian Regional Plan for Implementation of the Sendai Framework endorsed by
the Asian countries.
In this conference, senior-level delegations from Asian countries, representatives of UN
bodies and Disaster Management experts will participate.
Background
AMCDRR is a biennial conference jointly organized by different Asian countries and the UNISDR. It
was established in 2005. So far, 6 AMCDRR conferences have been organised. For the first time
India had also hosted the second AMCDRR in 2007 in New Delhi.

About Sendai Framework for Disaster Risk Reduction 2015-30


It is an international Treaty that was approved by UN member states in March 2015 at the
Third World Conference on Disaster Risk Reduction held in Sendai, Japan.
It is a voluntary and non-binding treaty which recognizes that the UN member State has the
primary role to reduce disaster risk. It has framework for 15-year i.e. 2015 to 2020.
It also calls for sharing the responsibility with other stakeholders including local government,
the private sector and other stakeholders.
It is successor of the Hyogo Framework for Action (20052015), which had been the most
encompassing international accord on disaster risk reduction.
It sets of common standards, a comprehensive framework with achievable targets, and a
legally-based instrument for disaster risk reduction.
It calls for adopting integrated and inclusive institutional measures for preventing
vulnerability to disaster, increase preparedness for response and recovery and strengthen
resilience.
Four specific priorities of Sendai Framework: (i) Understanding disaster risk (ii)
Strengthening disaster risk governance to manage disaster risk (iii) Investing in disaster risk
reduction for resilience (iv) Enhancing disaster preparedness for effective response and
recovery, reconstruction and rehabilitation.
2016 GKToday | All Rights Reserved | www.gktoday.in

43

Current Aairs: October 2016 [Date-wise Compendium]

Renowned Manipuri theatre legend Heisnam Kanhailal passes away


October 7, 2016

Renowned Manipuri theatre legend Heisnam Kanhailal passed away. He was 75.
He was actively been involved in threatre for the last 40 years in different roles of a playwright,
director and also an actor.

About Heisnam Kanhailal


Born on 17 January 1941 in Keisamthong Thangjam Lairak, Imphal.
He was the founder-director of Kalakshetra, the theatre laboratory in Manipur. He had
employed a very indigenous outlook of Manipuri culture in his theatre production.
His famous political play Pebet, styled to the tune of oral Manipuri folklores, was considered
as a revolutionary production.
Awards and Honours: Manipur state Award for Theatre, Sangeet Natak Akademi Award
(1985), Sangeet Natak Akademi Fellow (2011), META Life Time Achievement Award (2015)
and Padma Shri (2004) and Padma Bhushan (2016).

Maharashtra Government allocates land to LIGO India Project


October 7, 2016

Maharashtra Government has allocated 40.68 hectare land to Department of Atomic Energy (DAE)
to build LIGO (Laser Interferometer Gravitational Wave Observatory) India Project at Dudhala
village in Hingoli district.
With this, India will join elite league of countries consisting of US, UK, Italy, Germany and Japan
that support on-going research on gravitational waves.
Besides, it will be third such laboratory in the world and first outside US. The existing two LIGO
laboratories in US are located in Livingston, Louisiana and Hanford, Washington.
Key Facts
The LIGO-India Project will be piloted and overseen by Department of Atomic Energy
(DAE) and Department of Science and Technology (DST).
It will be international collaboration between the LIGO Laboratory of US and consortium of
three leading Indian institutions. They are
(i) Indores Raja Ramanna Centre for Advanced Technology (RRCAT): It will provide
its expertise in lasers and laser technology;
(ii) Punes Inter-University Centre for Astronomy and Astrophysics (IUCAA): It will
provide the scientific teams, scientific data computation and data acquisition;
(iii) Gandhinagars Institute for Plasma Research (IPR): It will contribute in cryogenic
and high vacuum systems for the prestigious project.
Background
2016 GKToday | All Rights Reserved | www.gktoday.in

44

Current Aairs: October 2016 [Date-wise Compendium]


In April 2016, India and US had signed an MoU to set up the LIGO Observatory. It was signed
between the scientists from the US National Science Foundation (NSF) and Indias Department of
Atomic Energy (DAE).
What are gravitational waves?
Gravitational waves are ripples in the curvature of spacetime which propagate as waves,
travelling outward from the source at the speed of light.
They transport energy as gravitational radiation and pass through matter without interacting
with it.
Gravitational waves were first predicted in 1916 by Albert Einstein on the basis of his Theory
of General Relativity.
Strongest sources of gravitational waves: They are among enigmatic objects in our
universe like black holes, supernova, neutron stars and Big Bang.
Significance: Information extracted by these transmitted waves will help to address unsolved
questions and mysteries of physics and astronomy.

Payment Banks need RBIs Prior product approval


October 7, 2016

The Reserve Bank of India (RBI) has notified entities that have been granted a payments bank (PB)
licence will need to take specific approval for products they would be offering to customers.
In this regard, RBI has issued separate operating guidelines for payments banks in view to focus on
financial inclusion.
Key Facts
Employee of Payment Bank should be available for sufficient duration at a fixed location to
attend customers. They must at least have 25% of access points in un-banked rural areas.
The main mandate of Payment Banks is to offer remittance services. They will be not allowed
to lend.
Payment Banks can also offer simple financial products like insurance and mutual funds.
The RBI may place suitable restrictions on the design, functioning, or other features of the
product of Payment Banks.
RBI may even discontinue the product launched by Payment banks if it feels that the product
is not suitable for customers.
RBI it will have no objection to payments banks making arrangements with other scheduled
commercial bank or small finance bank.
Background
RBI had granted in-principle licences to 11 payments banks in August 2015. While three out of 11
PBs have dropped out, others will have to start operations within 18 months of receiving in 2016 GKToday | All Rights Reserved | www.gktoday.in

45

Current Aairs: October 2016 [Date-wise Compendium]


principle approval.

India slams hold on declaring Masood Azhar global terrorist


October 7, 2016

India has reacted sharply to the extension of the technical hold on banning Jaish-e-Mohammed
(JeM) chief Masood Azhar by the United Nations 1267 Sanctions Committee.
It clearly mentioned that being a global body, UN will be sending a dangerous message if it fails to
act upon Indias demand for his designation as a global terrorist.
India also criticised the complete non-transparent and anonymous manner of designating individuals
by the UNSC 1267 Sanctions Committee.
It also mentioned that, UN Sanctions Committee has already proscribed Pakistan-based JeM a
terrorist organisation. However committee ignored the need to take action against the organisations
main leader, motivator and financier who continues his terrorist actions unhindered.
What is the case?
In March 2016, India told the 1267 Committee of UN to designate Masood Azhar as an
international terrorist citing to act against its main leader, financier and motivator of terrorist
organisation.
India had pressed case against Masood in UN following the January 2016 Pathankot attack
which it blamed on the outfit JeM. Masood was also blamed as the mastermind behind recent
September 2016 Uri attack in Jammu Kashmir.
However in March 2016, China a veto-wielding permanent member of UNSC had blocked
Indias move to put a ban on Azhar Masood under the al-Qaeda Sanctions Committee.
China was the sole member in the 15-nation UNSC to put a hold on Indias application while
the rest 14 members of the UNSC supported Indias bid.

About United Nations Security Council 1267 Committee


The UNSC 1267 Committee was established pursuant to resolution 1267 (1999). It is also
known as the AlQaida and Taliban Sanctions Committee.
It was established for the purpose of overseeing the implementation of sanctions measures
imposed on Taliban-controlled Afghanistan for its support of Osama bin Laden.
However in course of time, the 1267 sanctions regime has been modified and strengthened by
subsequent resolutions.
If an individual or terrorist organisation is included in this list, it helps in restricting their
movement, financial penalties and assets freeze among others.
The Committee comprises all 15 members of the UNSC and makes its decision by consensus
and secretly. If single member opposes it there is no consensus.
Thats why China opposition to Indias bid is not allowing UNSC to designate Masood Azhar
2016 GKToday | All Rights Reserved | www.gktoday.in

46

Current Aairs: October 2016 [Date-wise Compendium]


as an international terrorist and freeze his assets and travel ban.
The committee is being criticised for being non transparent and in recent time there is
demand for its reforms to address procedural shortcomings especially from India.

Union Government approves Rs.114 crore worth projects under HRIDAY Scheme
October 7, 2016

The Union Ministry of Urban Development has approved projects worth Rs.114 crore under
HRIDAY scheme for improving infrastructure facilities around core heritage sites in five cities.
These five cities are: Varnasi (Uttar Pradesh), Amritsar (Punjab), Dwaraka (Gujarat), Puri (Odisha)
and Warangal (Telangana).
Projects in these districts were selected based on recommendation of inter-Ministerial HRIDAY
National Empowered Committee.

About Heritage Infrastructure Development and Augmentation Yojana (HRIDAY)


HRIDAY is a Central Scheme that aims to preserve and rejuvenate the rich cultural heritage
of the country.
The scheme will be completely funded by the Central Government to create infrastructure
and provide facilities around the heritage sites to attract more tourists.
It aims to bring urban planning, economic growth and heritage conservation together for
heritage cities.
It seeks to promote an inclusive, sustainable and integrated development of heritage sites,
focusing on maintenance of monuments and advancement of their entire ecosystem.
Objectives of Scheme: (i) Bring urban planning, economic growth and heritage
conservation together for heritage cities. (ii) Beautification of heritage cities in an inclusive
and integrated manner with prime focus on livelihoods, skills, cleanliness, security, safety,
accessibility and faster service delivery. (iii) Guide conservation, restoration, future use and
development of heritage cities. (iv) Create improved connectivity platform and access to
tourists.
It will help to harness full potential of Indias tourism sector which has total 35 UNESCO
recognized natural, cultural and mixed heritage sites. India ranks second in Asia and fifth in
the world interms of heritage sites.

2016 Nobel Peace Prize awarded to Colombian President Juan Manuel Santos
October 7, 2016

The 2016 Nobel Peace Prize has been awarded to Colombian President Juan Manuel Santos. He was
selected for the prestigious award by the Norwegian Nobel Committee for his resolute efforts to
bring the more than 50-year-long civil war of Colombia to an end.
He is second Colombian to win Nobel Prize after writer Gabriel Gracia Marquez who had won the
2016 GKToday | All Rights Reserved | www.gktoday.in

47

Current Aairs: October 2016 [Date-wise Compendium]


literature Prize in 1982.
Santos is also second Latin American to receive Nobel Peace Prize after Guatemalas Rigoberta
Menchu had won it in 1992.
Peace Agreement
Manuel Santos had initiated the negotiations with the guerrilla (rebel) group Revolutionary Armed
Forces of Colombia (FARC). These negotiations had culminated into a peace agreement between the
Colombian government and the FARC guerrillas. However in October 2016, the peace deal was
rejected by a narrow majority by the Colombians when it was put to referendum.
About Juan Manuel Santos
Born on 10 August 1951 in Bogota, capital city of Colombia.
He was elected Colombian President for first time in 2010 and was re-elected in 2014.
Earlier he had served as defence minister from 2006 until 2009
Background
The civil war in Colombia is one of the longest civil wars in modern times and the sole
remaining armed conflict in the South America.
The five decade long civil war has cost the lives of at least 220 000 Colombians and displaced
close to six million people.
It was started in the 1960s as a rural uprising for land rights by FARC as communist-inspired
guerrilla movement.
The main aim of FARC was to reduce the gulf dividing rich and poor and land reform for
equality in the Andean country.
The conflict has drawn in various leftist rebel groups, drug gangs and right-wing
paramilitaries over the decades.
However in recent times, the FARC had shifted from their original ideology and were active
in illegal drug trade, kidnapping and extortion activities.

October 8, 2016
EC bars political parties from using public space for propagating symbol
October 8, 2016

The Election Commission of India (ECI) has issued directions barred political parties from using
public resources that amount to advertisement for parties or propagating their election symbols.
The ECI has issued these directions in the wake of the High Court July 2016 order and considering
the views of political parties.
Key Facts
ECI order bars political parties from using public space, public funds or government
machinery for carrying out activities propagate party election symbols or advertise them.
2016 GKToday | All Rights Reserved | www.gktoday.in

48

Current Aairs: October 2016 [Date-wise Compendium]


It has clarified that above directions violates the lawful direction of it within the meaning of
paragraph 16A of the Election Symbols (Reservation & Allotment) Order.
The clause 16A of the Order confers power on ECI to suspend or withdraw recognition of a
recognised political party fails to follow its lawful directions and instructions or to observe
Model Code of Conduct.
What do these directions mean?
Henceforth, no registered political party in the country shall either use or allow the use of any public
place or public funds or government machinery for carrying out any activity that would amount to
advertisement for the party or propagating partys allotted election symbol. It will further augment
ECIs goal of conducting free and fair election and level playing field for all stakeholders.
Background
The Delhi High Court in July 2016 had issued an order in Common Cause vs Bahujan Samaj Party
case requesting ECI to issue appropriate directions or guidelines for preventing the recognised
political party in power from using public resources for propagating the partys election symbol.
Delhi HC had mentioned that utilising public resources for promoting any political party or its
election symbol is antithetical to the concept of free and fair election and the principle of level
playing field for all stakeholders.

Election Commission partners with Facebook to encourage voter registration


October 8, 2016

The Election Commission of India (ECI) has collaborated with social networking giant Facebook to
register young voters in 5 states, which are going to polls in 2017.
These five states are Uttar Pradesh, Punjab, Goa, Manipur and Uttarakh.The purpose first-of-itskind voter registration drive is to encourage youth to participate in the democratic exercise.
Key Facts
The services of Facebook will be used to link all the eligible voters who are 18 years and
above to the National Voters Services Portal (NVSP).
Facebook users from these five states will receive a reminder in their Facebook accounts
News Feed sector to register to vote.
By clicking on the Register Now button, people will be directed to the NVSP which will
guide them through the registration process. Facebook has 148 million monthly active users
in India.

Maharashtra becomes 17th state to join UDAY Scheme


October 8, 2016

Maharashtra became 17th state to Central Governments Ujwal Discom Assurance Yojna (UDAY)
scheme.
2016 GKToday | All Rights Reserved | www.gktoday.in

49

Current Aairs: October 2016 [Date-wise Compendium]


It will help Maharashtra to reap benefits worth Rs 9,725 crore by way of cheaper funds, transmission
losses, energy efficiency and coal reforms during the period of the turnaround.
Under the UDAY Scheme, Maharashtra has committed to take more than 75% of the discoms noncapex debt of around 6,600 crore rupees in the current year.
The balance 25% debt will be converted into bonds or re-priced at cheaper rates which will reduce
the interest burden by 595 crore rupees.

About Ujwal DISCOM Assurance Yojna (UDAY)


UDAY Scheme was launched by Union Ministry of Power for financial restructuring of debt
of Power Distribution companies (DISCOMs).
It aims for financial revival and turnaround of DISCOMs and also ensures a sustainable
permanent solution to the problem.
Power DISCOMs of states by joining this scheme can convert their debt into bonds which
can be issued in market as well as roll out number of measures to improve efficiency at power
plants.
It will make DISCOMs financially and operationally healthy so they can supply adequate
power at affordable rates.
It will help in reduction in interest cost of DISCOMs and enforcing financial discipline in
DISCOMs through alignment with State finances.

SC stays commercial release of GM mustard


October 8, 2016

The Supreme Court has stayed the commercial release of Genetically Modified (GM) mustard crop
till October 17, 2016.
It has asked the Central Government to seek public opinion before releasing the variety for
cultivation purpose.
Order in this regard was issued by SC Bench of Chief Justice T.S. Thakur and Justice A.M.
Khanwilkar on petition filed by Aruna Rodrigues.
What petition says?
The petition had alleged that sowing of the GM Mustard seeds will be undertaken without
relevant tests and without entire bio-safety dossier for commercial launch GM mustard.
It also had urged the SC to prohibit open field trials and the commercial release of Herbicide
Tolerant (HT) crops, including HT Mustard DMH 11 and its parent lines/variants.
GM Mustard DMH-11
Mustard is one of Indias most important winter crops sown between mid-October and late
November. It a self-pollinating crop difficult to hybridise naturally as it cross-pollinate. It is
largest edible oil yielding crop of India.
2016 GKToday | All Rights Reserved | www.gktoday.in

50

Current Aairs: October 2016 [Date-wise Compendium]


DMH (Dhara Mustard Hybrid)-11 is genetically modified variety of mustard developed by
Centre for Genetic Manipulation of Crop Plants at Delhi University. It was Government
sponsored project.
But researchers at Delhi University have created hybridised mustard DMH-11 using barnase
/ barstar technology for genetic modification. It is Herbicide Tolerant (HT) crop.
In February 2016, the Genetic Engineering Appraisal Committee (GMEC) had allowed the
commercial production of another GM crop viz. Mustard DMH-11.
Arguments in Favour GM Mustard
DMH-11 yields about 30% more than a traditional reference mustard variety.
Help in boosting edible mustard oil production thus, reduce huge import bill for edible oil.
Help to boost government-led scientific researches in Agriculture.
Arguments against GM Mustard
Approval to GM mustard would open a gate to several genetically modified food crops.
Environmentalists are raising biosafety concern with GM crops as their introduction may
adversely affect environment, human and animal health.
As DMH-11 has external gene that makes the plant resistant to herbicide. Thus it will force
farmers to use only select brands of agro-chemicals.
Technical expert committee appointed by the Supreme Court in this regard earlier had found
that HT crops completely unsuitable in the Indian context.
The herbicide-resistant crops may adversely impact the manual labourers, for whom weeding
provides livelihood.

New termite species named Chiraharitae discovered


October 8, 2016

A new termite species Glyptotermes Chiraharitae was discovered at Kakkayam in the Malabar Wildlife
Sanctuary, Kerala.
It has been named Chiraharitae after the tropical evergreen forests of the Western Ghats, where it
was spotted.
Key Facts
Termites are of three types damp wood, dry wood and subterranean. Glyptotermes
Chiraharitae species are of the damp wood category.
They infest parts of woods with high moisture content, the decaying or rotting areas in
particular. They are exclusively wood dwelling and do not require any contact with soil.
The flying adults of this species are approximately 10 mm long, while the soldiers are around
9.5 mm long.
Its relatives are known to attack mango, sal, Rhododendron, banyan trees, Artocarpus, silver
2016 GKToday | All Rights Reserved | www.gktoday.in

51

Current Aairs: October 2016 [Date-wise Compendium]


oak, and jamun trees.
Compared with other insect groups, termites are not a multifarious speciose. In India there 285
species of termites and among them 61 are documented from Kerala.

India-Pakistan border to be sealed completely by 2018


October 8, 2016

Union Home Minister Rajnath Singh announced that India-Pakistan border will be completely
sealed by December 2018.
Announcement in this regard was made after Union Home Minister held meeting of representatives
of 4 states sharing border with Pakistan in Jaisalmer, Rajasthan.
Key Facts
Central government has formulated time-bound action plan to completely seal the entire
stretch of 3,323-km-long border between the two countries.
The process of sealing border will be monitored at Ministry of Home Affairs (MHA) level,
Border Security Force (BSF) level and chief secretary of states level.
Besides, a new concept of Border Security Grid (BSG) has been envisaged, in which all the
stake holders related to border security are involved. Based on their suggestions, a final shape
to BSG will be given.
Background
India-Pakistan share 3,323-km-long border which is termed as International Border. Border
between both countries was created based upon the Radcliffe line in 1947.
4 states share this international border with Pakistan. Of this 1,225 km falls in Jammu and
Kashmir (including Line of Control), 1,037 km in Rajasthan, 553 km in Punjab and 508 km in
Gujarat.
The Border Security Force (BSF) controls security of the 2,308 km-long border with Pakistan
from Gujarat to Jammu and Kashmir.
In Jammu, 192 km of International Border, referred to as a working boundary by Pakistan, is
manned by the BSF, while the remaining 8 km is secured by the Indian Army.
The 740 km Line of Control (LoC), running along Kashmir and separating Pakistan occupied
Kashmir (PoK) is entirely secured by the Army.

Adoption of GST tax will boost Indias medium-term growth: IMF


October 8, 2016

The International Monetary Fund (IMF) has asserted that adoption of GST (Goods and Services) tax
will boost Indias medium-term growth.
It was mentioned IMFs latest Asia Pacific regional economic update. It mentioned that India has
shown progress on reforms that could boost business investment and growth.
2016 GKToday | All Rights Reserved | www.gktoday.in

52

Current Aairs: October 2016 [Date-wise Compendium]


IMFs findings
Greater labour market flexibility and product market competition in India remain essential to
create jobs and raise growth.
Priorities also include effective implementation of the new corporate debt restructuring
mechanisms.
India has shown progress on reforms that could initiate business investment, including
already strong FDI inflows, which will further increase domestic demand.
Over the medium term, a number of Asian economies stand to benefit from a demographic
dividend.
In some Asian economies like India and Indonesia the working-age population continues to
grow, potentially helping sustain strong potential growth.
Indias growth continues to benefit from large improvement in terms of trade, positive policy
actions, including implementation of key structural reforms, enhancement in confidence and
gradual reduction of supply-side constraints.
What is GST?
GST is uniform single indirect tax regime throughout the country that has subsumed central
and state indirect taxes into single indirect tax.
It aims to remove indirect tax barriers across states and integrate the country into a single
common market, boosting government revenue and reducing business costs.
The Constitution (One Hundred and First Amendment) Act, 2016 gives the constitutional
status to GST.
For more information: Benefits of GST Regime

BRICS Contingent Reserve Arrangement declared operational


October 8, 2016

The Contingent Reserve Arrangement (CRA) of BRICS nations viz. Brazil, Russia, India, China and
South Africa was declared operational.
It was announced by Union Finance Minister Arun Jaitley who had chaired the Governing Council
meeting of the BRICS CRA in New York, United States.
What is BRICS Contingent Reserve Arrangement (CRA)?
CRA was established in 2015 during 7th BRICS summit in July 2015. The Treaty for the
establishment of BRICS CRA was signed at Fortaleza, Brazil in July 2014.
It is a framework that aims at providing support through additional liquidity and other means
to BRICS countries at a time of economic crisis.
It will be providing support to BRICS countries through liquidity and precautionary
instruments in response to actual or potential short-term balance of payments pressures.
2016 GKToday | All Rights Reserved | www.gktoday.in

53

Current Aairs: October 2016 [Date-wise Compendium]


The initial total committed resources of the CRA will be 100 billion dollars with individual
commitments as follows: China ($41 billion), India ($18 billion), Brazil ($18 billion), Russia
($18 billion) and South Africa ($5 billion).
The CRA is generally seen as a competitor to the International Monetary Fund (IMF) and
along with BRICS New Development Bank (NDB) is viewed as an example of increasing
South-South cooperation.
Both CRA and NDB were annoucned as part of BRICS Fortaleza Declaration announed
during 6th BRICS summit held in Fortaleza, Brazil in July 2014.
Commonwealth Finance Ministers Meeting
Union Finance Minister Arun Jaitley also had chaired the Commonwealth Finance Ministers
Meeting at the International Monetary Fund (IMF) headquarters in Washington DC, US.
During the meeting, two important issues were discussed and deliberated. They are (i) International
Taxation-a Commonwealth Conversation around the Panama Paper (ii) Economics of Climate
Change and Financing Climate Adaptation and Mitigation.

Indias first Medipark will be set up in Tamil Nadu


October 8, 2016

Indias first Medipark (Medical Park) will be set up in Tamil Nadu at Chengalpet near Chennai,
Tamil Nadu. It will boast of the first medical devices manufacturing park in the country.
In this regard, the Union Cabinet has allowed HLL Lifecare, a miniratna public sector company to
sub-lease over 300 acres of land for the purpose.
How it will be formed?
The Medipark will be funded through a Special Purpose Vehicle (SPV) with HLL being a 50%
shareholder.
HLL will sublease 330.10 acres of land to investors, through a transparent bidding process to
investors desirous to set up manufacturing units for medical equipment and devices.
It will be developed in phases with completion period of seven years. In the first phase,
physical infrastructure will be developed.
Importance
First manufacturing cluster in the medical technology sector in the country envisaged to
boost local manufacturing of hi-end products at a significantly lower cost.
Play a significant role in the development of medical devices and technology industry and
allied disciplines under the Make in India initiative
Help in providing in affordable healthcare delivery, particularly in diagnostic services to a
large section of people.
Significance
2016 GKToday | All Rights Reserved | www.gktoday.in

54

Current Aairs: October 2016 [Date-wise Compendium]


The first of its kind Medipark project will lay roadmap to reduce the dependence on imports of
medical equipment and devices. It will create a strong base for the growth of indigenous and
domestic industry by providing access to state-of-the-art infrastructure and technology.

Union Government set up Public Debt Management Cell


October 8, 2016

The Union Finance Ministry has set up a Public Debt Management Cell (PDMC) with the objective
of deepening bond markets in the country.
PDMC will streamline government borrowings and better cash management for deepening bond
markets.
Key Facts
PDMC is an interim arrangement and will be upgraded to a statutory Public Debt
Management Agency (PDMA). Thus, it will requisite preparatory work for PDMA.
It will allow separation of debt management functions from RBI to PDMA in a gradual and
seamless manner, without causing market disruption.
The Middle Office of the Budget Division in the Union Finance Ministry will be subsumed
into PDMC with immediate effect.
The Joint Secretary (Budget), Department of Economic Affairs of the Finance Ministry will
be the overall in-charge of the PDMC.
PDMC will have only advisory functions in order to avoid any conflict with the statutory
functions of RBI.
Functions of PDMC
Plan government borrowings, including market borrowings and other borrowings, like
Sovereign Gold Bond (SGB) issuance.
Manage governments liabilities, improve cash forecasting, monitor cash balances, foster a
liquid and efficient market for government securities.
Advise government on matters related to capital market operations, investment,
administration of interest rates on small savings etc.
Develop an Integrated Debt Database System (IDMS) as a centralised data base for all
liabilities of government, on a near real time basis. IDMC will be an integral part of PDMC.
Need
The setting up of the PDMA is top priority of government for bringing financial sector
reform.
With establishing PDMA, Government seeks to divest the RBI of its dual and often
conflicting roles as the banker and manager of the Central Governments borrowing.
It will also facilitate in better planning and management of domestic and foreign market
2016 GKToday | All Rights Reserved | www.gktoday.in

55

Current Aairs: October 2016 [Date-wise Compendium]


borrowings of Central Governemnt. It will help in strengthen bond market and help to
promote investment.
It will be in pursuance global practice of shifting public debt management from central bank
to a debt management office.
Background
Former Finance Minister Pranab Mukherjee was first announced the setting up of PDMA in Budget
2011-12. However, RBI was first to recognise the need for PDMA in its Annual Report 2000-01. It
was also supported by the Financial Sector Legislative Reforms Commission (2013).

Indian Pharma companies rank 19th in Biopharmaceutical Competitiveness & Investment


(BCI) Survey
October 8, 2016

Indian pharmaceutical companies have ranked low at 19th position with an overall score of 59 out of
100 among in the Biopharmaceutical Competitiveness & Investment (BCI) Survey.
The survey of 28 countries was commissioned by the Pharmaceutical Research and Manufacturers of
America and executed by the Pugatch Consilium. It was third edition of survey.
How Survey was undertaken?
Five metrics were used to determine rankings of countries. They are (i) scientific capabilities
and infrastructure; (ii) clinical research conditions and framework; (iii) regulatory system; (iv)
market access and financing; and (v) effective intellectual property protections.
It had looked at biomedical innovation and its influencers, like intellectual property
protection for investments in the pharmaceutical sector.
It had categorized countries into two groups viz. mature markets like UK US, and Germany,
and newcomers like China, India and Brazil.
Key Highlights of Survey
Mature markets provide advanced intellectual property protection, highly streamlined
regulatory framework and favourable tax conditions with some exceptions.
India, along with China, Argentina, Colombia, Russia, Turkey and Brazil based on their
overall score were labelled as trailing markets.
Singapore which has placed in the newcomers category with a score of 85 is only a point
behind the US, which is a mature market.
India scores low on almost all metrics except in the clinical research conditions and
framework and scientific capabilities and infrastructure.
Indias IP regime, including Section 3(d) of IP Act requiring biopharmaceutical inventions to
show enhanced efficacy, continues to affect its investment environment.
2016 GKToday | All Rights Reserved | www.gktoday.in

56

Current Aairs: October 2016 [Date-wise Compendium]

Indias first international arbitration centre inaugurated in Mumbai


October 8, 2016

The first International Arbitration Centre of India was inaugurated in Mumbai, Maharashtra to
provide an arbitration platform for Indian business houses to negotiate commercial disputes.
The Mumbai Centre for International Arbitration (MCIA) was launched by Maharashtra Chief
Minister Devendra Fadnavis.
Key Facts
With this Maharashtra becomes first state in the country to clear policy for institutional
arbitration.
It is also considered as a major step towards making Mumbai an International Financial
Services Centre (IFSC)
The MCIA will be an independent, not-for-profit organization (NGO) governed by a council
comprising eminent national and international legal luminaries.
It will provide world-class infrastructure for arbitration, 247 functionality and live
transcription services recording during arbitration proceedings for transparency.
It will be in sync with the Make in India campaign and will provide a time bound and costeffective facility to reinforce investor confidence.
Background
At present, in absence of an international arbitration centre in the country most of the
business disputes involving Indian parties land in Singapore or London International
Arbitration Centres.
Indian parties make up an estimated 30% of the arbitration cases handled by the Singapore
and London Arbitration Centres.
The total outflow of funds to resolve such cases along with logistics and other related
expenditure is around $5 billion. MCIA will significantly bring down this cost.

October 9-10, 2016


October 9: World Post Day
October 10, 2016

The World Post Day is observed each year on October 9 to spread awareness about the postal
services and their role in the everyday lives of people and businesses.
2016 Theme: Innovation, Integration and Inclusion.
In India, this day is celebrated by Department of Posts. For more than 150 years, India Post has been
the backbone of the countrys communication network. It is the largest postal network in the world
providing mail parcel, banking, money transfer, insurance and retail services.
Background
2016 GKToday | All Rights Reserved | www.gktoday.in

57

Current Aairs: October 2016 [Date-wise Compendium]


The World Post Day was instituted at the Universal Postal Union (UPU) Congress held in
Tokyo, Japan in 1969.
Significance of the day: The day is celebrated to mark anniversary of the establishment of
the Universal Postal Union (UPU) in 1874 in the Swiss Capital, Bern.

About Universal Postal Union (UPU)


UPU is a specialized agency of the UN that coordinates postal policies among member
nations, in addition to the worldwide postal system.
It was established in 1874 and is the second oldest international organization worldwide after
International Telecommunication Union (ITU) which was established in 1865.
Its headquarters are located in the Swiss capital Berne. It has 192 member countries.
UPU is the primary forum for cooperation between postal sector players among member
countries. It helps to ensure a truly universal network of up-to-date products and services.
It sets rules for international mail exchanges and makes recommendations for growth in mail,
financial and parcel services volumes and also to improve quality of service for customers.

October 10: World Mental Health Day


October 10, 2016

The World Mental Health Day (WMHD) is observed every year on 10 October to raise awareness
about mental health issues around the world and mobilizing efforts in support of mental health.
Observance of the day seeks to provides an opportunity for all stakeholders working on mental
health issues to talk about their work and measures to be taken to make mental health care a reality
for the people.
2016 theme: Psychological First Aid. It highlights importance of both psychological and social
support to persons suffering from mental health issues.
The 2016 WMHD campaign aims to encourage people to take personal ownership of their own
mental health and wellbeing. It focuses on a simple, personal mental health promise that can be made
by anyone, regardless of their own mental health.
Background: The World Mental Health Day is an initiative of the World Federation for Mental
Health (WFMH) and is recognised by World Health Organisation (WHO). It was first celebrated in
1992.

Union Government extends anti-dumping duty on certain Chinese products


October 10, 2016

The Union Government has extended anti-dumping duty on import of certain Chinese products,
used in garment, toys and footwear manufacturing, for another five years i.e. till 2021.
Decision in this regard was taken by Central Board of Excise and Customs (CBEC) based on
recommendations of the Directorate General of Anti-Dumping and Allied Duties (DGAD).
2016 GKToday | All Rights Reserved | www.gktoday.in

58

Current Aairs: October 2016 [Date-wise Compendium]


This extension will protect domestic industry from the harm caused by dumping (import) of the
cheap Chinese products.
Issue
The DGAD made the case for continuation of the anti-dumping duty on Chinese products
after its second sunset review of the anti-dumping duty in force on the imports.
Earlier in October 2010, the Revenue Department of the Unio Finance Ministry had extended
the levy (anti-dumping duty) till October 2015.
What is Anti-Dumping Duty?
Anti-Dumping Duty is a trade levy imposed by any government on imported products which
have prices less than their fair normal values in their domestic market.
Thus, it is protectionist tariff that seeks to stop dumping process where company exports a
product at a price lower than price it normally charged in domestic market of importing
countries.
Anti-Dumping Duty is imposed under the multilateral World Trade Organisation (WTO)
regime and varies from product to product and from country to country.
In India, anti-dumping duty is recommended by the Union Ministry of Commerce (i.e. by
DGAD), while the Union Finance Ministry imposes it.

Union Government to invite bids soon for 36 inland waterways


October 10, 2016

The Union Government is going to soon float tenders to invite bids for the development for 36
inland waterways projects in the first phase.
This move will help to tap the huge potential in the so-far-untapped inland waterways transport
segment despite India being endowed with natural rivers suitable for it.
Background
Earlier, Parliament had enacted a legislation allowing 106 rivers across the country to be
converted into National Waterways (NWs) in addition to the existing 5 such NWs.
The law was enacted in a bid to boost movement of goods and passengers via rivers and
reduce transportation costs substantially.
Advantages of Inland water transport
It is environment-friendly and cost-effective mode of transportation.
It will reduce the logistics costs significantly which are currently as high as 18% in India.
India has huge natural geographical potential for inland waterways as 14,500 km of river
channels are navigable.
Development inland waterways can spur industrial growth and tourism potential along the
waterway especially in plain areas of North India.
2016 GKToday | All Rights Reserved | www.gktoday.in

59

Current Aairs: October 2016 [Date-wise Compendium]


The statutory status to the inland waterways would pave for investments in inland waterways
transportation sector which has been neglected in the past.
It would provide a cheaper mode of transport and reducing traffic burden and congestion on
roads and railways.
Problems
Seasonal variations in water level especially in the rain-fed rivers of the peninsula may cause
hindrance to inland waterway projects.
Diversion of river water for irrigation, industrial and domestic purpose may affect the
viability of these projects.
Inland waterways may cause serious environmental threat to Gangetic river dolphins,
national Aquatic animal of India.

IAE to tie up with India for its global LED programme


October 10, 2016

The International Energy Agency (IAE) is going to partner with India to implement its Unnat Jyoti
by Affordable LEDs for All (UJALA) initiative globally for energy savings.
IAE has been impressed with performance of Indias UJALA programme in terms of vastly
improving access to LED lighting while reducing their cost drastically.
Key Facts
The UJALA scheme is implemented by the Energy Efficiency Services Limited (EESL), a joint
venture of PSUs under the Union Ministry of Power.
The price at which EESL has been purchasing LED lights to distribute under UJALA scheme
has been consistently falling over the last couple of years
Along with this, production has also been ramped up to about 4 crore per month from the 10
lakh a month in 2014 to support the implementation of scheme.

About Unnat Jyoti by Affordable LEDs for All (UJALA) scheme


UJALA scheme is LED-based Domestic Efficient Lighting Programme (DELP) that aims to
promote efficient lighting, reducing energy consumption and energy savings.
It was launched as National LED programme by Prime Minister Narendra Modi in January
2015. It was renamed UJALA in March 2016.
Under it, every grid-connected consumer having a metered connection from their respective
Electricity Distribution Company will get the LED bulbs at subsidized rates.
Benefits: (i) Electricity savings (ii) Reduction of load (iii) Reduction of consumer bills (iv)
Greenhouse gas emission reductions.
Note: LED bulbs have a very long life, almost 50 times more than ordinary bulbs. They are
8-10 times that of CFLs, therefore provide both energy and cost savings in medium term.
2016 GKToday | All Rights Reserved | www.gktoday.in

60

Current Aairs: October 2016 [Date-wise Compendium]


For more information: International Energy Agency

Hubble Space Telescope detects Great Balls of Fire


October 10, 2016

NASAs Hubble Space Telescope (HST) has detected Great Balls of Fire (GBF), a mysterious superhot blobs of gas seen near a dying star.
The plasma gas balls were observed near a red giant called V Hydrae, about 1,200 light years away
from Earth. Red giants are dying stars that are nearing the end of their fuel supplies and have begun
to expand and puff up.
Key Facts
HST had found that these GBFs from V Hydrae are double the mass of Mars and twice as hot
as surface of sun. GBF have continued once every 8.5 years for at least past 400 years.
They are moving so fast in space that they would take only half an hour cover distance
between moon and Earth.
If scientists are able to discover origin of these balls, it could explain other weird shapes seen
in the cloud of gas around dying stars, which is difficult for scientists to explain at present.

About Hubble Space Telescope (HST)


HST is a space telescope that was launched in 1990 by NASA in collaboration with European
Space Agency. It is named after the astronomer Edwin Hubble and is still in operation.
It is managed by NASAs Goddard Space Flight Center in Greenbelt, Maryland. Its operations
are conducted by Baltimore based Space Telescope Science Institute (STScI).
It is located in low Earth orbit outside the distortion of Earths atmosphere that allows it to
take extremely high-resolution images.
Its successor, James Webb Space Telescope (JWST), is scheduled for launch in 2018.

Janani Suraksha Yojana pays dividends: Study


October 10, 2016

As per the new study Janani Suraksha Yojana (JSY) has played important role in reducing
socioeconomic disparities existing in maternal care.
The study was conducted by researchers from Delhi based National Council of Applied Economic
Research.
This is for first time study has shown JSY has reduced socioeconomic inequalities in maternal care
compared to earlier studies which showed impact of JSY in reducing maternal mortality.
Key Findings
JSY has led to reduction of prevalent differences in access to maternal care between individual
people of higher or lower socioeconomic status.
It has led to enhancement in utilisation of health services among all groups especially among
2016 GKToday | All Rights Reserved | www.gktoday.in

61

Current Aairs: October 2016 [Date-wise Compendium]


the poorer and underserved sections in the rural areas.
Utilisation of all three maternal healthcare services was remarkably higher among illiterate or
less educated and poor women.
Usage of all three maternal healthcare services by the Dalit, Adivasis, OBC and Muslim
women increased between the surveys.
However, inequalities still exist in access to maternal care but JSY has narrowed gap in access
to healthcare between the marginalised group of women and financially better-off.
Women in their early 20s more likely avail maternal health care services as compared to their
older women.
The incidence of women availing maternal healthcare decreases with the increase in the
number of children.

Background
In India, high incidence of maternal mortality continues to plague. As per the latest report on
maternal health, India accounted for 15% the total maternal deaths in the world in 2015 second only
to Nigeria with 45,000 women dying during pregnancy or childbirth.

About Janani Suraksha Yojana (JSY)


The JSY was launched as part of the National Rural Health Mission (NRHM) in 2005 to
improve maternal and neonatal health by promotion of institutional deliveries (childbirth in
hospitals). It is a 100% centrally sponsored scheme (CSS). It integrates cash assistance with
delivery and post-delivery care.
Under it, ASHA (Accredited social health activist) is considered as a link between
Government and poor pregnant women for encourage institutional deliveries among poor
women.

Image Source: The Hindu


SC widens ambit of Domestic Violence Act, 2005
October 10, 2016

The Supreme Court in its landmark judgement has widened the scope of the Protection of Women
from Domestic Violence Act, 2005 by making it gender-neutral.
2016 GKToday | All Rights Reserved | www.gktoday.in

62

Current Aairs: October 2016 [Date-wise Compendium]


The SC ordered deletion of the words adult male before the word person in Section 2(q) of the Act
making it gender-neutral. The remaining part of the legislation would remain operative and was
kept untouched.
The Protection of Women from Domestic Violence Act, 2005 protects women from physical, sexual,
verbal, emotional and economic abuse at home.
Key Facts
The SC order paves way for prosecution of women and even non-adults for subjecting a
woman relative to violence and harassment.
It strikes down two words from Section 2(q) of Act which deals with respondents who can be
sued and prosecuted under it for harassing a married woman in her matrimonial home.
It also allows a woman to seek legal action against her daughter-in-law and even her minor
grandchildren for domestic violence.
Why SC ordered to delete words?
According to SC, microscopic difference between male and female, adult and non-adult is neither
real nor substantial. It also does not have any rational relation to the object of the legislation. The
words adult male also violated right to equality under Constitution. It is contrary to object of
affording protection to women who have suffered from domestic violence of any kind.
Background
The landmark verdict of Supreme Court came on an appeal against the Bombay High Court
judgement, which had resorted to the literal construction of the term adult male. Based on literal
construction of these words, Bombay High Court had discharged four persons, including a woman,
two girls and a minor boy of a family from a domestic violence case on the ground that they were not
adult male and hence cannot be prosecuted under the Act.

Indian researcher produces stable Perovskite Nanocrystal solar cells


October 10, 2016

For the first time, Indian researcher has successfully produced a stable, high-efficiency, all-inorganic
perovskite nanocrystal solar cells.
It was produced by Abhishek Swarnkar, a research scholar from the Department of Chemistry at
Punes Indian Institute of Science Education and Research (IISER).
Key Facts
The new inorganic perovskite nanocrystal material has 10.77% efficiency to convert sunlight
to electricity. It used cesium to produce the material of cesium lead iodide.
The nanocrystals of cesium lead iodide were reduced to nanometre range. It allowed the
material to absorb visible sunlight till 700 nm at ambient temperature.
These nanocrystals were found to be stable from 196 degree C to about +200 degree C. It
2016 GKToday | All Rights Reserved | www.gktoday.in

63

Current Aairs: October 2016 [Date-wise Compendium]


also converted sunlight to electricity by producing a high voltage of 1.23 volts.
These nanocrystals were assembled as a thin film. The thin film was used for making both
solar cells and red LEDs.
Significance of Research
This research on inorganic perovskite nanocrystal moves away from traditional research
based around a hybrid organic-inorganic halide perovskite material.
Though, this traditional hybrid material has high efficiency of over 22% but organic
component in it volatile.
This material becomes completely unstable under ambient conditions within a short span of
time which renders it unsuitable for commercial photovoltaic applications.

Oliver Hart and Bengt Holmstrom win 2016 Nobel Prize in Economics
October 10, 2016

Two economists Oliver Hart and Bengt Holmstrom have won the 2016 Nobel Memorial Prize in
Economic Sciences. Both of them will share 8 million kronor, or about $930,000.
Royal Swedish Academy of Sciences has selected them for their contributions to contract theory
which tells how contracts help people deal with conflicting interests.
Their individual theories on contract theory are valuable to the understanding of real-life contracts
and institutions, as well as potential pitfalls in contract design. For example, contract theory can be
used to analyze performance-based pay for CEOs or deductibles and co-pays for insurance.
Their work provided economists microecomic tools to understand interactions between entities,
such as design of performance incentives in firms, corporate governance, privatisation,
constitutional law and entrepreneur-investor relationships.
Oliver Hart: He was born in 1948 in London, UK. He holds Ph.D. from Princeton University, US.
Presently, he is Andrew E. Furer Professor of Economics at Harvard University, US.
Bengt Holmstrm: He was born in 1949 in Helsinki, Finland. He holds Ph.D. from Stanford
University, US. Presently he is Paul A. Samuelson Professor of Economics and Professor of
Economics and Management at Massachusetts Institute of Technology (MIT), US.

About Nobel Memorial Prize in Economic Sciences


In 1968, Swedens central bank had added the economic sciences prize as a memorial to Nobel. Thus,
economics award is not a Nobel Prize as the others prizes which were established by Swedish
industrialist Alfred Nobel in 1895.

October 11, 2016


October 11: International Day of the Girl Child
October 11, 2016

The International Day of the Girl Child (IDGC) is observed every year across the world on 11
2016 GKToday | All Rights Reserved | www.gktoday.in

64

Current Aairs: October 2016 [Date-wise Compendium]


October to recognize girls rights and the unique challenges girls face around the world.
Significance of Day: The observance of the day seeks to increase awareness of gender inequality
faced by girls worldwide based upon their gender and supports more opportunity for girls. This year
it was fifth edition of this day after it was observed for first time in 2012.
2016 Theme: Girls Progress = Goals Progress: What Counts for Girls. It calls for action for increased
investment in collecting and analyzing girl-relevant, girl-focused and sex-disaggregated data.
Background
The International Day of the Girl Child (IDGC) was instituted by the United Nations General
Assembly (UNGA) by adopting Canada backed Resolution in December 2011. The observation of
the day supports more opportunity for girls. It also increases awareness of gender inequality faced by
girls worldwide based upon their gender. This inequality includes areas such as right to education,
nutrition, medical care, legal rights and protection from discrimination, violence and unfree child
marriage.

4th BRICS Science, Technology and Innovation Ministerial Meeting held at Jaipur
October 11, 2016

The 4th BRICS Science, Technology and Innovation Ministerial Meeting was held at Jaipur,
Rajastan.
The purpose of the meeting was to further strengthen the collaboration amongst the BRICS
countries in the areas of Science, Technology and Innovation (STI).
Key Facts
The meeting adopted a five-pronged approach, viz. Institution Building, Implementation,
Integration, Innovation and Continuity.
In alignment with the theme of Indias Chairmanship of 8th BRICS Summit, Building,
Responsive Inclusive and Collective Solutions, Jaipur Declaration was unanimously
adopted.
The BRICS member countries have resolved to intensify, diversify and institutionalize STI
cooperation through the BRICS innovation and research initiative.
Indias proposal to establish a BRICS Science and Technology driven Entrepreneurship and
Innovation Partnership Programme to harness innovativeness of youth was also agreed.

Himansh, Indias remote, high-altitude station opens in Himalayas


October 11, 2016

A high altitude glaciological research station in Himalaya called Himansh (meaning a slice of ice)
began functioning above 13,500 ft (4000 m) in a remote region in Spiti, Himachal Pradesh.
Researchers will use this station as a base for undertaking surveys that would digitize the glacier
motion and snow cover variations with exceptional precision.
2016 GKToday | All Rights Reserved | www.gktoday.in

65

Current Aairs: October 2016 [Date-wise Compendium]


Key Facts
Himansh station will provide much needed fillip to the scientific research on Himalayan
glaciers and its hydrological contribution.
The research lab has been established by the National Centre for Antarctic and Ocean
Research (NCAOR) in Spiti Valley, one of the most uninhabited parts of the country
The station houses instruments to quantify glacier melting and its relation to changing
climate.
Some of the instruments available at this research facility include automatic weather stations,
ground penetrating radars, geodetic GPS systems and other sophisticated facilities.
It will also serve as the base for Terrestrial Laser Scanners (TLS) and Unmanned Aerial
Vehicles (UAVs) for undertaking surveys.
Significance
The Himalayan region has the largest concentration of glaciers outside the polar caps. It is
called the Water Tower of Asia.
It is source of 10 major river systems that provide irrigation, drinking water and power for
over 700 million people (10% of worlds population) living in India, Pakistan and Bangladesh.
Thus, understanding behaviour of these Himalayan glaciers and their contribution to
sustainable supply of water for mankind and agriculture is one of grand challenges of Indian
scientific community.

Russia, Turkey agree to intensify military, intelligence contacts


October 11, 2016

The Russia and Turkey have agreed to intensify military and intelligence contacts. They also have
agreed on the need for aid to get to the northern Syrian city of Aleppo.
Decision in this regard was taken after meeting between Turkish President Recep Tayyip Erdogan,
Russian President Vladimir Putin on sidelines of World Energy Congress in Istanbul.
Key Facts
The two countries also have signed a deal to construct two TurkStream gas pipeline pipelines
to send Russian gas under the Black Sea to Turkey.
Turkey will use one pipeline for its domestic consumption and the other will supply
southeastern Europe, bypassing Ukraine.
Improvement in Relations
This was the first time Russian President Putin visited Turkey after their relations were
strained after Turkey, a NATO member had downed a Russian war plane in November 2015
alongside the Syrian border.
Since then relations between both countries had ebbed after Russia had imposed economic
2016 GKToday | All Rights Reserved | www.gktoday.in

66

Current Aairs: October 2016 [Date-wise Compendium]


sanctions against Turkey.
Turkey is massively dependent on energy imports from Russia for its energy security.
Besides, first nuclear power station of Turkey is also being built with Russian help.
Syria
However, both nations still differ over Syrian strategy and plan. Russia has always stood by
Assad regime in Syria throughout the years of unrest.
Russia had further intensified its support to regime by launching an air campaign against
rebels. Contrastingly, Turkey always stood against Assad and helped the rebel factions.
Turkeys decision was seen counterproductive as it had further strengthened the stronghold
of ISIS in Syria and surrounding region and also intensified the Syrian civil war and leading
to spread of terrorism and refugee crisis.

Union Government to invest Rs 15000 crore to increase Panipat oil refinery capacity
October 11, 2016

The Union Government has decided to invest 15,000 crore rupees to increase the capacity of Panipat
refinery in Haryana to 25 Million Tonnes from existing 15 Million Tonnes.
It was announced by Union Minister of State (MoS) for Petroleum and Natural Gas, Dharmendra
Pradhan at a function in Panipat.
Key Facts
The expansion of Panipat refinery will help raise specifications of fuel quality from BS-IV to
BS-VI to meet future demand.
It will also create employment opportunities for the people and also help in raising quality of
fuel in the country by the year 2020.
Besides, Indian Oil Corporation (IOC) will also set up bio-ethanol plant in Panipat to
generate alternative fuel from agricultural residue which would boost agriculture sector.
What is Bioethanol?
It is an alcohol derived by process of fermentation mostly from carbohydrates of agricultural
residue and feedstocks.
As a quasi-renewable energy, ethonal can be blended with petrol or diesel making it a
sustainable transport fuel. It will help to reducing emissions and dependency on imported
fuel.
Benefits of bio-ethanol plant
Help in solving the chronic problem of straw burning of leftover agro-based produce especially from
wheat and rice feedstocks. Benefit farmers economically, as they would be paid for their agro-based
produce to extract bio-ethanol. It would also help in preventing the loss of fertility of soil and
damage to environment by reducing air pollution.
2016 GKToday | All Rights Reserved | www.gktoday.in

67

Current Aairs: October 2016 [Date-wise Compendium]

Union Government to commemorate birth centenary of Nanaji Deshmukh


October 11, 2016

The Union Government is going to commemorate the birth centenary of social activist and former
RSS leader Nanaji Deshmukh from October 11, 2016 to October 11, 2017.
It was announced by the Union Culture Minister Mahesh Sharma during inaugural programme
organised on the eve of the birth centenary celebrations.

Who is Nanaji Deshmukh?


Nanaji Deshmukh was a social activist who had worked in the fields of education, health, and
rural self-reliance. He was a leader of Bharatiya Jana Sangh and RSS.
He was born as Chandikadas Amritrao Deshmukh on October 11, 1916 in Kadoli in
Maharashtras Hingoli district and passed away on February 27, 2010.
He was inspired by Lokamanya Tilak and his nationalist ideology. He was associated with Dr.
Keshav Baliram Hedgewar, founding Sarsanghachalak of the RSS.
He also had actively participated in Bhoodan Movement started by Vinoba Bhave. He also had
supported the Total Revolution movement of J P Narayan.
He was one of main architects of the Janata Party. He had won in the 1977 Lok election from
Balrampur Lok Sabha constituency of Uttar Pradesh.
He had established Chitarkoot Gramoday Vishwavidyalaya in Chitrakoot, Indias first rural
University and had served as its Chancellor
He had done pioneering work towards the anti-poverty and minimum needs programme. He
had implemented the philosophy of integral humanism to improve the living standards of
more than 150 villages of bundelkhand.
He was nominated to Rajya Sabha in the 1999 in recognition of his services to the nation. He
was awarded Padma Vibhushan in 1999.

October 12, 2016


Abdelilah Bekirane elected as Prime minister of Morocco
October 12, 2016

Abdelilah Bekirane was re-appointed as the Prime Minister of Morocco for second term. Benkirane
has been serving as Prime Minister since November 2011.
It was officially announced by Mohammed VI, the king of Morocco after Benkiranes Islamic Justice
and Development Party (PJD) emerged victorious in recent general election.
Now PM Bekirane will now reach out to other parties to form a coalition government as he does not
have absolute majority in lower house.
Background
In recently concluded general election the PJD party had emerged single largest party by
2016 GKToday | All Rights Reserved | www.gktoday.in

68

Current Aairs: October 2016 [Date-wise Compendium]


winning 125 seats.
While the opposition Authenticity and Modernity Party came second with 102 seats.
Moroccos oldest party Istiqlal (Independence) Party emerged third winning 46 seats.
Morocco has multi-party system which makes it impossible for any political party to win an
absolute majority in the lower house of 395 members.
Of the total 395 members, 305 are elected in multi-seat constituencies from electoral lists,
while of the remaining 90 seats, 60 seats are reserved for a national list of women. Reest 30
seats are set aside for the young candidates below the age of 35.
About Abdelilah Bekirane
Born in 1954 in Rabat, Morocco. He holds graduate degree in Physics. He is former Managing
Editor of Al-Islah, Ar-Raya and Attajdid newspapers.
In 2016, he was elected for a fifth term in Parliament and represents Sal constituency since
November 1997. Since 2008, he is secretary-general of the Justice and Development Party of
Morocco.

Jitu Rai wins Champion of Champions pistol award of ISSF


October 12, 2016

Ace Indian Shooter Jitu Rai has won the 2016 Champion of Champions pistol Award of the
International Shooting Sport Federation (ISSF) for pistol shooting.
He won the award consisting cash prize of 5000 in the World Cup Final held in Bologna, Italy. In
the 10 m Air Pistol Champions Trophy final Jitu Rail defeated Damir Mikec of Serbia by 29.6 to 28.3
points. Olympic champion Anna Korakaki finished third in this event.
10 m Air Pistol Champions Trophy
It was the mixed knock-out competition meant only for the medallists in the World Cup Final. In
this event shooters are eliminated after the first four shots, one each after every successive shot. The
last two compete on a three-shot format.
Other Awards
In the rifle event category, the honour went to Sergey Kamenskiy of Russia. He had defeated twotime Olympian Du Li of China by 31.7 to 31.6 points. London Olympics champion Yi Siling of China
took the third place.

Harikatha: traditional art of story-telling in South India


October 12, 2016

Harikatha, a traditional art story-telling was in news for its revival due to patronage from various
cultural organisation.
Harikatha is a form of Hindu religious discourse in which the storyteller explores a religious theme,
usually based on the life of a saint or a story from an Indian epic.
2016 GKToday | All Rights Reserved | www.gktoday.in

69

Current Aairs: October 2016 [Date-wise Compendium]


What is Harikatha?
Harikatha is an art form composed of storytelling, music, drama, dance, poetry, and
philosophy in South India especially in villages of Andhra Pradesh, Karnataka and Tamil
Nadu.
It had originated from Ajjada village of Srikakulam in Coastal Andhra. Its aim is to imbue
truth and righteousness in the minds of people and sow the seeds of devotion in them.
It also aims to educate people about knowledge of atman (self) through stories and show them
the path of liberation.
Harikatha at its peak was a popular medium of entertainment, which had helped to transmit
cultural, educational and religious values to the masses.
Style of Harikatha
Harikatha involves the narration of a story, intermingled with various songs relating to the
story. The subject for the Harikatha story may be any Hindu religious theme.
Usually the narration of story involves numerous sub-plots and anecdotes, which are used to
emphasise various aspects of the main story.
The main story teller is usually assisted by one or more co-singers, who elaborate the songs
and Mridangam accompanist. The storyteller uses a pair of cymbals to keep beat.

Telangana Government creates 21 new districts


October 12, 2016

Telangana Government in its the biggest administrative reform has created 21 new districts. With
this reorganisation, the total number of districts in the State went up to 31 from 10.
The 21 new districts are: Siddipet, Jayashankar, Jangaon, Jagtial, Yadadri, Warangal (Rural),
Peddapally, Sangareddy, Kamareddy, Mancheriyal, Kothagudem, Vikarabad, Rajanna, Asifabad,
Suryapet, Wanaparthy, Nirmal, Mahabubabad, Nagarkurnool, Jogulamba and Malkajgiri.
Earlier 10 districts are: Hyderabad, Mahabubnagar, Ranga Reddy, Karimnagar, Medak, Adilabad,
Warangal, Nizamabad, Khammam and Nalgonda.
Background
Earlier, Telangana government had decided to issue an ordinace to amend some provisions of the
Telangana District (Formation) Act, 1974 to abridge the process for increasing the number of
districts in state beyond 27. Besides, formation of new districts it also has called for reconstitution of
revenue divisions and other administrative units along with the new districts.
Why new districts were formed?
Telangana came into existence as the 29th State of India on June 2, 2014 after it was carved
out of Andhra Pradesh. Since then it was having 10 districts.
With this administrative reform, each district on average will have a population of two lakhs
2016 GKToday | All Rights Reserved | www.gktoday.in

70

Current Aairs: October 2016 [Date-wise Compendium]


to four lakhs except in Hyderabad where the population will be little more
Through small districts, state government seek to deliver people-centric welfare schemes and
governance to all citizens in the state with greater focus. This system has been adopted based
on governance models of Scandinavian countries.

Union Government to adopt three-pronged approach to spur internal trade


October 12, 2016

The Union Government may soon consider adopting a three-pronged approach to revitalise
domestic retail and wholesale trade.
The three-pronged approach includes (i) Establishing a regulatory body for national internal trade
(retail and wholesale) (ii) Comprehensive domestic trade policy and (ii) Board for Internal Trade.
Need for new approach
Domestic trade mainly comprises non-corporate small businesses providing employment to
an estimated 460 million people.
Currently, there is no single regulatory body or Ministry for such six crore small businesses in
the country that have collective annual turnover of around Rs. 30 lakh crore.
The current approach policy regime and the environment is considered only conducive for
foreign retail giants and their Indian partners or counterparts
Implications
The proposed three-pronged policy approach will address need of these small businesses with
an aim to generate employment, improve financial inclusion of small traders.
It will also ensure ease of doing business by eliminating multiplicity of approvals, improve
competiveness of economy and boost overall demand to boost growth.
Besides, the new strategy will also help to link Central Governments flagship initiatives such
as Digital India, Skill India and Make In India with these small businesses.

China raps US over Asia interventions


October 12, 2016

China has rapped United States for its interventions in Asia as tensions are growing between the two
powers over territorial disputes in the South China Sea (SCS).
In this regard, Chinas Defence Minister has levied thinly veiled criticism at recent US involvement
in Asias trouble spots.

China-US Issue
China has claimed its territorial rights over almost the entire South China Sea region while
other countries Malaysia, Brunei, Philippines, Vietnam and Taiwan also have rival claims.
To support its rights, China also has built artificial islands capable of supporting military
facilities the strategically vital South China Sea.
2016 GKToday | All Rights Reserved | www.gktoday.in

71

Current Aairs: October 2016 [Date-wise Compendium]


Earlier to oppose Chinas moves, US had conducted freedom of navigation operations close to
artificial islands built by China in the SCS as it pose a challenge to freedom of navigation.
Besides, it also had agreed to deploy a missile defence system in South Korea following
repeated nuclear and missile tests by Chinas ally North Korea.
US also has sought to pivot to Asia by increasing its military and economic engagement
with countries in region which is seen as its move to contain Chinas growing power.
In July 2016, an international Permanent Court of Arbitration (PCA) in its pronouncement
had ruled that China has no legal basis over the entire SCS. However, China has rejected PCA
verdict.

India-Indonesia bilateral maritime exercise commences at Belawan, Indonesia


October 12, 2016

The 28th India and Indonesia Coordinated Patrol (CORPAT) and Second Bilateral Maritime
Exercise have commenced at Belawan, Indonesia in the Andaman Sea.
The 17-day-long bilateral exercise (from 10 to 27, October, 2016) will demonstrate Indias
commitment to its ties with Indonesia and to maritime security in Indian Ocean Region (IOR).
Key Facts
The bilateral maritime exercise and CORPAT will see participation by one warship and one
maritime patrol aircraft from navies of both countries.
India will be represented by INS Karmuk, an indigenously built missile corvette, based under
Andaman and Nicobar Command alongwith a Dornier Maritime Patrol Aircraft.
These naval interactions will provide opportunities for both countries for extensive
operational and training engagements. Besides, it will also contribute substantively to the
maintenance of good order at sea.
The bilateral naval exercise also seeks to strengthen existing bonds of friendship between
both countries. It will also underscore Indias partnership and solidarity with friendly
countries of the region.
Backgfround
Defence relations between India and Indonesia are growing steadily with regular joint
activities and interactions between the Armed Forces of the two countries.
The navies of both countries have been carrying out CORPAT on respective sides of the
International Maritime Boundary Line (IMBL) twice a year since 2002.
The main aim of CORPAT is to keep the vital part of the Indian Ocean Region (IOR) safe and
secure for commercial shipping, International trade and legitimate marine activities.
The CORPAT has helped to strengthened understanding and interoperability between the
two navies and also has promoted net maritime security in the region.
2016 GKToday | All Rights Reserved | www.gktoday.in

72

Current Aairs: October 2016 [Date-wise Compendium]

NASA develops electroactive bandage to speed up wound healing


October 12, 2016

The NASA (National Aeronautics and Space Administration) has developed a new high-tech
electroactive bandage that uses electricity to significantly promote healing of injured wounds.
The high-tech bandage creates an electric charge to promote the healing process of wounds in space
especially in conditions of non-Earth gravity.
Need
In conditions of non-Earth gravity, human blood displays quite different behaviour from that
on Earth. In case of injury in space, wounds heal more slowly
Considering the survival risks due to injury and the cost of space missions, healing wounds as
fast as possible is crucial.
Key Facts
The electroactive bandage uses a new material called polyvinylidene fluoride (PVDF) which
can be stimulated by pressure of cell growth and body heat.
The new material generates a small amount of electricity when interacts with another
surface, including human skin.
When this electroactive bandage is applied to an external wound site, it utilises low level
electrical stimulation to promote wound healing.
The bandage speeds the wounds healing process and minimises infection and related
complications such as amputation or illness.
Potential Applications: This bandage could be used by astronauts in space, military
personnel wounded in field, patients who have undergone surgery or who have suffered a
serious wound.

CERC appointed committee suggests overhaul in transmission planning


October 12, 2016

The Central Electricity Regulatory Commission (CERC) appointed committee has submitted its
recommendations. It was headed by power system expert Mata Prasad.
The committee has suggested an overhaul in transmission planning to facilitate transfer of power on
economic principles.
Key Recommendations
Transmission planning: It must be aligned to meet customer aspirations as opposed to
existing system where transmission is associated with long-term power purchase agreements
(PPAs).
It can should be done basis of projected load of states and anticipated generation scenario
based on economic principles of merit order operation.
2016 GKToday | All Rights Reserved | www.gktoday.in

73

Current Aairs: October 2016 [Date-wise Compendium]


Renewable energy sources transmission system: To be planned by central transmission
utility (CTU) based on estimated capacity additions in perspective plan and renewable
purchase obligations of each state.
Promote of power market: Transmission corridor allocation must be done suitably made.
5% of each flow gate may be reserved for day-ahead collective transactions.
This flow gate may be released for contingency market in case of non-utilisation of corridor
by power exchanges. The percentage of reservation may be reviewed after 1 year of
operation.
System studies: They must be carried out for various generation and load scenarios during
peak and off-peak hours. It should also consider renewable capacity addition and scheduling
of various generating stations that dont have any PPAs.
Creation of a central repository of generators: It should be created in the Central
Electricity Authority of India (CEA).In this case any generation project developer proposing
to set up a new generation plant must register itself.
For accurate demand forecasting: Hand-holding of states by CEA and CTU for accurate
demand forecasting. States must procure software for short-term, medium-term and longterm demand forecasting.

October 13, 2016


Finance Ministry constitutes committee to consolidate regulation of pension products
October 13, 2016

The Union Finance Ministry has constituted a high-level committee to consolidate the regulation of
pension products that is currently being done by three different watchdogs including insurance and
stock market regulators.
The committee would have representatives from all financial sector regulators SEBI, IRDA, RBI and
PFRDA. Its mandate will be to look into the issue of bringing these companies which are offering
pension plans under different regulators under the purview of PFRDA.
Note
PFRDA: Pension Fund Regulatory and Development Authority.
SEBI: Securities and Exchange Board of India.
IRDA: Insurance Regulatory and Development Authority of India.
RBI: Reserve Bank of India.
Why there is need to consolidate regulation of pension products?
The PFRDA Act says that PFRDA will be the pension regulator in the country. Currently, pension
products floated by insurance companies and those sold by mutual funds are under purview of IRDA
2016 GKToday | All Rights Reserved | www.gktoday.in

74

Current Aairs: October 2016 [Date-wise Compendium]


and SEBI respectively. Thus, there are cases of overlapping functions performed by financial sector
regulators SEBI, IRDA, RBI and PFRDA in case of floating different pension products. For instance
PFRDA is regulating all pension products in the country. However, insurers and mutual funds
continue to sell pension products outside PFRDAs watch. The PFRDA Act says that PFRDA will be
the pension regulator in the country.

About Pension Fund Regulatory and Development Authority (PFRDA)


PFRDA is a statuary pension regulatory authority established in 2003 under the PFRDA Act.
It functions under the aegis of Union Ministry of Finance, Department of Financial Services.
PFRDA promotes old age income security by establishing, developing and regulating pension
funds.
It also protects interests of subscribers to schemes of pension funds and related matters.
It is responsible for appointment of various intermediate agencies such as Central Record
Keeping Agency (CRA), Custodian, Pension Fund Managers, NPS Trustee Bank, etc.

Union Cabinet approves MoU between India and Hungary on cooperation in field of water
management
October 13, 2016

The Union Cabinet has given its approval for signing of a Memorandum of Understanding (MoU)
between India and Hungary on cooperation in the field of water management.
The MoU will be signed between Union Ministry of Water Resources, River Development and
Ganga Rejuvenation and the Ministry of Interior, Government of Hungary.
This is for the first time India is entering into a MoU with Hungary with wide-ranging areas on
water sector.
Key Features of MoU
Both the countries will cooperate in the field of river basin management, integrated water
resources management, irrigation technology innovation, efficiency in water supply and flood
& drought management to improve the socio-economic conditions of the people.
They will also exchange of scientific delegations and experts in the field of water resources
development and management for joint activities.
Benefits
Enhance bilateral cooperation in the field of water management, on the basis of equality and mutual
benefits. Encourage the development of bilateral relations between private and public organizations
concerning water resources of both the countries.

First World Tsunami Awareness Day to be celebrated at AMCDRR 2016


October 13, 2016

The first World Tsunami Awareness Day will be celebrated on November 5, 2016 at the Asian
2016 GKToday | All Rights Reserved | www.gktoday.in

75

Current Aairs: October 2016 [Date-wise Compendium]


Ministerial Conference for Disaster Risk Reduction (AMCDRR) 2016.
The conference will organised from November 3 to 5, 2016 in New Delhi by the Union Government
in collaboration with United Nations Office for Disaster Risk Reduction (UNISDR).
To commemorate the occasion, an event will be organised with Disaster Risk Reduction (DRR)
Champions at the AMCDRR 2016.

About World Tsunami Awareness Day


The World Tsunami Awareness Day was instituted by the United Nations General Assembly
(UNGA) by adopting a resolution proposed by Japan.
The objective of the day is to spread awareness among people across the world in matters
related to the dangers of tsunami and stress on the importance of early warning systems in
order to mitigate damage from the devastating natural calamity. It also seeks to revive
traditional knowledge about tsunamis.
Background: November 5 was selected by UNGA to coincide with the annual anniversary of
the 1854 Inamura-no-hi (Fire of Inamura) event. On this day in 1854, a villager in Wakayama
Prefecture, Japan through his quick action had saved countless lives when he set fire to
sheaves of rice on the top of a hill, thus warning people of the imminent danger of a tsunami.
This was the first documented instance of a tsunami early warning.
Indias initiatives
Following the devastating Indian Ocean Tsunami of 2004, Central Government established
an Indian Tsunami Early Warning Centre (ITEWC). It is operational since October 2007.
ITEWC was established under the Indian National Centre for Ocean Information Services
(INCOIS) at Hyderabad.
The Centre has state-of-the-art infrastructure for generating and disseminating tsunami
bulletins for the entire Indian Ocean region.
Besides, India along with 23 other Indian Ocean countries had participated in a tsunami mock
drill in September 2016.

Union Cabinet approves establishment of IIM in Jammu


October 13, 2016

The Union Cabinet has approved the establishment and operationalisation of Indian Institute of
Management (IIM) at Jammu.
It will become operational from the Academic Year 2016-17 at the transit or temporary campus at
Old Government College of Engineering & Technology.
Key Facts
For the initial four years from 2016 to 2020, the IIM will function in the temporary campus.
The student strength intake in Post Graduate Diploma Programme (PGDP) in Management
2016 GKToday | All Rights Reserved | www.gktoday.in

76

Current Aairs: October 2016 [Date-wise Compendium]


for year 2016-17 will be 54. It will be progressively increased to 120 in 4th year.
Meanwhile, steps will also be taken up for setting up campus at Jammu and an out-campus in
Kashmir region.
The Detailed Project Report under preparation for the permanent campuses and the process
for setting up of the campuses would start after it.
The Union Cabinet also approved formation of an IIM Jammu Society under the Societies
Registration Act, 1860. IIM Jammu will be run and managed by the Society with a Board of
Governors (BOGs) to be constituted by the Central Government. BOGs will administer the Institute
and would be responsible for establishment and operationalisation of the Institute.
Comment
It was announced as part of Prime Ministers development package for Jammu & Kashmir. The IMM
coupled with opening of IIT at Jammu, NIT Srinagar and two new AIIMS institutions, one each in
Jammu region and Kashmir region will help in meeting the requirement of high quality living and
education in Jammu & Kashmir.
Background
IIM are the countrys premier institutions imparting best quality education in management.
At present, there are total nineteen IIMs.
Out of these, thirteen IIMs are located at Ahmedabad, Kolkata, Bengaluru, Indore, Lucknow,
Raipur, Kozhikode, Rohtak, Ranchi, Shillong, Kashipur, Trichy, Udaipur.
New six IIMs were started in 2015 and they are located at Amritsar, Nagpur, Sirmaur,
Sambalpur, Bodhgaya and Vishakhapatnam.

Four PSBs may struggle to pay AT1 bond coupons


October 13, 2016

Four public sector banks (PSBs) may struggle to make coupon payments on their additional tier 1
(AT1) bond as they have reported heavy losses due to a surge in bad loans.
In this case coupon payment is an annual interest paid on the face value of a bond. It is expressed as a
percentage. AT1 bond is issued under Basel III capital regulations.
Why PSBs finding difficult to pay them?
The main reasons that may affect ability of PSBs to pay coupon on AT1 bonds are decline in
profitability and increasing losses that may wipe out their revenue reserves.
Governments position
Union Government has committed capital support to these PSBs on the coupon on AT1 bonds.
However, this support can only be serviced through PSBs current years profit or from revenue
reserves. Thus, any capital infusion by the government alone cannot help the banks to service
coupon on these bonds.
2016 GKToday | All Rights Reserved | www.gktoday.in

77

Current Aairs: October 2016 [Date-wise Compendium]


What are Additional Tier 1 (AT1) Bonds?
AT1 bonds are the hybrid bonds that combine debt and equity elements. They are also called
as contingent convertible capital instruments (CoCos).
AT1 or Cocos bonds have their roots in financial crisis when governments were forced to
bail out banks. They are the riskiest debt issued by banks and do not have any set maturity
date.
The defining characteristic of AT1 or Cocos bond is that it may be converted into shares
when certain conditions are met.
For example, when a company runs into trouble, the owners lose their stake and the debt
becomes equity, lenders turns into owners. But in case of banks such negotiations are not
possible. The coco bonds are designed to anticipate that process and transform automatically
from debt to equity.

October 13: International Day for Disaster Reduction


October 13, 2016

The International Day for Disaster Reduction (IDDR) is observed annually on 13 October across the
world encourage citizens and governments to take part in building more disaster resilient
communities and nations.
Significance of the day: It seeks to spread awareness about reining the risks of disasters around the
world and also reduce exposure of people and communities to disasters.
2016 Theme: Live to Tell: Raising Awareness, Reducing Mortality.
The 2016 edition of IDDR marks the launch of the new Sendai Framework for Disaster Risk
Reduction by United Nations Office for Disaster Risk Reduction (UNISDR). UNISDRs campaign
for the day is Sendai Seven that seeks to create a wave of awareness about actions taken to reduce
mortality around the world.
Background
The International Day for Disaster Reduction was instituted by UN General Assembly (UNGA) in
1989 to promote a global culture of risk-awareness and disaster reduction. Earlier, this day was
celebrated annually on the second Wednesday of October. But in 2009, the UNGA formally
designated 13 October as the annual date by adopting Resolution 64/200.
What is Sendai Framework?
The Sendai Framework is a 15-year voluntary, non-binding agreement which recognizes that
the countries have the primary role to reduce disaster risk.
It also recognises that this responsibility of countries should be shared with other
stakeholders including local government, private sector and other stakeholders.
It is the successor agreement to the Hyogo Framework for Action (20052015), which was
2016 GKToday | All Rights Reserved | www.gktoday.in

78

Current Aairs: October 2016 [Date-wise Compendium]


the most encompassing international accord on disaster risk reduction to date.
The Sendai Framework also seeks to promote best practices at the international, regional and
national level across all sectors, to reduce disaster risk and disaster losses. The framework has
seven targets and first of it is reducing disaster mortality.
The four priority themes of the Sendai Framework are: (i) Understanding disaster risk, (ii)
improving disaster risk governance, (iii) investing in disaster risk reduction (through
structural and non-structural measures) and (iv) disaster preparedness, early warning and
building back better in the aftermath of a disaster.

Draft National Water Framework Bill, 2016 suggests basin-level management


October 13, 2016

The Central Government has brought final draft of the National Water Framework Bill, 2016 to
provide uniform national legal framework to manage water in a better and efficient way
The comprehensive draft Bill proposes model law for all states. However, water being a State subject
under VII Schedule of constitution the law will be not binding on States for adoption.
Need
The water shortage problem is escalating and country has witnessed acute drought situation in
certain parts. In future, such situations may increase backdrop of climate change. Besides, presently
in absence of institutional arrangement there are inter-state water disputes because states do not
their contributions to a rivers catchment area to resolve conflicts.
Key Features of Bill
Every person has a right to sufficient quantity of safe water for life within easy reach of the
household regardless of his/her socio-economic factors.
All basin states have equitable rights over the use of river water provided such use does not
violate the right to water for life of any person in the river basin.
States must recognise the principle that the rivers are public trustees and not owned by the
basin-States.
All the basin States are equal in rights and status, and there is no hierarchy of rights among
them. Here equality of rights means not equal but equitable shares in river waters.
Managing water at river basin-level and right measurement of States contribution to river
system to in order to resolve conflicts.
Establishing River Basin Authority (RBA) for each inter-State basin to ensure optimum and
sustainable development of rivers and valleys.
Establishing institutional arrangements to deal with inter-state water disputes in order to
obviate disputes through negotiations, mediation or conciliation.
Proposes other mechanisms such as National water quality and footprint standards,
2016 GKToday | All Rights Reserved | www.gktoday.in

79

Current Aairs: October 2016 [Date-wise Compendium]


Integrated river basin development and management plan and Graded pricing system.

IISER researchers use human hair to produce cheaper cathodes for solar cells
October 13, 2016

Researchers from Kolkata based Indian Institute of Science Education and Research (IISER) have
produced cost-effective, metal-free cathodes using human hair for use in solar cells.
Significance: This is the first instance where a bio-waste-derived electrode (in this case human hair)
has been used as cathode in a quantum dot sensitised solar cell device.
Key Facts
IISER researchers have developed graphitic porous carbon cathode by cleaning and drying
human hair and treating with sulphuric acid to achieve precarbonisation.
It was then heated at different temperatures in the presence of inert gas for six hours to
carbonise and bring better electrical conductivity for efficient charge transfer.
This efficient green cathode achieves highly catalytic graphitic porous carbon stage at
optimum temperature of 850 degrees C.
Advantages
Producing graphitic porous carbon cathode using human hair is simple, quick and
inexpensive. It has potential to bring down the cost of solar cells.
The bio-waste-derived cathode has higher efficiency to convert visible sunlight to electricity.
It also enhances the power conversion efficiencies.
It generates high open-circuit voltage compared to conventional platinum and activated
carbon cathodes.
It is metal-free cathode, while commonly used cathode is made of platinum metal and metal
sulfides.

CCEA approves revision of ethanol price for supply to Public Sector OMCs
October 13, 2016

The Cabinet Committee on Economic Affairs (CCEA) has approved the mechanism for revision of
ethanol price for supply to Public Sector Oil Marketing Companies (OMCs).
The revised administered price will facilitate OMCs to carry out the Ethanol Blended Petrol (EBP)
Programme for the next sugar season 2016-17.
Key facts
The revision of ethanol prices will facilitate continuation of Government policy in providing
price stability and remunerative prices for ethanol suppliers.
Charges will be paid to the ethanol suppliers as per actuals in case of Excise Duty and GST/
VAT and transportation charges as decided by the OMCs.
However, these prices of ethanol can be reviewed and suitably revised by Central
2016 GKToday | All Rights Reserved | www.gktoday.in

80

Current Aairs: October 2016 [Date-wise Compendium]


Government at any time during ethanol supply period (from 1 December, 2016 to 30
November, 2017) depending upon prevailing economic situation and other relevant factors.
Background
The Ethanol Blended Petrol (EBP) Programme was launched by the Central Government in
2003 to promote the use of alternative and environment friendly fuels.
It was intervention sought to reduce import dependency for energy requirements and
environment friendly measure to reduce vehicular pollution.
However, since 2006, OMCs were not able to meet required quantity of ethanol demand
against the tenders floated by them. The various constraints for it were like State Specific
issues, Supplier related issues including Pricing issues of ethanol.
In order to augment the supply of ethanol, a new mechanism for pricing of ethanol was
placed in where Government decided delivered price of ethanol at OMC depots. The decision
has helped in significantly improving the supply of ethanol.
What is Ethanol Blending?
The ethanol blending is process of mixing petrol with ethanol. The mixture is called as
Ethanol Fuel / Gasohol which is considered as a quasi-renewable energy.
Ethanol is biofuel derived from Sugarcane molasses (by-product in the conversion of
sugarcane to sugar), corn, sorghum etc.
In India, practice of blending ethanol was started in 2001. Ethanol blending for first time was
mention in the Auto fuel policy of 2003.
Later, the National Policy on Bio-fuels, 2009 made mandatory for oil companies to sell petrol
blended with at least 5% of ethanol.
Benefits of ethanol blending
It reduces the vehicular emissions especially carbon monoxide emissions.
It is cheaper than petrol as it is cheaper to manufacture.
It decreases a nation dependence on foreign oil.
Ethanol has a higher octane rating than ethanol-free petrol.
In case of India, ethanol production can give higher sugarcane price for farmers which can
help in rural prosperity.

October 14, 2016


Union Cabinet approves MoU between EXIM bank and NDB
October 14, 2016

The Union Cabinet has given its approval for signing of a MoU on General Cooperation with New
Development Bank (NDB) or BRICS Bank and Export Import Bank of India (EXIM Bank). The
MoU will be through the BRICS Interbank Cooperation Mechanism.
2016 GKToday | All Rights Reserved | www.gktoday.in

81

Current Aairs: October 2016 [Date-wise Compendium]


The MoU will help enhance trade and economic relations among the BRICS countries. However
there is no financial implication involved with signing of the MoU.
What is Purpose of MoU?
This MoU is a non-binding umbrella agreement. It aims at establishing a cooperation framework in
accordance with the national laws and regulations. Besides it also seeks for skills transfer and
knowledge sharing amongst the signatories.
Why this MoU was signed?
The establishment of the NDB reflects the close relations among the BRICS countries and
provides a powerful instrument for increasing their economic cooperation among them.
The signing of MoU is necessary keeping in view the strategic relevance of cooperation for
sustainable development and inclusive economic growth among BRICS countries.
It also reflects cooperation extended by the BRICS members in various forms for promoting
and facilitating trade of goods and services as well as investments in mutual projects.
Background
The BRICS Interbank Co-operation Mechanism was established by the five banks from the BRIC
nations to enhance trade and economic relations amongst the BRIC countries, and enterprises. It
proposes to sign a MOU on General Co-operation with the NDB. BRICS countries (Brazil, Russia,
India, China and South Africa) are developing countries, collectively accounting for 43% of the
worlds population, 15% of global trade and 20% of the worlds investment flows.

Antonio Guterres elected as new Secretary-General of UN


October 14, 2016

The United Nations General Assembly (UNGA) has formally elected Antonio Guterres (67) as the
new Secretary-General of the United Nations.
In this regard, 193 member states of UN adopted by acclamation a resolution for appointing Antonio
Guterres. He will have five-year term beginning January 1, 2017.
He will succeed retiring Ban Ki-moon who has completed two five-year terms as the worlds top
diplomat. Earlier in October 2016, Antonio Guterres had won unanimous support from UN Security
Council (UNSC).
Who is Antonio Guterres?
Antonio Guterres is Portuguese politician and diplomat.
He had entered politics in 1976 during Portugals first democratic election after the
Carnation Revolution which had ended five decades of dictatorship.
He was Prime Minister of Portugal from 1995 to 2002.
He was the United Nations High Commissioner for Refugees (UNHCR) from June 2005 to
December 2015.
2016 GKToday | All Rights Reserved | www.gktoday.in

82

Current Aairs: October 2016 [Date-wise Compendium]


He led the UNHCR through during one of the worlds worst present refugee crises, including
those in Syria, Afghanistan, and Iraq.

About Secretary-General of United Nations


The UN Secretary-General is the head of the United Nations Secretariat. He is de facto
spokesperson and leader of the UN.
The role Secretary-General has been mentioned in Chapter XV (Articles 97 to 101) of UN
Charter. The post lasts for five years and is limited to a maximum of two terms.
Summary
The post of Secretary-General of United Nations is not as powerful, but he serves as the UNs top
diplomat and chief administrative officer. He has to run an unwieldy bureaucracy and manage the
competing demands of the worlds big powers. His role is to step in both publicly and privately to
prevent escalation of international disputes.

India receives first Iranian oil parcel for emergency reserves


October 14, 2016

India has received the first parcel of crude oil from Iran to partly fill the strategic storage in southern
India as part of the Strategic Petroleum Reserves (SPRs) storage.
The first parcel of 2 million barrels of Iranian oil was imported by Mangalore Refinery and
Petrochemicals Ltd (MRPL) with the help of very large crude carrier (VLCC) Dino.
India is planning to fill half of the storage with 6 million barrels of Iranian oil while continuing talks
with United Arab Emirates and Saudi Arabia for the remainder. The second parcel of such reserve
will be procured by Bharat Petroleum Corp.
Why it is important for India to have strategic storage?
India imports around 80% of its crude oil requirement from the international market. Most of
these imports come from Middle East which is unstable region.
Any price aberrations in crude oil prices have a direct impact on the Indian economy
especially upon the current account and inflation amounting to strategic risks.
Thus, it is necessary for India to have SPRs which will act as a valuable buffer against short
term oil supply disruptions and global price shocks.
Government Proposals
Since 2003, the Central Government has commissioned the strategic oil reserves to maintain
nations energy as well as economic security.
The erstwhile Planning Commission in its Integrated Energy Policy, 2006 also had
recommended to maintain a reserve equivalent to 90 days of oil imports for strategic-cumbuffer stock purposes.
The recommendation was based on identification that any supply, market and technical risks
2016 GKToday | All Rights Reserved | www.gktoday.in

83

Current Aairs: October 2016 [Date-wise Compendium]


in global supply chain of oil may become major threat to Indias energy security.
Present Status
India is building emergency SPR storages in vast underground caverns at three locations in southern
India to store about 5.33 million tonnes of crude oil enough crude to cover almost two weeks of
demand. They are located are in Visakhapatnam, Mangalore and Padur.

Bob Dylan wins 2016 Nobel Prize in Literature


October 14, 2016

American singer-songwriter, artist and writer Bob Dylan (75) has won the 2016 Nobel Prize in
Literature. He is the first American to win the prize since novelist Toni Morrison in 1993.
The Swedish Academy has selected Dylan for this prestigious award for creating new poetic
expressions within the great American song tradition.

About Bob Dylan


Born as Robert Allen Zimmerman on May 24, 1941 in Duluth, Minnesota.
His debut album was Bob Dylan released in 1962. He had adopted the name Dylan after the
poet Dylan Thomas.
He is one of the best-selling artists (musician) of all time and has sold more than 100 million
records.
He has 58 singles, 37 studio albums, 11 live albums and 6 collaborations to his name.
Besides, his lyrics have incorporated various political, social, philosophical and literary
influences.
His early songs such as The Times They Are a-Changin and Blowin in the Wind were the
anthems for the American civil rights and anti-war movements.
He has published six books of drawings and paintings, and his work has been exhibited in
major art galleries.
Awards and Honours: 11 Grammy Awards, Golden Globe Award and Academy Award
(Oscar). The Pulitzer Prize jury had awarded him a special citation in 2008 for his profound
impact on popular music and American culture.

About Nobel Prize in Literature


The Nobel Prize in Literature is one of the five Nobel Prizes established by the will of Alfred
Nobel in 1895. It is awarded since 1901.
It is awarded annually to an author from any country who has produced the most outstanding
work in an ideal direction in the field of literature.
The award carries a gold medal, a diploma bearing a citation, and a sum of money.
Recent winners of Nobel Prize for Literature: Svetiana Alexievich (2015), Patrick Modiano
(2014), Alice Munro (Canada), Yo Yan (China).
2016 GKToday | All Rights Reserved | www.gktoday.in

84

Current Aairs: October 2016 [Date-wise Compendium]

India ranked 97th in 2016 Global Hunger Index


October 14, 2016

India has been ranked low 97th among the 118 countries surveyed in 2016 Global Hunger Index
(GHI). In 2016 GHI, India has scored low 28.5 on a 0-100 point scale of the index.
It describes Indias hunger situation as serious. The index was released by the International Food
Policy Research Institute (IFPRI).
What is Global Hunger Index (GHI)?
The GHI is a multidimensional statistical tool used to describe the state of countries hunger
situation. It is released annually by IFPRI since 2006.
It ranks countries on a 100-point scale. Zero on the scale is the best score (no hunger), and
100 is the worst.
It highlights successes and failures in hunger reduction and provides insights into the drivers
of hunger. Thus, GHI aims to trigger actions to reduce hunger.
The GHI is calculated by taking into account four indicator parameters. They are (i)
Undernourished population (1/3rd weight), (ii) Child wasting (1/6th weight), (iii) Child
stunting (1/6th weight) and (iii) Infant mortality rate (1/3rd weight).
Stunting: Deficiency in height in relation to age, reflects chronic undernutrition. Wasting:
Low weight in relation to a childs height, reflects acute undernutrition.
Key highlights of 2016 GHI
There is widespread and chronic lack of balanced food in India. About 15% of under-5
children are `wasted

and 39% of under-5 children are stunted.


The under-5 mortality rate is 4.8% in India, partially reflecting the fatal synergy of inadequate
nutrition and unhealthy environments.
2016 GKToday | All Rights Reserved | www.gktoday.in

85

Current Aairs: October 2016 [Date-wise Compendium]


Estimated 15% population is undernourished and lacks in adequate food intake, both in
quantity and quality.
Though India runs two of the worlds biggest childrens nutrition programmes malnutrition
continues to haunt India.
These two programmes are ICDS (Integrated Child Development Services) for children
under 6 years and the mid-day meal programme for school going kids up to the age of 14/
The main reasons for the sorry state in India are endemic poverty, unemployment, lack of
sanitation and safe drinking water, and lack of effective healthcare.
Indias Neighbours

22 MPs from smaller parties, independents in Rajya Sabha recognised as a consolidated


bloc
October 14, 2016

Vice-President and Rajya Sabha Chairman Hamid Ansari has recognised a group of 22 MPs
belonging to smaller parties and certain Independents as a consolidated bloc the United Group.
With this recognition the United Group, will be the third largest group of MPs in the Rajya Sabha,
after the Congress and the BJP.
This is only the third time in the history of Indian Parliament consolidated bloc has been recognised,
the first was in 1983 and the second in 1990.
In 1983, the first such consolidated group was called United Associations of Members was recognised
by the then Rajya Sabha Chairman M.Hidayatullah. In 1990, the then chairman of Rajya Sabha
2016 GKToday | All Rights Reserved | www.gktoday.in

86

Current Aairs: October 2016 [Date-wise Compendium]


recognised organised group of Parliamentarians and was renamed as the United Group.
What are benefits for consolidated bloc?
The newly recognised bloc will find a place in the Business Advisory Committee (BAC) that
decides time allotment. Thus, MPs of bloc will secure more time to speak in House debates.
At present, time allotted to parties to speak on debates depends entirely on their strength in
the House.
Earlier due to their solitary or numerically lean status, members of this bloc had as little as
three minutes of speech time.
Thus, a grouping of this kind will make it possible for a party like Sikkim Democratic Front
with a single MP to speak for as much time as say, a Samajwadi Party with 19 MPs in Rajya
Sabha.

About Rajya Sabha


The Council of States or Rajya Sabha is the upper house of the Parliament. It is permanent
house of the Parliament and unlike the Lok Sabha it is not subject to dissolution.
The origin of the Rajya Sabha as the second chamber can be traced to the MontagueChelmsford Report of 1918 and the Government of India Act, 1919.
As per Article 80 of Constitution, the maximum strength of Rajya Sabha is 250, out of which
238 are representatives of States and of two Union Territories having legislatures (Delhi and
Puducherry).
These members are indirectly elected by state and territorial legislatures using single
transferable votes.
Remaining 12 members are nominated by the President for their contributions to art,
literature, science, and social services.
Members of Rajya Sabha have six-year term and one third of the members retire every two
years.
The Vice President of India is the ex-officio Chairman of the Rajya Sabha, who presides over
its sessions.

India to eliminate use of HFC-23 by 2030


October 14, 2016

India has decided to eliminate use of HFC-23, a greenhouse gas (GHG) that harms the ozone layer by
2030. With this, India, is taking the lead on tackling climate change.
It was announced during the 28th Meeting of the Parties to the Montreal Protocol at Kigali in
Rwanda. In this meeting, final negotiations are taking place to substantially reduce the use of HFCs
(hydrofluorocarbons) by 2030.
Key Facts
2016 GKToday | All Rights Reserved | www.gktoday.in

87

Current Aairs: October 2016 [Date-wise Compendium]


The elimination will potentially check emissions of HFC-23 equivalent to 100 million tonnes
of CO2 over the next 15 years.
Indian companies will not be compensated for the costs involved in ensuring that these gases
are not released.
This move is considered as a major break away from the concept of financial assistance for
every action on environment in which India earlier had shown the lead.
What is HFC-23?
HFC-23 is a by-product of HCFC-22 (Hydrochloroflurocarbon-22), which is used in industrial
refrigeration. It is a potent greenhouse gas (GHG) with global warming potential of 14,800 times
more than that of CO2.
What is Montreal Protocol?
The Montreal Protocol, is an international treaty designed to protect the ozone layer. It came
into force in 1989.
It aims at reducing the production and consumption of ozone depleting substances (ODS) in
order to protect the earths fragile ozone layer.
It has been ratified by 197 parties making it universally ratified protocol in United Nations
history.
It is also highly successful international arrangement, as it has phased-out more than 95% of
the ODS so far in its main mandate less than 30 years of its existence.

India, Russia ink agreement to set up 25 Agro Irradiation Centres


October 14, 2016

India and Russia have signed an agreement for cooperation in the development of a network of 25
integrated infrastructure Agro irradiation centres.
The agreement was signed between Indian Agricultural Association, Hindustan Agro Co-Op Ltd
(HACL) and United Innovation Corporation (UIC), a subsidiary of ROSATOM State Atomic Energy
Corporation of Russia.
As part of the agreement, India-Russia joint venture (JV) will be formed in which HACL will have
the majority 51% stakeholding. UIC will be a technology partner and will help set up the irradiation
centres. In first phase 7 centers will be set up in Maharashtra in Sindhurg, Satara, Solapur, JNPT,
Jalana, Napur and Pune.
Importance of Agreement
For the first time paves the way for establishment of network of integrated infrastructure
irradiation centres in India to reduce agricultural losses.
These irradiation centres will help in treatment of perishable food items to improve shelf life
and cut post-harvest losses.
2016 GKToday | All Rights Reserved | www.gktoday.in

88

Current Aairs: October 2016 [Date-wise Compendium]


It will further increase basket perishable items for irradiation process on a commercial scale
ranging from flowers to fish.
Expand India-Russia cooperation in civil nuclear energy.
Why India needs strong network of Agro Irradiation Centres?
In India post-harvest losses in food and food grains are around 40-50%. It is primarily due to
microbiological contamination, insect infestation, physiological changes due to sprouting and
ripening, and poor shelf life. The wastage of fruits and vegetables including cereals, meat, pulses and
flowers results in the annual loss estimated to be Rs. 2,50,000 crores. There is presence of few low
level irradiation plants in the country which are not adequate. So to reduce the losses in food and
food grains and meet the demand it is necessary to increase strong network of agro irradiation
centres.
What is food irradiation?
Food irradiation is a technology in which food products are subjected to a low dosage of
radiation to treat them for germs and insects, increasing their longevity and shelf life.
It is the application of ionizing radiation to food in which energy is transmitted without
direct contact in the targeted food.
The radiation can be emitted by a radioactive substance or generated electrically. The
irradiated food does not become radioactive.
Radioisotopes are used as the source for the gamma rays. Generally cobalt-60 radioisotope is
used as a radioactive source for gamma irradiation.
Then International Atomic Energy Agency (IAEA) recommends the irradiation doses for
Food irradiation.
Benefits: Irradiation is equivalent to pasteurization for solid foods, but it is not the same as
sterilization.
It does not reduce the nutritional value of food products and does not change their
organoleptic properties and appearance.
The irradiation treatment of food and foodgrains preserves them for longer time, prevent the
spread of invasive pests, delay/eliminate ripening or sprouting and reduce the risk of food
borne illness.

October 15, 2016


Union Government to use post offices to distribute pulses
October 15, 2016

The Union Government has decided to utilise the vast network of post offices across the country to
sell subsidised pulses.
The decision was taken during the meeting of the inter-ministerial committee on prices of essential
2016 GKToday | All Rights Reserved | www.gktoday.in

89

Current Aairs: October 2016 [Date-wise Compendium]


commodities headed by Consumer Affairs Secretary.
The committee reviewed
The availability and prices of essential commodities, especially pulses.
The prices of pulses, sugar and other essential commodities.
The committee decided to release more gram (chana) from buffer stock to ensure its availability
through retail distribution at reasonable prices during the festive season.
Why post offices will distribute pulses?
The purpose of this move is to ensure that there is no shortage of the pulses during the
ongoing festival season.
The postal networks will fill the gap of absence of government outlets in the States for the
distribution of pulses, mainly, tur, urad and chana.
It will help government to leverage strong network of around 1.54 lakh post offices in the
country, of which 1.39 lakh are in rural areas.
It will help government to release more pulses from buffer stock and sell it at subsidised
prices to protect consumers from high prices.
In order to protect consumers from high prices, Central Government has been releasing tur and urad
from its buffer stocks to state governments as well as government agencies like NAFED and Mother
Dairy for retail distribution at a subsidised rate.

Women of India Festival 2016 of organic products inaugurated in New Delhi


October 15, 2016

The Women of India Festival 2016 of organic products was inaugurated by the Union Minister of
Women & Child Development in New Delhi
The 10 day festival has been organised by the Union Ministry of Women & Child Development to
benefit women entrepreneurs, and also actively promote organic food and products.
Key Facts
The Women of India Festival is intended to highlight the health and environmental
advantages of organic goods, provide a platform for women engaged in this economy.
It also seeks to encourage the development of sustainable and easily accessible sales outlets for
organic producers from remote areas.
This years Festival will have 230 stalls and about 372 participants. The list includes producers
from 23 states/UTs.
The festival will see display of large selection of organic products ranging from food, fabrics
and furniture to personal care, wellness and other products.

Ajay Kumar Bhalla appointed as DG of Directorate General of Foreign Trade


October 15, 2016

2016 GKToday | All Rights Reserved | www.gktoday.in

90

Current Aairs: October 2016 [Date-wise Compendium]


Senior IAS officer Ajay Kumar Bhalla has been appointed as new Director General of Directorate
General of Foreign Trade (DGFT).
He will succeed Anup Wadhawan, who has been appointed as Additional Secretary in the Union
Ministry of Commerce.
He is a 1984 batch IAS officer of Assam-Meghalaya cadre. Presently, he is the Additional Secretary in
the Union Ministry of Commerce.

About Directorate General of Foreign Trade (DGFT)


The DGFT is the nodal agency of the Union Ministry of Commerce and Industry, responsible
for execution of the import and export Policies of India.
It is entrusted with responsibilities for formulating and implementing the foreign trade policy
with the main objective of promoting Indias exports.
DGFT plays a very important role in the development of trading relations with various other
nations.
It also plays important role in improving not only the economic growth but also provides a
certain impetus needed in the trade industry.

Bangladesh, China agree to elevate their ties to strategic partnership


October 15, 2016

Bangladesh and China have agreed to elevate their bilateral relations to strategic partnership. It was
announced during Chinese President Xi Jinpings visit Bangladesh.
This is the first official state visit by Chinese head of state to Bangladesh in three decades since
President Li Xiannians visit in March 1986.
Highlights of Chinese President Xi Jinpings visit
The two countries also have agreed to jointly advance Chinas One belt and road (OBOR)
initiative. With this Bangladesh, formally joined Chinas OBOR initiative.
They also agreed to establish institutional cooperation in areas of maritime issues and
counter-terrorism.
They also agreed to increase high level exchanges and strategic communication between
them.
Both countries signed of 26 agreements on different sectors. Through these agreements
consensus was reached to work together in trade and investment and other key areas such as
infrastructure, energy and power, agriculture, ICT and transportation.
Comment
Geo-politically, Bangladesh is witnessing growing rivalry between China and India,
particularly in the era of weakening US role in the region. In this scenario, it signifies
growing importance of the Bangladesh in South Asias economics and geo-politics.
2016 GKToday | All Rights Reserved | www.gktoday.in

91

Current Aairs: October 2016 [Date-wise Compendium]


Bangladesh maintains very strong military ties with China. Even Bangladeshs economic
relations with China have traditionally been dominated by trade. The bilateral trade between
China and Bangladesh was about $12.5 billion in 2014 and Bangladeshs export to China has
not exceeded even $1 billion.
However, in this new type of emerging big power relationship in the South Asia region
relatively smaller countries like Bangladesh are adjusting and maintaining balanced economic
and geo-political relations, particularly with China and India.
What is One Belt, One Road (OBOR) Initiative?
OBOR initiative is Chinas ambitious development strategy and framework focuses on
connectivity and cooperation among countries primarily between China and rest of Eurasia.
It consists of two main components, the land-based Silk Road Economic Belt (SREB) and
oceangoing 21st Century Maritime Silk Road (MSR).
OBOR initiative part of Chinas revived 21st century Silk Road diplomacy that seeks to push
it to take a bigger role in global affairs as a major global power.

15 October: International Day of Rural Women


October 15, 2016

The International Day of Rural Women is observed annually across the world on 15 October to
recognize the critical role and contribution of rural women.
Significance of the day: Highlights the critical role and contribution of rural women, including
indigenous women in enhancing rural and agricultural development, eradicating rural poverty and
improving food security.
2016 Theme: Climate is changing. Food and agriculture must too. It tries to bring focus on farmers,
pastoralists and fishers who are standing on the frontlines of food insecurity due to climate changed
due to temperatures rise.
The United Nations also calls for empowering rural women as a pre-requisite for fulfilling the vision
of the Sustainable Development Goals (SGD) which aims at ending poverty and hunger, achieve food
security and empower all women and girls.
Why International Day of Rural Women is celebrated?
Rural women, make up over a quarter of the total world population. Majority of them depend
on natural resources and agriculture for their livelihood.
In developing countries, rural women represent approximately 43% of the agricultural labour
force. They produce, prepare and process much of the food available, thereby giving them
primary responsibility for food security.
Thus it is important to recognise the contribution and significant role played by these rural
women in food security and poverty elevation.
2016 GKToday | All Rights Reserved | www.gktoday.in

92

Current Aairs: October 2016 [Date-wise Compendium]


It is also necessary for ensuring rural womens access to productive agricultural resources
contributes to decreasing world hunger and poverty.
Background: The first International Day of Rural Women was observed on 15 October 2008. The
day was constituted by the UN General Assembly by passing resolution 62/136 in December 2007.

National Seismic Programme launched in Odisha


October 15, 2016

The Union Government has launched National Seismic Programme (NSP) to trace hydrocarbon
resources like oil and natural gas in Mahanadi basin.
It was launched by Union Minister of State (MoS) for Petroleum and Natural Gas at Taranga village
under Soro block in Balasore district of Odisha.
Significance
Mahanadi basin has been chosen as the first location for the roll out of this national-level
programme. The outcome from the NSP help in better understanding of the sub-surface with
petroleum systems across wider geological ages especially in sedimentary basins to explore
hydrocarbon reserves in Odisha.
What is National Seismic Programme (NSP)?
NSP aims to undertake a fresh appraisal in all sedimentary basins across India in order to have
better understanding of the hydrocarbon potential of the country.
Under this programme, high-resolution 2D seismic Acquisition, Processing and
Interpretation (API) survey will be conducted across India especially in the sedimentary
basins.
The survey will be conducted by state owned Oil and Natural Gas Corporation (ONGC) and
Oil India Limited (OIL). The survey project will be completed by March 2019.
ONGC will conduct the survey of around 40,835 Line Kilo Meter (LKM) in onland part of 26
sedimentary basins in 18 states/UTs including Mahanadi basin.
While OIL will carry out the survey of 7,408 LKM in the North eastern states of Assam,
Manipur, Arunachal Pradesh, Mizoram and Nagaland.

IISc researchers develop self-powered UV photodetector for self-charging energy storage


devices
October 15, 2016

The researchers from the Indian Institute of Science (IISc), Bengaluru have developed a costeffective, high-performance, self-powered UV photodetector.
The self-powered UV photodetector can use the harvested optical energy for direct self-charging of
energy storage devices such as supercapacitor. It can also be used for operating electronic devices in
the absence of external power source.
2016 GKToday | All Rights Reserved | www.gktoday.in

93

Current Aairs: October 2016 [Date-wise Compendium]


What is Photodetector?
Photodector is a device capable of sensing (detecting) or responding to electromagnetic energy,
typically light by using the electrical effect of individual photons.
What researchers have developed?
The researchers have developed the photodetector by integrating semiconducting vanadium
doped zinc oxide (VZnO) nanoflakes with a conducting polymer.
The zinc oxide (ZnO) is the base material for UV detection which can be doped with
vanadium to produce photodetectors that are self-powered.
The photodetector has superior performance in terms of faster detection of photo signals in
the order of milliseconds even when UV light intensity is low.
How it works?
When doped with vanadium, the microstructure of ZnO changes from nanorods to closelypacked nanoflakes. It causes increase in the surface area to the volume of the material.
It also creates surface defects within the band gap of ZnO, which helps in trapping the UV
radiation that falls on the nanoflakes.
These nanorods are one-dimensional and cause more light reflection from top surface. When
UV light enters into pores it undergoes multiple reflections and finally gets absorbed.
What are potential benefits?
The nanoflake (VZnO) produces five times more photocurrent, compared with ZnO, which
generates only 40 nA photocurrent.
Once these nanoflakes are hydrogenated, the current generation capacity further increased to
1,000 nA.
When these hydrogenated nanoflakes exposed to UV light, they detect photo signal within
milliseconds, which is nearly 100 times faster than conventional UV photodetectors.
VZnO nanoflakes structure has 98% light harvesting efficiency which is much higher than
84% seen in ZnO.

2nd Rashtriya Sanskriti Mahotsav begins in New Delhi


October 15, 2016

The second edition of Rashtriya Sanskriti Mahotsav (RSM)-2016 was inaugurated by Union Home
Minister Rajnath Singh at Indira Gandhi National Centre for Arts in New Delhi.
The 10-day event aims to showcase the rich cultural heritage of India in all its rich and varied
dimensions.
Key Facts
RSM 2016 will showcase Handicrafts, Painting, Cuisine, Photography, Sculpture,
Documentation and Performing Arts-Folk, Tribal, Classical and Contemporary in one place.
2016 GKToday | All Rights Reserved | www.gktoday.in

94

Current Aairs: October 2016 [Date-wise Compendium]


Nearly 2,000 artists from across the country will showcase Indias cultural heritage through
performances, arts during the cultural extravaganza.
It is one of the five such festivals planned in this financial year in different parts of India
including Bangalore, Varanasi and Jammu and Kashmir.
Under this cultural event, seven zonal cultural centres have been given slots to present their
arts during the festival.
It will showcase an amalgamation of Ek Bharat, Srestha Bharat, Hamaari Sanskriti, Hamaari
Pahchaan and Swachh India.

Brazil wins inaugural BRICS U-17 Football Tournament


October 15, 2016

Brazil has won the inaugural BRICS U-17 Football Tournament 2016 held at Goa on the occasion of
the 8th BRICS hosted in India.
In the final match, Brazil defeated South Africa by 5-1 goals. For Brazil goals were scored by Paulo
Henrique Samapaio Filho (24th Minute), Vinicius de Oliveira (34th minute), Victor de Oliveira (40th
& 61st minute). For South Africa only goal was scored by Smiso Bophela .
Earlier in the match for the third place between Russia and China, Russia had defeated China by 2-1
goals.

About BRICS U-17 Football Tournament


The BRICS U-17 Football Cup is a under 17 Football tournament played among teams of five
BRICS countries viz. Brazil, Russia, India, China and South Africa.
It was launched in October 2016 and seeks to transcend the traditional spheres of interaction
and opens new vistas in the field of sports.
The tournament will take place once every year and following the lines of the tournaments
under-17 FIFA. Each edition will be held in a BRICS nation.
In it, all five teams shall play each other in a round-robin before the top two teams qualify for
the final and the third and fourth place teams play in the third-place match.

Union Government launches satellite-based illegal Mining Detection System


October 15, 2016

Union Government has launched Mining Surveillance System (MSS), a pan-India surveillance
network to check illegal mining using latest satellite technology.
MSS is a satellite-based monitoring system which aims to check illegal mining activity through
automatic remote-sensing detection technology in order to establish a regime of responsive mineral
administration.
Mining Surveillance System (MSS)
Developed by Indian Bureau of Mines (IBM) in coordination with Bhaskaracharya Institute
2016 GKToday | All Rights Reserved | www.gktoday.in

95

Current Aairs: October 2016 [Date-wise Compendium]


for Space Applications and Geo-informatics (BISAG), Gandhinagar.
One of the first such surveillance systems developed in the world using space technology. It
has been developed under the Digital India Programme.
Why there is need of such system?
There are in total 3843 mining leases of major minerals across India, out of which 1710 are working
mines and 2133 are non-working mines. Presently, there is absence of robust mechanism of
monitoring of illegal mining activity. Current system of monitoring is completely based on local
complaints and unconfirmed information which is not transparent and quicker response and action.
How MSS works?
In the MSS, Khasra maps of mining leases have been geo-referenced and are superimposed
on latest satellite remote sensing scenes obtained from CARTOSAT & USGS.
It has capacity to check any unusual activity such illegal mining at a region of 500 meters
around the existing mining lease boundary.
It has been incorporated with automatic software which leverages image processing
technology to generate automatic triggers of unauthorized activities.
These automatic triggers will be studied at a Remote Sensing Control Centre of IBM and
then transmitted to the district level mining officials for field verification.
Besides, a user-friendly mobile app also has been created for these officials to submit
compliance reports of their inspections.
The mobile app aims to establish a participative monitoring system in which the citizens also
can use this app and report unusual mining activity.
MSS also has an executive dashboard to work as a decision support system. Using it officials
at all levels can track the current status of mapping of the mining leases, status of inspections
and penalty levied etc. for all major mineral mining leases across the country.
Advantages of remote sensing technology based monitoring system
Transparent as public will be provided an access to the system.
Deterrence Effect as continues monitoring will be from space.
Bias-free and Independent as the system is based on technology evidences and has no human
interference.
Quicker Response and Action as the mining areas will be monitored regularly and the
sensitive areas will be monitored more frequently.
Effective Follow-up as action will be taken on triggers that will be followed-up and
monitored at various levels

2016 GKToday | All Rights Reserved | www.gktoday.in

96

Current Aairs: October 2016 [Date-wise Compendium]

India, Russia sign 16 agreements across diverse sectors


October 15, 2016

India and Russia have signed 16 agreements across diverse sectors including in defence,
infrastructure, energy, space, ship building sectors etc.
They were signed following delegation-level talks co-chaired by Prime Minister Narendra Modi and
Russian President Vladimir Putin during the 17th India-Russia Annual Bilateral Summit in Goa.
Some of the signed agreements are
Agreement on cooperation in International Information Security.
MoU for expansion of bilateral trade and economic cooperation.
Shareholder agreement for establishing a Joint Venture to manufacture Ka-226T helicopter
in India.
MoU for developing smart cities in Andhra Pradesh and Haryana and for developing
transport logistics systems for such cities.
MoU between Gazprom and Engineers India Limited (EIL) for the joint study of a gas
pipeline to India from Russia and other areas of cooperation.
Cooperation Agreement between Rosneft Oil Company and ONGC Videsh Limited (OVL) in
the area of Education and Training.
MoU for setting up an investment fund of $1 billion by the National Investment and
Infrastructure Fund (NIIF) and the Russian Direct Investment Fund (RDIF).
MoU for Cooperation between Russian and Indian Railways in increasing the speed of trains
between Nagpur- Hyderabad/ Secundrabad.
MoU between ROSCOSMOS and ISRO on Mutual Allocation of Ground Measurement
Gathering Stations for NAvIC and GLONASS.
Inter-governmental agreements (IGAs) for the procurement of S-400 Air Defence System
and construction of 1135.6 series of frigates.
MoU between Indias Department of Science and Technology and Russias Federal Agency
for Scientific Organisations.

October 16, 2016


Rosneft-led consortium buys Indias Essar Oil for $13 billion
October 16, 2016

Russian consortium led energy giant Rosneft Oil Company has agreed to acquire Indias second
biggest private oil firm Essar Oil in an all-cash deal valued at about US $13 billion.
The consortium includes Rosneft Oil Company, Netherlands-based commodities trader Trafigura
and private investment group United Capital Partners.
The business transaction was announced in the presence of Prime Minister Narendra Modi and
2016 GKToday | All Rights Reserved | www.gktoday.in

97

Current Aairs: October 2016 [Date-wise Compendium]


Russian President Vladimir Putin at the 8th BRICS Summit in Goa.
Under this deal
Rosneft bought a 49% stake in Essar Oils refinery, port and petrol pumps, while Trafigura
Group Pte and United Capital Partners split 49% equity equally.
It includes $10.9 billion for Essar Oils Vadinar refinery (Indias second-largest refinery with a
capacity of 20 million tonnes) and $2 billion for a port terminal that helps feed refinery.
It also includes Essar Oils debt of $4.5 billion and about $2 billion debt with the Port
Company and power plant.
What is significance of the deal?
This acquisition is the largest ever foreign acquisition in India and also is the single largest
foreign investment in the Indian refining sector
It is Russias largest outbound or foreign acquisition deal. It significant milestone for Rosneft
as it is entering India, one of the most promising and fast-growing world markets.
With this, Rosneft becomes third international player after Royal Dutch and Shell and British
Petroleum to enter Indian fuel retailing market.
It also proves the attractiveness of the Indian energy market to foreign investors as India is
one of the fastest growing fuel consuming economies in the world
How India and Russia will benefit?
It will also help to strengthen the existing ties between Russia, one of worlds largest oil
producer and India, one of the worlds fastest growing fuel consumer.
Russia is keen to deepen its Soviet-era economic ties with India at a time when its own
economy is stagnant and is hit by Western sanctions over the Crimean crisis.

About Essar Group


Essar Group is an Indian conglomerate led by Shashi Ruia and Ravi Ruia. It controls number of
world-class assets diversified across the core sectors of Energy, Metals & Mining, Infrastructure and
Services. It also occupies a strategic position in the global oil market and owns world-class refining
and infrastructure assets.

Russia, India ink defence deal on S-400 Air Defence Systems


October 16, 2016

India has signed a defence deal worth nearly Rs 39,000 crore with Russia to purchase five state of art
S-400 Triumf air defence systems.
Besides, India also announced to collaborate in making four stealth frigates and setting up a joint
production facility for making Kamov helicopters under Make in India.
In this regard, inter-governmental agreements (IGAs) were signed between both countries following
talks between Prime Minister Narendra Modi and Russian President Vladimir Putin on the sidelines
2016 GKToday | All Rights Reserved | www.gktoday.in

98

Current Aairs: October 2016 [Date-wise Compendium]


of the 8th BRICS Summit in Goa.
What is S-400 Triumf Air Defence System?
An S-400 Triumf long-range air defence missile system is one of the most advanced longrange defence systems in the world. It is referred to as SA-21 Growler by NATO.
It is an upgraded version of the S-300 defence system and is manufactured by Almaz-Antey. It
has been in service in Russia since 2007.
It is capable of firing three types of missiles. Thus, it creates a layered defence, and
simultaneously engaging 36 targets.
It has capability to destroy incoming all airborne targets at ranges of up to 400 km. With it,
India can destroy Pakistani aircrafts even when they are in their airspace.
The five S-400 systems will be operated by the Indian Air Force (IAF). India is the second
export customer, after China to purchase this most advance defence system.
Ka-226T helicopters
Kamov 226T is a light multipurpose helicopter designed for work in difficult conditions like
high mountains, hot climate and marine areas.
It allows for reconnaissance, monitoring, targeting and transportation. It also boasts of
incredibly precise hovering ability, excellent maneuverability and high safety standards.
The helicopters will be made by Hindustan Aeronautics Limited (HAL) with Russian
cooperation under Make In India. It will replace the aging Cheetah and Chetak choppers.
After induction, Kamov helicopters will operate at 20,000 feet and alsol help to maintain the
forward posts on the Siachen glacier.
Frigates
India and Russia will built four stealth frigates which will be improved versions of Krivak or
Talwar class stealth frigates
Two frigates will be built in Russia and other two will be constructed at an Indian shipyard
with Russian cooperation.

Saraswati River existed: KS Valdiya committee


October 16, 2016

The K.S. Valdiya committee in its report submitted to the Union Water Resources Ministry has
concluded that Saraswati River so far considered mythical, existed.
The committee comprising of geologists, archaeologists and hydrologists say that they have found
evidence of the course of the river Saraswati.
Earlier, River Saraswati was considered as a mythological river and has been mentioned in
the ancient Hindu scriptures like the Rig Veda, Ramayana, Mahabharata and others giving credence
that it existed during those times.
2016 GKToday | All Rights Reserved | www.gktoday.in

99

Current Aairs: October 2016 [Date-wise Compendium]


Committee found that
Saraswati River originated in the Himalayas. It was approximately 4,000 km in length and
flowed through

India and Pakistan before meeting Arabian Sea through Rann of Kutch.
Two-third stretch of river measuring nearly 3000 km in length fell in India and reaming onethird of the river stretch fell in present-day Pakistan.
The river had two branches viz. (i) Western Branch: It represented by the Himalayan-born
Satluj River of the past which flowed through the channels of present-day GhaggarPatialiwali rivulets. (ii) Eastern Branch: Palaeochannels (remnants of defunct rivers)
Sarsuti-Markanda rivulets in Haryana were courses of eastern branch of river, known as
Tons-Yamuna.
The confluence of the branches was near Shatrana, 25 km south of Patiala. From this
confluence point river flow crossed dessert (Rann of Kutch) and meet gulf of western sea.
What is basis for conclusion?
The conclusion of the expert committee was based on the six-month research. During the
research the committee had came across an unique palaeochannel (path abandoned by river
when it changes course) relating to present Sarsuti, Ghaggar, Nara and Hakra rivers
They also had studied piles of sediments, their features and shapes which appeared to have
been brought by the big river and were considered reminiscent to ones found in present-day
Ganga, Ghaggar and Yamuna.

16 October: World Food Day


October 17, 2016

The World Food Day is observed globally on 16 October to mark foundation of Food and
Agriculture Organisation (FAO) of the United Nations in 1945.
2016 GKToday | All Rights Reserved | www.gktoday.in

100

Current Aairs: October 2016 [Date-wise Compendium]


Significance of Day: It aims to raise public awareness about hunger challenges and encourage
people around the world to take action in the fight against hunger.
2016 Theme: Climate is changing. Food and agriculture must too. It highlights the issue of food security
related to climate change. It calls adopting sustainable practices for growing more food with less area
of land and use natural resources wisely.
Background: The World Food Day was established in November 1979 by FAOs Member
Countries at the 20th General Conference of FAO to raise awareness of the issues behind poverty
and hunger. For the first time it was celebrated on 16 October 1981 and the idea of celebrating the
WFD worldwide was mooted by Hungarian Delegation to FAO.

About Food and Agriculture Organisation (FAO)


FAO is an agency of the United Nations that leads international efforts to defeat hunger. Its
parent organization is UN Economic and Social Council (UNESC).
It is neutral forum serving both developed and developing countries where all nations meet as
equals to negotiate agreements and debate policy.
Established: 16 October 1945.
Headquarters: Rome, Italy.
Motto: Let there be bread.
Members: 194 member states, along with the European Union (member organization)
FAO meet the demands posed by major global trends in agricultural development and
challenges faced by member nations.
It helps countries in transition modernize and improve agriculture, forestry and fisheries
practices, ensuring good nutrition and food security for all.

8th BRICS Summit concludes in Goa


October 17, 2016

The eighth BRICS summit was held in Goa from 15 to 16 October 2016. The summit concluded with
adaptation of Goa Declaration.
The two day summit was attended by the Prime Minister Narendra Modi, Chinese President Xi
Jinping, Russian President Vladimir Putin, Brazilian President Michel Temer and South African
President Jacob Zuma. The theme for the summit was Building Responsive, Inclusive and Collective
Solutions.
Key Highlights of Goa Declaration
BRICS nations condemns terrorism in all its forms and stressed that there can be no
justification for such acts.
International terrorism, especially the Islamic State (IS) is an unprecedented threat to
2016 GKToday | All Rights Reserved | www.gktoday.in

101

Current Aairs: October 2016 [Date-wise Compendium]


international peace and security.
BRICS nations reaffirmed the commitment to increase effectiveness of the UN counter
terrorism framework.
Need for adaptation of Comprehensive Convention on International Terrorism (CCIT) in
the UN General Assembly.
Urgent need to reforms of the United Nations, including UN Security Council, to increase
representation of developing countries.
Resolution of civil war in Syria, in accordance with the legitimate aspirations of the people
of Syria and action against U.N.-designated terrorist groups like IS and Jabhat al-Nusra.
All nations must counter radicalism and block sources of financing international terrorism,
dismantling terrorist bases and countering misuse of the Internet including social media.
Appreciated progress in implementation of Strategy for BRICS Economic Partnership and
emphasise importance of BRICS Roadmap for Trade, Economic and Investment Cooperation
until 2020.
Welcomed adoption of 2030 Agenda for Sustainable Development and its Sustainable
Development Goals.
Reiterated determination to use all policy tools to achieve the goal of sustainable and inclusive
growth.
The ninth BRICS Summit will be hosted by China in 2017.
Besides, first-ever BRICS-BIMSTEC Outreach Summit was also held on side-lines of 2016 BRICS
Summit. Leaders from both forums planned to jointly explore possibilities of expanding ties between
them on issues such as terrorism, economy, trade and connectivity.
BIMSTEC stands for Bay of Bengal Initiative for Multi-Sectoral Technical and Economic Cooperation.
BIMSTEC member countries are Bangladesh, India, Myanmar, Sri Lanka, Thailand, Bhutan and
Nepal.
About BRICS
BRICS is the acronym for an association of five major emerging national economies: Brazil,
Russia, India, China and South Africa.
It was established in2009. Originally it was known as BRIC before the inclusion of South
Africa in 2011. The first formal summit was held in Yekaterinburg, Russia.
The BRICS members are all developing or newly industrialised countries and all five are G-20
members.
They are distinguished by their large, fast-growing economies and significant influence on
regional and global affairs.
2016 GKToday | All Rights Reserved | www.gktoday.in

102

Current Aairs: October 2016 [Date-wise Compendium]

17 October: International Day for the Eradication of Poverty


October 17, 2016

The International Day for the Eradication of Poverty is observed globally on 17 October for
eradication of poverty throughout the world.
Significance of Day: It seeks to promote awareness among people to eradicate poverty and
destitution around the world particularly in developing countries
2016 Theme: Moving from humiliation and exclusion to participation: Ending poverty in all its forms. It
highlights importance to recognize and address the humiliation and exclusion suffered by many
people living in poverty.
Background: The United Nations had declared 17 October as the International Day for the
Eradication of Poverty by adopting a resolution on 22 December 1992. For the first time, it was
observed in 1993.

Japanese Encephalitis claims 54 lives in Odishas Malkangiri district


October 17, 2016

The Japanese Encephalitis has claimed 54 lives in Odishas backward Malkangiri district in just a
more than a month.
The State Government is monitoring and overseeing the measures to check the spread of Japanese
Encephalitis.
What is Japanese Encephalitis?
Japanese encephalitis (JE) is a mosquito-borne flavivirus. It belongs to the same genus as
dengue, yellow fever and West Nile viruses.
The first case of JE was documented in 1871 in Japan.
Target: JE primarily affects children. Most adults in endemic countries have natural
immunity after childhood infection, but individuals of any age may be affected.

2016 GKToday | All Rights Reserved | www.gktoday.in

103

Current Aairs: October 2016 [Date-wise Compendium]

Transmission: It is transmitted by rice field breeding mosquitoes (primarily Culex


tritaeniorhynchus group).
The mosquitoes transmit JE by feeding on domestic pigs and wild birds infected with the
Japanese encephalitis virus (JEV). It is not transmitted from person-to-person.
Disease outbreaks: Major JE outbreaks occur every 2-15 years. JE transmission mainly
intensifies during the rainy season, during which vector populations increase.
Signs and symptoms: Most JE infections are mild (fever and headache) or without apparent
symptoms, but it may result in severe clinical illness.
Severe infection is marked by quick onset, headache, high fever, neck stiffness, disorientation,
stupor, occasional convulsions (especially in infants) etc.
Treatment: There is no specific therapy. Intensive supportive therapy is indicated.

BRICS to set up credit rating agency


October 17, 2016

The five-nation group of emerging economies BRICS has agreed to set up an independent BRICS
Rating Agency in its efforts to challenge western hegemony in the world of finance.
It was announced during the 8th BRICS summit held in Goa, India. BRICS comprises five emerging
economies Brazil, Russia, India, China and South Africa.
Key Facts
The BRICS Rating Agency will be based on market-oriented principles to strengthen the
global governance architecture.
2016 GKToday | All Rights Reserved | www.gktoday.in

104

Current Aairs: October 2016 [Date-wise Compendium]


It will be launched as per the shared vision of the BRICS nations for institution-building to
transform global financial architecture based on the principles of fairness and equity.
It will assist BRICS and other countries to rate infrastructure and sustainable projects in the
emerging economies. Thus, it will further bridge the gap in global financial architecture
Why there is need to establish BRICS Rating Agency?
At present, western developed countries have hegemony in the world of finance. Three global
agencies (S&P, Fitch, Moodys) based in western countries account for 90% of the rating market.
Emerging economies claim that western ratings firms are biased, pessimistic on the developing
countries and optimistic on developed nations. They also have concerns over methodologies of the
three global agencies. BRICS members Russia in particular and China have been perturbed by these
western ratings firms. Russia alleges that the western firms had deliberately lowered its rating after
the Ukraine (Crimean) crisis.

Historic Global Greenhouse gas emission Agreement signed in Kigali


October 17, 2016

A historic global climate deal was reached in Kigali, Rwanda at the Twenty-Eighth Meeting of the
Parties to the Montreal Protocol on Substances that Deplete the Ozone Layer (MOP28).
The so called Kigali Amendment which amends the 1987 Montreal Protocol aims to phase out
Hydrofluorocarbons (HFCs), a family of potent greenhouse gases by the late 2040s.
Under Kigali Amendment, in all 197 countries, including India have agreed to a timeline to reduce
the use of HFCs by roughly 85% of their baselines by 2045.
What are Hydrofluorocarbons (HFCs)?
HFCs are a family of greenhouse gases (GHGs) that are largely used in refrigerants in home,
car air-conditioners and air sprays etc.
These factory-made gases had replaced CFCs under the 1987 Montreal Protocol to protect
Earths fragile protective Ozone layer and heal the ozone hole over the Antartica.
Why they are harmful?
In recent times, it was found that HFCs have several thousand times capacity in retaining heat
in the atmosphere compared to carbon dioxide (CO2), a potent GHG.
Thus, it can be said that HFCs have helped ozone layer but exacerbated global warming.
Currently, HFCs are currently the worlds fastest GHGs, with emissions increasing by up to
10% each year.
What is significance of the Kigali Amendment?
The Kigali Amendment amends the 1987 Montreal Protocol that was designed to close
growing ozone hole in by banning ozone-depleting coolants like chlorofluorocarbons (CFCs).
Thus, amended Montreal Protocol which was initially conceived only to plug gases that were
2016 GKToday | All Rights Reserved | www.gktoday.in

105

Current Aairs: October 2016 [Date-wise Compendium]


destroying the ozone layer now includes HFCs responsible for global warming.
This move will help to prevent a potential 0.5 degree Celsius rise in global temperature by the
end of the century.
The Kigali Agreement or amended Montreal Protocol for HFCs reduction will be binding
on countries from 2019.
It also has provisions for penalties for non-compliance. Under it, developed countries will
also provide enhanced funding support estimated at billions of dollars globally.
The exact amount of additional funding from developed countries will be agreed at the next
Meeting of the Parties in Montreal in 2017.
Different timelines under Kigali Amendment
All signatory countries have been divided into three groups with different timelines to go
about reductions of HFCs.
First group: It includes richest countries like US and those in European Union (EU). They
will freeze production and consumption of HFCs by 2018. They will reduce them to about
15% of 2012 levels by 2036.
Second group: It includes countries like China, Brazil and all of Africa etc. They will freeze
HFC use by 2024 and cut it to 20% of 2021 levels by 2045.
Third group: It includes countries India, Pakistan, Pakistan, Iran, Saudi Arabia etc. They will
be freezing HFC use by 2028 and reducing it to about 15% of 2025 levels by 2047.
How it is different from Paris agreement?
The Paris agreement which will come into force by 2020 is not legally binding on
countries to cut their emissions.
The Kigali Amendment is considered absolutely vital for reaching the Paris Agreement target
of keeping global temperature rise to below 2-degree Celsius compared to pre-industrial
times.

China successfully launches its longest ever manned mission to space


October 17, 2016

China has successfully launched longest-ever manned mission by taking two astronauts Jing Haipeng
(50) and Chen Dong (37) into the orbit.
They were launched on board of Shenzhou-11 (heavenly vessel) spacecraft from the Jiuquan Satellite
Launch Centre near the Gobi Desert in northwest China. The spacecraft was put into orbit by a
Long March-2F carrier rocket.
Key Facts
Both astronauts will spend a month aboard an experimental space laboratory Tiangong-2
2016 GKToday | All Rights Reserved | www.gktoday.in

106

Current Aairs: October 2016 [Date-wise Compendium]


(Heavenly Palace 2).
During the mission, they will conduct aerospace medical experiments, space science
experiments and in-orbit maintenance with human participation.
They will also undertake ultrasound tests during space travel for the first time, cultivate
plants in space etc.
The Shenzhou-11 mission is Chinas sixth manned spacecraft. It will be first time that
Chinese astronauts will stay in orbit for medium term.
Earlier in 2013, three Chinese astronauts in a manned space mission had spent 15 days in
orbit and were docked with a space laboratory Tiangong 1.
Comment
Tiangong-2 was launched in September 2016 as part of Chinas efforts to set up its own permanent
manned space station by 2022. It is Chinas second experimental space station module and had
upgraded habitat with improved life support systems, power, communications and research
equipments. By launching longest-ever manned space mission, China has moved a step closer to
establish its permanent space station. After completion, it will make China, only the country to have
space station facility is space as the current in-service International Space Station (ISS) retires by
2024.

National Summit on Fortification of Food inaugurated in New Delhi


October 17, 2016

The National Summit on Fortification of Food was recently inaugurated in New Delhi to address
interventions in combating micronutrient malnutrition in the country.
The two day summit was co-hosted by the Food Safety and Standards Authority of India (FSSAI) in
partnership with related central ministries/departments and development partners.
The Summit had brought together experts from nutrition and development communities as well as
representatives from state governments, technical supporters, academics and donors to beneficiaries.
What is Food fortification?
Food fortification or enrichment is the process of adding micronutrients i.e. essential trace
elements and vitamins into the food.
It is an integrated approach to prevent micronutrient deficiencies and complements other
approaches to improve health and nutrition.
Food fortification does not require changes in existing food habits and patterns nor individual
compliance. It does not alter characteristics of food and is socio-culturally acceptable.
It can be introduced quickly and can produce nutritional benefits and improve health of
people in a short period of time. It also safe and cost effective.
This enrichment process has proven as an effective strategy to meet the nutritional needs of a
2016 GKToday | All Rights Reserved | www.gktoday.in

107

Current Aairs: October 2016 [Date-wise Compendium]


large number of people across various sections of the society, including the poor and
underprivileged, pregnant women and young children.
What is present status of food fortification in India?
The FSSAI has formulated a Food Safety and Standards (Fortification of Foods) Regulations,
2016, a comprehensive regulation on fortification of foods in the country.
These regulations set standard guidelines for food fortification and encourage the production,
manufacture, distribution, sale and consumption of fortified foods.
They also assign specific role of FSSAI in promotion for food fortification and to make
fortification mandatory.

BIMSTEC countries express commitment to combat terrorism in all its forms


October 17, 2016

The BIMSTEC (Bay of Bengal Initiative for Multi-Sectoral Technical and Economic Cooperation)
countries have expressed commitment to combat terrorism in all its forms and manifestations.
In this regard, seven nations of BIMSTEC countries have issues Leaders Retreat Outcome Document
emphasising that there can be no justification for acts of terror.
Key Highlights of document
BIMSTEC countries strongly condemned the recent terror attacks in the region.
The states who encourage and support provide sanctuary to terrorists should be identified
and held accountable. Strong measures against such states must be taken.
They recognised the need for urgent measures to counter and prevent the spread of
terrorism, violent extremism and radicalization.
They renewed their commitment to the early conclusion of BIMSTEC Free Trade Area
negotiations, and direct the Trade Negotiating Committee and Working Groups to expedite
the finalization of its constituent Agreements.
They agreed to expedite the signing of the BIMSTEC Convention on Mutual Assistance in
Criminal Matters.
They also called for early ratification of the BIMSTEC Convention on Cooperation in
Combating International Terrorism, Transnational Organized Crime and Illicit Drug
Trafficking.
They also stressed the necessity of pursuing sustainable development and of the
implementation of the Paris Agreement on Climate Change.
They also recognized that cooperation in sustainable development of fisheries in Bay of
Bengal region for ensuring food security and improving livelihoods of people.

About BIMSTEC
BIMSTEC (Bay of Bengal Initiative on Multi Sectoral Technical and Economic Cooperation)
2016 GKToday | All Rights Reserved | www.gktoday.in

108

Current Aairs: October 2016 [Date-wise Compendium]


is the sub-regional group of seven countries in South Asia and South East Asia lying in the
littoral and adjacent areas of the Bay of Bengal constituting a contiguous regional unity.
They are India, Nepal, Bangladesh, Bhutan, Sri Lanka (from South Asia) and Myanmar,
Thailand (from South East Asia).
Established: 6 June 1997 through the Bangkok Declaration.
Headquarters: Dhaka, Bangladesh.
Objectives: Technological and economical cooperation among south Asian and south East
Asian countries along the coast of the Bay of Bengal.
It also covers cooperation in commerce, technology, investment, agriculture, tourism, human
resource development, fisheries, transport and communication, textiles, leather etc.

October 17, 18, 2016


Indian wrestlers win four medals at 2016 TAFISA World Games
October 18, 2016

Four Indian wrestlers have won four medals individually including a gold medal, at the sixth edition
of the TAFISA World Games held in Jakarta, Indonesia.
Wrestler Dalmiya had won gold medal in 60kg category by defeating Azerbaijans Muhammad Sahan
by 4-1 points in the final match.
Other medal winning Wrestlers are
Luv Singh: He won the silver medal in the 80 kg category after he was defeated by
Muhammad Alim of Azerbaijan by 1-5 points in the final match.
Naveen Kumar: He won the bronze medal in 90 Kg category after he defeated Lithuanias
Oleg by 5-2 points.
Josil: He won the bronze medal in the super heavyweight category by defeating Mustafa
Sultani of Afghanistan by 3-1 points.
Besides, Aarti Chaudhari (29), Vishruti Waghmare (16) and Siddhi Jadhav (10) won three gold
medals in Jeet Kune-do, a form of martial arts.

About Association for International Sport for All (TAFISA)


The TAFISA is the leading international Sport for All organisation with more than 260 members in
over 150 countries on all continents. Its vision is to achieve an Active World by globally promoting
and facilitating access for every person to Sport for All and physical activity.

Eminent writer C Radhakrishnan wins Mathrubhumi Literary Award 2016


October 18, 2016

Eminent writer C Radhakrishnan was selected for the prestigious Mathrubhumi Literary Award for
the year 2016.
He was selected by a panel chaired by novelist M Mukundan in recognition of his contribution to the
2016 GKToday | All Rights Reserved | www.gktoday.in

109

Current Aairs: October 2016 [Date-wise Compendium]


field of literature.

About C Radhakrishnan
He is a writer and film director in Malayalam language. He was born in February 1939in
Ponnani taluk in Malappuram district of Kerala.
Currently, he is editor of Malayalam magazine Piravi. Earlier he had served as chief editor of
Madhyamam daily from 1999 to 2001.
He has directed films like Agni (1978), Kanalattam (1979), Pushyaragam (1979) and
Ottayadippaathakal (1990).
Awards and Honours: He has been bestowed with National Literary Academy of India
(1989), Kerala Literary Academy (1962) etc.

About Mathrubhumi Literary Award


Mathrubhumi Literary Award is a literary award bestowed upon writers for his overall
contribution to the Malayalam literature.
The award was instituted in 2001 by leading Malayalam daily Mathrubhumi. It carries sum of
2 lakh, a plaque and citation.

Amandeep Singh Gill appointed as Indias Ambassador to UN Conference on Disarmament


October 18, 2016

Senior IFS officer Amandeep Singh Gill was appointed as the Indias Ambassador to the UN
Conference on Disarmament, Geneva.
He is an Indian Foreign Service (IFS) officer of 1992 batch. He had represented India on the UN
Secretary Generals Panel on Missiles from 2007 to 2008. Besides, he was also member of the Indian
delegation to the Conference on Disarmament during the negotiations on the Comprehensive
Nuclear Test Ban Treaty (CTBT).
What is UN Conference on Disarmament?
The Conference on Disarmament (CD) is not formally a United Nations (UN) organization.
It is linked to the UN through a personal representative of the UN Secretary-General
It is a forum established in 1979 by the international community to negotiate multilateral
arms control and disarmament agreements.
The conference has 65 members represent all areas of the world, including all known
nuclear-weapon states (including India, Pakistan, Israel and North Korea)
Resolutions adopted by the UN General Assembly often request conference to consider
specific disarmament matters. In turn, conference annually reports its activities to UNGA.
In the 1990s, the Conference had held intensive efforts to draft CTBT text and its two
annexes, but it did not succeed in reaching consensus on the adoption of the text.
Currently it conducts discussion on Fissile Material Cut-off Treaty (FMCT), a pact to prevent
2016 GKToday | All Rights Reserved | www.gktoday.in

110

Current Aairs: October 2016 [Date-wise Compendium]


an arms race in outer space, nuclear disarmament and negative security assurances (NSA).

Ashok Leyland rolls out first India-made electric bus


October 18, 2016

Conglomerate Hinduja Groups flagship company Ashok Leyland has unveiled countrys first 100 per
cent indigenously made electric bus which are non-polluting and have zero-emissions.
These buses were unveiled in lines with National Electric Mobility Plan which aims at a 20%
penetration of electric/hybrid (eco-friendly) vehicles by 2020. They enjoy a subsidy under the
Central governments Faster Adoption and Manufacturing of (Hybrid) and Electric Vehicles
(FAME) scheme.
Key Facts
These electric buses will be offered on multiple platforms and have seating capacity ranging
from 35 to 65. They can travel 150 km on a single charge.
They have been integrated with a fire detection and suppression system. They are built on
mass-market platform that will enable operator to cater to customers in city centres with
minimal operational and maintenance costs.
These buses can be used in heritage sites, metro stations, tarmacs, rural areas, hill stations and
by IT firms and educational institutions.
Besides, state Transport Undertakings can also use them for environment friendly mass
public transportation.
Way Forward
In India, electric buses in future will have a big leap in mass public transport. It would support the
government initiative of reducing fuel import bill. Besides, it will help to curb air pollution as these
buses have zero tailpipe emissions and lower noise pollution.
What is National Electric Mobility Mission Plan (NEMMP) 2020?
The NEMMP 2020 is one of most ambitious initiatives undertaken by Central Government
to promote hybrid and electric vehicles in the country to achieve national fuel security.
It has set an ambitious target to achieve 6-7 million sales of hybrid and electric vehicles year
on year from 2020 onwards.
The FAME India Scheme was launched under this mission to provide fiscal and monetary
incentives to electric and hybrid vehicles ranging from two wheelers to buses.

India completes nuclear triad by commissioning INS Arihant


October 18, 2016

India has completed its nuclear triad by inducting the first indigenously built strategic nuclear
submarine INS Arihant into service.
With this, India becomes sixth country to have a nuclear triad i.e. capable of delivering nuclear
2016 GKToday | All Rights Reserved | www.gktoday.in

111

Current Aairs: October 2016 [Date-wise Compendium]


weapons by aircraft, ballistic missiles and submarine launched missiles. Other countries having
nuclear triad are Russia, United States, United Kingdom, France and China.
INS Arihant was formally commissioned by Navy Chief Admiral Sunil Lanba in August 2016.
However, it was kept secret owing to strategic significance.
What are features of INS Arihant?
INS Arihant is nuclear submarine capable of carrying nuclear tipped ballistic missiles. Thus it
belongs to class

referred to as Ship Submersible Ballistic Nuclear (SSBN).


Its design is based on the Russian Akula-1 class submarine. It was built with Russias help
under Advanced Technology Vessel (ATV) project started in the 1980s.
It weighs 6000 tonnes and is 112 metre long. It is powered by 83 MW pressurised light water
nuclear reactor with enriched uranium fuel.
It will be armed with the K-15 Sagarika missiles with a range of 750 km. Later it will be
armed with much longer range K-4 missiles (3,500 km range) being developed by DRDO.
The vessel was launched in 2009 by then Prime Minister Manmohan Singh. Since then, it had
undergone extensive sea trials and reactor on board went critical in 2013.
What is Ship Submersible Ballistic Nuclear (SSBN)?
SSBNs are designed to prowl the deep ocean waters and carry nuclear weapons. They provide a
nation with an assured second strike capability i.e. capability to strike back after being hit by nuclear
weapons first. In case of India, second strike capability is particularly important as it had committed
2016 GKToday | All Rights Reserved | www.gktoday.in

112

Current Aairs: October 2016 [Date-wise Compendium]


to a No-First-Use policy as part of its nuclear doctrine.
Nuclear triad capability of India
India is sixth Nation in the world to possess Nuclear Triad. It means that India is capable of
delivering nuclear weapons by aircraft, land based ballistic missiles and submarine launched
missiles.
India Army has strong arsenal of land based ICBM (intercontinental ballistic missile). It
includes Agni series, Prithvi series, Prahar Missile, Shaurya Missile, supersonic Brahmos and
subsonic Nirbhay missiles.
Indian Airforce (IAF) operates Jaguars aircrafts which are designed for deep penetration
strike and can carry nuclear bomb. Besides, IAF also has SU 30 MKI and Rafale aircrafts
which can also be used to deliver nuclear weapons.
Indian Navy now has its own nuclear powered ballistic missile submarine INS Arihant
capable of delivering nuclear weapons.

PM Narendra Modi inaugurates three mega hydro projects in Himachal Pradesh


October 18, 2016

Prime Minister Narendra Modi inaugurated three hydro-electric projects (HEPs) with a generating
capacity of 1,732 MW in Himachal Pradesh.
All the three power projects are equipped with generating equipment supplied and commissioned by
state owned Bharat Heavy Electricals Limited (BHEL).
These three hydro-electric projects are
Koldam HEP: It is 4200 MW capacity project executed by National Thermal Power
Corporation (NTPC).
Parbati HEP: It is Stage-III project with a capacity of 530 MW of National Hydropower
Corporation (NHPC).
Rampur HEP: It is 412 MW capacity project of Sutlej Jal Vidyut Nigam (SJVN).
These three projects will collectively play an important role in boosting the Hydro-power sector of
Himachal Pradesh.
BHELs contribution
BHEL has a vast experience in hydro-electric projects (HEPs). It has contracted more than 500
hydro-generating sets with a cumulative capacity of more than 29,000 MW of various ratings in
India and abroad. BHELs hydro installations are in operation in India and also across the world in
Bhutan, Malaysia, Azerbaijan, Nepal, Tajikistan, Taiwan, Rwanda and Vietnam. Presently, BHEL is
executing HEPs of around 3,300 MW in the country in addition to major hydropower projects in DR
Congo and Bhutan.
Significance of hydro-projects
2016 GKToday | All Rights Reserved | www.gktoday.in

113

Current Aairs: October 2016 [Date-wise Compendium]


Hydropower generated by hydro-projects is renewable source of energy as it uses and not consumes
the water for generation of electricity. It does not emit greenhouse gas emission and helps in
achieving a low carbon path. India has hydropower potential of around 1,45,000 MW and at 60%
load factor, it can meet the demand of around 85, 000 MW. So far, around 26% of Hydropower
potential has been exploited in India.

PM launches National SC/ST hub and Zero Defect Zero Effect scheme
October 18, 2016

Prime Minister Narendra Modi launched the National SC/ST hub and the Zero Defect, Zero Effect
(ZED) scheme for Micro, Small and Medium Enterprises (MSMEs) at Ludhiana in Punjab.
Besides, he also presented National Awards to MSMEs. He also distributed 500 traditional wooden
charkhas (spinning wheels) among women.
What is National SC/ST hub?
The objective of the SC/ST (Schedule Castes/Schedule Tribes) Hub is to provide professional
support to entrepreneurs from the SC/ST.
It also seeks to promote enterprise culture and entrepreneurship among the SC/ST
population and to enable them to participate more effectively in public procurement.
It will work towards strengthening market access/linkage, capacity building, monitoring,
sharing industry-best practices and leveraging financial support schemes.
Public Procurement Policy, 2012: It specifies that 4% of procurement done by ministries,
departments and CPSEs should be done from enterprises owned by SC/ST entrepreneurs.
What is Zero Defect, Zero Effect (ZED) scheme?
ZED Scheme aims to rate and handhold all MSMEs to deliver top quality products using clean
technology. It will have sector-specific parameters for each industry.
MSME sector is crucial for the economic progress of India and this scheme will help to match
global quality control standards.
The slogan of Zero Defect, Zero Effect (ZED) was first mentioned by PM Narendra Modi in his
Independence Day speech in 2014. It was given for producing high quality manufacturing products
with a minimal negative impact on environment.
Comment
ZED Scheme is meant to raise quality levels in unregulated MSME sector which is engine of
growth for Indian economy. MSME sector drives almost 38% of nations GDP and around
employs 110 million employees.
The scheme will be cornerstone of the Central Governments flagship Make in India
programme, which is aimed at turning India into a global manufacturing hub, generating
jobs, boosting growth and increase incomes.
2016 GKToday | All Rights Reserved | www.gktoday.in

114

Current Aairs: October 2016 [Date-wise Compendium]

October 19, 2016


Delhis air quality very poor: SAFAR
October 19, 2016

According to System of Air Quality and Weather Forecasting and Research (SAFAR) national capital
New Delhis air quality was recorded in the very poor category.
The average (24-hour rolling) of PM2.5 and PM10, suspended respirable pollutants, were recorded
as 120.8 and 248 micrograms per cubic metre as against safe limits of 60 and 100.
The poor air quality may lead to respiratory problems, irritation to eyes and skin, blurring of vision
and headache.
The major reasons for the poor air quality may due to the favourable cold weather conditions
coupled with very light wind, vehicular pollution and farm fires in neighbouring states of Punjab,
Haryana.
What is System of Air Quality and Weather Forecasting and Research (SAFAR)?
SAFAR was introduced by Union Ministry of Earth Sciences (MoES) for greater
metropolitan cities of India to provide location specific information on air quality in near real
time.
It was developed by Indian Institute of Tropical Meteorology (IITM), Pune along with India
Meteorological Department (IMD) and National Centre for Medium Range Weather
Forecasting (NCMRWF).
The main objective of SAFAR project is to increase awareness among general public
regarding the air quality.
The purpose awareness for general public is to take appropriate mitigation measures and
systematic action for betterment of air quality and related health issues.

Saina Nehwal appointed member of IOCs Athletes Commission


October 19, 2016

Ace Indian shuttler Saina Nehwal has been appointed as a member of the International Olympic
Committees (IOC) Athletes Commission. It is rare honour for an Indian sportsperson.
In this regard, Saina has received a letter to the effect from the IOC President Thomas Bach
mentioning that she has been appointed in consultation with Chair of Athletes Commission.
The Athletes Commission of IOC is chaired by Angela Ruggiero. It comprises nine vice presidents
and 10 other members.

About Saina Nehwal


She is the first Indian to win Olympics medal in Badminton and had achieved this feat by
winning the Bronze medal at the London Olympics 2012.
She is the first Indian to win the World Junior Badminton Championships and was also the
2016 GKToday | All Rights Reserved | www.gktoday.in

115

Current Aairs: October 2016 [Date-wise Compendium]


first Indian to win a Super Series tournament, by winning Indonesia Open in 2009.
Awards: Government of India has awarded her with Arjuna Award (2009), Rajiv Gandhi
Khel Ratna (20092010), Padma Shri (2010) and Padma Bhushan (2016).
What is International Olympic Committees (IOC) Athletes Commission?
The IOC Athletes Commission is a body of sportspersons maintained by the IOC for the
purpose of representing athletes.
It was established in 1981 and its creation was codified by the IOC in Rule 21 of the Olympic
Charter.
The athletes commission works as a consultative body and act as link between active athletes
and the IOC.
It makes recommendations to the IOCs executive bodies. Its chair serves as a member of the
IOC Executive Board.
It meets once or twice a year, and also organises an Athletes Forum once every two years.

Government funds Ro Pax Ferry Service in Gulf of Cambay under Sagarmala Programme
October 19, 2016

The Union Ministry of Shipping has sanctioned the Capital Dredging Project of Ro Pax Ferry
Services between Gogha & Dahej, in Gulf of Cambay in Gujarat under Sagarmala programme.
The total project cost is estimated to be Rs 234 Crore of which 50% will be funded by the Union
Government under the Sagarmala programme.
Significance of Project
This is first of its kind project in India as it will be executed in the area of worlds 2nd highest
tidal range.
This project on completion will result in reduction in motorable distance of 231 kms between
Gogha and Dahej in Gujarat to mere 31 kms.
It will pave way for launching Indias first roll on-roll off (Ro-Ro) vessel passenger ferry
services on this route by April 2017.
Besides, it will reduce the travel time to 1 hour from 7 hours. It will also result in savings in
fuel, reduction in CO2 emission and reduction in road congestion.
The project will open up new avenues in coastal shipping & tourism and help in socioeconomic development of proximate areas.
It will also help in utilisation of inland waterways through River Narmada for shipping goods
from industries located upstream.
What is Sagarmala Programme?
The Sagarmala Programme is Union Governments flagship strategic, customer-oriented
initiative to achieve the broad objective of promoting port-led economic development in
2016 GKToday | All Rights Reserved | www.gktoday.in

116

Current Aairs: October 2016 [Date-wise Compendium]


India.
The ambitious programme seeks to harness vast potential of Indias 7,500 km long coastline,
14,500 km of potentially navigable waterways and strategic location on key international
maritime trade routes.
It envisages the growth of coastal and inland shipping as a major mode of transport for
carriage of goods and people along the coastal and riverine economic centres.
Sagarmala when integrated with the development of inland waterways will help to reduce
cost and time for transporting goods, benefiting industries and export/import trade.
Gulf of Cambay
The Gulf of Cambay (also known as Gulf of Khambhat) is an inlet of the Arabian Sea in the state of
Gujarat along the west coast of India. It divides the Kathiawar peninsula to the west from the eastern
part of Gujarat state. Narmada, Tapti, Mahi and Sabarmati rivers drain into it.

Union Government proposes 4 GST slabs


October 19, 2016

The Union Government has proposed a four-slab rate structure for the new indirect tax regime i.e.
Goods & Services Tax (GST) ranging from 0 to 26 per cent.
It was proposed at the meeting of the GST Council headed by Union Finance Minister Arun Jaitley
and included representatives from all states in New Delhi.
Outcomes of Meeting based on consensus
Compensation of states due to loss of revenue after implementation of GST from 1 April
2017.
Secular growth rate of 14% will be taken into consideration for calculating the revenue of
each state in the first five years of implementation of the GST.
Base year for calculating the revenue of states will be 2015-16. States getting lower revenue
will be compensated by the Centre.
These states will be compensated fund to be created from the Cess on top of the GST on
ultra-luxury items and demerit goods.
The rate structure will be revenue-neutral so that there is no need to burden consumers with
additional tax. It will not lead to further inflation.
Good items along with other 50% items of common usage to be exempted from the tax to
keep the inflation under check.
Proposed Four GST slabs
The GST is proposed to be levied at 6% (lower rate), 12% (Standard 1 rate), 18% (Standard 2
rate) and 26% (Higher rates) on the goods and services.
It will be 0% on host of goods and services, including food, health and education services, and
2016 GKToday | All Rights Reserved | www.gktoday.in

117

Current Aairs: October 2016 [Date-wise Compendium]


26% on luxury items, such as fast-moving consumer goods and consumer durables.
On consumption of ultra-luxury items and demerit goods, such as big cars and tobacco
products, cess must be imposed over and above a 26% GST rate.
What is GST Council?
As per Article 279A of the Constitution, GST Council will be a joint forum of the Centre and the
States. It shall consist (i) Union Finance Minister (Chairperson). (ii) The Union Minister of State
(MoS) in-charge of Revenue of finance (Member) and (ii) The Minister In-charge of taxation or
finance or any other Minister nominated by each State Government (Members).
What are the functions of GST Council?
The functions of GST Council are mention in Article 279A (4) of the Constitution. The Council will
make recommendations to Union and States on important issues related to GST, like (i) Goods and
services that may be subjected or exempted from GST. (ii) Model GST Laws. (iii) Principles that
govern Place of Supply, threshold limits, GST rates. (iv) GST rates will including the floor rates with
bands and (v) Special rates for raising additional resources during natural disasters/ calamities,
special provisions for certain States, etc.

NPL Scientists produce electricity from water without using energy


October 19, 2016

Scientists have developed a novel way using of producing electricity from water hydroelectric cell at
room temperature without using any power or chemicals.
The major breakthrough was developed by team of scientists led by Dr. RK Kotnala from Delhis
National Physical Laboratory (NPL).
How does it works?
In this new method, scientist had used zinc and silver as electrodes to make a cell that
produces electricity. They had used nanoporous magnesium ferrite to split water into
hydroxide (OH) and hydronium (H3O) ions.
As magnesium has high affinity for hydroxide, it spontaneously splits water into hydroxide
and hydronium ions. The H3O ions get trapped inside the nanopores of magnesium ferrite
and generate an electric field. The electric field helps in further dissociation of water.
To further enhance the activity of magnesium ferrite, about 20% of magnesium is replaced
with lithium. The substitution of lithium at magnesium site increases the sensitivity of
magnesium ferrite.
This is helpful in dissociating water at room temperature as the electrons get trapped in the
oxygen deficient sites.
Significance: The hydroelectric cell using magnesium ferrite of 1 sq. inch size can produce 8 mA
current and 0.98 volt. Further if these four cells [of 2-inch diameter] are connected in series the
2016 GKToday | All Rights Reserved | www.gktoday.in

118

Current Aairs: October 2016 [Date-wise Compendium]


voltage increases to 3.70 volts and can operate a small plastic fan or a LED light of 1 watt.

Auto manufacturers to provide emission and noise pollution details for every vehicle
October 19, 2016

The Union Ministry of Road Transport & Highways has made it mandatory for all automobile
manufacturers to provide emission and noise pollution details for every vehicle they produce by
April 2017.
In this regard, the ministry has amended Form 22 under the Central Motor Vehicles Act, 1989
through which manufactures provide the initial certification of compliance of vehicles.
Basically, it will include pollution standards, safety standards of component quality and roadworthiness certificate for all vehicles.
With this mandatory measure, the Union Ministry of Road Transport & Highways in future intends
to award five star ratings to vehicles based on their emission and noise pollution standards.
What are provisions of amended Form 22?
Mandatory for all automobile manufacturers to provide emission and noise pollution details
for every vehicle including makers of electric rickshaws and electric carts.
The Form will include the brand, engine number, chassis number and emission norm of the
vehicle. It will apply to all vehicles including petrol, CNG, LPG, electric, diesel and hybrid.
Automobile manufacturers must specify the levels of each pollutant like carbon monoxide,
hydro carbon, nonmethane HC, NOx, HC+NOx, PM etc. for petrol and diesel vehicles.
They should also specify the sound level for horn and pass-by noise values of all vehicles.
Earlier, Form 22 only certified that the vehicle in question complied with the provisions of the
Motor Vehicles Act, 1989 and rules thereunder, including the relevant emission norms.

Indian Navy commissions INS Tihayu


October 19, 2016

Indian Navy has commissioned INS Tihayu, a Car Nicobar Class water jet fast attack craft (WJFAC).
It was commissioned by the Flag-Officer-Commanding-in-Chief of the Eastern Naval Command
Vice-Admiral H.C.S. Bisht in Visakhapatnam, Andhra Pradesh.

About INS Tahiyu


INS Tahiyu is the sixth WJFAC Car Nicobar Class vessel to be commissioned and allotted to
the Eastern Fleet of the Indian Navy.
It has been built by Kolkata-based shipyard Garden Reach Shipbuilders and Engineers
(GRSE). Overall, INS Tahiyu is the 20th vessel in this class to be built by the shipyard.
INS Tahiyu weighs 315-tonne and can achieve a top operating speed of 35 knots per hour. It
is an improved version of the earlier vessels.
It has been fitted with three water jet propulsion systems, powered by marine diesel engines
2016 GKToday | All Rights Reserved | www.gktoday.in

119

Current Aairs: October 2016 [Date-wise Compendium]


generating 2720 KW of power.
It also has been loaded with an indigenously-built 30 mm CRN gun, an assorted variant of
machine guns and shoulder-launched IGLA surface to air missile.
What are Car Nicobar Class WJFACs?
The Car Nicobar class water jet fast attack crafts are high-speed offshore patrol vessels built
by Garden Reach Shipbuilders and Engineers (GRSE) for the Indian Navy.
The class and its vessels are named after Car Nicobar island part of Andaman and Nicobar
They are the first water jet propelled vessels of the Indian Navy.
The vessels are designed by GRSE as a cost-effective platform for patrol, anti-piracy and
rescue operations in Indias Exclusive Economic Zone (EEZ).

SC asks whether mass religious appeals for electoral gains can be categorised corrupt
practice
October 19, 2016

The seven-judge Constitution Bench of Supreme Court has questioned whether the practice of using
the mass religious appeal by leaders to canvas votes for candidates amounts to a corrupt electoral
practice.
The SC bench led by Chief Justice Chief Justice is re-considering its 1995 verdict which had held that
canvassing votes in name of Hindutva/Hinduism wasnt a corrupt electoral practice, as Hinduism
was not a religion but a way of life in India.
Background
Recently, the Constitution Bench of SC raised question of using the mass appeal of religious
while testing the limits of Section 123 of Representation of the People Act.
It was looking into the various means by which misuse of religion or faith of the masses for
electoral gains can be categorised as a corrupt practice.
It was also looking into electoral practices of political parties and candidates to rope in clerics
or priests to flex their religious sway over particular religious community to swing votes.
What 1995 verdict says?
The Representation of the People Act bars candidates and political parties to appeal appeal in
the name of religion. If found guilty for violation, the candidate can be disqualified.
The 1995 judgment delivered by Justice JS Verma had seeking votes in the name of Hinduism
is not a corrupt practice under Section 123 of Representation of the People Act.

Milk from Tasmanian devils could fight superbugs: Study


October 19, 2016

Scientists from University of Sydney (Australia) have found that mothers milk from marsupials (also
known as Tasmanian devils) could help to deadly superbugs which resist antibiotics.
2016 GKToday | All Rights Reserved | www.gktoday.in

120

Current Aairs: October 2016 [Date-wise Compendium]


They have found that peptides in the marsupials milk killed resistant bacteria, including methicillinresistant golden staph bacteria and enterococcus that is resistant to powerful antibiotic vancomycin.
Scientist are hopeful marsupial peptides could eventually be used to develop new antibiotics for
humans to aid the battle against superbugs.
What are Superbugs?
Superbugs are deadly bacteria which cannot be treated by current antibiotics and other drugs.
According to recent British study, Superbugs could kill up to 10 million people globally by 2050.
What Scientists have found?
Marsupials have more peptides than other mammals. It has six peptides whereas humans
have only one of this type of peptide.
These peptides give underdeveloped young marsupials an immature immune system when
they are born.
This natural immune system helps young marsupials to survive growth in their mothers
bacteria-filled pouch.
It should be noted that because of their biology, marsupials carry their young in a pouch after
birth to complete their development.
Using marsupials peptides, scientists had artificially created the antimicrobial peptides called
cathelicidins after extracting the sequence from the devils genome.
It was found that these artificially created the antimicrobial peptides killed the resistant
bacteria and other bacteria.

About Tasmanian devil


The Tasmanian devil is a carnivorous marsupial belonging to the family Dasyuridae. They are
endemic in the wild only in Tasmania, island state of Australia.
It is largest carnivorous marsupial in the world. It has stocky and muscular build, black fur,
pungent odour, keen sense of smell and extremely loud and disturbing screech.
Its large head and neck allow it to generate strongest bites per unit body mass among any
extant mammal land predator.

October 20, 2016


World Bank pledges $2 billion aid to Bangladesh to tackle climate change
October 20, 2016

The World Bank has pledged 2 billion US dollars aid package to Bangladesh to tackle climate change
over the next three years. The aid package is part of the World Banks fund for the poorest countries.
Bangladesh is extremely vulnerable to climate change and accounts for almost 70% of all storm surges
in the world. The aid package aims to further develop the resilience of the people at a time when
climate change is putting their lives and livelihoods at greater risk.
2016 GKToday | All Rights Reserved | www.gktoday.in

121

Current Aairs: October 2016 [Date-wise Compendium]


Background
This is the second large investment announcement made by the World Bank President Jim Yong
Kim during his 2-day visit to Bangladesh. Earlier he had announced to provide US $1 billion aid to
Bangladesh over the next three years to fight child malnutrition.
Comment
The World Bank has provided Bangladesh more than $24 billion in financing in the last 45 years.
Bangladesh is the largest recipient of International Development Association (IDA) assistance,
having received over 24 billion US dollars since Independence.

About World Bank


The World Bank is an international financial institution that provides loans to developing
countries for capital programs.
It comprises two institutions: the International Bank for Reconstruction and Development
(IBRD), and the International Development Association (IDA).
Formation: July 1944.
Headquarters: Washington DC, United States.
Motto: Working for a World Free of Poverty.
Membership: 189 countries (IBRD), 173 countries (IDA)
The World Bank is one of the five components of the World Bank Group, which is part of
the United Nations system.

Sir David Cox honoured with first International Prize in Statistics


October 20, 2016

Noted British statistician Sir David Cox (92) was awarded inaugural recipient of the International
Prize in Statistics.
The International Prize in Statistics Foundation has bestowed this award on Sir David Cox in
recognition of Survival Analysis Model (or Cox Model) Applied in Medicine, Science, and
Engineering.

About Sir David Cox


Sir David Cox is renowned statistician born on 15 July 1924 in Birmingham, England.
He is known for developing the proportional hazards model i.e. Cox Model in 1992. His
model is widely used in analysis of survival data which enables researchers to more easily
identify risks of specific factors for mortality or other survival outcomes among groups of
patients with disparate characteristics.
He is a foreign associate of the US National Academy of Sciences and fellow of the Royal
Society, an honorary fellow of the British Academy.
He has served as President of Bernoulli Society, Royal Statistical Society and International
2016 GKToday | All Rights Reserved | www.gktoday.in

122

Current Aairs: October 2016 [Date-wise Compendium]


Statistical Institute.
Awards and Honours: Copley Medal (2010), he was knighted in 1985.

About International Prize in Statistics


The prestigious award given by the International Prize in Statistics Foundation. It recognizes
a major achievement of an individual or team in the field of statistics.
The Prize aims at promoting understanding of the growing importance and diverse ways
statistics, probability and data analysis. It carries monetary award of US 75000 dollars.

India, Algeria mull fertilizer Joint Venture


October 20, 2016

India has shown keen desire to initiate a joint venture (JV) arrangement with Algeria for a multibillion dollar fertilizer plant located in the North African country.
The proposal in this regard has moved to front burner following a meeting between Algerian
Minister for Industry and Mining and Indian Minister of State for Chemicals in Algiers.
However no specific agreements were signed between both courtiers at the end of Vice-President
Hamid Ansaris three-day visit to Algeria.
Why India is forming JV with Algeria?
Phosphorus is one of three important elements in the chemical fertilizers that are most important in
plant nutrition along with nitrogen and potassium. India, which has up to 96% phosphate
dependency. Currently, India imports raw phosphate resources from a number of African countries
including Tunisia, Egypt, Jordan and Syria and also from Russia, Canada and Israel among others.
For India, forming JV Algeria will have major favourable food security implications and also could
ease the fertilizer subsidy burden.
India-Algeria
Indias bilateral trade with Algeria currently stands at 1.5 billion dollars a year with the trade balance
in Algerias favour given Indias import of oil and gas. The major item of exports from India to
Algeria has been automobiles.
India-Hungary
Earlier, Vice-President Hamid Ansari had visited Hungary on as part of this two-nation trip
(Hungary and Algeria). Both countries had signed two agreements including in the field of water
management.

EC hosts first ever International Conference on Voter Education


October 20, 2016

The Election Commission of India hosted the first ever International Conference on Voter
Education in New Delhi.
The three day (from 19 to 21 October, 2016) conference was inaugurated by the Chief Election
2016 GKToday | All Rights Reserved | www.gktoday.in

123

Current Aairs: October 2016 [Date-wise Compendium]


Commissioner (CEC) Dr Nasim Zaidi.
Key Facts
The topic of the first of its kind conference is Voter Education for Inclusive, Informed and
Ethical Participation. 27 countries and 5 international organizations are participating in it.
It has been organized with an aim to share best practices, initiatives and policies of voter
education by election management bodies.
It aims at achieving a resolution to strengthen inclusive, ethical and informed electoral
participation among member countries with the help of voter education and outreach.
It will deliberate upon five topics ranging from Role of Information Communication
Technology in Voter Education, Electoral Literacy to Enhanced participation by special
categories of voters and Strategies for Informed and Ethical Voting.
The Conference will see the launch of the ambitious VoICE.NET project, a Global
Knowledge Network on voter education.
Besides, an exhibition showcasing Voter Education tools and materials will be also organized
on the occasion.

India, China conduct second Joint tactical Exercise in Ladakh


October 20, 2016

India and China have conducted second joint military exercise Sino India Cooperation 2016 in
Ladakh region along the Line of Actual Control (LAC) in Jammu and Kashmir.
The exercise compliments Hand in Hand series of India-China joint military exercises that aims to
enhance cooperation and maintain peace and tranquility along border areas of both countries.
Key Facts
The theme of the day long joint exercise was based on Humanitarian Aid and Disaster Relief
(HADR). In it a fictitious situation of earthquake striking an Indian Border village was
painted.
Thereafter joint teams of both countries carried out rescue operations, evacuation and
rendering of medical assistance.
The exercise not only refined the drills to provide succor to the border population in case of
natural calamity but has also increased the level of trust and cooperation between the border
guarding forces of both countries along the LAC in Eastern Ladakh.
Background
The second edition of the exercise was conducted as part of ongoing initiative between both
countries to enhance interaction and cooperation under provisions of Border Defence
Cooperation Agreement (BDCA), 2013.
The first Joint Exercise under BDCA, 2013 was held in February 2016 in the area of Border
2016 GKToday | All Rights Reserved | www.gktoday.in

124

Current Aairs: October 2016 [Date-wise Compendium]


Personnel Meeting Hut at Chushul Garrison of Eastern Ladakh, along with Chinese troops of
Moldo Garrison.
Comment
This is for first time both countries have conducted military exercises in state of Jammu and
Kashmir in Eastern Ladakh amidst growing uneasiness in the bilateral relationship.
The exercise also comes in the backdrop of stalemate over Indias multilateral disagreements
with China over blacklisting of Pakistan-based terrorist group Jaish-e-Mohammed (JeM)
chief Masood Azhar at the United Nations and membership of the Nuclear Suppliers Group
(NSG).

India, Myanmar signed 3 MoUs including Cooperation in field of Power Sector


October 20, 2016

India and Myanmar have signed three Memorandum of Understanding (MoUs) to boost bilateral
relations between two neighbouring nations.
These MoUs were signed in New Delhi after delegation level talks between Prime Minister Narendra
Modi with the visiting State Counsellor of Myanmar Aung San Suu Kyi.
Signed MoUs are
MoU for designing an academic and professional building programme for insurance industry
of Myanmar.
MoU on Cooperation in the field of Power Sector. Under it India has offered to enhance
power supply from Moreh in Manipur to Tamu in Myanmar.
MoU on Banking Supervision. It was signed between Reserve Bank of India (RBI) and
Central Bank of Myanmar.
Besides, India extended support to Myanmar for a better connected future. Both countries agreed to
cooperate in security and strategic issues. They also agreed to enhance engagement in several areas
including to develop a mutually beneficial arrangement for trade in pulses.

India signs pact with Russia to lease second nuclear submarine


October 20, 2016

India has signed agreement with Russia to lease second Akula-II class nuclear attack submarine
(SSN). The deal is believed to be worth about $2 billion.
The lease to transfer a multipurpose Project 971 nuclear submarine to India from the Russian
Navy was signed in Goa on the sidelines of 8th BRICS Summit. The second nuclear submarine is
expected to arrive in Indian waters in 2020-21.
Background
India had earlier leased first Akula-II class nuclear attack submarine (SSN) for a period of 10
years from Russia.
2016 GKToday | All Rights Reserved | www.gktoday.in

125

Current Aairs: October 2016 [Date-wise Compendium]


It was inducted into service as INS Chakra (formerly known as K-152 Nerpa) in April 2012.
Its lease expires in 2021.
Since then, India had expressed interest in leasing at least one more submarine to train Navy
crew in complex submarine operations to have a large fleet of nuclear submarines.
Akula-class submarines
Akula-class submarines of Project 97 (NATO name Akula-II) weigh around 8,140 tonnes. They are
considered one of the most silent SSNs in service and are capable of sailing at speeds up to 35 knots.
They are nuclear powered but are armed with conventional land attack missiles.
INS Arihant
Earlier in August 2016, India had quietly inducted its first indigenously built nuclear ballistic
missile submarine (SSBN) Arihant into service completing its nuclear triad.
Besides, follow-on submarines are also planned and at least two are in various stages of
construction which will give India strategic foothold in the Indian Ocean Region (IOR).
The second indigenous nuclear submarine of the same class, INS Aridhaman, will be
commissioned in 2018.
Comment
Pakistan does not have a nuclear submarine and lacks a nuclear triad. China has four nuclearpowered submarines in operation and in a couple of years this number that will go up to six. Chinas
Nuclear-submarines carry a version of the Dong Feng nuclear missile that has a range of up to 7,000
km.

IUNC to declare Kashmirs Red Stag as Critically Endangered


October 20, 2016

The International Union for Conservation of Nature (IUCN) is going to declare the Kashmiri Red
Stag (also known as Hangul) as a Critically Endangered species.
The critically endangered status to the Kashmiri Red Stag will help it to get more protection and
enhance the conservation efforts to increase its rapidly declining population.

About Kashmiri Red Stag


The Kashmir Stag or Hangul is a subspecies of elk native to India.
Earlier it was believed that it is a subspecies of red deer. But mitochondrial DNA genetic
studies have revealed that it is part of the Asian clade of elk.
It is found in dense riverine forests in the high valleys and mountains of Kashmir Valley and
northern Chamba district of Himachal Pradesh.
As per Bombay Natural History Society (BNHS) nearly 3000 to 5000 Hanguls existed around
the 1940s.
But at present, only about 150 of them survive within its last bastion in Dachigam National
2016 GKToday | All Rights Reserved | www.gktoday.in

126

Current Aairs: October 2016 [Date-wise Compendium]


Park located on foothills of Zabarwan range on the outskirts of Srinagar, J&K.
Protection status: It has been listed under Schedule-I of the Wildlife (Protection) Act, 1972
and J&K Wildlife (Protection) Act, 1978. It also has been listed among the top 15 species of
high conservation priority by the Central Government.
Reasons for decline in population: (i) habitat destruction, (ii) over-grazing by domestic
livestock, and (iii) poaching.

About International Union for Conservation of Nature (IUCN)


The IUCN is the worlds oldest and largest global environmental organisation working in the
field of nature conservation and sustainable use of natural resources.
It was founded in 1948. It is headquartered in Geneva, Switzerland.
It is a leading non-governmental authority on the environment and sustainable development.
It is also involved in data gathering and analysis, research, field projects, advocacy, lobbying
and education.
IUCN is best known to the wider public for compiling and publishing the IUCN Red List,
which assesses the conservation status of species worldwide.

Okha-Kanalus, Porbandar-Wansjaliya sections of Gujarat become Green Train Corridors


October 20, 2016

The Union Ministry of Railways has declared the Okha-Kanalus and the Porbandar-Wansjaliya
railway sections of Gujarat as Green Train Corridors.
All trains passing on these 175-km long lines (141-km-long Okha-Kanalus route and the 34-km-long
Porbandar-Wansjaliya route) are now equipped with bio-toilets.
On these routes, about 29 trains consisting of nearly 700 coaches have been equipped with bio-toilets
to prevent open discharge free zone on tracks as part of the Swachh Bharat Mission.
What are Green Train Corridors?
Green Train Corridors are sections of the railways which will be free of human waste
discharge on the tracks. Trains running on these corridors will be equipped with bio-toilets.
Thus, it will completely stop discharge of human waste from trains onto the ground which in
turn would help in improving cleanliness and hygiene.
The 114-km long Rameswaram-Manamadurai section of Tamil Nadu was made the
Indias first Green Rail Corridor in July 2016
Background
The Union Ministry of Railways has taken up a mammoth task of equipping human discharge free
bio-toilets in all its coaches by 2021-22 in order to contribute to mission Swachh Bharat Abhiyan. It
aims at completely stopping discharge of human waste from trains onto the ground in order
improving cleanliness and hygiene
2016 GKToday | All Rights Reserved | www.gktoday.in

127

Current Aairs: October 2016 [Date-wise Compendium]


What are Biotoilets?
The environment-friendly bio-toilets for passenger coaches were developed jointly by Indian
Railways and Defence Research and Development Organisation (DRDO).
In the bio-toilet fitted coaches, human waste is collected in biodigester tanks below the toilets
and is decomposed by a consortium of anaerobic bacteria.
By the process of hydrolysis, acetogenesis, acidogenesis and methanogenesis, the anaerobic
bacteria converts human faecal matter into water and small amount of gases (including
methane).

No relook into allocation of Krishna water: Tribunal


October 20, 2016

The Krishna Water Disputes Tribunal II (KWDT-II) headed by Justice Brijesh Kumar, has decided
to maintain status quo on the allocation of Krishna River water among all four riparian states viz.
Maharashtra, Karnataka, Telangana and Andhra Pradesh. However, tribunal decided to confine the
reallocation of Krishna water to Andhra Pradesh and Telangana.
What is the case?
The KWDT-II pronounced its judgement on the litigation prompted by Andhra Pradesh and
Telangana. Both states in their petition had sought fresh allocation of Krishna River water among all
four riparian states. They had urged that Section 89 in the Andhra Pradesh Reorganisation Act, 2014
meant redistribution of Krishna water among all the four riparian States not just between both of
them.
What was KWDT-II verdict?
In its verdict, the tribunal said that Section 89 the Andhra Pradesh Reorganisation Act, 2014 was not
applicable to all four riparian states but is meant only for Telangana and Andhra Pradesh. Thus,
there was no need to re-allocate of Krishna River water among all four riparian states. The river
water should be re-allocated between Telangana and Andhra Pradesh, from the share of undivided
Andhra Pradesh.
What is Krishna Water Disputes Tribunal (KWDT)?
Krishna Water Disputes Tribunal (KWDT) is a tribunal set up by the Central Government in 1969
under the Interstate River Water Disputes Act, 1956 to resolve the disputes between the states of
Karnataka, Maharashtra and then undivided Andhra Pradesh over sharing of Krishna river water.
About Krishna River
The Krishna River is the second biggest river in peninsular India after Goadavri River. It originates
near Mahabaleshwar (Satara) in Maharashtra. It then runs from four states Maharashtra (303 km),
North Karnataka (480 km) and the rest of its 1300 km journey in Telangana and Andhra Pradesh
before it empties into the Bay of Bengal.
2016 GKToday | All Rights Reserved | www.gktoday.in

128

Current Aairs: October 2016 [Date-wise Compendium]


What are Water Disputes Tribunals?
As per the Article 262 of the Constitution, Parliament may by law provide for the
adjudication of any dispute or complaint with respect to the use, distribution or control of the
waters of, any interstate rivers or river valley.
In pursuance of this article, the parliament had enacted Inter-State Water Disputes Act, 1956
to govern the inter-State river water disputes. Under this law, water disputes tribunals are
constituted for the adjudication of the interstate water dispute.
Article 262 vs Article 131 of the Constitution
The Article 131 of the Constitution which deals with the Supreme Courts exclusive
jurisdiction to adjudicate disputes between States is not an unrestricted one.
The apex court can exercise its jurisdiction under Article 131 in an inter-State dispute,
provided other Articles in the Constitution allowed it to do so.
Article 262 (2) of the Constitution allows Parliament, by law, to bar the Supreme Court from
interfering in an inter-State water dispute.
It means that award of water tribunal appointed under a law to resolve inter-State water
dispute would be final and deemed to have the force of a Supreme Court order or decree.
Thus, it can be said that the award of the tribunal formed under the Inter-State Water
Disputes Act, 1956 eclipses the Supreme Courts jurisdiction.

October 21, 2016


SBI declares Silchar as its 5th zone in North East
October 21, 2016

The State Bank of India (SBI) has declared Silchar as its 5th zone in North East to facilitate banking
issues for the customers of the region.
The 5th zone will further help to cater the banking needs of the people of Mizoram, Tripura and
Manipur along with the Barak Valley of North east India. SBI also opened its administrative office at
Silchar, Assam.
Other four zones of SBI in North East are Guwahati, Shillong, Dibrugarh and Jorhat. Earlier, Silchar
was under Guwahati zone. However, due to difficult terrain of the region it was tough on the part of
the bank to address the complaints and issues of the customers in the region.
Presently, SBI has 736 branches across North East together with 3,000 ATMs and 3,000 customer
service points for customer facilitation.

Maharashtra Government launches Kaushalya Setu skill-development programme


October 21, 2016

Maharashtra government has launched Kaushalya Setu, a skill development programme for
students.
2016 GKToday | All Rights Reserved | www.gktoday.in

129

Current Aairs: October 2016 [Date-wise Compendium]


It was launched by Chief Minister Devendra Fadnavis and Union Minister of State Skill
Development and Entrepreneurship (Independent Charge) Rajiv Pratap Rudy.
Under the Kaushalya Setu initiative
Students of Maharashtra State Board of Secondary & Higher Secondary Education who failed
to clear Class X or SSC exam, will be accommodated in skill development courses.
The main aim of the initiative to compensate loss of one year of students who have failed
matriculation examination by providing skill-oriented training.
It will give a big boost to Central Governments Skill India Mission-2020 by providing skill
development courses to students.

Railway ministry inks MoU for Smart Railway Stations


October 21, 2016

The Union Ministry of Railways and Union Urban Development Ministry have inked Memorandum
of Understanding (MoU) for smart railway stations.
The validity of the MoU is for the five years and can be extended with the consent of both the
ministries.
Under this initiative
Railway stations and surrounding areas will be redeveloped under the Smart City plan.
500 railway stations in the country will be smarten up with better passenger amenities,
integrated public transport hubs and easy access.
Initially, 100 railway stations and an adjoining area of 300-800 acres will be redeveloped in
Smart Cities and Atal Mission for Rejuvenation and Urban Transformation (AMRUT)
Schemes. Later the scope of the MoU will be extended to over 500 cities in time.
Countries like Germany, Japan, France, South Korea, UK and Belgium have shown interest
in the redevelopment of railway stations.
To begin with, Railway stations in 10 cities will be taken up for the redevelopment with the
involvement of the National Buildings Construction Corporation (NBCC).
They are Varanasi, Bhubaneswwar, Sarai Rohilla (Delhi), Varnasi, Lucknow, Jaipur, Kota,
Thane, Margao (Goa), Tirupati and Puducherry.

Triple talaq a highly misused custom: NCW


October 21, 2016

The National Commission of Women (NCW) has held that triple talaq was a highly misused
custom and Government should scrap it to protect the rights of Muslim women.
NCW clearly mentioned that the traditional custom of triple talaq should be banned in an effort to
protect the rights of Muslim women and it cannot be linked to the Uniform Civil Code. According
to the commission, Muslim women feels disempowered because of the practice of triple talaq.
2016 GKToday | All Rights Reserved | www.gktoday.in

130

Current Aairs: October 2016 [Date-wise Compendium]


What is triple talaq?
Triple Talaq or Oral talaq is a procedure of divorce mentioned under the Sharia Law which is a
body of the Islamic law. Under this, a husband can divorce his wife by pronouncing Talaq thrice.
Why triple talaq should be abolished?
The practice of triple talaq has enabled husbands to divorce their wives arbitrarily, devoid of any
substantiation. It impact adversely on the right of women to a life of dignity. The triple talaq has
been abolished in 21 Islamic theocratic countries including Pakistan. It is also against constitutional
principles such as gender equality, secularism, international laws etc.
What is Governments position?
Recently Central Government in affidavit submitted to Supreme Court held that the practice of
Triple talaq is against the principles of gender justice, gender equality and dignity enshrined in the
Constitution. This pernicious practice is not integral to the right to freedom of religion. Triple talaq
is not an essential religious practice in Islam and violates right to equality, non-discrimination on the
grounds of sex and the right to live with dignity to women in par with men guaranteed in the
Constitution.

About National Commission for Women (NCW)


The NCW is a statutory body generally concerned with advising the government on all policy
matters affecting women.
It was established in January 1992 under the National Commission for Women Act, 1990 as
per the provisions of the Indian Constitution.
The objective of the NCW is to represent the rights of women in the country and to provide
a voice for their issues and concerns.

Pakistan warns India against breaching Indus Water Treaty


October 21, 2016

Pakistan has warned India that it will take appropriate action if India violates the Indus Water Treaty
(IWT) unilaterally.
It also mentioned that it was closely monitoring the situation after India decided to suspend all talks
with Pakistan on IWT in the wake of the Uri attack.
Why Indus River is important for Pakistan?
Around 65% of the Pakistans geographical area including entire Pakistans Punjab province is part of
Indus basin. The water of Indus River is lifeline of Pakistan and is source of irrigation, hydro power
and drinking water for millions of Pakistanis. If the water from rivers is stopped by India it can
surely cut lifeline of Pakistan.
What is Indus Waters Treaty (IWT)?
IWT is a water-distribution treaty between India and Pakistan which was brokered by the
2016 GKToday | All Rights Reserved | www.gktoday.in

131

Current Aairs: October 2016 [Date-wise Compendium]


World Bank (then the International Bank for Reconstruction and Development).
The treaty deals with sharing of water of Indus water system having six rivers Beas, Ravi,
Sutlej, Indus, Chenab and Jhelum between the two countries.
It was signed by then Prime Minister Jawaharlal Nehru and President of Pakistan Ayub Khan
in Karachi on September 19, 1960.
As per treaty, control over three eastern rivers Ravi, Beas and Sutlej was given to India.
While control over three western rivers Indus, Jhelum and Chenab was given to Pakistan.
It allows India to use only 20% of the water of Indus river, which flows through it first, for
irrigation, power generation and transport.
Most disagreements and disputes have been settled via legal procedures, provided for within
the framework of the treaty.
Under it Permanent Indus Commission (PIC) was set up as a bilateral commission to
implement and manage the Treaty. The Commission solves disputes arising over water
sharing. Besides, treaty also provides arbitration mechanism to solve disputes amicably.
A Permanent Indus Commission was set up as a bilateral commission to implement and
manage the Treaty. The Commission solves disputes arising over water sharing
It is most successful water treaty in world. Even, it has survived India-Pakistan wars of 1965,
1971 and the 1999 Kargil standoff besides Kashmir insurgency since 1990.

For more information: Recent clamour about abrogation of Indus Water Treaty
2016 GKToday | All Rights Reserved | www.gktoday.in

132

Current Aairs: October 2016 [Date-wise Compendium]

Protein-enriched rice variety developed by IGKV Chattisgarh


October 21, 2016

Researchers from Indira Gandhi Krishi Vishwavidyalaya (IGKV),Chattisgarh have developed a


protein-enriched rice variety.
The new developed rice variety has over 10% protein content, which is 3% more than what is found
in any popular variety and has 30 PPM (parts per million) zinc content.
Key facts
The protein-enriched rice variety was developed by researchers from IGKVs Department of
Plant Molecular Biology and Biotechnology.
They had worked for seven years to develop the variety of rice that is rich in protein along
with high zinc content.
This protein rice variety will play a crucial role in addressing issues of protein deficiencies
and it will serve as a boon for malnourished population, especially children, in tribaldominated Chhattisgarh.
Zinc-enriched variety of rice
Earlier in May 2015, scientists from Indira Gandhi Agriculture University (IGAU), Raipur
(Chhattisgarh) had developed a high zinc-enriched called Chhattisgarh Zinc Rice-1. It was the
first zinc biofortified rice variety developed in India.
Advantages
Zinc and other mineral content are not lost upon polishing. The rice has long self-life and used after
keeping it for long time. Even it tastes just as good as the conventional variety.
Why is zinc so important to the body?
Zinc is important to the body because over 300 enzymes in our body use it as an essential
component in their action.
Zinc is essential in supporting our immune system, in synthesising and degrading DNA, in
wound healing and several other activities.
Our body requires very small amount of Zinc, but if the level falls down to below normal it
may result into growth retardation, diarrhoea, eye and skin lesions and loss of appetite.
Background
As per a survey conducted in 2015 very high rate of malnutrition was found among preschool
children in tribal areas across Chhattisgarh which is considered rice bowl of the country.
The survey had showed that over five lakh children in the state are underweight and stunted
growth especially in the tribal districts.

IIL develops CYSVAX, worlds first vaccine to fight against tapeworms in pig
October 21, 2016

2016 GKToday | All Rights Reserved | www.gktoday.in

133

Current Aairs: October 2016 [Date-wise Compendium]


The Indian Immunologicals Limited (IIL) has launched CYSVAX, worlds first vaccine to fight
against Taenia solium Tapeworms in pigs.
The vaccine has significant potential benefit to reduce the incidence of epilepsy in humans.
Key Facts
The CYSVAX is a recombinant porcine (Pigs) Cysticercosis Vaccine.
The vaccine has potential to break the Cysticercosis disease parasites life-cycle in pigs which
is considered to be one of the important reasons for epilepsy in humans.
What is Cysticercosis?
Cysticercosis is a parasitic disease caused by pork tapeworm named Taenia solium. WHO
has designated it as one of 17 Neglected tropical diseases worldwide.
It is a zoonotic parasite disease involving pigs as intermediate hosts because it is caused by
ingesting the eggs of the tapeworm Taenia solium.
Humans are usually infected by the accidental consumption of eggs present in the undercooked pork, vegetables and greens that have been improperly washed.
In humans it develops into cysts in central nervous system and cause Neuro-cysticercosis,
considered as one of the important reasons for epilepsy in humans.
What are reasons for the spread of disease?
The primary breeding grounds for tapeworms Taenia solium are open defecation and unhygienic pig
rearing. Cases of Cysticercosis in India have been reported in North Uttar Pradesh and NorthEastern states.
About Indian Immunologicals Limited (IIL)
IIL is Hyderabad-based subsidiary of National Dairy Development Board (NDDB). It is a market
leader in veterinary biologicals and is also big player in animal and human healthcare sector in
country.

Union Government inks loan pact with World Bank for Eastern Freight Corridor
October 21, 2016

The Union Government has signed $650 million loan agreement with the World Bank for
construction of the Eastern Dedicated Freight Corridor (EDFC)-III project.
The agreement was signed between International Bank for Reconstruction and Development
(IRBD), part of World Bank Group to lend $650 million, third tranche for construction EDFC to
Dedicated Freight Corridor Corporation of India (DFCCIL).
Features of EDFC-III project
The project aims to enhance rail transport capacity, improve service quality and boost freight
carriage on the 401-km- long Ludhiana-Khurja section of the EDFC.
On completion it will directly benefit the power and heavy manufacturing industries located
2016 GKToday | All Rights Reserved | www.gktoday.in

134

Current Aairs: October 2016 [Date-wise Compendium]


in the Northern and Eastern parts of India.
This project will provide these industries smooth railway network for efficient transportation
of their raw materials along with distribution processed and consumer goods.
In addition, railway passengers will also be benefitted as the existing passengers lines would
get decongested.
Background
The first two phases of the EDFC are already being implemented by the DFCCIL with the help of
financial assistance provided by the World Bank. The IRDB had provided loans worth $975 million
and $1,100 million for the Dadri-Khurja-Kanpur and and Kanpur-Mughal Sarai stretches
respectively of the Eastern Rail Corridor.
What is Eastern Dedicated Freight Corridor (EDFC)?
EDFC is 1,840 km long freight corridor of Indian Railways extending from Ludhiana in
Punjab to Kolkata in West Bengal.
The objective of the EDFC is to augment railway freight carrying capacity along the railway
corridor between Ludhiana and Kolkata.
The project will benefit industries of northern and eastern India, which rely on railway
network for transportation of material inputs and processed goods.
It is also going to help to reduce greenhouse gases (GHGs) emissions, as trains complying on
this dedicated freight corridor will operate entirely on electricity.

SC freezes BCCIs transactions with state cricket bodies


October 21, 2016

Supreme Court has frozen all financial transactions between the BCCI (Board of Control for Cricket
in India) and state cricket associations.
The apex court directed BCCI, the apex cricket body not to disburse any funds to state cricket
associations till they resolve to abide by Justice RM Lodha Committee recommendations.
Besides, the Supreme Court also asked the Lodha Committee to appoint an independent auditor to
verify the BCCIs accounts. It was also asked to set a threshold value for various contracts the BCCI
enters into, and all contracts in excess of that amount would need the committees approval.
What is the issue?
The Lodha committee appointed by the Supreme Court had recommended a complete
overhaul the cricket, the most popular sport in India.
The committee had recommended sweeping reforms in BCCIs administrative and
governance structures from top to grassroot levels and on the issue of transparency.
The most important set of recommendations were aimed at transforming the entire power
structure in the BCCI.
2016 GKToday | All Rights Reserved | www.gktoday.in

135

Current Aairs: October 2016 [Date-wise Compendium]


However, BCCIs position was that it could not implement the Committees
recommendations as majority of its state associations are not agreeing to do so.
These state associations were reluctant to accept Committees recommendations such as onestate-one-vote policy, the age cap of 70 years for administrators and the limit of three, threeyear terms with cooling-off periods in between for office bearers.

October 22, 2016


BCCI approves use of DRS in India-England Test series
October 22, 2016

The Board of Control for Cricket in India (BCCI) has given its consent for using the Decision
Review System (DRS) in the forthcoming five-Test series against England in November 2016.
BCCI will use the DRS on a trial basis to evaluate the improvements and changes made to the system
over a period of time.
Earlier, BCCI as well as many members of the current Indian cricket team including Captain
Mahendra Singh Dhoni had objected used of DRS as is not 100% accurate.
What changes have been made?
The technology approved by Massachusetts Institute of Technology (MIT). It includes introduction
of ultramotion cameras with better frame rates for accurate ball tracking and Ultra edge, a sound
based technology for determining the frame of impact and minimising manual intervention. A data
retrieval system also has been included as a further safeguard.
What is Decision Review System (DRS)?
DRS is technology-based system used in sport of cricket for the sole purpose of reviewing
controversial decisions made by on-field umpires as to whether or not a batsman had been
dismissed. The system was first tested in an India v Sri Lanka match in 2008.
It was officially launched by the International Cricket Council (ICC) in 2009 during the first
Test match between New Zealand and Pakistan.
Initially, ICC had made the DRS mandatory in all international matches, but later made its
use optional, so that the system would only be used if both teams agree.
How does the Decision Review System (DRS) work?
The DRS comprises a combination of several off-field technologies. Using these off-field
technologies, the third umpire informs the on-field umpire whether to overturn or uphold
their original decision.
The process of checking a DRS it starts with (i) Checking of No-ball (ii) Inside Edge (iii) Pitch
of Ball (iv) Location of Ball while hitting the pads (v) Ball hitting the stumps
First two things are checked manually by 3rd Umpire.
The rule to give out you require this three things (i) Ball should pitch In-Line of stumps or
2016 GKToday | All Rights Reserved | www.gktoday.in

136

Current Aairs: October 2016 [Date-wise Compendium]


Outside off (ii) Impact of ball while hitting the pads should be In-Line of stumps (iii) Ball
should be hitting stumps.
What are off-field technologies of Decision Review System?
Off-field technologies of DRS includes (i) footage from TV cameras placed around the ground; (ii)
Hot Spot, an infrared camera system that highlights parts of the bat and body that have been hit by
the ball; (iii) Hawkeye, a ball-tracking system usually used to check leg before wicket decisions; and
(iv) Realtime Snickometre, a waveform of the sound captured by a stump microphone.

Smooth-coated otter sighted for first time in Krishna sanctuary


October 22, 2016

Smooth-coated otter was sighted for the first time in the mangrove forest near the Krishna wildlife
sanctuary (KWS) in Krishna district, Andhra Pradesh.
In all, seven otters were sighted in the mangrove forests and brackish water channels of
Eelachetladibba and Lankevennedibba and other areas outside the KWS. The presence of the otter is
a key indicator for rise of the mangrove cover in the KWS.

About smooth-coated Otter


The smooth-coated Otter (Lutrogale perspicillata) is a species of otter, the only extant
representative of the genus Lutrogale. Otter is a carnivorous mammals.
The fur of this species is smoother and shorter than that of other otters. It is distributed
throughout South Asia and South East Asia.
The otter lives in rivers, lakes, peat swamp forests, mangroves and estuaries. It uses swamps
as natal den sites and nursery during the breeding season in early winter.
Otters are social and hunt in groups. The male otter is polygamous and mates with up to the
four females.
It has been categorised vulnerable in the red data book of International Union for
Conservation of Nature (IUCN).

ICGS Aryaman and Atulya Commissioned into Indian Coast Guard


October 22, 2016

The Indian Coast Guard Ships (ICGSs) Aryaman and Atulya were commissioned into Indian Coast
Guard in Kochi, Kerala.
These ships are the eighteenth and nineteenth in the series of twenty Fast Patrol Vessels (FPVs)
designed and built by Cochin Shipyard Limited.
Key Facts
The ships have been christened ICGS Aryaman and Atulya. Aryaman literally meaning is
Possessor of Greatness and Atulya meaning is Unrivalled.
Both FPVs are 50m long and have displace of 317 tonnes of water. They can achieve a
2016 GKToday | All Rights Reserved | www.gktoday.in

137

Current Aairs: October 2016 [Date-wise Compendium]


maximum speed of 33 knots with range of 1500 nautical miles.
They are equipped with state-of-the-art weaponry, advanced communication and
navigational equipment.
These FPVs are ideal platform for undertaking multifarious close-coast missions such as
surveillance, interdiction and search and rescue.
These ships have onboard Integrated Bridge Management System (IBMS) and Integrated
Machinery Control System (IMCS).
Aryaman will be based at Kochi under the administrative control of the Commander, Coast
Guard Region (West).
Atulya will be based at Visakhapatnam under the administrative control of the Commander,
Coast Guard Region (East).

About Indian Coast Guard (ICG)


ICG is Indias statutory multi-mission paramilitary organization that protects countrys
maritime interests and enforces maritime law.
It was established on 18 August, 1978 by the Coast Guard Act, 1978 and operates under the
aegis of Union Ministry of Defence. Its headquarters are in New Delhi.
ICG has jurisdiction over the territorial waters of India including its contiguous zone and
exclusive economic zone (EEZ).

Union Government launches UDAN Scheme for Regional Air Connectivity


October 22, 2016

The Union Government has launched the regional air connectivity scheme UDAN (Ude Desh ka
Aam naagrik) which seeks to get more people to fly in the smaller towns.
The scheme was launched by the Union Civil Aviation Minister Ashok Gajapathi Raju in New Delhi.
It will be rolled out by January, 2017 and will be in operation for a period of 10 years.
What is UDAN (Ude Desh ka Aam naagrik) Scheme?
The UDAN scheme seeks to provide connectivity to un-served and under-served airports of
the country through revival of existing air-strips and airports.
This first-of-its-kind scheme will ensure affordability, connectivity, growth and
development. It aims to increase ticketing volume from 80 million to 300 million by 2022.
Under it regional connectivity will be developed on market-based mechanism under which
Airlines will bid for seat subsidies.
Airline operators will bid for up to 40 subsidised seats and minimum seats will be 9. There
will be 50% seats on market based pricing.
It will create affordable yet economically viable and profitable flights on regional routes so
that flying becomes affordable to the common man even in small towns.
2016 GKToday | All Rights Reserved | www.gktoday.in

138

Current Aairs: October 2016 [Date-wise Compendium]


Under it, airlines will have complete freedom to enter into code sharing with larger airlines
for connectivity and they will be exempted from various airport charges.
Airlines will have exclusive rights for three years to fly on a particular regional route. On
these routes for regional flights Airfares will be capped at 2500 rupees for an hours flight.
How the scheme will achieve targets ?
Central and State governments and airport operators will provide a financial stimulus in the
form of concessions to airlines.
The mechanism of Viability Gap Funding (VGF) will be provided to interested airlines to
kick-off operations from such airports so that the passenger fares are kept affordable.
Regional Connectivity Fund (RCF) will be created to meet the VGF requirements under the
scheme. The RCF levy per departure will be applied to certain domestic flights.
What are various facilities available for Airline and helicopter operators?
Airline and helicopter operators interested under the scheme can start operations on unconnected routes by submitting proposals to the Implementing Agency.
The can seek various concessions apart from the Viability Gap Funding (VGF).
The un-connected routes proposed under the scheme will be offered through competitive
bidding and will be awarded to participant quoting the lowest VGF per Seat.
The successful bidder will have exclusive rights to operate the route for a period of three
years.
After a three year grace period, such support will be withdrawn as the route is expected to
become self-sustainable.
Selected airline operator will have to provide a minimum of 9 and a maximum of 40 UDAN
Seats (subsidized rates) on the UDAN Flights for operations.
Airport operators will not impose any Landing and Parking charge and Terminal Navigation
Landing Charges.
What is role of States in the UDAN Scheme?
The selection of airports where UDAN operations begins will be done in consultation with
State Government and after confirmation of their concessions.
The scheme will also help in starting operations on un-served airports and revival of the
dysfunctional airports.
Way forward
The UDAN Scheme is likely to a give a major fillip helicopters and small aircraft operations
along with tourism and employment generation in the hinterland.
It will also help to significantly reduce travel timings in remote and hilly regions, as well as
islands and other areas of the country.
2016 GKToday | All Rights Reserved | www.gktoday.in

139

Current Aairs: October 2016 [Date-wise Compendium]

Global Conference to strengthen Arbitration & Enforcement held in New Delhi


October 22, 2016

The first ever global conference to strengthen Arbitration and Enforcement in India was held in
New Delhi.
The 3-day Conference was inaugurated by President Pranab Mukherjee on 21 October 2016.
Key Facts
The conference aims to provide impetus to commercial arbitration in country or faster and
efficient dispute resolution outside the court room.
It was started with title National Initiative On Strengthening Arbitration and Enforcement.
It was held to change the face of dispute resolution in India.
For the first time six leading international arbitral institutions and all major industry
associations had participated in it.
Discussions on Indian Arbitration Act, Case Management of Arbitration, Setting up an world
class autonomous arbitration institution and court support for arbitration were held.
Background
The conference comes the backdrop of the immense losses suffered by the business enterprises and
the economy at large, due to backlogs involved in dispute resolution in Indian courts. This has kept
money and resources of the businesses trapped till the final resolution of the dispute is resolved. It is
considered as major disincentive for foreign companies coming to invest in India and also affected
ease of doing business environment of the country.

NABARD sanctions Rs. 19,702 crore loan to National Water Development Agency
October 22, 2016

The National Bank for Agriculture and Rural Development (NABARD) has sanctioned loan worth
Rs. 19,702 crore to National Water Development Agency (NWDA).
The loan will be provided through NABARDs Long Term Irrigation Fund (LTIF) for 50 irrigation
projects from 11 States.
Key Facts
This loan will be part of the Central share of assistance to State Governments.
It will ensure front loading of resources so that the identified incomplete irrigation projects
under Pradhan Mandtri Krishi Sinchai Yojna (PMKSY) are executed in time.
Loan released by NABARD to NWDA would be disbursed to the respective State
Governments as Central Share in the projects sanctioned.
The LTIF was set up by NABARD as a sequel to the initiatives announced by the Union Finance
Minister Arun Jaitley for irrigation purpose in his Budget Speech 2016-17.
What will be significance of these irrigation projects?
2016 GKToday | All Rights Reserved | www.gktoday.in

140

Current Aairs: October 2016 [Date-wise Compendium]


The loan funds for these irrigation projects will help to create additional irrigation potential of 39.14
lakh hectares in 11 States. Thus, it will support governments vision to double the farmers income by
2022 as providing assured irrigation to farmers is one of the important prerequisites to achieve that
target.

About National Water Development Agency (NWDA)


NWDA is the agency of the Union Ministry of Water Resources. It was set up in July, 1982 as
autonomous society under the Societies Registration Act, 1860.
It was established to carry out the water balance and other studies on a scientific and realistic
basis for optimum utilisation of Water Resources of the Peninsular rivers system.
Besides it was tasked to prepare feasibility reports and give concrete shape to Peninsular
Rivers Development Component of National Perspective.
In 1990, it was also entrusted with the task of Himalayan Rivers Development Component of
National Perspectives.
Recently, the functions of NWDA have been further modified and the work of preparation of
detailed Project Reports (DPR) of various link proposals and Pre-feasibility Reports.
It has been also entrusted to prepare feasibility reports of intra-State links as proposed by the
States.

India wins 2016 Kabaddi World Cup


October 22, 2016

India has won the 2016 Kabaddi World Cup by defeating Iran by 38-29 score in the final match
played at the Arena By TransStadia in Ahmedabad (Gujarat.
It was Indias third consecutive Kabaddi World Cup in the Standard Style version of Kabaddi. Earlier
India had won in 2004 and 2007.
Captain of Indian team: Anup Kumar
Captain of Iranian team: Meraj Sheykh

About Kabaddi World Cup


Kabaddi World Cup is standard style indoor international kabaddi competition conducted by the
International Kabaddi Federation (IKF).
The competition has been previously contested in 2004, 2007 and 2016. All the three
tournaments have been won by India by defeating Iran in the finals.

Arena By TransStadia
It is Indias first-ever convertible stadium. It uses a globally patented technology, which converts an
outdoor stadium into an indoor arena within 6 minutes at a touch of a button. It can house 14 sports,
excluding cricket and has seating capacity of 20,000.
2016 GKToday | All Rights Reserved | www.gktoday.in

141

Current Aairs: October 2016 [Date-wise Compendium]

October 23-24, 2016


Western Zonal Council Meeting held in Mumbai
October 24, 2016

The 22nd meeting of the Western Zonal Council was recently held at Mumbai, Maharashtra under
the chairmanship of Union Home Minister Rajnath Singh.
The Western Zonal Council consist of the States of Maharashtra, Gujarat, Goa and Union
Territories of Daman & Diu and Dadra & Nagar Haveli
Key facts
The meeting was attended by Chief Ministers and ministers of these States and Union
Territories administrators as well as senior officers of Union and State Governments.
The Council reviewed issues related to internal security, coastal security, issuance of biometric identity cards to fishermen and card readers, modernization of police force,
formulation of plans for countering terrorism etc.
It also reviewed other important issues such as Pradhan Mantri Awas Yojna (Urban) Mission,
providing shelters to the urban homeless, and surplus lands of Union Government for
Housing for All: 2022.
Issues related to checking pollution in Daman Ganga River and Kolak River due to discharge
of untreated effluents from Vapi industrial area was also discused.
What are Zonal Councils?
Zonal Councils were set up under the States Reorganization Act, 1956 (so it is not
constitutional body) to foster Inter-State co-operation and co-ordination among the States.
Currently, there are total five zonal councils viz. Northern, Western, Eastern, Central and
Southern.
The Zonal Councils are mandated to discuss and make recommendations on any matter of
common interest in field of social and economic planning, linguistic minorities, border
disputes or inter-State transport etc.
Thus, Zonal Councils are regional fora of cooperative endeavor for States linked with each
other economically, culturally and politically.
They are specially meant for looking after the interests of respective Zones and focus
attention on specific issues taking into account regional factors, while keeping national
perspective in view.
Note
The north eastern states are not included in these five zonal Councils. Their special problems
are looked after by North Eastern Council, set up under the North Eastern Council Act, 1972.
It is also headed by Union Home Minister.
2016 GKToday | All Rights Reserved | www.gktoday.in

142

Current Aairs: October 2016 [Date-wise Compendium]


The North Eastern Council includes Assam, Arunachal Pradesh, Manipur, Tripura, Mizoram,
Meghalaya and Nagaland. Sikkim was included in the North Eastern Council in 2002 and
excluded from the eastern council.

ITBP celebrates its 55th raising day


October 24, 2016

The Indo Tibetan Border Police (ITBP) celebrated its 55th raising day on 24 October. The force was
raised on 24th October in 1962.
ITBP is deployed on border guarding duties from Karakoram Pass in Ladakh to Jachep La in
Arunachal Pradesh covering 3488 kilometer of India-China Border. It is manning Border Outposts
on altitudes raging from 9000 feet to 18700 feet in the Western, Middle and Eastern Sector of the
India-China Border.

About Indo-Tibetan Border Police (ITBP)


ITBP is one of the eight statutory Central Armed Police Forces of India. It was established on
24 October 1962, under the CRPF Act, in the wake of India- China War of 1962.
Later Parliament enacted the Indo-Tibetan Border Police Force Act, 1992 to provide full
autonomy to ITBP. It operates under the aegis of Union Ministry of Home Affairs
At present ITBP has strong force of about 50,000 personnel to secure the 3,488 km border
with China along Tibet Autonomous Region.
ITBP force is also engage in a variety of internal security tasks including civil Medical Camp,
disaster management, nuclear, biological and chemical disasters and UN peacekeeping
missions around the world.

October 24: United Nations Day


October 24, 2016

The United Nations Day is being observed globally on October 24. The day marks the anniversary of
the entry into force of the UN Charter (the founding document of UNO) in 1945.
It entered into force on 24 October 1945 after it was ratified by the original five permanent members
of the Security CouncilChina, France, Union of Soviet Socialist Republics (now Russia), United
Kingdom, and United States and majority of the other signatories.
What is significance of UN Charter?
With the ratification of UN Charter, the United Nations officially came into existence. It is a
constituent treaty and all signatory members are bound by its articles. Furthermore, Article 103 of
the UN Charter states that obligations to the United Nations prevail over all other treaty obligations.
Most countries in the world have now ratified the Charter.
2016 Celebrations
This years UN Day will be used to highlight concrete actions people can take to help achieve the
2016 GKToday | All Rights Reserved | www.gktoday.in

143

Current Aairs: October 2016 [Date-wise Compendium]


Sustainable Development Goals. Besides, UN Day Concert is also organised in the United Nations
General Assembly Hall with the theme Freedom First.
Background
The United Nations General Assembly had declared 24 October as the United Nations Day by
adopting Resolution 2782 in 1971. The day is devoted making people aware of the aims and
achievements of the United Nations Organization. This day is part of United Nations Week, which
runs from 20 to 26 October.

India falls short in female literacy compared to its neighbours: Study


October 24, 2016

As per new study, Indias school education system is under-performing in terms of quality in female
literacy when compared to its neighbours, Pakistan, Bangladesh and Nepal.
These findings were released by International Commission on Financing Global Education
Opportunity (or Education Commission). It shows that Indias education system is underperforming.
What the study says?
The proportion of women who completed five years of primary schooling in India and were
literate was 48%, much less than 92% in Nepal, 74% in Pakistan and 54% in Bangladesh.

The female literacy rates in India went up by one to 15% after completing two years of
schooling. Corresponding numbers for Pakistan and Nepal were three to 31% and 11 to 47%
respectively.
Around the world, female literacy rates are improving. However, it is not clear if that is
because of improvement in school quality.
India ranks low in global indices of female literacy as well. African countries Rwanda,
Ethiopia and Tanzania all rank higher than India.
India was ranked 38th among the 51 developing countries for female literacy rates when
countries are ranked by earliest grade at which at least of the women are literate.
2016 GKToday | All Rights Reserved | www.gktoday.in

144

Current Aairs: October 2016 [Date-wise Compendium]


How study was conducted?
For this research, researchers had devised a way to measure the quality of education around
the world, with a specific focus on girls.
They had used data from nationally representative Demographic and Health Surveys (DHS)
which is one of the most comparable data sources on living standards in developing world.

World Bank must aid countries to manage shift away from coal
October 24, 2016

The global development lenders like the World Bank and Asian Development Bank (ABD) must
provide help countries including India to shift away from coal for energy purpose.
It was proposed by the World Coal Association to finance countries to help them to shift from their
to more efficient technologies so that they can meet their COP21 commitments.
Why should global development lenders provide aid?
Coal is backbone of countries energy mix to meet their energy demands. Even if countries
push towards renewable energy they are not going to do away with coal in overall energy
mix.
In absence of any funding countries invest in inefficient sub-critical thermal plants, which
have much higher CO2 and particulate matter (PM) emissions. Thus, leading to higher
emissions.
The aid global development lenders will facilitate countries to adopt Super critical and ultrasuper critical (USC) plants technologies.
What are benefits of aid from global development lenders?
Super critical and ultra-super critical (USC) plants technologies have capability to
substantially reduce CO2 emissions and virtually eliminate PM emissions.
It will help countries in reducing their emissions from coal, rather than reducing coal itself
and meet target provided in Intended Nationally Determined Contributions (INDCs).
Thus some target of Paris agreement on Climate Change about reducing the emissions from
coal power plants will be achieved.

Government to provide higher subsidy for airlines connecting two regional airports
October 24, 2016

The Union Government will provide higher a subsidy to airlines that operate flights between two
regional airports under the Regional Connectivity Scheme.
The subsidy amount given to airlines that operate between two unserved or underserved airports
will be 10% higher than sum offered to airlines that connect only one regional airport.
Key Facts
2016 GKToday | All Rights Reserved | www.gktoday.in

145

Current Aairs: October 2016 [Date-wise Compendium]


The subsidy will be provided under the UDAN (Ude Desh ka Aam naagrik) scheme for threeyears so that the route would become viable.
It will be provided to airlines to fund their losses to enable them to offer airfares at Rs. 2,500
for an hours flight on half the seats under the Regional Connectivity Scheme.
The subsidy amount for airlines connecting to one regional airport will vary between Rs.
2,350 and Rs. 5,100 per seat depending upon distance covered between two destinations.
Besides, airlines can also transfer their rights or contract to get subsidy from government and
fly on regional routes to another airline operating a similar type of aircraft..
Other decisions taken by Government to boost Regional Connectivity
The Union Civil Aviation Ministry has scrapped its earlier proposal to reduce the subsidy
amount based on improved passenger load factor.
Earlier, the draft scheme had proposed that if an airline manages to fill 90% of its seats by
operating regional flights during its first year of operations, the subsidy amount in
subsequent year would be reduced by half as the route would become viable.
Besides, Union Government has also reduced the bank guarantee to be provided by airlines to
fly on each regional route.
Now, Airlines will be required to submit a bank guarantee equivalent to 5% of the total
subsidy amount.

October 25, 2016


PM Narendra Modi launches Urja Ganga in Varanasi
October 25, 2016

Prime Minister Narendra Modi has laid the foundation stone of Urja Ganga, the highly ambitious gas
pipeline project in Varanasi, Uttar Pradesh.
The gas pipeline project aims to provide piped cooking (PNG) gas to residents of the eastern region
of the country and CNG gas for the vehicles.
Key Facts
The project is being implemented by state-run gas utility GAIL. It envisages laying a 2,050km pipeline connecting Jagdishpur (UP) to Haldia (West Bengal) by 2018.
From Varanasis perspective, 50,000 households and 20,000 vehicles will get cleaner and
cheaper fuel PNG

2016 GKToday | All Rights Reserved | www.gktoday.in

146

Current Aairs: October 2016 [Date-wise Compendium]

and CNG gas respectively.


The project is considered as a major step towards collective growth and development of the
Eastern region of India. Under it, overall 20 lakh households will get PNG connections.
The Urja Ganga project also augments existing GAILs network of trunk pipelines covering
the length of around 11,000 km by 2540 km.
Besides, under this project work on 2540-km long Jagdishpur-Haldia and Bokaro-Dhamra
Natural Gas pipeline project will begin and will be completed between 2018 and 2020.
Seven East India cities Varanasi, Jamshedpur, Patna, Ranchi, Kolkata, Bhubaneswar, Cuttack
will be the major beneficiary of this network development.
Besides, LNG terminal at Dhamra will provide clean fuel to the Industrial Development of
the Eastern states of Uttar Pradesh, Jharkhand, Bihar, West Bengal and Odisha.
25 industrial clusters in these 5 states will be developed using gas from this pipeline. Besides,
40 districts and 2600 villages will benefit from this project.
It will also help in revival of defunct fertilizer plants in Barauni in Bihar, Gorakhpur in UP,
Sindri in Jharkhand and Durgapur in West Bengal by supplying gas.

ISRO starts landing tests for Chandrayaan-2 mission


October 25, 2016

The Indian Space Research Organsiation (ISRO) has started a series of ground and aerial tests linked
to the critical Moon landing of Chandrayaan-2.
These tests were started as ISROs new site at Challakere in Chitradurga district, 400 km from
Bengaluru. This site has simulated craters.
ISRO Satellite Centre (ISAC) is the lead centre for the second Moon mission. It has artificially
2016 GKToday | All Rights Reserved | www.gktoday.in

147

Current Aairs: October 2016 [Date-wise Compendium]


created close to ten craters to simulate the lunar terrain and test the Landers sensors.
ISAC will conduct many tests including on avionics and electronics, testing the Landers legs,
followed by a combined full test.

About Chandrayaan-2 mission


Chandrayaan-2 mission is tentatively set for late 2017 or early 2018. The mission includes an
Orbiter, a Lander and a Rover.
The Orbiter spacecraft will be launched from Sriharikota will travel to the Moon and release
the Lander, which will in turn deploy a tiny Rover to roam the lunar surface.
The mission includes soft-landing on Moon and moving a rover on its surface. All three
project components will be sending data and pictures to Earth.
Some facts about Chandrayaan-1 mission
Chandrayaan-1 was Indias first lunar probe. It was launched by the ISRO in October 2008
and operated until August 2009.
The spacecraft was launched by PSLV-C11 from Satish Dhawan Space Centre at Sriharikota,
Andhra Pradesh. It had completed 312 days in orbit. It made more than 3,400 orbits around
the moon.
The vehicle was successfully inserted into lunar orbit on 8 November 2008. The probe had
impacted near Shackleton Crater ejecting underground soil that could be analysed for the
presence of water or ice.
It had sent more than 70,000 images of the lunar surface which provided breathtaking views
of lunar mountains and craters, especially craters in the permanently shadowed areas of the
Moons polar region.

CO2 level reaches record high in 2016: WMO


October 25, 2016

According to the World Meteorological Organizations (WMO) annual Greenhouse Gas Bulletin
level of carbon dioxide (CO2) emissions in the atmosphere has reached a record high.
The globally averaged concentration of CO2 in the atmosphere reached to 400 parts per million
(ppm) for the first time in 2015. It is expected to surge again to new records in 2016 on the back of
the very powerful El Nio event.
Key Facts
This is for the first time CO2 levels have reached the 400 ppm barrier on a global average
basis for the entire year.
Besides, longest-established GHGs monitoring station at Mauna Loa, Hawaii predicts that
CO2 concentrations will stay above 400 ppm for whole of 2016 and not dip below that level
for many generations.
2016 GKToday | All Rights Reserved | www.gktoday.in

148

Current Aairs: October 2016 [Date-wise Compendium]


There was a 37% increase in radiative forcing (the warming effect on our climate) because of
long-lived GHGs such as CO2, methane and nitrous oxide (N2O) released from industrial,
agricultural and domestic activities between 1990 and 2015.
What is relation between CO2 emissions and El Nio event?
According to WMO, the growth spurt in CO2 was fuelled by the El Nio event, which
started in 2015 and had a strong impact well into 2016.
The El Nio event had triggered droughts in tropical regions and reduced the capacity of
sinks like forests, vegetation and the oceans to absorb CO2.
These sinks currently absorb about half of CO2 emissions but now there is a risk that they
may become saturated.
Once these sinks become saturated it will increase the fraction of emitted CO2 which stays in
the atmosphere.

About World Meteorological Organization (WMO)


WMO is an intergovernmental organization and specialised agency of the UN for
meteorology (weather and climate), operational hydrology and related geophysical sciences. It
is a member of the United Nations Development Group.
Established: It had originated from the International Meteorological Organization (IMO),
which was founded in 1873.
Membership: 191 Member States and Territories.
Headquarters: Geneva, Switzerland.

ECI amends election rules to allow e-transfer of postal ballot


October 25, 2016

The Election Commission of India (ECI) has amended the Conduct of Election Rules, 1961 to
facilitate the e-transfer of postal ballot.
The new rules empower a returning officer in any constituency to send postal ballots to an eligible
voter by electronic means as specified by the ECI.
In India, postal ballots have played a critical role in extending the electoral process to voters who are
unable to exercise their franchise due to either the nature of their job or geographical location of
their posting.
How the e-transfer of postal ballot will work?
Under it, the returning officer can send ballot through a web portal with a One Time Password
(OTP) to voters. The voter needs to download the ballot for voting. The voter after voting will
physically return the ballot through post. It should be noted that two-way e-voting has not been
recommended by the Election Commission, citing security and secrecy concerns.
What is significance of the new rule?
2016 GKToday | All Rights Reserved | www.gktoday.in

149

Current Aairs: October 2016 [Date-wise Compendium]


The e-transfer of postal ballot will replace existing system in which postal ballots were sent through
the Department of Posts. It will ease the logistical issues involved in ensuring that the ballot paper of
the constituency, where a voter is eligible to vote, is sent in time. Thus, it will cut time in
transmission of the ballot paper and help the EC overcome logistical problems.

Cyrus Mistry removed as Chairman of Tata Group


October 25, 2016

The board of Indias largest conglomerate Tata Sons Ltd has removed Cyrus Pallonji Mistry (48) as
Chairman. However he will remain a director of the individual companies.
The board has replaced him with his predecessor Ratan Tata as interim chairman for four months.
Mistry had shortest tenure as Chairman so far at the group.
Mistry had replaced Ratan Tata as chairman in December 2012. He was only the sixth group
Chairman in nearly 15 decades and the first from outside the Tata family to be appointed to top most
post of Tata group.
Besides, the board also named a five-member search committee to choose a successor within four
months. It has given time of four months to choose a successor.

About Cyrus Pallonji Mistry


Cyrus Pallonji Mistry is son of the billionaire Pallonji Shapoorji Mistry.
Prior to his appointment as chairman he was deputy chairman of Tata Sons. He had remained
the director of Tata Sons since 2006.
Billionaire Pallonji Mistry, along with his sons, Shapoor Mistry and Cyrus Mistry are owners
of 18.5% stake in Tata group. They are largest shareholders on individual basis of group.
He also had served as chairman of Tata Motors, Tata Chemicals, Tata Steel and Tata Global
Beverages.

PM Narendra Modi inaugurates first National Tribal Carnival


October 25, 2016

Prime Minister Narendra Modi inaugurated the first National Tribal Carnival in New Delhi to
promote a sense of inclusiveness among the tribals.
The main purpose of this four-day carnival is to promote a sense of inclusiveness among the tribals.
It will also showcase and promote various facets of tribal culture on a large scale.
Key Facts
In this carnival around 1,600 tribal artists and around 8,000 tribal delegates from across the
country are taking part.
Besides, eminent tribal personalities who have excelled in various fields like arts and culture,
literature, sports, academics and medicine will also participate.
The idea behind the carnival is to preserve and promote various facets of the tribal life
2016 GKToday | All Rights Reserved | www.gktoday.in

150

Current Aairs: October 2016 [Date-wise Compendium]


relating to culture, tradition, customs and their skills.
Besides it also seeks to expose it to the general public with a view to utilise the potential for
overall holistic development of the Scheduled Tribes.
The carnival will host activities like displaying documents on traditional socio-culture
aspects, exhibition of art/artifacts, cultural performances. It will also demonstration of skills
like paintings, traditional healing practices, sports, etc.
It will also host workshops on issues like implementation of Panchayats (Extension to the
Scheduled Areas) Act, 1996 (PESA), Forest Rights Act (FRA), 2006.

Chinas Space Lab Tiangong-2 launches Micro-Satellite Banxing-2


October 25, 2016

Chinas experimental space lab Tiangong-2 orbiting the Earth with two astronauts on board has
successfully launched a micro-satellite Banxing-2.
The two astronauts were launched into space onboard of the Shenzhou-11 spacecraft carried two
astronauts on 11 October 2016. It docked with Tiangong-2 on 13 October 2016.

About Banxing-2
The micro-satellite Banxing-2 is roughly the size of a desktop printer and the media has
nicknamed Selfie Stick. It weighs 47 kilogrammes.
It has series of visible light cameras, including a 25 megapixel camera and wide-angle imagers.
It also has an infrared camera that is temperature-sensitive.
Its mission is to take photographs of Tiangong II and the Shenzhou 11 spacecraft, which has
been docked with the lab.
The micro-satellite has three solar panels which generates enough power to adjust its orbit to
shoot pictures of the lab and spacecraft.
its predecessor, Banxing-1, accomplished the same mission for Shenzhou VII in 2008.
Banxing-2 is new model of its predecessor Banxing-1 which had accomplished the same
mission for Shenzhou VII in 2008. However Banxing-1 is smaller and has a higher capacity.
What are Micro-satellites?
They weigh around 500 to 100 kilograms. They are usually cheaper, faster and more advanced than
traditional satellites. The commercial potential has attracted much attention from businesses.

October 26, 2016


Uranus may have two more moons: Study
October 26, 2016

According to study conducted by researchers from University of Idaho, US Uranus may have two
tiny, previously undiscovered moons orbiting near two of the planets rings.
These two moons were detected by researchers after analysing decades-old images of Uranus icy
2016 GKToday | All Rights Reserved | www.gktoday.in

151

Current Aairs: October 2016 [Date-wise Compendium]


rings taken by NASAs Voyager 2 spacecraft which had flown by the planet 30 years ago.
What researchers have found?
Scientists have found that the pattern in Uranus rings was similar to moon-related structures
in Saturns rings called moonlet wakes.
They have estimate the hypothesised moonlets in Uranus rings may be four to 14 kilometres
in diameter. It means they are as small as some identified moons of Saturn.
How researchers discovered these moons?
They had analysed decades-old images of Uranus icy rings taken by NASAs Voyager 2
spacecraft which had flown by the planet 30 years ago.
During their analysis they had noticed the amount of ring material on the edge of the alpha
ring one of the brightest of Uranus multiple rings varied periodically.
They also had found similar, even more promising pattern occurred in the same part of the
neighbouring beta ring.
They also had analysed radio occultations made when Voyager 2 sent radio waves through
the rings to be detected back on Earth.
They also had analysed stellar occultations made when spacecraft measured light of
background stars shining through the rings, which helped to show how much material they
contain.
What is significance of this research?
Uranian moons are especially hard to spot because their surfaces are covered in dark material. These
findings could help explain some characteristics of Uranus rings, which are strangely narrow
compared to Saturns moons. It will also help to explain that if moonlets exist, they may be acting as
shepherd moons, helping to keep the rings from spreading out. Two of Uranus 27 known moons,
Cordelia and Ophelia act as shepherds to Uranus epsilon ring.

About Uranus
Uranus is the seventh planet from the Sun. It has the third-largest planetary radius and
fourth-largest planetary mass in the Solar System.
Uranus is similar in composition to Neptune and are classified as ice giants to distinguish
them from the gas giants.
Its primary composition of hydrogen and helium, but it also contains more ices such as water,
ammonia, and methane, along with traces of other hydrocarbons
Every planet in our solar system except for Venus and Uranus rotates counter-clockwise as seen
from above the North Pole; that is from west to east.

2016 GKToday | All Rights Reserved | www.gktoday.in

152

Current Aairs: October 2016 [Date-wise Compendium]

Uttarakhand High court bans sale of whitener


October 26, 2016

The Uttarakhand High Court ordered state government to completely ban on the sale of whitener in
the state following the death of a teenager from sniffing an excess of it.
The High Court issued the order by taking suo motu cognisance of the death of the 14-year-old boy
due to sniffing excess whitener for a kick.
Besides, the high court also ordered that items such as Feviquick, Iodex should not be sold to
children below 18 years of age in view of prevalent abuse of these among teenagers.
What are whiteners?
Whiteners are mainly used for erasing ink on paper and as shoe whitener. It contains volatile
aliphatic petroleum hydrocarbons such toluene and trychloroethane which are intoxicating
chemicals.
What is whitener-inhalation addiction?
In whitener-inhalation addiction, people inhale the whitener i.e. intoxicating chemicals
present in it.
By doing this for some time, it makes the sniffer dizzy, high and disconnected. It has a
hallucinatory effect on the sniffer. Thus, it is the first step to drug addiction or alcoholism.
The whitener abuse is generally seen among the youths. The problem has been compounded
by its easy procurement, negligible cost and its lack of odour.
What are side-effects of whitener-inhalation addiction on health?
Whiteners contain hydrocarbons which is deadly solvents that can infuse easily with the
blood and can affect the central nervous system of a person.
Some of the reported symptoms of solvent addiction are uncharacteristic behaviours such as
mood swings and insomnia and it may results in kidney, liver and lung damage
What measures need to be taken to stop it?
Government should completely ban the sale of products that contain intoxicating chemicals,
like the whitener.

Union Government forms panel to resolve telecom penalty row


October 26, 2016

The Union Telecom Ministry has formed a committee to look into the regulator TRAIs
recommendation of a Rs. 3,050 crore penalty on top three telcos Airtel, Vodafone and Idea.
The committee has been asked to give a report on the recommendations soon likely to be before 15
November 2016.
What is the case?
Earlier TRAI had recommended Department of Telecom to impose penalty on these three
2016 GKToday | All Rights Reserved | www.gktoday.in

153

Current Aairs: October 2016 [Date-wise Compendium]


incumbents telecom companies for denying interconnection points to Reliance Jio.
According to TRAI, these three companies have violated licence norms by denying adequate
interconnection points to Reliance Jio Infocomm.
Their actions also have stifled the competition and are seen as anti-consumer and against
public interest.
The Reliance Jio had alleged that due to inadequate interconnection points its subscribers
were unable to make calls to other networks.
TRAI had recommended fine for Airtel and Vodafone to Rs.1,050 crore for 21 service areas
(all, expect Jammu and Kashmir) and for Idea the penalty was pegged at Rs 950 crore for 19
service areas (all, except for Himachal Pradesh, Jammu and Kashmir and Northeastern
States).

Paul Beatty wins 2016 Man Booker Prize


October 26, 2016

Paul Beatty (54) has won the 2016 Man Booker Prize. With this he becomes the first US author to
win this prestigious literary award.
He was bestowed with this award for his racial satirical novel The Sellout at a ceremony held in
Londons Guildhall. His novel tells the story of a young black man who tries to reinstate slavery and
racial segregation in a suburb of Los Angeles.

About Paul Beatty


He was born in Los Angeles in 1962.
He had received an MFA in creative writing from Brooklyn College and an MA in psychology
from Boston University.
His novels are The White Boy Shuffle (1996), Tuff (2000), Slumberland (2008) and The Sellout
(2015).
In 2016, he was bestowed with the National Book Critics Circle Award (Fiction) for his novel
The Sellout.

About Man Booker Prize


The Man Booker Prize worlds most prestigious English-language literary award. It was
launched in 1969 and presented by the Man Group.
The award aims at promoting the finest literary work in fiction by rewarding the best novel
of the year written in English and published in the United Kingdom.
It carries cash prize of 50,000 pounds. The authors are shortlisted by judges from a wide
range of disciplines and all with a passion for quality fiction.
The 2015 Man Booker Prize was conferred on Jamaican author Marlon James for his book A
Brief History of Seven Killings.
2016 GKToday | All Rights Reserved | www.gktoday.in

154

Current Aairs: October 2016 [Date-wise Compendium]

India-Sri Lanka joint military exercise Mitra Shakti 2016 begins


October 26, 2016

The fourth edition of the India-Sri Lanka Joint Military Exercise MITRA SHAKTI 2016 began at
Sinha Regimental Centre in Ambepussa, Sri Lanka.
The main focus of this edition of joint exercise is to enhance inter-operability while carrying out
Counter Terrorism (CT) and Counter Insurgency (CI) operations under the UN Mandate.
Key Facts
Under this mandate the comprehensive training programme spanning fourteen days (24
October to 06 November 2016) has been drawn up.
In this edition of exercise, India is being represented by a platoon from the RAJPUTANA
RIFLES Regiment and Sri Lankan Army is represented by a platoon from the Sinha
Regiment.
The bilateral military exercise will be conducted in two stages in which both armies will get
familiar each others respective methodology of such operations, arms and equipment and the
command and control systems.
It will also graduate towards tactical understanding to enhance inter-operability while
carrying out Counter Terrorism (CT)/ Counter Insurgency (CI) operations.
Background
Mitra Shakti series of bilateral exercises is one of the major bilateral defence cooperation initiatives
between India and Sri Lanka since 2013. The previous edition of the exercise between both nations
was successfully conducted in September 2015 at Pune, Mahrashtra.

India ranks 87th on the WEFs Global Gender Gap Report 2016
October 26, 2016

India has been ranked 87th out of 144 countries on the World Economic Forums (WEF) Global
Gender Gap Report 2016. India has climbed 21 spots from 108th position in 2015.
The report measures gender gap as progress towards parity between men and women in four areas
(i) Educational attainment, (ii) Health and survival, (iii) Economic opportunity and (iv) Political
empowerment.
Key Highlights of the report
Top 5 Countries in 2016 Report: Iceland (1st), Finland (2nd), Norway (3rd), Sweden (4th)
and Rwanda

2016 GKToday | All Rights Reserved | www.gktoday.in

155

Current Aairs: October 2016 [Date-wise Compendium]

(5th).
India related facts: In this edition India has overtaken China which is ranked 99th.
The improvement in Indias ranking is driven largely by major improvements in education,
where it has managed to close its gap entirely in primary and secondary education.
In case of education attainment, India has made considerable strides moving up from 125th
rank in 2015 to 113th in 2016.
On economic participation and opportunity too, India has moved up to 136th rank in 2016,
from 139th in the year 2015.
On health and survival, India has made little progress moving up by one place to 142nd rank
compared to 141st in 2015. On political empowerment, India continues to be ranked 9th in
the world.
Overall Global Facts: The global march towards parity in key economic pillar has slowed
down dramatically. This gap stands at 59% which is now larger than at any point since 2008.
As a consequence, global economic gender gap will now not close until year 2186.
Comment
The gender gap in India has narrowed down. But India still remains one of the worst countries in the
2016 GKToday | All Rights Reserved | www.gktoday.in

156

Current Aairs: October 2016 [Date-wise Compendium]


world for women in terms of labour force participation, income levels as well as health and survival.
India has closed its gender gap by 2% in a year (2016), but much work remains to be done to
empower women in the economic sphere. India continues to rank third-lowest in the world on
Health and Survival, remaining the worlds least-improved country over the past decade.

India ranks 130th in 2017 Ease of Doing Business index


October 26, 2016

India has been placed at 130th position among the 190 countries in the recently released World
Banks ease of doing business index for the year 2017.
The index was released as part of the World Banks annual report Doing Business 2017: Equal
Opportunity for All. This report had revised Indias rank to 131 from earlier 130th for the year
2016. Thus, India has improved its place by one spot in the 2017 index and its place remained
unchanged from the previous original ranking of 130 in the year 2016.
What is ease of doing business index?
The ease of doing business index is annually released by World Bank in its Ease of Doing
Business Report. It was introduced in 2004.
In this index, ranking of country is based on index averages the countrys percentile rankings
on 10 indicators each having equal weightage.
A higher ranking of country in this list means that its regulatory environment is more
conducive and favourable for the starting and operation of firms.
10 indicators are starting business, getting electricity, dealing with construction permits,
registering property, protecting investors, getting credit, employing workers, trading across
borders, paying taxes, enforcing contracts and resolving insolvency.
Key Highlights of 2017 Report
Top 10 Countries: New Zealand (1st), Singapore (2nd), Denmark (3rd), Hong Kong (4th),
South Korea (5th), Norway (6th), UK (7th), US (8th), Sweden (9th) and Macedonia (10th).
BIRCS Nations: India ranks lowest in Doing Business among the BRICS nations. Russia (40),
South Africa (74), China (78), Brazil (123) and India (130).
Indias Neighbours: Bhutan (73), China (78), Nepal (107), Sri Lanka (110), Pakistan (144)
and Bangladesh (176).
India Related Facts: In the ranking, India has made a substantial improvement in some
areas such as electricity connection. But it has sliped in other areas, including payment of
taxes and enforcing contracts.
India has embarked on a fast-paced reform path and has acknowledged a number of
substantial improvements.
2016 GKToday | All Rights Reserved | www.gktoday.in

157

Current Aairs: October 2016 [Date-wise Compendium]


Some the improvement mentioned are electricity connections to businesses, paying taxes,
electronic system for paying employee state insurance contributions, the Companies
(Amendment) Act, electronic filing of integrated customs declarations, passage of the
commercial courts and the Insolvency and Bankruptcy Code.
Global Facts: Economies in all regions are implementing reforms easing the process of doing
business.
However, Europe and Central Asia continues to be the region with the highest share of
economies implementing at least one reform.
96% of economies in the Europe and Central region have implemented at least one business
regulatory reform.
A record 137 economies around the world have adopted key reforms that make it easier to
start and operate small and medium-sized businesses.
Developing countries carried out more than 75% of the 283 reforms in the past year, with
Sub-Saharan Africa accounting for over one-quarter of all reforms.

Government launches International Household Survey Network Toolkit


October 26, 2016

The Union Minister for Statistics and Programme Implementation, D.V. Sadananda Gowda has
launched the International Household Survey Network (IHSN) Toolkit.
He also had launched new Website of the Ministry of Statistics and Programme Implementation in
New Delhi.
What is International Household Survey Network (IHSN) Toolkit?
The IHSN is a standard bench mark which can be used as an active management tool for
better integration of administrative structures and statistical access for policy making.
2016 GKToday | All Rights Reserved | www.gktoday.in

158

Current Aairs: October 2016 [Date-wise Compendium]


It is about latest trends & developments of data archives, storage and dissemination
technology tools available to user. It is designed to address technical issues facing data
producers.
Components of Toolkit are (i) Metadata Editor: It allows user to add survey metadata and
create the ddi.xml and as Nesstar study document. (ii) CD-ROM Builder: It allows user to
generate HTML output from the study that can be published on the Internet or a CD for
dissemination. (iii) NADA: It is search engine that allows user to import the ddi.xml and
search for variables and view metadata.

Life expectancy highest in Jammu and Kashmir: RGI Data


October 26, 2016

According to the data given by the Registrar General of India (RGI), Jammu and Kashmir has the
highest life expectancy (i.e. post-60 life expectancy), except life expectancy at birth.
With this, Jammu and Kashmir has surpassed Kerala which used to be the leader in life expectancy in
almost all categories of age till 2010.
What is data about?
RGIs Sample Registration System (SRS) had published state level life expectancy data
prepared following surveys done between 2010 and 2014.
The data was released for different ages: 0 (at birth), 1, 5, 10, 20, 30, 40, 50, 60 and 70. It had
defined life expectancy as estimate of average number of additional years that a person of a
given age can expect to live.
However this statistical system was confined only to 21 bigger states and UTs and it did not
produce life expectancy numbers on 15 small states and UTs such as Arunachal Pradesh,
Lakshadweep.
Key Facts
The data shows that Kerala still has the highest overall life expectancy at birth at 74.9 years. It
is 72 for men and 77.8 for women.
Delhi is second, with an overall life expectancy at birth at 73.2 years. It is 72 for men and 74.7
for women.
Jammu and Kashmir stands at third and it had the second-highest life expectancy at birth. It is
behind only Kerala even during 2006-10.

October 27, 2016


IPS officer Karnal Singh appointed as Director of Enforcement Directorate
October 27, 2016

The Union Government has appointed Senior IPS officer Karnal Singh as the Director of
Enforcement Directorate (ED).
2016 GKToday | All Rights Reserved | www.gktoday.in

159

Current Aairs: October 2016 [Date-wise Compendium]


Decision in this regard was taken by the Appointments Committee of the Cabinet (ACC) headed by
Prime Minister Narendra Modi. He will be in office till 31 August 2017.
Karnal Singh is 1984-batch IPS officer of the Union territories cadre. Prior to this appointment, he
was holding additional charge as Director of ED for over a year.
What is Enforcement Directorate (ED)?
The Economic Enforcement is economic intelligence and law enforcement agency agency
responsible for enforcing economic laws and fighting economic crime in India.
It functions under the aegis of the Department of Revenue, Union Ministry of Finance.
It has a mandate to enforce two of most stringent laws in the country. They are Prevention of
Money Laundering Act (PMLA) and Foreign Exchange Management Act (FEMA) to check
black money and hawala trade cases.

One-man judicial Committee on OROP submits report to Defence Minister


October 27, 2016

The one-man judicial committee on One Rank One Pension (OROP) set up by the Union
Government, has submitted its report to Defence Minister Manohar Parrikar.
The Union had appointed the committee under the Chairmanship of retired Chief Justice of Patna
High Court L Narasimha Reddy in December 2015. It was tasked to look into the anomalies arising
out of implementation of OROP.
The committee had held hearings at around 20 cities and towns across the country and interacted
with cross sections of ex-servicemen as well as their associations. It also had received 704
representations from individuals and various ex-servicemen associations and had held extensive
interactions with all stakeholders.
What is One Rank One Pension (OROP)?
The OROP scheme aims to provide ex-servicemen of same rank and same length of service uniform
pension regardless of date of retirement, provided that the ex-servicemen concerned had retired
from the same rank and they had served for an equal number of years. Under this scheme, pension
will be re-fixed every five years of ex-servicemen and future enhancements in rates of pension will
be automatically passed to the past pensioners.

Union Government launches Biotech-KISAN and Cattle Genomics to empower farmers


October 27, 2016

The Union Ministry of Science and Technology has launched two new schemes Biotech-KISAN
and Cattle Genomics in order to apply science to boost rural economy.
The intent of these programmes is to form a network to farmers directly with the scientists and
experts. Thus they seek to empower farmers, especially women farmers.
Biotech-KISAN (Krishi Innovation Science Application Network) Scheme
2016 GKToday | All Rights Reserved | www.gktoday.in

160

Current Aairs: October 2016 [Date-wise Compendium]


Its purpose is to connect farmers, scientist and science institution across country. Under it,
fellowship will be given to women farmers for training and education in farm practice.
Under it scientists will spend time on farms and link communication tools to soil, water seed
and market.
The main aim of the scheme is to understand individual problems of the small holding
farmers and provide ready solutions.
It will be implemented in 15 agro-climatic zones of India in phased manner with objective of
linking new technology to farm by understanding problem of local farmer.
Cattle Genomics Scheme
The scheme aims at boosting selective breeding of the native livestock more accurately to
ensure high-yielding, disease-resistant, resilient livestock.
Under it, government will undertake an ambitious project of genome sequencing of 40
registered indigenous cattle breeds of India.
Besides, a high-density DNA chips will be developed under this scheme to reduce the cost and
time interval of breeding of the native livestock.
Genome selection will use information on variations in DNA sequences between animals to
predict the breeding value more accurately. Thus, help to transform livestock breeding.

India, New Zealand ink three agreements including avoidance of double taxation
October 27, 2016

India and New Zealand signed three agreements in the areas of double taxation avoidance (DTA),
sports and food security to carry forward the ties between both sides.
The agreements were signed in the presence of Prime Minister Narendra Modi and his New Zealand
counterpart John Key after the delegation-level talks in New Delhi.
Both countries have reached an understanding on further cooperation on cyber security, counterterrorism, customs, education and food safety
Signed agreements are
Arrangement between the Ministry for Primary Industries of New Zealand regarding Food
Safety Cooperation and Food Safety and Standards Authority of India (FSSAI)
MoU on cooperation in the field of youth affairs and sports. It was signed between the
Ministry of Youth Affairs and Sports of India and Sport New Zealand.
Protocol to the convention for the avoidance of double taxation and the prevention of fiscal
evasion with respect to taxes on income.
Besides, both countries also agreed to
Establish Bilateral Ministerial Dialogue between two Foreign Ministries.
Establish Annual Foreign Ministry Consultations at Senior Officials Level.
2016 GKToday | All Rights Reserved | www.gktoday.in

161

Current Aairs: October 2016 [Date-wise Compendium]


Cooperation and Dialogue on Cyber Issues.
Support for NSG
During this visit of New Zealand PM, India failed to get outright support of New Zealand for its bid
for Nuclear Supplier Group (NSG) membership. India had clearly notified New Zealand that its entry
to the NSG was tied to its need for clean energy and climate change commitments. Thus, it indicates
that New Zealand is yet to change its position of admitting only signatories of Nuclear NonProliferation Treaty (NPT) countries in NSG, a group of 48 countries which works by consensus.
Why New Zealand does not support Indias bid to NSG?
New Zealand is part of a group called the New Agenda for Coalition which promotes the NPT and
pushes for nuclear disarmament worldwide.
Comment
The official state visit New Zealand PM John Key comes in run-up to a crucial NSG
Consultative Group (CG) meeting to be held in Vienna in November 2016.
This meeting will specifically to consider whether countries that havent signed on to the
NPT can be considered for membership.
Earlier in June 2016, Indias membership bid to NSG had failed to make headway in Seoul
(South Korea) after it was opposed by China and other countries.
New Zealand was also among the countries led by China that have demanded to set criteria
for non-signatories of the NPT for joining NSG.
What is New Agenda for Coalition (NAC)?
NAC is a geographically dispersed group of middle power countries that promotes the NPT and
pushes for nuclear disarmament worldwide. It consists of Brazil, Egypt, Ireland, Mexico, New
Zealand and South Africa. The group was officially launched in Dublin (Ireland) in June 1998 in
response to the North-South divide that stymied talks on nuclear disarmament and non-proliferation
within the framework of the NPT.

Union Government constitutes Munialappa Committee to monitor bird flu situation


October 27, 2016

The Union Agricultural Ministry has constituted a High-Level Committee to monitor the bird flu
situation and help state governments take steps to contain the disease.
The High-Level Committee will be headed by Joint Commissioner in the Department of Animal
Husbandry, Dairy and Fisheries (DADF) Munialappa.
Key Facts
The committee will have representatives from the Union Ministry of Health, Union Ministry
of Environment, Forests and Climate Change (MoEFCC), Department of Agriculture
Research and Extension (DARE) and the Delhi Government.
2016 GKToday | All Rights Reserved | www.gktoday.in

162

Current Aairs: October 2016 [Date-wise Compendium]


Besides monitoring the situation, the committee will assist the state governments in taking
steps to contain the spread of avian influenza.
The state government will be assisted by the National Institute of High Security Animal
Diseases (NIHSAD), Indian Veterinary Research Institute and four regional laboratories. In
case of an emergency response, they will get samples tested.
Background
The committee was constituted after mortality among the birds in National Zoological Park, Delhi
NCR and other parts of the country due to Avian Influenza Virus (H5N1) was found.
What is Avian influenza?
Avian influenza, commonly called bird flu is an infectious viral disease of birds. Thus, it is
zoonotic disease.
It is an infectious viral disease of birds and can sometimes spread to domestic poultry and
cause large-scale outbreaks of serious disease.
Human Infections: Most avian influenza viruses do not infect humans. There is no evidence
that the disease can be spread to people through properly cooked food.
However A(H7N9) and A(H5N1) virus strains have caused serious infections in people. The
infections in humans have been associated with direct or indirect contact with infected live or
dead poultry.
Bird flu symptoms in Humans: Fever, sore throat, cough, muscle, body aches, nausea. It
can lead to pneumonia, severe breathing problems and acute respiratory distress syndrome.
Treatment: Controlling the spread of disease in animals is the first step in decreasing risks to
humans.
Human infections with bird flu viruses usually can be treated with the same prescription
drugs that are used to treat human seasonal flu viruses.

Union Cabinet approves agreement between India and Estonia on the transfer of sentenced
persons
October 27, 2016

The Union Cabinet has given its approval for signing and ratification of an Agreement between
India and Estonia on the transfer of sentenced persons.
This agreement will facilitate the Indian prisoners imprisoned in Estonia or vice-versa for serving
remaining part of their sentence in their country and be near to their families. It will also facilitate
their social rehabilitation.
Background
India had enacted Repatriation of Prisoners Act, 2003 under which foreign prisoners are transferred
to country of their origin to serve the remaining part of their sentence and transfer of prisoners of
2016 GKToday | All Rights Reserved | www.gktoday.in

163

Current Aairs: October 2016 [Date-wise Compendium]


Indian origin convicted by a foreign court to serve their sentence in India. The transfer of such
prisoners to their own native countries was to facilitate their social rehabilitation. India has so far
signed bilateral Agreements on Transfer of Sentenced Persons with 27 countries in pursuance of the
Repatriation of Prisoners Act, 2003. So far total 65 prisoners have been exchanged, out of which 55
were Indians.

Union Cabinet gives approval to IISER, Berhampur in Odisha


October 27, 2016

The Union Cabinet has given its ex-post facto approval to formation of the Indian Institute of
Science Education & Research (IISER) at Berhampur in Odisha.
IISER, Berhampur will be established under the Societies Registration Act, 1860 and it will start of
its 2016-17 academic session from 1 August 2016.
It will being its operations from a transit or temporary campus from the Academic Year 2016-17 for
the initial three years i.e. till 2016-19.
Union Cabinet also gave approval for introduction of National Institutes of Technology, Science
Education and Research (Second Amendment) Bill, 2016.
This bill will pave way for inclusion of IISER, Tirupati and IISER, Berhampur in Second Schedule of
National Institutes of Technology, Science Education and Research (NITSER) Act, 2007.
Background
The IISER have been declared as Institutes of National Importance under the NITSER Act,
2012. These Institutes are envisaged to carry out research in areas of science and provide
quality science education at Under-Graduate and Post-Graduate level.
Earlier, there were five IISERs at Pune, Kolkata, Bhopal, Mohali and Thiruvananthapuram.
In 2015, new IISER was started at Tirupati.
IISER, Berhampur comes in line with the announcement made by Union Finance Minister in
his Budget Speech (2015) for setting up of an IISER in Odisha.

RBI allows startups to raise $3 million via ECBs annually


October 27, 2016

The Reserve Bank of India (RBI) has permitted startups to raise external commercial borrowings
(ECBs) of up to $3 million in a financial year for three year tenure
The new rules issued by RBI aims at boosting innovation and promoting job creation in the country.
It will apply to startups looking to raise foreign borrowings and restrictions on such funds will be
kept minimum.
Key Facts
Under the ECB route, borrowing of startups should be denominated in any freely convertible
currency or in Indian Rupees (INR) or a combination thereof.
2016 GKToday | All Rights Reserved | www.gktoday.in

164

Current Aairs: October 2016 [Date-wise Compendium]


In case of borrowing in INR, the non-resident lender, should mobilise INR through
swaps/outright sale undertaken through bank in India.
Under this, Funds can be raised with a minimum maturity of 3 years. There will no costceiling or restriction on the end use of the funds raised.
The borrowing can be in form of loans or non-convertible, optionally convertible or partially
convertible preference shares and minimum average maturity period will be 3 years.
The ECBs can be raised from a country which is either a member of Financial Action Task
Force (FATF) or either through FATF-Style Regional Bodies.
Overseas branches and subsidiaries of Indian banks and overseas wholly-owned subsidiary or
joint venture of an Indian company will not be considered as recognised lender.
What is External Commercial Borrowings (ECBs)?
Any money borrowed from foreign sources for financing the commercial activities in India
are called ECBs. The Central Government permits ECBs as a source of finance for Indian
Corporates for expansion of existing capacity as well as for fresh investment.
Thus, ECBs are defined as money borrowed from foreign resources including the following:
(i) Commercial bank loans (ii) Buyers credit and suppliers credit (iii) Securitised instruments
such as Floating Rate Notes and Fixed Rate Bonds etc. (iv) Credit from official export credit
agencies and commercial borrowings from the private sector window of Multilateral
Financial Institutions such as World Bank, ADB, AFIC, CDC, etc.
How ECB is different from FDI?
In case of Foreign Direct Investment, the foreign money is used to finance the Equity Capital. But in
case ECBs, foreign money is used to finance any kind of funding other than Equity.

October 28, 2016


Andhra Pradesh and Singapore inks FinTech Cooperation agreement
October 28, 2016

Andhra Pradesh government has signed a FinTech Cooperation agreement with Monetary
Authority of Singapore (MAS) to promote innovation financial services in their respective markets.
The agreement was signed in the presence of Chief Minister N Chandrababu Naidu and MAS Chief
FinTech Officer, Sopnendu Mohanty.
Under the agreement
MAS and Andhra Pradesh government will explore joint innovation projects on technologies
such as digital payments and blockchain.
They will also collaborate on the development of education programmes and curricula on
fintech (financial technologies).
They also agreed to discuss emerging FinTech trends and exchange views on regulatory
2016 GKToday | All Rights Reserved | www.gktoday.in

165

Current Aairs: October 2016 [Date-wise Compendium]


issues related to innovations in financial services.
Both signatory parties would enable creation of a niche, first-of-its-kind ecosystem in
Visakhapatnam that was emerging as the FinTech Valley of India.
The knowledge exchange will not only help to create high-quality job opportunities in
FinTech but also help provide market access for implementation in both countries.

Two-thirds of wild animals may go extinct by 2020


October 28, 2016

According to recently released study, global wildlife populations have fallen by 58% since 1970 and if
the trend continues then two-thirds of wild animals may go extinct by 2020
The study was published as The Living Planet assessment by the Zoological Society of London (ZSL)
and the World Wildlife Fund (WWF).
What the study says?
It suggests that animals living in lakes, rivers and wetlands are suffering the biggest losses.
Human activity, including habitat loss, wildlife trade, pollution and climate change
contributed to the decline in global wildlife populations.
It also concluded that vertebrate populations are declining by an average of 2% each year.
How the study was conducted?
The report in its analysis had looked at 3,700 different species of birds, mammals, fish,
amphibians and reptiles, about 6% of total number of vertebrate species in the world.
It had also collected data from government statistics, peer-reviewed studies and surveys
collated by conservation groups and NGOs.
They had included any species with population data going back to 1970, with two or more
time points in the study.
Then using this data researchers had analysed how the population sizes had changed over
time.
Some of this information was weighted to take into account the groups of animals that had a
great deal of data or very little data.
Background
The Living Planet Report is published every two years. It aims to provide an assessment of
the state of the worlds wildlife.
The last report was published in 2014. It had estimated that the worlds wildlife populations
had halved over the last 40 years.

2 Yazidi women Nadia Murad Basee and Lamiya Aji Bashar win 2016 Sakharov Human Right
Prize
October 28, 2016

2016 GKToday | All Rights Reserved | www.gktoday.in

166

Current Aairs: October 2016 [Date-wise Compendium]


Two Yazidi women Nadia Murad Basee and Lamiya Aji Bashar have been selected for European
Unions prestigious Sakharov Prize for Human Rights for year 2016.
Both of them were among thousands of Yazidi girls and women who were abducted by Islamic State
(IS) militants in Iraq and forced into sexual slavery in 2014. But both survived and had escaped sexual
enslavement. Now they campaign for the Yazidi community.
They also have become figureheads for effort to protect Yazidis, followers of an ancient religion with
more than half a million believers concentrated in northern Iraq.

About Sakharov Prize


The Sakharov Prize for Freedom of Thought is annual award given by the European
Parliament.
It is bestowed on individuals or organisations who have dedicated their lives to the defence of
human rights and freedom of thought.
It was established in December 1988 and is named after Soviet scientist (physicist) and
dissident Andrei Sakharov.
The first prize was jointly awarded to Nelson Mandela and Russian human rights campaigner
Anatoly Marchenko.
It is awarded annually on or around December 10 (also celebrated as Human Rights Day), day
on which UN General Assembly ratified Universal Declaration of Human Rights in 1948.

28 October: National Ayurveda Day


October 28, 2016

The first National Ayurveda Day was observed on 28 October on the occasion of Dhanwantari
Jayanti. The theme for this year is Ayurveda for Prevention and Control of Diabetes.
Ayurveda is the traditional Hindu system of medicine based on the idea of balance in bodily systems
and uses diet, herbal treatment, and yogic breathing. It has been incorporated in Atharva Veda, the
last of the four Vedas.
To celebrate this day, the Union Ministry of AYUSH had organized a day-long national seminar on
Prevention and Control of Diabetes through Ayurveda in New Delhi.
Besides, the Ministry also had launched Mission Madhumeha through Ayurveda on the occasion.
The Mission Madhumeha will be implemented throughout the country through a specially designed
National Treatment Protocol for effective management of Diabetes through Ayurveda. The national
treatment Protocol was also released on occasion of National Seminar.
Background
Union Government has decided to celebrate National Ayurveda Day every year on the occasion of
Dhanwantari Jayanti. This day marks birth date of Lord Dhanvantari, which occurs before two days
before Diwali on Dhanteras. Dhanvantari is the form of Lord Vishnu. He is considered as the ideal
2016 GKToday | All Rights Reserved | www.gktoday.in

167

Current Aairs: October 2016 [Date-wise Compendium]


doctor and manifestation of compassion, representing the wisdom of ayurveda in practical
application.

Union Cabinet approves establishment of National Academic Depository


October 28, 2016

The Union Cabinet approved establishment and operationalisation of a National Academic


Depository (NAD).
Under it all academic degrees, certificates and awards in the country will be made digitally available
for verification on single spot.
Key Facts
The NAD will be established and operationalised within the next three months. It will be
rolled out throughout the country in 2017-18.
It will be operationalised by NSDL Database Management Limited (NDML) and CDSL
Ventures, Limited (CVL).
Background
The Finance Ministers Budget Speech (2016) had incorporated commitment to establish a Digital
Depository for school learning certificates, degrees and other academic awards of Higher Education
Institutions, on the pattern of a Securities Depository.
How it will work?
Academic institutions will be directed to upload and authenticate all documents in digital
form into the NAD system
The NAD will register educational institutions/eligibility assessment bodies/boards, students
and other users/verifying entities like employer companies, banks, government agencies and
academic institutions.
It will provide digital or a printed copy of the academic award with security features to the
students or other authorized users.
Besides, NAD will also verify academic awards online on the same day of request initiated by
any authorized user.
Requests for access to academic awards will be on basis of consent of the owner. For ex,
access from potential employers and academic institutions will be only on the basis of consent
of the student.
NAD will maintain the authenticity, integrity and confidentiality of its database. It will root
out fake degrees.

Union Cabinet approves new Agreement on Trade, Commerce and Transit between India
and Bhutan
October 28, 2016

2016 GKToday | All Rights Reserved | www.gktoday.in

168

Current Aairs: October 2016 [Date-wise Compendium]


The Union Cabinet has approved a new agreement on Trade, Commerce and Transit between India
and Bhutan.
The Agreement provides for a free trade regime between two countries. It also provides duty free
transit of Bhutanese merchandise for trade with third countries. As per the agreement, bilateral trade
between both neighbouring countries will continue to be transacted in Indian Rupees and Bhutanese
Ngultrums.
Background
The agreement on Trade, Commerce and Transit between India and Bhutan was renewed on in July
2006 for ten years. The validity of this agreement was extended, with effect from July 2016, for one
year or till the new agreement comes into force.
How trade relations between India and Bhutan are governed?
The bilateral trade relations between India and Bhutan are governed by the Agreement on Trade,
Commerce and Transit. It facilitates mechanism for a free trade regime between two countries, and
duty free transit of Bhutanese merchandise for trade with third countries.
The bilateral trade between both countries had grown by 55% year-on-year in FY 2016 to $750
million. During this period, Indias exports have increased 40.4% to $469 million, while imports from
Bhutan rose 87% to $281 million.

Union Government gives in-principle approval for strategic sale in some PSUs
October 28, 2016

The Union Cabinet has given its in-principle approval for strategic sale of over a dozen public sector
undertakings (PSUs).
This decision was taken based on the recommendation of NITI (National Institution for
Transforming India) Aayogs proposal on disinvestment and strategic sale of sick PSUs.
Key Facts
The strategic disinvestment of such public sector units with the transfer of management
control to a private entity will be taken up subsequently on a case-by-case basis.
It will be undertaken after consultations with their respective administrative ministries
without any timeline.
The Union Government had set a target to raise Rs. 20,500 crore in 2017-18 through
strategic sales of PSUs and another Rs. 36,000 crore from sale of minority stakes in PSUs.
There is no specific timeline set for disinvestment and strategic sale of sick PSUs. Each PSU
will be considered on its own merit with the timing of the sale to be decided accordingly.
Union Government will follow settled valuation procedures in a transparent process for such
transactions.
What is strategic disinvestment?
2016 GKToday | All Rights Reserved | www.gktoday.in

169

Current Aairs: October 2016 [Date-wise Compendium]


In Strategic disinvestment the management control and a significant proportion of a PSUs
share goes to a private sector strategic partner.
Thus, strategic disinvestment of a PSU is different from the ordinary disinvestment in which
management of PSU is retained with Government.
According to the Department of Disinvestment, in the strategic disinvestment of a PSU, the
transaction has two elements: (i) Transfer of a block of shares to a Strategic Partner and (ii)
Transfer of management control to the Strategic Partner.

Microsoft launches Indias first Cybersecurity Engagement Center in Delhi


October 28, 2016

Technology giant Microsoft has launched its first full-scale Cybersecurity Engagement Center
(CSEC) in Delhi.
It is first-of-its-kind Cybersecurity Centre in India and overall 7th in the world including in Tokyo,
Seoul, Beijing, Singapore, Berlin and Washington DC.
Key Facts
The CSEC will bring together Microsoft capabilities to foster deeper Cybersecurity
collaborations with public and private sector organisations.
The centre will also build a trusted and secure computing environment, a critical enabler for
Indias digital transformation.
The CESC will offer their expertise and work with partners to detect and take evasive
measures to tackle cyber threats in India.
The centre is a part of Microsoft Consultancy Services (MCS), a dedicated response team
based in India that will offer security consultancy services to business organisations.
It will offer services such as cyber monitoring, threat detection using machine learning based
technology and take evasive measures to combat cyber threat.

Antarticas Ross Sea declared as worlds largest Marine Protected Area


October 28, 2016

Antarticas Ross Sea was declared as worlds largest Marine Protected Area (MPA) to protect the
Earths most pristine marine ecosystem.
It was declared after landmark international agreement among 24 countries and European Union in
Commission for the Conservation of Antarctic Marine Living Resources (CCAMLR) meeting in
Hobart, Australia.
Key Facts
The Ross Sea is one of the last intact marine ecosystems in the world. It covers 1.6 million
square kilometers.
2016 GKToday | All Rights Reserved | www.gktoday.in

170

Current Aairs: October 2016 [Date-wise Compendium]

MPA status to Ross Sea will bring blanket ban on commercial fishing across about three
quarters of its area for 35 years.
Ross Sea in the Southern Ocean is home to 38% of the worlds Adelie penguins, 30% of
worlds Antarctic petrels and around 6% of worlds population of Antarctic minke whales.
It is also home to huge numbers of krill, a staple food for species including whales and seals.
What is Commission for the Conservation of Antarctic Marine Living Resources
(CCAMLR)?
The CCAMLR was established by international convention in 1982 with the objective of
conserving Antarctic marine life. It is part of the Antarctic Treaty System.
It was signed by 14 states and has been ratified by 35 states (including India) and the
European Community
Headquarters: Tasmania, Australia.
Goal: Preserve marine life and environmental integrity in and near Antarctica
The CCAMLR was established in response to increasing commercial interest in Antarctic
krill resources, a keystone component of the Antarctic ecosystem and over-exploitation of
several other marine resources in the Southern Ocean.

India, Russia agree to double the range of BrahMos Missile


October 28, 2016

India and Russia have agreed to double the range of the BrahMos supersonic cruise missile from 290
km to over 600 km.
The decision in this regard was taken during the meeting between Prime Minister Narendra Modi
and Russian President Vladimir Putin in Goa on the sidelines of the BRICS summit.
Why this decision was taken?
Earlier, India was denied access to the missile technology with range over 300 km as it was not
2016 GKToday | All Rights Reserved | www.gktoday.in

171

Current Aairs: October 2016 [Date-wise Compendium]


member state of the Missile Technology Control Regime (MTCR). But Indias recent accession to
MTCR, allowed Russia to provide the critical systems and technology to extend the range of the
missile beyond 300km.
Benefits of extending range: It will enhance the stand-off capability and the operational radius in
striking targets of the missile. It will have major force multiplier due its speed and accuracy.
Background
India and Russia had teamed up to develop the BrahMos missile in 1998. It is based on the Russian
Yakhont anti-ship missile. Its range was limited to 290 km as Russia was a member of MTCR but
India was not. Due to restrictions under MTCR, Russia was not able provide the critical systems of
the missile to increase its range.
What are features of BrahMos Missile?
BrahMos is one of its kind supersonic cruise missile. Named from the names of two rivers,
Indias Brahmaputra River and Russias Moskva River.
It operates on fire and forget principal and is capable of being launched from land, sea, subsea and air against sea and land targets.
It is capable of carrying a warhead of 300 kilogram and has top supersonic speed of Mach 3
(that is, three times the speed of sound).
It is two-stage missile, the first one being solid and the second one ramjet liquid propellant.
The BrahMos Missile already been deployed by Indian Army and Indian Navy in anti-ship and
precision strike roles respectively. The air version is at present undergoing testing.
What is Missile Technology Control Regime (MTCR)?
MTCR was established by G-7 countries in 1987. It is only an informal political understanding and
not an official treaty with legally binding obligations. The aim of the MTCR is to restrict the
proliferation of missiles, unmanned air vehicles (UAVs), complete rocket systems and related
technology for those systems capable of carrying a 500 kilogram payload for at least 300 kms, as well
as systems intended for the delivery of weapons of mass destruction (WMDs).

2016 GKToday | All Rights Reserved | www.gktoday.in

172

Current Aairs: October 2016 [Date-wise Compendium]

October 29, 2016


Union Government recognizes Boxing Federation of India as National Sports Federation for
Boxing
October 29, 2016

The Union Ministry of Youth Affairs and Sports has recognized Boxing Federation of India (BFI) as
the National Sports Federation (NSF) for Boxing.
This decision will give massive fillip to the recently-formed BFI and also give a boost to the sport of
boxing which had suffered due to absence of an NSF in the discipline since 2012.
The BFI was formed recently and had conducted its elections of its office bearers in September, 2016.
AIBA, the International Body for boxing had recognized these newly elected office bearers of BFI.
Considering the election report and other documents submitted by BFI, the Ministry of Youth
Affairs & Sports granted of recognition as NSF for Boxing
Comment
The official recognition of BFI ended the administrative vacuum that had hampered the sport of
boxing since 2012. Since 2012, Indian boxing had been enduring a turbulent time after the now
2016 GKToday | All Rights Reserved | www.gktoday.in

173

Current Aairs: October 2016 [Date-wise Compendium]


defunct Indian Amateur Boxing Federation was terminated for manipulation in elections. The
recognition of BFI will also help Indian boxers prepare for 2020 Olympics with more national &
international exposure, and training.

Sunil Bharti Mittal appointed as Chairman of GSMA


October 29, 2016

The Founder and Chairman of Bharti Enterprises Sunil Bharti Mittal was appointed as Chairman of
GSM Association (GSMA).
He succeeds Jon Fredrik Baksaas who will step down at the end of 2016. In his new role, Mittal will
oversee the strategic direction of the organisation.
Besides, Mari-Nolle Jego-Laveissiere, Executive Vice President, Innovation, Orange Group was reelected as Deputy Chairman of GSM Association. It also elected the new members of the 26-member
Board of the company.

About Sunil Bharti Mittal


Sunil Mittal is Indian telecom tycoon, entrepreneur and philanthropist.
He is the founder, chairman and Group CEO of Bharti Enterprises, which has interests in
telecom, financial services, retail and agri business.
The Groups flagship company Bharti Airtel is Indias largest telecom company and worlds
third largest mobile operator. It has its presence in 20 countries across Asia and Africa and
customer base of over 275 million.
Presently, he serves on many international bodies and think-tanks and currently is the Chair
of the World Economic Forums (WEF) Telecommunications Steering Committee.
He is listed as the eight richest person in India by Forbes with a Net worth of 7 billion dollars.
In 2007, he was awarded the Padma Bhushan, Indias third highest civilian honour.

About the GSM Association


The GSMA represents the interests of mobile operators worldwide. It unites nearly 800
operators with almost 300 companies in the broader mobile ecosystem.
The GSMA Board has 26 members, including 25 operator representatives from worlds
largest operator groups as well as smaller, independent operators.
It produces industry-leading events such as Mobile World Congress Shanghai, Mobile World
Congress, Mobile World Congress Americas and the Mobile 360 Series of conferences.

DoPT launches Employees Online Application


October 29, 2016

The Department of Personnel & Training (DoPT) under the Ministry of Personnel, Public
Grievances and Pensions has launched Employees Online (EO) App.
The mobile application aims to update on Appointments Committee of the Cabinet (ACC)
2016 GKToday | All Rights Reserved | www.gktoday.in

174

Current Aairs: October 2016 [Date-wise Compendium]


appointments and postings on real time basis and to bring transparency.
The EO App has been developed keeping in mind the spirit of maximum Governance, minimum
Government and meet the need of the hour to bring high-tech systems in the governance.
Key Features of App
Enable its users, to stay updated on real time basis with appointments and postings approved
by the ACC and vacancies at senior level in the Central Government.
Eliminate the information asymmetry in appointments and postings and will reduce
speculations regarding transfers and postings in the Union Government.
Make the system completely transparent as all the relevant orders and notifications will now
be instantly available in the public domain.
Provide personal records like Annual Performance Appraisal Report (APAR), Immovable
Property Return (IPR), Executive Record (ER) sheet to IAS officers on Pan India basis and
officers serving under Central Staffing Scheme.

Himachal Pradesh declared Indias second Open Defecation Free State


October 29, 2016

The State of Himachal Pradesh was officially declared Open Defecation Free (ODF). Every individual
household in the state has functional toilet under the Swachh Bharat Mission.
With this, Himachal Pradesh becomes the second State in the country after Sikkim to achieve the
feat. However, among bigger states Himachal Pradesh is the first state to become ODF.
Himachal Pradesh has successfully achieved total rural sanitation coverage of 100% in the state.
Besides, all 12 districts in state were declared as well as verified as ODF.
This status will further entitle Himachal Pradesh to receive World Bank funding under Rs 9,000
crore project to sustain sanitation campaign.
What is Open defecation?
Open defecation means people defecate in open spaces such as fields, bushes, forests, open
bodies of water rather than using the toilet.
Hazards of Open Defecation: It is important factor in causing various diseases like intestinal
worm infections, diarrhoea, polio, hepatitis etc.
Prevention: Creating awareness about adverse impact of open defecation on health,
changing behavioural habit of people, building of community toilets etc.

Union Government reconstitutes Inter-State Council


October 29, 2016

The Union Government has reconstituted the Inter State Council (ISC) and the Standing Committee
of the ISC under Clause 2 of the Inter State Council order,1990.
The Inter-State Council is a constitutional body to facilitate coordination between states and the
2016 GKToday | All Rights Reserved | www.gktoday.in

175

Current Aairs: October 2016 [Date-wise Compendium]


centre. It will be headed by the Prime Minister Narendra Modi.
Composition of reconstituted Inter State Council (ISC)
Members: Chief Ministers of all states and Union Territories (having legislative Assembly),
Six Union Ministers.
Besides, administrators of Union Territories which do not have legislative Assembly are also
members of the Council.
Six Union Ministers: Home Minister Rajnath Singh, Finance Minister Arun Jaitley,
Defence Minister Manohar Parrikar, External Affairs Minister Sushma Swaraj, Minister for
Information and Broadcasting M Venkiah Naidu and Road Transport Highways and
Shipping Minister Nitin Gadkari.
The Standing Committee of the Inter State Council will be headed by Union Minister of Home
Affairs Rajnath Singh
What is the inter-state council?
The Inter-State Council is a constitutional body to facilitate coordination between states
and the centre.
It is a recommendatory body to investigate and discuss subjects, in which some or all of the
states or the Central government have a common interest.
It is set up on the basis of provisions in Article 263 of the Constitution of India by a
Presidential Order, 1990 based on the recommendation of Sarkaria Commission.
Not a permanent constitutional body
The Inter-state council is not a permanent constitutional body for coordination between the states
and Central government. Rather, President can establish it at any time if it appears to him that the
public interests would be served by the establishment of such a council.
What is the composition of the inter-state council?
The Council is headed by Prime Minister (Chairman). Besides, Chief Ministers of all states and UTs
(having legislative Assembly) are its members. Six Union ministers of cabinet rank nominated by the
Prime Minister are also members.

For more information: Inter-state council

October 30-31, 2016

EU, Canada sign CETA


October 31, 2016

The European Union (EU) and Canada have signed Comprehensive Economic and Trade
Agreement (CETA), a landmark trade deal.
The deal was signed by Canadian Prime Minister Justin Trudeau and top European Union officials in
Brussels, Belgium. Now all 28 European Union states are required to endorse it.
2016 GKToday | All Rights Reserved | www.gktoday.in

176

Current Aairs: October 2016 [Date-wise Compendium]


What is Comprehensive Economic and Trade Agreement (CETA)?
CETA is a free free-trade agreement (FTA) between Canada and the EC. It aims to revoke
roughly 9,000 tariffs, covering many industrial goods and agricultural and food items.
It links single European Union market with Canada, worlds 10th largest economy. It will add
around 11.6 billion to the EU economy and around 8.2 billion to Canadas economy
It also promises to open up competition and cooperation in the services sector ranging from
regulations, to financial services, and telecoms.
It remove customs duties, open-up the services market, end restrictions on access to public
contracts, offer predictable conditions for investors and help prevent illegal copying of EU
innovations and traditional products.

First meeting of NCTF held in New Delhi


October 31, 2016

The first meeting of National Committee on Trade Facilitation (NCTF) was held in New Delhi to
discuss ways for developing roadmap for implementation of WTOs trade facilitation agreement.
It was headed by Cabinet Secretary P K Sinha and was attended by all 24 members along with
secretaries of various ministries and departments like Home, Shipping, Civil Aviation.
What is National Committee on Trade Facilitation (NCTF)?
The NCTF is a mandatory requirement under the WTOs Trade Facilitation Agreement (TFA). It is
a national level body to facilitate domestic co-ordination and implementation of TFA provisions. It
aims to develop the pan-India road map for trade facilitation. It will have three tier structure with
main national committee for monitoring implementation of TFA.
What is Trade Facilitation Agreement (TFA)?
TFA is the WTOs first-ever multilateral accord that contains provisions for expediting the
movement, clearance and release of goods, including goods in transit. It also sets out measures for
effective cooperation between customs and other appropriate authorities on trade facilitation and
customs compliance issues.

October 31: World Cities Day


October 31, 2016

The World is observing Cities Day globally on 31st October to work together for a planned and
sustainable urban life in the wake of rapid growth of population and problems.
The day aims to promote international communitys interest in global urbanization, push forward
cooperation among countries in meeting opportunities and addressing challenges of urbanization,
and contributing to sustainable urban development around the world.
2016 Theme: Inclusive Cities, Shared Development. It highlights the role of urbanization as a source of
global development and social inclusion.
2016 GKToday | All Rights Reserved | www.gktoday.in

177

Current Aairs: October 2016 [Date-wise Compendium]


Background
The General Assembly designated 31st October as World Cities Day by passing resolution 68/239 in
December 2013. This day was constituted in recognition of the significance of urban basic services as
a foundation for the overall social and economic development. The general theme of World Cities
Day is Better City, Better Life, to promote successes of urbanization and address specific challenges
resulting from urbanization.

October 31: Rashtriya Ekta Diwas


October 31, 2016

Rashtriya Ekta Diwas (National Unity Day) is observed across India on 31 October to commemorate
birth anniversary of Independent Indias first Deputy Prime Minister Sardar Vallabhai Patel.
Observance of the day provides an opportunity to re-affirm inherent strength and resilience of our
nation to withstand actual and potential threats to unity, integrity and security of our country.
2016 Theme: Integration of India.
To mark this occasion various activities and events are being organised across ministries and
departments all over the country. Besides, Run for Unity was organised in New Delhi.
The idea of Rashtriya Ekta Diwas was mooted by PM Narendra Modi in October 2014 to mark the
birth anniversary of countrys first Home Minister Sardar Vallabhbhai Patel.

About Sardar Vallabhbhai Patel


He was born on 31 October, 1875 at Karamsand, Gujarat. He is known as the Iron Man of
India or Bismarck of India.
By profession, he was a barrister and had successfully practiced law. Later he left law practice
and joined the Indian national movement under leadership of Mahatma Gandhi.
In Indias independence struggle, he played a major role in organizing peasants movements
on Gandhian principles in Bardoli and Kheda in Gujarat and during the Quit India
movement.
His biggest contribution to India was his role in the integration of the over 500 princely states
with India during 1947-49.
He is also remembered as the Patron Saint of Indias civil servants for establishing modern
All-India services.

Compendium of Multiple Choice Questions


Below 230 Questions were published in GKTodays Current Aairs Quiz Section during month of
October 2016.

1. What is the theme of 2016 International Translation Day (ITD)?


[A] Translate with respect for everyone
[B] Enthusiasm in translation and interpreting
2016 GKToday | All Rights Reserved | www.gktoday.in

178

Current Aairs: October 2016 [Date-wise Compendium]


[C] Translation and interpreting: connecting worlds
[D] Working together with joy

Answer:Translation and interpreting: connecting worlds


Explanation:
The International Translation Day (ITD) is observed every year on September 30 to show the
solidarity of the worldwide translation community in an eort to promote the translation
profession among all the people and not only in one country but in the entire world. The 2016
theme is Translation and Interpreting: Connecting worlds.

2. Who has been appointed as the new chairman of Press Trust of India (PTI)?
[A] Viveck Goenka
[B] Riyadh Mathew
[C] Mahendra Mohan Gupta
[D] Hormusji N Cama

Answer:Riyadh Mathew
Explanation:
Riyadh Mathew, the director of Malayala Manorama, has been appointed as the new Chairman of
Press Trust of India (PTI). He succeeded Hormusji N. Cama.

3. Who has been appointed as the new director of Nehru Memorial Museum & Library (NMML)?
[A] Shakti Sinha
[B] Bhanu Singh
[C] Mahesh Rangarajan
[D] M L Mehta

Answer:Shakti Sinha
Explanation:
Shakti Sinha, the retired IAS ocer, has been appointed as the new director of Nehru Memorial
Museum & Library (NMML). The post was lying vacant since 2015.

4. Who has been named for 2016 Lata Mangeshkar Award for Lifetime Achievement?
[A] Vinod Tawde
[B] Mohammad Safi
[C] Uttam Singh
[D] Prabhakar Jog

Answer:Uttam Singh
Explanation:
The noted music director and violinist, Uttam Singh has been named for 2016 Lata Mangeshkar
Award for Lifetime Achievement for his contributions in the eld of music by the Maharashtra
2016 GKToday | All Rights Reserved | www.gktoday.in

179

Current Aairs: October 2016 [Date-wise Compendium]


government. The award carries a citation, a trophy and Rs. 5 lakh in cash.

5. Who has been honoured with the 2015 Moortidevi award?


[A] Shanti Prasad Sahu
[B] Kolakaluri Enoch
[C] Kapila Vatsyayan
[D] C Radhakrishnan

Answer:Kolakaluri Enoch
Explanation:
The telugu author and eminent scholar, Prof. Kolakaluri Enoch has been honoured with the 29th
Moortidevi Award 2015 for his novel Ananta Jeevanam. The novel portrays the struggle of a
weak, downtrodden and common man from Rayalseema in the face of a cyclone.

6. Who has become first Indian woman to cross 60m in javelian throw?
[A] Seema Antil
[B] Annu Rani
[C] H M Jyothi
[D] Sindoor Gujar

Answer: Annu Rani


Explanation:
Annu Rani from Merrut has broke her own national record and became the rst Indian woman to
cross the 60m mark by throwing javelin 60.01 m at the 56th Open National Athletics
Championships in Lucknow, Uttar Pradesh.

7. The book All That Man Is has been authored by whom?


[A] David Szalay
[B] Milind Dsouza
[C] Latheesh Mohan
[D] P Lankesh

Answer:David Szalay
Explanation:
The book All That Man Is has been authored by David Szalay. The novel is a collection of short
stories, which is a portrait of masculinity and is shortlisted for 2016 Man Booker Prize.

8. Which state has topped list of endemic flowering plant in India, as per the recently released
publication of Botanical Survey of India (BSI) titled Endemic Vascular Plants of India?
[A] Kerala
[B] Tamil Nadu
2016 GKToday | All Rights Reserved | www.gktoday.in

180

Current Aairs: October 2016 [Date-wise Compendium]


[C] Manipur
[D] Maharashtra

Answer:Tamil Nadu
Explanation:
Tamil Nadu has topped the list of endemic owering plant in India, according to the recently
released publication of Botanical Survey of India (BSI) titled Endemic Vascular Plants of
India.As per the latest publication, Tamil Nadu accounts for the highest number of species with
410, followed by Kerala (357 species) and Maharashtra (278 species).

9. Who is the newly elected president of Indian Newspaper Society (INS)?


[A] Somesh Sharma
[B] Kiran B Vadodaria
[C] P V Chandran
[D] Ravindra Kumar

Answer:Somesh Sharma
Explanation:
Somesh Sharma of Rashtradoot Saptahik has been elected as the new President of the Indian
Newspaper Society (INS) for 2016-17. He succeeded P V Chandran of Grehalakshmi
(Mathrubhumi Group). The INS is a kind of pressure group which works to protect the interest of
newspaper industry in particular and print media in general. Its headquarters are at Ra Marg,
New Delhi.

10. Abhaneri festival has started in which state of India?


[A] Andhra Pradesh
[B] Kerala
[C] Himachal Pradesh
[D] Rajasthan

Answer:Rajasthan
Explanation:
The Abhaneri festival has started at the Abhaneri village in Dausa district, Rajasthan from
October 1, 2016 to promote rural tourism in the area. The 2-day festival will witness artistes
performing Shehnai Vaadan, puppet show, Behrupia, Raavanhatha, among others at Chand Baori
and Harshat Mata temple. The Department of Tourism, Government of Rajasthan in collaboration
with the Dausa District Administration organises the annual event.

11. What is the theme of 2016 United Nations (UN) International day of Older Persons?
[A] Longevity: Shaping the Future
[B] Take A Stand Against Ageism
[C] The Growing Opportunities & Challenges of Global Ageing
2016 GKToday | All Rights Reserved | www.gktoday.in

181

Current Aairs: October 2016 [Date-wise Compendium]


[D] Leaving No One Behind: Promoting a Society for All

Answer:Take A Stand Against Ageism


Explanation:
The United Nations (UN) International day of Older Persons is observed every year on October 1
to raise awareness about issues aecting the elderly and to appreciate the contributions that
older people make to society.The 2016 theme is Take a Stand Against Ageism.

12. The powerful Typhoon Chaba is expected to hit which of the following countries?
[A] Japan
[B] New Zealand
[C] United States
[D] United Kingdom

Answer:Japan
Explanation:
The very strong Typhoon Chaba is expected to hit the southern islands of Japan, with a speed of
290 km/ hr. The typhoon is likely to threaten lives and bring damage to property across the
Ryukyu Islands and mainland Japan.

13. Who will be the chief guest for the 2016 Republic Day of India?
[A] Khalifa bin Zayed Al Nahyan
[B] Maktoum bin Rashid Al Maktoum
[C] Sheikh Mohammed bin Zayed Al Nahyan
[D] Zayed bin Sultan Al Nahyan

Answer:Sheikh Mohammed bin Zayed Al Nahyan


Explanation:
Sheikh Mohammed bin Zayed Al Nahyan, the crown prince of Abu Dhabi, will be the chief guest
for the 68th Republic Day celebrations of India on January 26, 2017.

14. Which Indian-American professor has been named as the 2016 class of the Jerger Future
Leaders of Audiology by the American Academy of Audiology?
[A] Vinaya Manchaiah
[B] Madhumitha Kumari
[C] Nidhi Sharma
[D] Nishant Prakash

Answer:Vinaya Manchaiah
Explanation:
Dr. Vinaya Manchaiah, an Indian-American associate professor at Lamar University in Texas, has
been named as the 2016 class of the Jerger Future Leaders of Audiology by the American
2016 GKToday | All Rights Reserved | www.gktoday.in

182

Current Aairs: October 2016 [Date-wise Compendium]


Academy of Audiology. The American Academy of Audiology is the worlds largest professional
organization of audiologists. The members look to provide hearing care services through
education, research, and increased public awareness of hearing and balance disorders.
Audiology is a branch of science that studies hearing, balance and related disorders.

15. Which short film has won the first prize at the Swachh Bharat Short Film Festival (SBSFF)?
[A] Chembuku Moodindi
[B] Murga
[C] Sarkarmi Rati Wadho
[D] Nahna Doot

Answer:Murga
Explanation:
The short lm Murga, which was directed by young lmmaker Katyayan Shivpuri from
Maharashtra, has won the rst prize at the Swachh Bharat Short Film Festival (SBSFF). The short
lm promotes the idea of clean India in which Murga as the metaphor depicts the victims that
citizens have made of themselves and of the children by not keeping the surroundings clean.
Minister of Information and Broadcasting, M Venkaiah Naidu awarded Katyayan with a certicate
and a cash prize of Rs 10 lakh.

16. Who has won the 2016 Nobel Prize in Physiology or Medicine?
[A] Shinya Yamanaka
[B] Youyou Tu
[C] Yoshinori Ohsumi
[D] Christian de Duve

Answer:Yoshinori Ohsumi
Explanation:
The Japanese scientist, Dr. Yoshinori Ohsumi has won the 2016 Nobel Prize in Physiology or
Medicine for his discoveries of mechanisms for autophagy, a fundamental process for degrading
and recycling cellular components. Dr Ohsumis work is important because it helps to explain
what goes wrong in a range of illnesses, from cancer to Parkinsons. The prize was awarded by
the Nobel Assembly at the Karolinska Institute in Stockholm, Sweden.

17. Who has been appointed as the full-time member of 21st Law Commission of India?
[A] Abhay Bhardwaj
[B] A P Shah
[C] S Sivakumar
[D] Abhay Bhardwaj

Answer:S Sivakumar
Explanation:
Prof. S Sivakumar, a professor at the Indian Law Institute in New Delhi, has been appointed as

2016 GKToday | All Rights Reserved | www.gktoday.in

183

Current Aairs: October 2016 [Date-wise Compendium]


the full-time Member of the 21st Law Commission of India (LCI). The 21st LCT was constituted by
the Union Government for a period of 3 years from September 1, 2015 and will have its tenure
only till August 31, 2018. The present Law Commission has not submitted any report till date.
Justice Balbir Singh Chauhan, the former Supreme Court (SC) judge, is the Chairman of the
commission.

18. The World Vegetarian Day (WVD) is observed on which date?


[A] October 7
[B] October 3
[C] October 4
[D] October 1

Answer:October 1
Explanation:
The World Vegetarian Day (WVD) is observed every year on October 1 to promote the joy,
compassion and life-enhancing possibilities of vegetarianism.

19. Which countrys currency has officially joined elite Club of International Monetary Fund (IMF)
reserve currencies?
[A] Dinar
[B] Ruble
[C] Yuan
[D] Rupee

Answer:Yuan
Explanation:
The Chinese currency Yuan has ocially become a member of the International Monetary Fund
(IMF)s basket of global reserve currencies. With this, yuan has joined the U.S. dollar, the euro,
the yen and British pound in the IMFs Special Drawing Rights (SDR) basket, which determines
currencies that countries can receive as part of IMF loans. The yuan is the rst to be added to
SDR since the euro in 1999. China will use this opportunity to further deepen economic reforms
and open up the sector to promote global growth.

20. The book Far and Away:Reporting From the Brink of Change has been authored by whom?
[A] Steve Donoghue
[B] Salman Rushdie
[C] Andrew Solomon
[D] Joshua Hammer

Answer:Andrew Solomon
Explanation:
The book Far and Away:Reporting From the Brink of Change has been authored by Andrew

2016 GKToday | All Rights Reserved | www.gktoday.in

184

Current Aairs: October 2016 [Date-wise Compendium]


Solomon. It is a big, sumptuous collection of those pieces of reportage and travel writing. It is
inspired by love, uncertainty about home and a celebration of freedom which valuably warns that
freedom must sometimes be learned.

21. Who is the newly appointed CMD of Power Finance Corporation (PFC)?
[A] Rajeev Sharma
[B] M K Goel
[C] Mahendra Goyal
[D] K K Singhal

Answer:Rajeev Sharma
Explanation:
Rajeev Sharma is the newly appointed Chairman and Managing Director (CMD) of state-run
Power Finance Corporation (PFC). He succeeded M K Goel who retired on September 30, 2016 on
attaining the age of superannuation. Prior to this appointment, Sharma was the CMD of Rural
Electrication Corporation (REC). The headquarters of PFC are at New Delhi.

22. Who has won 2016 Malaysia Grand Prix Formula One World Championship?
[A] Nico Rosberg
[B] Daniel Ricciardo
[C] Kimi Raikkonen
[D] Max Verstappen

Answer:Daniel Ricciardo
Explanation:
Daniel Ricciardo has won 2016 Malaysia Grand Prix Formula One World Championship at Sepang
International Circuit in Malaysia. He is an Australian racing driver who is currently competing in
Formula One for Red Bull Racing.

23. The World Animal Day (WAD) is observed on which day?


[A] October 3
[B] October 5
[C] October 4
[D] October 6

Answer:October 4
Explanation:
The World Animal Day (WAD) is observed every year on October 4 to raise the status of animals
in the society. It is done by encouraging individuals, animal welfare organisations, community
groups, youth and childrens clubs and businesses to organise events in celebration of this day.

24. Which state has become the first Indian state to implement Direct Benefit Transfer (DBT) in
Kerosene?
2016 GKToday | All Rights Reserved | www.gktoday.in

185

Current Aairs: October 2016 [Date-wise Compendium]


[A] Assam
[B] Andhra Pradesh
[C] Haryana
[D] Jharkhand

Answer:Jharkhand
Explanation:
Jharkhand has become the rst Indian state to implement Direct Benet Transfer (DBT) in
Kerosene. The scheme is being implemented in 4 identied districts of the State from October 1,
2016. These districts are Chatra, Hazaribagh, Khunti and Jamtara. Under the DBT Scheme, PDS
kerosene is being sold at non-subsidised price and the subsidy is being transferred to consumers
directly into their bank accounts. The purpose of the initiative is to rationalize subsidy and
plugging the leakages.

25. What is the Indias GDP growth forecast for 2016, as per World Banks latest report South Asia
Economic Focus?
[A] 7.6%
[B] 7.7%
[C] 7.8%
[D] 7.9%

Answer:7.6%
Explanation:
As per World Banks latest released report South Asia Economic Focus in Washington, Indias
GDP growth will remain strong at 7.6 % in 2016 and 7.7 % in 2017. As per latest report, Indias
economic growth remained robust and is expected to support continued poverty reduction.
Though, the report stated that India faces the challenge of further accelerating the
responsiveness of poverty reduction to growth, promoting inclusion, and extending gains to a
broader range of human development outcomes related to health, nutrition, education and
gender.

26. Which Indian film-maker has been conferred with the French honour of Knight in the National
Order of Merit?
[A] Mani Ratnam
[B] Prajna Chowta
[C] Girish Karnad
[D] Nitin Bose

Answer:Prajna Chowta
Explanation:
Prajna Chowta has been conferred with the Knight in the National Order of Merit (Chevalier de
lOrdre National du Merite), one of the highest civilian recognitions of the French government, in

2016 GKToday | All Rights Reserved | www.gktoday.in

186

Current Aairs: October 2016 [Date-wise Compendium]


recognition of a life devoted to caring for wild Asian elephants. She is the Indian lm-maker,
elephant researcher and the founder of Aane Mane Foundation, which for the past 16 years has
been researching and conserving wild Asian elephants. As a researcher, she was also
instrumental in developing an online monitoring system and one of the rst GPS collars for
elephants in India.

27. Which Indian-origin personality has won 2016 Google Science Fair?
[A] Advay Ramesh
[B] Pulak Kulshrestha
[C] Kiara Nirghin
[D] Pramath Jain

Answer:Kiara Nirghin
Explanation:
Kiara Nirghin (16), an Indian-origin South African teen, has won a $50,000 scholarship at the
2016 Google Science Fair held in United States for her work using orange peel to develop a
cheaper super-absorbent material to help retain water in soil. She is grade 11 student at private
school St Martins. The Google Science Fair is a programme for any budding scientists between
the ages of 13 to 18, who are invited to solve worlds biggest challenges using science and
technology.

28. What is the theme of 2016 World Habitat Day (WHD)?


[A] Cities and Climate Change
[B] Public Spaces for All
[C] Housing at the Centre
[D] Cities, magnets of hope

Answer:Housing at the Centre


Explanation:
The World Habitat Day (WHD) is observed every year on the rst Monday of October across the
world to raise awareness about need for aordable housing for all in urban areas, cities and
towns. The 2016 WHD was observed on October 3 with theme Housing at the Centre. It aims to
raise awareness about need for aordable housing for all in urban areas, cities and towns.

29. Which of the following personalities have won the 2016 Nobel Prize in Physics?
[A] David Thouless, Duncan Haldane and Michael Kosterlitz
[B] Serge Haroche and David J Wineland
[C] Andre Geim and Konstantin Novoselov
[D] Saul Perlmutter, Brian P Schmidt and Adam Riess

Answer:David Thouless, Duncan Haldane and Michael Kosterlitz


Explanation:
The British-born scientists, David J. Thouless, F. Duncan M. Haldane and J. Michael Kosterlitz have

2016 GKToday | All Rights Reserved | www.gktoday.in

187

Current Aairs: October 2016 [Date-wise Compendium]


been awarded the 2016 Nobel Prize in physics for theoretical discoveries of topological phase
transitions and topological phases of matter. The discoveries have paved the way for designing
new materials with all sorts of novel properties. All three laureates are conducting their research
at universities in the United States. They will share the 8m kronor (727,000) prize; it was
announced by the Royal Academy of Sciences in Stockholm, Sweden.

30. The first-ever BRICS trade fair will be held in which city of India?
[A] Pune
[B] New Delhi
[C] Indore
[D] Chennai

Answer:New Delhi
Explanation:
The rst-ever BRICS trade fair & exhibition will be held at Pragati Maidan in New Delhi from
October 12-14 to boost trade among the BRICS nations. The theme of the fair is Building BRICS
Innovation for Collaboration. The trade fair will showcase about 20 key sectors viz. aerospace,
agro-processing, auto, chemicals, green energy, healthcare, railways, textiles, infrastructure, IT,
engineering goods, tourism, gems & jewellery and skill development. The BRICS trade fair will
also focus on Start-ups and bring together young entrepreneurs from across BRICS nations on
this platform.

31. What is the current repo rate, as per the recently released 4th bi-monthly monetary policy
statement for the year 2016-17?
[A] 6.00%
[B] 6.50%
[C] 6.25%
[D] 6.75%

Answer:6.25%
Explanation:
The Reserve Bank of India (RBI) has recently released its 4th bi-monthly monetary policy
statement for the year 2016-17. As per the monetary policy statement, the policy repo rate
under the Liquidity Adjustment Facility (LAF) has cut by 25 bps to 6.25 %, the reverse repo rate
under the LAF stands adjusted to 5.75 % and the Marginal Standing Facility (MSF) rate and the
Bank Rate to 6.75 % and also Statutory Liquidity Ratio (SLR) is at 20.75 %. The decision to cut
interest rates is consistent with the aim of achieving a midterm ination target of 4 % within a
band of + or 2 %. Though, the RBI has kept the Cash Reserve Ratio (CRR) of scheduled banks
unchanged at 4.0 % of Net Demand and Time Liabilities (NDTL). It is rst time that the newly
constituted Monetary Policy Committee (MPC) has made Indias rst collective interest rate
decision. The next monetary policy review will be announced on December 7, 2016.

32. What is the theme of 2016 World Space Week (WSW)?


[A] Space for Education
2016 GKToday | All Rights Reserved | www.gktoday.in

188

Current Aairs: October 2016 [Date-wise Compendium]


[B] Exploring the Universe
[C] 50 Years in Space
[D] Remote Sensing: Enabling our Future

Answer:Remote Sensing: Enabling our Future


Explanation:
The World Space Week (WSW) is an annual event i.e. observed from 4-10 October every year in
various parts of the world including Europe and Asia. It is ocially dened as an international
celebration of science and technology and their contribution to the betterment of the human
condition. The 2016 theme is Remote Sensing: Enabling our Future, which celebrates Earth
observation from space for the betterment of the human race.

33. Who has become brand ambassador of Keralas Haritha Keralam project?
[A] Jency Anthony
[B] K J Yesudas
[C] A R Rahman
[D] Vijay Prakash

Answer:K J Yesudas
Explanation:
Dr. K J Yesudas, the veteran Carnatic vocalist and playback singer, has become brand
ambassador of Haritha Keralam (Green Kerala) project of the Kerala government. The project is a
massive initiative envisaged for a garbage-free and clean state.

34. Which union minister has launched the Indian Bridge Management System (IBMS) in New
Delhi?
[A] Nitin Gadkari
[B] Rajnath Singh
[C] Kalraj Mishra
[D] Anant Geete

Answer:Nitin Gadkari
Explanation:
Nitin Jairam Gadkari, the Union Minister of Road Transport and Highways, has recently launched
the Indian Bridge Management System (IBMS) in New Delhi to ensure proper and timely upkeep
of bridges. IBMS is developed to create an inventory of all bridges in the country and rate their
structural condition so that timely repair and rehabilitation work can be carried out based on the
criticality of the structure. IBMS is the largest platform in the world owned by a single owner,
with database that could exceed 1,50,000 bridge structures.

35. Who will be the first female President of Estonia?


[A] Mart Helme
2016 GKToday | All Rights Reserved | www.gktoday.in

189

Current Aairs: October 2016 [Date-wise Compendium]


[B] Kersti Kaljulaid
[C] Eiki Nestor
[D] Marina Kaljurand

Answer:Kersti Kaljulaid
Explanation:
Kersti Kaljulaid, a former member of the European Court of Auditors, has been elected as the
President of Estonia by the parliament. With this, she will be the rst female president of the
Baltic country Estonia. She will succeed the current President Toomas Hendrik Ilves who is
stepping down next week after two ve-year terms in the ceremonial post.

36. Which bank has became the first domestic bank of India to open a branch in Yangon, Myanmar?
[A] Punjab National Bank
[B] State Bank of India
[C] Bank of Baroda
[D] Dena Bank

Answer:State Bank of India


Explanation:
The State Bank of India (SBI) has become the rst domestic bank of India to open a branch in the
capital city of Yangon, Myanmar. Ghanshyam Srivastava has been appointed as the chief
executive of SBI Yangon branch. The Yangon branch is the 54th foreign branch of SBI, which has
extended the global presence of SBI in 37 countries through 198 oces.

37. Which committee has been constituted to examine causes of vacant seats in IITs, NITs?
[A] Pradeep Mathur committee
[B] Partha Pratim Chakraborty committee
[C] Devang Khakhar committee
[D] Pradipta Banerji committee

Answer:Partha Pratim Chakraborty committee


Explanation:
The Ministry of Human Resource Development (HRD) has constituted a 3-member committee
headed by IIT-Kharagpur Director Partha Pratim Chakraborty to suggest ways to reduce the
number of seats falling vacant at IITs and NITs every year. The move comes in the wake of
almost 3,000 seats falling vacant in 2016 despite six rounds of joint counselling for the IITs, NITs
and centrally funded technical institutions.The committee has been asked to consider a number
of options, including looking at whether scrapping of fee refund in case a candidate does not join
the institute, would help improve the situation. It will submit its report within 3 weeks.

38. Who has been awarded the 2016 John F. Richards Prize?
[A] Farina Mir
2016 GKToday | All Rights Reserved | www.gktoday.in

190

Current Aairs: October 2016 [Date-wise Compendium]


[B] Nayanjot Lahiri
[C] Sunil Amrith
[D] Azfar Moin

Answer:Nayanjot Lahiri
Explanation:
Prof. Nayanjot Lahiri has been awarded the 2016 John F. Richards Prize for her book Ashoka in
Ancient India. The book has been critically acclaimed for its riveting account of charismatic
emperor Ashoka who spoke to his people through his edicts, his victories and loss that left a
legacy that surpassed him. Lahiri is a professor of History at the Ashoka University in Haryana.
The John F. Richards Prize is awarded annually by the American Historical Association (AHA) to
the best book in South Asian history published in English. The prize will be awarded at the
Associations 131st Annual Meeting in January, 2017.

39. The book Modis Midas Touch in Foreign Policy has been authored by whom?
[A] Richard Eaton
[B] Tarun Vijay
[C] Surendra Kumar
[D] Mahendra Jogi

Answer:Surendra Kumar
Explanation:
The book Modis Midas Touch in Foreign Policy has been authored by former Ambassador
Surendra Kumar. The book has recently launched by M. Venkaiah Naidu, the Union Minister for
Information and Broadcasting, in New Delhi. The book outlines the progress and the fast imprint
on the conduct of Indias Foreign Policy by the Prime Minister Narendra Modi in the last two and a
half years. It maps the dierent policy initiatives focusing on the vision of developing India into a
strong economically developed nation.

40. What is the theme of 2016 World Teachers Day (WTD)?


[A] Valuing Teachers, Improving their Status
[B] Empowering teachers, building sustainable societies
[C] Invest in the future, invest in teachers!
[D] A Call for Teachers

Answer:Valuing Teachers, Improving their Status


Explanation:
The World Teachers Day (WTD) is observed every year on October 5 to mobilize support for
teachers and to ensure that the needs of future generations will continue to be met by teachers.
The 2016 theme is Valuing Teachers, Improving their Status.

41. Which of the following personalities have won the 2016 Nobel Prize in Chemistry?
[A] William Moerner, Eric Betzig and Stefan Hell
2016 GKToday | All Rights Reserved | www.gktoday.in

191

Current Aairs: October 2016 [Date-wise Compendium]


[B] Fraser Stoddart, Jean-Pierre Sauvage and Bernard Feringa
[C] Arieh Warshel, Michael Levitt and Martin Karplus
[D] Paul Modrich, Tomas Lindahl and Aziz Sancar

Answer:Fraser Stoddart, Jean-Pierre Sauvage and Bernard Feringa


Explanation:
Jean-Pierre Sauvage (from France), Sir J. Fraser Stoddart (from Scotland) and Bernard L. Feringa
(from the Netherlands) have been awarded the 2016 Nobel Prize in Chemistry for the design and
synthesis of molecular machines- the worlds smallest machines. The prize recognizes their
success in linking molecules together to design everything from motors to a car and muscles on
a tiny scale. They will share the 8m kronor (727,000) prize; it was announced by the Royal
Academy of Sciences in Stockholm, Sweden.

42. What is the Indias growth forecast for FY17, as per recently released IMFs World Economic
Outlook (WEO) report?
[A] 7.4%
[B] 7.6%
[C] 7.8%
[D] 8.0%

Answer:7.6%
Explanation:
As per latest revised edition of World Economic Outlook (WEO) report of International Monetary
Fund (IMF), Indias GDP growth forecast has been raised by 0.2 % to 7.6 % for FY17 (2016-17 )
and FY18 (2017-18). According to report, Indias economy continued to recover strongly,
beneting from a large improvement in the terms of trade, eective policy actions and stronger
external buers, which have helped boost sentiment. Beside this, the report has also higlighted
that the Indian government must have to continue to reform its tax system and eliminate
subsidies to provide more resources for investments in infrastructure, education and healthcare.

43. Which union ministry has set up a Public Debt Management Cell (PDMC) in New Delhi?
[A] Ministry of Law and Justice
[B] Ministry of Labour and Employment
[C] Ministry of Home Affairs
[D] Ministry of Finance

Answer:Ministry of Finance
Explanation:
The Union Finance Ministry has set up a Public Debt Management Cell (PDMC) to streamline
government borrowings and better cash management with the overall objective of deepening
bond markets. The PDMC, to be housed at RBIs Delhi oce, is an interim arrangement and will
be upgraded to a statutory Public Debt Management Agency (PDMA) in about 2 years. The PDMC
will have only advisory functions to avoid any conict with the statutory functions of RBI. PDMC
2016 GKToday | All Rights Reserved | www.gktoday.in

192

Current Aairs: October 2016 [Date-wise Compendium]


has been tasked to plan government borrowings, including market borrowings and other
borrowings, like Sovereign Gold Bond issuance. It will also advise government on matters related
to investment, capital market operations, administration of interest rates on small savings
among others.

44. Yusuf Arakkal, who passed away recently, was a well-known personality of which field?
[A] Painting
[B] Sports
[C] Politics
[D] Journalism

Answer:Painting
Explanation:
Yusuf Arakkal (71), who recently passed away in Bengaluru, was a Malayali painter and was
known for his contribution to the world of abstract and contemporary art. He was also known for
his works depicting social issues. He recently released a book title Faces of Creativity, which is
a comprehensive collection of portraits of fellow artists. It is rst of its kind that an Indian artist
had done portraits of other artists.

45. The 2016 World Economic Forum (WEF) India Economic Summit has started in which city of
India?
[A] Pune
[B] New Delhi
[C] Ahmedabad
[D] Chandigarh

Answer:New Delhi
Explanation:
The 2016 World Economic Forum (WEF) India Economic Summit has been inaugurated by
Commerce and Industry Minister Nirmala Sitharaman in New Delhi on October 6, 2016. The
theme of the summit is Fostering an Inclusive India through Digital Transformation. The 2-day
summit will focus on issues like digital transformation and regional economic integration. At least
600 participants from a little over 30 countries will take part.

46. Which Indian mountaineer has recently scaled the Mountain Cho Oyu?
[A] H P S Ahluwalia
[B] Mastan Babu Malli
[C] Arjun Vajpai
[D] Jamling Tenzing Norgay

Answer:Arjun Vajpai
Explanation:
2016 GKToday | All Rights Reserved | www.gktoday.in

193

Current Aairs: October 2016 [Date-wise Compendium]

Arjun Vajpai (23), the professional Indian mountaineer, has recently scaled the worlds 6th
highest mountain Cho Oyu (approx 8,188 metres), which is wedged between the China-Tibet
border. It stands 20 km west of Mount Everest. With this, he became the youngest in the world to
scale the Mount Cho Oyu among others. With the Cho Oyu summit, Arjun has climbed ve of the
14 highest mountains in the world that tower over 8000 metres and holds world records as the
youngest summiteer on Mount Lhotse, Mount Manaslu and Mount Makalu.

47. Who will become the new Secretary General of the United Nations?
[A] Matthew Rycroft
[B] Anibal Cavaco Silva
[C] Antonio Guterres
[D] Vitaly Churkin

Answer:Antonio Guterres
Explanation:
Antonio Guterres, the former Prime Minister of Portugal, will become the 9th new Secretary
General of the United Nations after the UN Security Council declared him to be the unanimous
choice in the 6th straw poll. Guterres, who led the UN refugee agency for 10 years, will succeed
Ban Ki-moon in 2017.

48. Who will be new chairperson of the Central Board of Direct Taxes (CBDT)?
[A] Atulesh Jindal
[B] Sushil Chandra
[C] Nilesh Chand
[D] Shankar Gora

Answer:Sushil Chandra
Explanation:
Sushil Chandra, a 1980 batch ocer of Indian Revenue Service (Income Tax cadre), has been
appointed as the new chairperson of the Central Board of Direct Taxes (CBDT), the apex policy
making body for the Income Tax department. He will replace Rani Singh Nair, who retires on
October 31, 2016. Currently, Chandra is the member (Investigation) of the CBDT.

49. The 2016 World Sustainable Development Summit (WSDS) has started in which city of India?
[A] New Delhi
[B] Pune
[C] Bhopal
[D] Jaipur

Answer:New Delhi
Explanation:
The rst edition of the World Sustainable Development Summit (WSDS) 2016 has been

2016 GKToday | All Rights Reserved | www.gktoday.in

194

Current Aairs: October 2016 [Date-wise Compendium]


inaugurated by President Pranab Mukherjee on October 5 in New Delhi. The 4-day summit is
organized by The Energy and Research Institute (TERI) with theme Beyond 2015: People, Planet
and Progress. It will be one of the rst international platforms to discuss the new agenda post
the adoption of the Sustainable Development Goals (SDG) and the signing of the Paris
Agreement. The attempt will be to initiate discussions on methodologies to be adopted to chalk a
path to realize the goals beyond 2015. The summit would emphasis on the need to limit the rise
of climate change.

50. The book Night of Fire has been authored by whom?


[A] Jung Chang
[B] Colin Thubron
[C] Arundhati Roy
[D] Tim Tigner

Answer:Colin Thubron
Explanation:
The book Night of Fire has been authored by Colin Thubron. The novel is a ctive masterpiece
of exquisite beauty and lingering mystery.

51. Which Indian shooter has clinched silver in the 50 metre pistol event of the ISSF World Cup
2016 ?
[A] Gagan Narang
[B] Ronjan Sodhi
[C] Jitu Rai
[D] Abhinav Bindra

Answer:Jitu Rai
Explanation:
Jitu Rai, an ace Indian shooter, has clinched a silver medal in the 50 metre pistol category of the
ISSF World Cup 2016 in the nal at Bologna, Italy. In this event, Wei Pang of China has won gold
and Giuseppe Giordano of Italy has won bronze medal.

52. The book The Ministry of Utmost Happiness has been authored by whom?
[A] Arundhati Roy
[B] Vikram Seth
[C] Kiran Desai
[D] Aravind Adiga

Answer:Arundhati Roy
Explanation:
The book The Ministry of Utmost Happiness has been authored by Arundhati Roy, which will
release in June 2017. It is ction-based novel and will be published by Hamish Hamilton UK and

2016 GKToday | All Rights Reserved | www.gktoday.in

195

Current Aairs: October 2016 [Date-wise Compendium]


Penguin India.

53. Which of the following personalities has won the Nobel Peace Prize for 2016?
[A] Martti Ahtisaari
[B] Juan Manuel Santos
[C] Shinzo Abe
[D] Shirin Ebadi

Answer:Juan Manuel Santos


Explanation:
Juan Manuel Santos, the Colombian President, has been awarded the Nobel Peace Prize for 2016
for his resolute eorts to bring the countrys more than 50-year-long civil war to an end, a war
that has cost the lives of at least 220 000 Colombians and displaced close to 6 million people.
The award should also be seen as a tribute to the Colombian people who, despite great
hardships and abuses, have not given up hope of a just peace and to all the parties who have
contributed to the peace process.

54. The Commonwealth Finance Ministers Meeting 2016 has been held in which of the following
cities?
[A] Geneva
[B] New Delhi
[C] London
[D] Washington

Answer:Washington
Explanation:
Arun Jaitley, the Union Finance Minister, has chaired the Commonwealth Finance Ministers
Meeting at the International Monetary Fund (IMF) headquarters in Washington DC, United States.
During the meeting, two important issues- the Economics of Climate Change and Financing
Climate Adaptation and Mitigation and International Taxation-a Commonwealth Conversation
around the Panama Papers were discussed and deliberated. Apart from this, Jaitley also chaired
the Governing Council meeting of the BRICSContingent Reserve Arrangement (CRA). The CRA is
a framework for the provision of support through liquidity and precautionary instruments in
response to actual orpotential short-term balance of payments pressures.

55. Which state chief minister of India has been honoured with the 2016 Sustainable Development
Leadership Award (SDLA)?
[A] Okram Ibobi Singh
[B] T R Zeliang
[C] Pawan Chamling
[D] Naveen Patnaik

Answer:Pawan Chamling
2016 GKToday | All Rights Reserved | www.gktoday.in

196

Current Aairs: October 2016 [Date-wise Compendium]


Explanation:
Pawan Chamling, the Chief Minister of Sikkim, has been honoured with the prestigious 2016
Sustainable Development Leadership Award (SDLA) of the The Energy and Resources Institute
(TERI) in recognition of his vision and leadership in environment and sustainable development
leading to the establishment of Sikkim as the rst and only organic state in India. Chamling was
presented with the award by President Pranab Mukherjee during World Sustainable Development
Summit (WSDS) at Vigyan Bhawan in New Delhi.

56. Indias First Medical Park will be set up at which of the following states?
[A] Uttar Pradesh
[B] Tamil Nadu
[C] Odisha
[D] Arunachal Pradesh

Answer:Tamil Nadu
Explanation:
Recently, the Union Government has approved HLL Lifecare (mini ratna Public Sector
Undertaking (PSU) ) to sub-lease 330.10 acres of land to set up Indias rst medical technology
manufacturing park (Medipark) at Chengalpattu in Chennai,Tamil Nadu. The Medipark project will
boost local manufacturing of hi-end products at a signicantly lower cost, resulting in aordable
healthcare delivery, particular in diagnostic services to a large section of people. It is expected to
be completed over 7 years.

57. The Election Commission of India (ECI) has collaborated with which social networking site to
register young voters?
[A] Instagram
[B] Facebook
[C] Twitter
[D] YouTube

Answer:Facebook
Explanation:
In a rst-of-its-kind initiative, the Election Commission of India (ECI) has collaborated with
Facebook to register young voters in 5 states, which are going to polls in 2017. The purpose of a
voter registration drive is to encourage youth to participate in the democratic exercise. From
October 6 to 9, Facebook users in Uttar Pradesh, Punjab, Goa, Manipur and Uttarakhand who are
18 years and above will receive a reminder in their News Feed to register to vote. By clicking on
the Register Now button, people will be directed to the National Voters Services Portal which
will guide them through the registration process.

58. Which Indian city will host the 2016 Asian Ministerial Conference on Disaster Risk Reduction
(AMCDRR)?
[A] Lucknow
2016 GKToday | All Rights Reserved | www.gktoday.in

197

Current Aairs: October 2016 [Date-wise Compendium]


[B] Ahmedabad
[C] New Delhi
[D] Pune

Answer:New Delhi
Explanation:
The 2016 Asian Ministerial Conference on Disaster Risk Reduction (AMCDRR) will be hosted by
the Government of India (GoI) in New Delhi on November 2-3, 2016. Prime Minister Narendra
Modi will inaugurate the conference that will be held in collaboration with the United Nations
Oce for Disaster Risk Reduction (UNISDR). The purpose of the conference is to transform the
commitments of governments and stakeholders during the Sendai Conference into national and
local action.

59. Which committee has been constituted to assess ground situation at Cauvery basin?
[A] MN Rao committee
[B] KK Paul committee
[C] GS Jha committee
[D] Raghuveer Bose committee

Answer:GS Jha committee


Explanation:
The Union Ministry for Water Resources, River Development and Ganga Rejuvenation has
constituted a high-level technical team to visit the river Cauvery basin area in Karnataka and
Tamil Nadu to assess the ground situation. GS Jha, the Chairman of Central Water Commission
(CWC), will head the Cauvery supervisory committee and will submit its report to the Supreme
Court by October 17, 2016.

60. The 2016 BRICS Science & Technology Senior Official Meeting (SOM) has been held at which
city of India?
[A] New Delhi
[B] Jaipur
[C] Bhopal
[D] Udiapur

Answer:Jaipur
Explanation:
The 6th BRICS Science & Technology Senior Ocial Meeting (SOM) has held at Jaipur, Rajasthan
on October 7, 2016 to provide an overarching vision and to set priorities for cooperation & joint
action amongst BRICS countries in the elds of Science, Technology and Innovation.

61. Heisnam Kanhailal, who passed away recently, was related to which field?
[A] Theatre
2016 GKToday | All Rights Reserved | www.gktoday.in

198

Current Aairs: October 2016 [Date-wise Compendium]


[B] Science
[C] Sports
[D] Politics

Answer:Theatre
Explanation:
Heisnam Kanhailal (75), the Manipur-based theatre legend, has recently passed away Imphal,
Manipur. Kanhailal was the founder-director of Kalakshetra Manipur, a theatre laboratory that
explores a new vocabulary in the existing language of theatre. He had contributed to the eld of
experimental theatre for more than 40 years and given it his own local perspective.

62. The Batukamma festival is celebrated in which state of India?


[A] Karnataka
[B] Telangana
[C] Jharkhand
[D] Kerala

Answer:Telangana
Explanation:
The Bathukamma festival is a oral festival, which is celebrated every year by the Hindu women
of Telangana. The 9-day festival indicates the beginning of Sharath Ruthu.This year, Telangana
women have established a new world record with the largest participation ever in the traditional
Batukamma festival. A record 9,292 women performed the customary dance around the 20-foot
Batukamma ower arrangement set in the centre of the Lal Bahadur Stadium in Hyderabad,
seeking blessings and benevolence of the deity for the happiness and prosperity of the State.
The spectacular performance garnered the State a place in the prestigious Guinness Book of
Records. With this, Telangana women have broken the earlier record set by 5,211 Kerala women
for the largest Onam congregation.

63. The Indian Air Force (IAF) Day is celebrated on which date?
[A] October 9
[B] October 7
[C] October 8
[D] October 10

Answer:October 8
Explanation:
The Indian Air Force (IAF) Day is celebrated every year on October 8. Recently, the 84th
anniversary of the IAF has been celebrated with traditional pomp and gaiety at the National
Defence Academy (NDA), Khadakwasla near Pune on October 8, 2016 .The IAF is the worlds 4th
largest air force with a primary responsibility to secure Indian airspace and to conduct aerial
warfare during a conict. It was ocially established on October 8, 1932 as an auxiliary air force
of the British Empire and the prex Royal was added in 1945 in recognition of its services during

2016 GKToday | All Rights Reserved | www.gktoday.in

199

Current Aairs: October 2016 [Date-wise Compendium]


World War II. After India became independent from the United Kingdom (UK) in 1947, the Royal
Indian Air Force served the Dominion of India, with the prex being dropped when India became
a republic in 1950.

64. Indias first-ever international arbitration centre has set up in which city?
[A] Pune
[B] Mumbai
[C] New Delhi
[D] Kochi

Answer:Mumbai
Explanation:
Indias rst-ever international arbitration centre has set up in Mumbai, Maharashtra on October
8, 2016. The Mumbai Centre for International Arbitration (MCIA) has been launched by
Maharashtra Chief Minister Devendra Fadnavis, which will help in resolving disputes worldwide
between dierent companies or individuals. The world-class facility will bring greater
transparency, accountability and credibility to the process of resolving overseas arbitration. Till
now, the closest arbitration centre for India was in Singapore.

65. What is the Indias rank in the Biopharmaceutical Competitiveness & Investment (BCI) Survey?
[A] 28th
[B] 33rd
[C] 10th
[D] 19th

Answer:19th
Explanation:
India has been ranked 19th out of 28 nations in the Biopharmaceutical Competitiveness &
Investment (BCI) Survey, with an overall score of 59 out of 100. India scores low on almost all
metrics except for partial step-ups on two counts: scientic capabilities and infrastructure, and
clinical research conditions and framework. The list has topped by United States, followed by UK
and Switzerland. The ranking is part of the 3rd edition of the BCI commissioned by the
Pharmaceutical Research and Manufacturers of America and executed by the Pugatch Consilium.
It looks at biomedical innovation and its inuencers, like intellectual property protection for
investments in the sector.

66. What is the theme of 2016 World Post Day (WPD)?


[A] Tell us about the world you want to grow up in
[B] Write a letter describing how music can touch lives
[C] The Post: Delivering for People and Businesses Daily
[D] Innovation, Integration and Inclusion

Answer:Innovation, Integration and Inclusion


2016 GKToday | All Rights Reserved | www.gktoday.in

200

Current Aairs: October 2016 [Date-wise Compendium]


Explanation:
The World Post Day (WPD) is observed each year on October 9 to commemorate the
establishment of the Universal Postal Union (UPU) in 1874 in Bern, Switzerland. The 2016 theme
is Innovation, Integration and Inclusion.

67. Doordarshan has signed MoU with which institute for transmission of Gyan Darshan channels?
[A] IIT Bombay
[B] Amity University
[C] IGNOU
[D] IIM Ahmedabad

Answer:IGNOU
Explanation:
Doordarshan has recently signed an agreement with Indira Gandhi National Open University
(IGNOU) regarding the transmission of 4 Gyan Darshan Educational channels. The Gyan Darshan
Bouquet of Educational TV Channels is an educational media initiative of MHRD in collaboration
with the Ministry of Information and Broadcasting (MIB), Prasar Bharati & ISRO with IGNOU as the
nodal agency. Now, IGNOU needs to apply for a fresh license to start the up-linking of its
channels for GSAT-10 satellite, as under the Uplinking and Downlinking Guidelines of the I&B
Ministry, it is not eligible for the grant of a license.

68. Who has won 2016 Formula One Japanese Grand Prix?
[A] Kimi Raikkonen
[B] Nico Rosberg
[C] Max Verstappen
[D] Sebastian Vettel

Answer:Nico Rosberg
Explanation:
Nico Rosberg, a German Formula One driver for Mercedes Formula One team, has won the 2016
Formula 1 Emirates Japanese Grand Prix at Mie Prefecture, Japan.

69. PR Sreejesh is associated with which sports?


[A] Badminton
[B] Cricket
[C] Hockey
[D] Table Tennis

Answer:Hockey
Explanation:
Parattu Raveendran Sreejesh is an Indian eld hockey player and plays as a goalkeeper in the
Indian national team. He will lead an 18-member Indian mens hockey team in the 4th Asian

2016 GKToday | All Rights Reserved | www.gktoday.in

201

Current Aairs: October 2016 [Date-wise Compendium]


Champions Trophy (ACT) to be held in Kuantan, Malaysia from October 20 to 30, 2016.

70. The book Old Path White Clouds: Walking in the Footsteps of the Buddha has been authored
by whom?
[A] PV Narasimha Rao
[B] Vinay Sitapati
[C] Thich Nhat Hanh
[D] Dalai Lama

Answer:Thich Nhat Hanh


Explanation:
The book Old Path White Clouds: Walking in the Footsteps of the Buddha has been authored by
Thich Nhat Hanh. It is the beautiful classic recounting of the life and teachings of Gautama
Buddha. The book traces the Buddhas life slowly and gently over the course of 80 years, partly
through the eyes of Svasti, the bualo boy, and partly through the eyes of the Buddha himself.

71. What is the theme of 2016 World Mental Health Day?


[A] Psychological First Aid
[B] Dignity in mental health
[C] Depression: A Global Crisis
[D] Health: Mental Will Power

Answer:Psychological First Aid


Explanation:
The World Mental Health Day is observed every year on October 10 to raise awareness about
mental health issues around the world and mobilizing eorts in support of mental health. The
2016 theme is Psychological First Aid. The day provides an opportunity for all stakeholders
working on mental health issues to talk about their work and what more needs to be done to
make mental health care a reality for the people.

72. Lalima Abhiyan will be launch by the which state government to make state free from anaemia?
[A] Odisha
[B] Assam
[C] Madhya Pradesh
[D] Kerala

Answer:Madhya Pradesh
Explanation:
The Madhya Pradesh government will launch a scheme Lalima Abhiyan to make state free from
anaemia, which will be eective from November 1, 2016. Under the campaign, free iron folic acid
tablets will be provided in aaganwadis, academic institutions and hospitals.

73. The 2016 International Gita festival will be held in which state of India?
2016 GKToday | All Rights Reserved | www.gktoday.in

202

Current Aairs: October 2016 [Date-wise Compendium]


[A] Madhya Pradesh
[B] Haryana
[C] Uttar Pradesh
[D] Rajasthan

Answer:Haryana
Explanation:
The 2016 International Gita festival will be held at Kurukshetra in Haryana on December 6 to
take the eternal message of the holy book to the international stage. The 5-day festival will be
inaugurated by President Pranab Mukherjee. A number of cultural performances, art shows,
literary events and festivities will mark the celebrations up to December 10. Approx 18,000
students from all over Haryana will recite the shlokas of the Gita at the sacred Brahma Sarovar in
Kurukshetra.

74. Who has been appointed as Chinas new Ambassador to India?


[A] Sun Weidong
[B] Luo Zhaohui
[C] Ma Qiang
[D] Le Yucheng

Answer:Luo Zhaohui
Explanation:
The Chinese President Xi Jinping has appointed Luo Zhaohui as Chinas new Ambassador to India.
Prior to this appointment, Luo was Chinas Ambassador to Canada from 2014 to 2016. He
succeeded Le Yucheng, who left New Delhi in April 2016.

75. The book Half Lion: How P V Narasimha Rao Transformed India has been authored by whom?
[A] R B Gaur
[B] Sajid Khan
[C] Vinay Sitapati
[D] Ashok Chawla

Answer:Vinay Sitapati
Explanation:
The book Half Lion: How P.V. Narasimha Rao Transformed India has been authored by Vinay
Sitapati. It is a well-researched, well written book that traces the life journey of P.V. Narasimha
Rao, about whom not much has been in the public domain so far. The book lls a gap in the
evolution of Indias polity and the oce of the Prime Minister.

76. Who has won the 2016 China Open Womens Singles Tennis tournament ?
[A] Agnieszka Radwanska
[B] Petra Kvitova
2016 GKToday | All Rights Reserved | www.gktoday.in

203

Current Aairs: October 2016 [Date-wise Compendium]


[C] Simona Halep
[D] Venus Williams

Answer:Agnieszka Radwanska
Explanation:
Agnieszka Radwanska, a German professional tennis player, has won the 2016 China Open
Womens Singles Tennis tournament by defeating Johanna Konta in the nal by 64, 62 in
Beijing, China.

77. Which of the following personalities have won the 2016 Nobel Prize in Economics?
[A] Eugene Fama and Robert Shiller
[B] Angus Deaton and Jean Tirole
[C] lvin Roth and Lloyd Shapley
[D] Oliver Hart and Bengt Holmstrom

Answer:Oliver Hart and Bengt Holmstrom


Explanation:
UK-born Oliver Hart and Bengt Holmstrom of Finland have won the 2016 Nobel Prize in
Economics for their contributions to contract theory. Their theories are valuable to the
understanding of real-life contracts and institutions, as well as potential pitfalls in contract
design. They will share the 8m kronor ($924,000) prize; it was announced by the Royal Swedish
Academy of Sciences in Stockholm, Sweden.

78. Harika Dronavalli is associated with which sports?


[A] Badminton
[B] Table Tennis
[C] Chess
[D] Boxing

Answer:Chess
Explanation:
Harika Dronavalli, an Indian chess grandmaster, has recently defeated World No. 1 Hou Yifan of
China to be at joint 4th position in the 2016 Isle of Man (IoM) International Chess Tournament.

79. Which sportsperson has won the 2016 Ekalabya Award?


[A] Ajinkya Rahane
[B] PV Sindhu
[C] Mary Kom
[D] Srabani Nanda

Answer:Srabani Nanda
Explanation:
2016 GKToday | All Rights Reserved | www.gktoday.in

204

Current Aairs: October 2016 [Date-wise Compendium]

Srabani Nanda, an Indian ace sprinter, has won the 24th Ekalabya Award for the year 2016 in
recognition of her sporting excellence. Among her recent achievements, she won gold in 200m,
silver in 100m and 4x100m relay in 12th South Asian Games held in Guwahati in February 2016.
The Ekalabya award is instituted by Odisha-based Indian Metals Public Charitable Trust (IMPaCT).
It is the annual sports award for outstanding young sportsperson of Odisha which carries a cash
prize of Rs. 5 lakh.

80. Who has won 2016 China Open Mens Singles Tennis tournament?
[A] Andy Murray
[B] Novak Djokovic
[C] Rafael Nadal
[D] Grigor Dimitrov

Answer:Andy Murray
Explanation:
Andy Murray, a Scottish professional tennis player, has won his rst China Open Mens Singles
Tennis tournament and the 40th of his career by defeating Grigor Dimitrov of Bulgaria in the nal
by 6-4, 7-6 in Beijing, China.

81. What is the theme of 2016 International Day of the Girl Child (IDGC)?
[A] Ending Child Marriage
[B] Girls Progress = Goals Progress: What Counts for Girls
[C] The Power of the Adolescent Girl: Vision for 2030
[D] Innovating for Girls Education

Answer:Girls Progress = Goals Progress: What Counts for Girls


Explanation:
The International Day of the Girl Child (IDGC) is celebrated annually on October 11 to recognize
girls rights and the unique challenges girls face around the world. The 2016 theme is Girls
Progress = Goals Progress: What Counts for Girls.

82. Where are the headquarters of the International Energy Agency (IEA)?
[A] Paris
[B] Geneva
[C] London
[D] New York

Answer:Paris
Explanation:
The International Energy Agency (IAE) is a Paris-based autonomous intergovernmental
organization, which works to accelerate the global energy transition, providing research,
statistics, indepth analysis & policy recommendations. It was established in the framework of

2016 GKToday | All Rights Reserved | www.gktoday.in

205

Current Aairs: October 2016 [Date-wise Compendium]


the Organisation for Economic Co-operation and Development (OECD) in 1974 in the wake of the
1973 oil crisis. Recently, tt is in news because IAE will tie-up with India to implement its Unnat
Jyoti by Aordable LEDs for All (UJALA) initiative globally for energy savings. The UJALA scheme is
executed by the Energy Eciency Services Limited (EESL), a joint venture of PSUs under the
Union Ministry of Power.

83. Who is the newly elected Prime Minister of Morocco?


[A] Hamid Chabat
[B] Abbas El Fassi
[C] Abdelilah Bekirane
[D] Mustapha Ramid

Answer:Abdelilah Bekirane
Explanation:
Abdelilah Bekirane has been re-appointed as the Moroccos Prime minister for another term. It is
ocially announced by Mohammed VI, the king of Morocco, after Benkiranes PJD party emerged
with the most number of seats in the 395-seat lower house of parliament. Benkirane, who is the
leader of Islamic Justice and Development Party, has been serving as Prime Minister in a coalition
government since 2011.

84. How much amount has been donated by India to the Nelson Mandela Foundation to help it in its
welfare work for the people of South Africa?
[A] Rs. 8.4 million
[B] Rs. 9.3 million
[C] Rs. 10.0 million
[D] Rs. 11.9 million

Answer:Rs. 9.3 million


Explanation:
The Government of India (GoI) has donated Rs. 9.3 million (ZAR 2 million) to the Nelson Mandela
Foundation to help it in its welfare work for the people of South Africa. The Nelson Mandela
Foundation was established in 1999 after the then President Nelson Mandela stepped down as
the President of South Africa.

85. Which state has launched Duck Boats to boost tourist footfall?
[A] Kerala
[B] Maharashtra
[C] Goa
[D] Tamil Nadu

Answer:Goa
Explanation:
2016 GKToday | All Rights Reserved | www.gktoday.in

206

Current Aairs: October 2016 [Date-wise Compendium]

The Goa Tourism department has launched the vessels popularly known as Duck Boats to boost
tourist footfall. It is a rst of its kind facility used for civilian purpose (joy rides) in India. The fulledged operations of the joy rides will begin in November 2016. The vessels have been
assembled in collaboration with Advanced Amphibious Design Inc Honolulu USA using a
methodology and design which is cost eective, safe, durable and enjoyable.The maximum
capacity for each vehicle is 32 seats. The circuit tours will operate from Panjim to Old Goa and
Dr. Salim Ali bird sanctuary to showcase the natural and cultural treasures of Goa.

86. The International Dussehra festival has started in which state of India?
[A] Karnataka
[B] Rajasthan
[C] Kerala
[D] Himachal Pradesh

Answer:Himachal Pradesh
Explanation:
The International Dussehra festival has started at in the popular Dhalpur maidan in Kullu,
Himachal Pradesh. The 7-day festival will begin with Rath Yatra, followed by Nati dance, food,
music, rituals, folk avour of the festivity, etc. According to legend, in the 17th century, Raja
Jagat Singh installed an idol of Raghunath on his throne as a mark of penance. After this, god
Raghunath was declared as the ruling deity of the Valley. This deity is taken in a Ratha to the
Dussehra.

87. The Worlds first-ever Bionic Olympics has started in which country?
[A] Malaysia
[B] Indonesia
[C] China
[D] Switzerland

Answer:Switzerland
Explanation:
The Worlds rst-ever Cybathlon championship for disabled athletes has started at Zurich in
Switzerland. The championship is popularly known as Bionic Olympics, it diers from the
Paralympics as it sees participants compete against each other using the latest assistive
technologies such as robotic prostheses, brain-computer interfaces and powered exoskeletons. It
is organized by the Swiss Federal Institute of Technology ETH Zurich.

88. Aaron Pryor, who passed away recently, was related to which sports?
[A] Boxing
[B] Badminton
[C] Wrestling
[D] Swimming
2016 GKToday | All Rights Reserved | www.gktoday.in

207

Current Aairs: October 2016 [Date-wise Compendium]


Answer:Boxing
Explanation:
Aaron Pryor (60), an American boxer, has recently passed away in Cincinnati, Ohio. He was
known as the Hawk and was a crowd favorite who fought with a frenetic style.

89. Ruthvika Shivani is associated with which sports?


[A] Tennis
[B] Badminton
[C] Chess
[D] Hockey

Answer:Badminton
Explanation:
Indian shuttler Ruthvika Shivani Gadde has recently won the 2016 womens singles Russian
Open Badminton Grand Prix title after defeating Russias Evgeniya Kosetskaya in the nal by
21-10, 21-13 at Vladivostok, Russia.

90. The book The Greatest Bengali Stories Ever Told has been authored by whom?
[A] Urmila Das
[B] Vijay Kumar
[C] Amar Mitra
[D] Arunava Sinha

Answer:Arunava Sinha
Explanation:
The book The Greatest Bengali Stories Ever Told has been authored by Arunava Sinha. The
author has taken the works of prominent Bengali authors and done a very meaningful translation
of the book.

91. The first BRICS Trade Fair and Exhibition has started in which city of India?
[A] Kochi
[B] New Delhi
[C] Bhopal
[D] Indore

Answer:New Delhi
Explanation:
The rst BRICS Trade Fair and Exhibition has started at Pragati Maidan in New Delhi from October
12, 2016 to boost trade among the BRICS nations. The fair is a platform for BRICS countries viz.
Brazil, Russia, India, China and South Africa to exhibit state-of-the-art technologies and advances
made in industrial development. The theme of the trade fair is Building BRICS Innovation for
Collaboration. The focus of the 3-day fair is to showcase some of the best technologies available
2016 GKToday | All Rights Reserved | www.gktoday.in

208

Current Aairs: October 2016 [Date-wise Compendium]


across BRICS nations to address BRICS countries main development challenges. It will also focus
on Start-ups, to bring together young entrepreneurs from across BRICS nations on this platform.
There is no entry fee for the general public.

92. The 2nd Rashtriya Sanskriti Mahotsav (RSM)-2016 will be held in which city of India?
[A] Ahmedabad
[B] Mumbai
[C] Pune
[D] New Delhi

Answer:New Delhi
Explanation:
The second Rashtriya Sanskriti Mahotsav (RSM)-2016 will be held from October 15 to 23 in the
premises of Indira Gandhi National Centre for Arts (IGNCA), New Delhi. It will showcase the rich
cultural heritage of India in all its rich and varied dimensions, viz Handicrafts, Cuisine, Painting,
Sculpture, Photography, Documentation and Performing Arts-Folk, Tribal, Classical and
Contemporary- all in one place.

93. The book The Story of a New Name has been authored by whom?
[A] Nayanjot Lahiri
[B] Uma Mahadevan
[C] Elena Ferrante
[D] Jennifer Das

Answer:Elena Ferrante
Explanation:
The book The Story of a New Name has been authored by Elena Ferrante. It is about the
womens fraught relationship and shifting fortunes are the life forces.

94. The first-ever World Tsunami Awareness Day (WTAD) will be observed on which day?
[A] October 15
[B] October 21
[C] November 12
[D] November 5

Answer:November 5
Explanation:
The rst-ever World Tsunami Awareness Day (WTAD) will be observed on November 5, 2016 to
spread awareness among people across the world in matters related to the dangers of tsunami
and shall stress on the importance of early warning systems in order to mitigate damage from
the often devastating natural hazard. It also aims at reviving traditional knowledge about
tsunamis.

2016 GKToday | All Rights Reserved | www.gktoday.in

209

Current Aairs: October 2016 [Date-wise Compendium]


95. The 2016 SAARC Sufi festival will be held in which of the following countries?
[A] India
[B] Maldives
[C] Afghanistan
[D] Pakistan

Answer:India
Explanation:
The 3rd edition of the SAARC Su festival will be held at the Diggi palace in Jaipur from October
14-16, 2016. The event is being organized by SAARC Writers and Literature (FOSWAL). In it,
SAARC Su scholars, poets, Su musicians, singers and dancers and performers will participate
excluding Pakistan. The festival would be a melange of Su dance and music performances,
poetry, book readings, book launches and interactive sessions. The topics like Susm and its
inter religious links, Su-Bhakti harmony, promoting su tenets for world peace, teachings of
Susm and application in todays life, women in Susm, etc will be discussed.

96. Himansh Indias remote and high altitude research station has recently opened in which state of
India?
[A] Sikkim
[B] Jammu & Kashmir
[C] Uttarakhand
[D] Himachal Pradesh

Answer:Himachal Pradesh
Explanation:
Indias remote and high altitude research station Himansh has recently opened at a remote
region in Spiti, Himachal Pradesh above 13,500 ft (> 4000 m). It is established by National
Centre for Antarctic and Ocean Research (NCAOR), Goa, under the Ministry of Earth Sciences to
better study and quanties the Himalayan glacier responses towards the climate change. The
research station will provide the much needed llip to the scientic research on Himalayan
glaciers and its hydrological contribution.

97. What is the Indias rank in the 2016 Global Hunger Index (GHI)?
[A] 97th
[B] 88th
[C] 110th
[D] 65th

Answer:97th
Explanation:
India has been ranked 97th out of 118 countries in the 2016 Global Hunger Index (GHI). The
focus of the 2016 GHI is to present a multidimensional measure of national, regional, and global

2016 GKToday | All Rights Reserved | www.gktoday.in

210

Current Aairs: October 2016 [Date-wise Compendium]


hunger, focusing on how the world can get to Zero Hunger by 2030.The GHI is a
multidimensional statistical tool used to describe the state of countries hunger situation. It
describes Indias hunger situation as serious. The Index is released by the International Food
Policy Research Institute (IFPRI).

98. Indias first-ever convertible stadium is located in which of the following cities?
[A] Mumbai
[B] Ahmedabad
[C] Mohali
[D] Kolkata

Answer:Ahmedabad
Explanation:
Recently, the 2016 Kabaddi World Cup (KWC) has started on October 7 at the Arena By
TransStadia in Ahmedabad, which is Indias rst-ever convertible stadium. The Arena by
TransStadia uses a globally patented technology, which converts an outdoor stadium into an
indoor arena within 6 minutes at a touch of a button. The facility has a seating capacity of
20,000 and can house 14 sports, excluding cricket.

99. The book The Legend Of Lakshmi Prasad has been authored by which Bollywood icon?
[A] Shahrukh Khan
[B] Deepika Padukone
[C] Twinkle Khanna
[D] Dhanush

Answer:Twinkle Khanna
Explanation:
The book The Legend Of Lakshmi Prasad has been authored by Twinkle Khanna, which will be
launch on November 2016.

100. Andrzej Wajda, who passed away recently, was a famous film-director of which country?
[A] Australia
[B] Canada
[C] Poland
[D] Germany

Answer:Poland
Explanation:
Andrzej Wajda (90) was a renowned Polish lm and theatre director, who passed away recently
in Warsaw, Poland. He was an Academy Award-wining director and was known for the movies
such as The Promised Land, Man of Iron and Katyn.

101. The 2016 BRICS trade ministers has started in which city of India?
2016 GKToday | All Rights Reserved | www.gktoday.in

211

Current Aairs: October 2016 [Date-wise Compendium]


[A] Udaipur
[B] Indore
[C] New Delhi
[D] Lucknow

Answer:New Delhi
Explanation:
The 2016 BRICS trade ministers has started in New Delhi to discuss ways to further enhance
economic cooperation. During the meeting, leaders of Brazil, Russia, India, China and South
Africa will adopt the framework for co-operation dealing in small and medium-sized enterprises
(SMEs), services, intellectual property rights, trade promotion, non-tari measures resolution and
standardisation.

102. What is the theme of 2016 World Sight Day (WSD)?


[A] Universal Eye Health
[B] No more Avoidable Blindness
[C] Stronger Together
[D] Eye care for all

Answer:Stronger Together
Explanation:
The World Sight Day (WSD) is an annual day of awareness held on the 2nd Thursday of October,
to focus global attention on blindness and vision impairment. The 2016 WSD is on October 13
with theme is Stronger Together. On this day, Government of India (GoI) has set up free eye
check up and treatments facilities in the designated primary health centres, community health
centres, medical colleges, regional institutes of ophthalmology and selected NGOs all over the
country.

103. What is the theme of 2016 International Day for Disaster Reduction (IDDR)?
[A] My City is Getting Ready!
[B] Live to Tell
[C] Living with Disability and Disasters
[D] Resilience is for Life

Answer:Live to Tell
Explanation:
The International Day for Disaster Reduction (IDDR) is observed every year on October 13 to
encourage every individual, community and government to take part in building more disaster
resilient communities and nations. It is a day to promote a global culture of risk-awareness and
disaster reduction. The 2016 theme is Live to Tell: Raising Awareness, Reducing Mortality,
which targets to reduce mortality globally.

104. Where are the headquarters of the United Nations Office for Disaster Risk Reduction
2016 GKToday | All Rights Reserved | www.gktoday.in

212

Current Aairs: October 2016 [Date-wise Compendium]


(UNISDR)?
[A] Paris
[B] Geneva
[C] New York
[D] Berlin

Answer:Geneva
Explanation:
The United Nations Oce for Disaster Risk Reduction (UNISDR) is part of the UN Secretariat and
its functions span the social, economic, environmental and humanitarian elds. Its headquarters
is at Geneva, Switzerland. It is in news recently because on 2016 International Day for Disaster
Reduction (IDDR), UNISDR has launched the new Sendai Seven campaign- 7 Targets, 7 Years,
an advocacy initiative to encourage implementation of the Sendai Framework for Disaster Risk
Reduction with the goal of saving lives, reducing disaster losses and improving management of
disaster risk.

105. Indian Railways has recently signed pact with which country for high-speed trains?
[A] France
[B] Italy
[C] Japan
[D] Germany

Answer:Germany
Explanation:
Indian Railways has recently signed pact with Germany for running high-speed trains in India.
The pact will deepen the collaboration and strengthen ties between the two countries for
bilateral cooperation in the rail sector.

106. Dario Fo, who passed away recently, was the Nobel prize winning playwright and actor of
which country?
[A] France
[B] Italy
[C] Germany
[D] United Kingdom

Answer:Italy
Explanation:
Dario Fo (90), the Nobel prize winning Italian playwright and actor, has recently passed away at
Milan, Italy. He was best known for his cutting political satire in plays such as Accidental Death
of an Anarchist and Cant Pay, Wont Pay.

107. The book Jihad in My Saffron Garden has been authored by whom?
2016 GKToday | All Rights Reserved | www.gktoday.in

213

Current Aairs: October 2016 [Date-wise Compendium]


[A] Anurag Kashyap
[B] Roxy Arora
[C] Kasturi Chauhan
[D] Bandita Ghosh

Answer:Roxy Arora
Explanation:
The book Jihad in My Saron Garden has been authored by Dr. Roxy Arora. It is the story of a
fearless woman who crusades to restore Kashmiriyat in a region which has been described as
paradise on earth. At the same time the tale contains the message of world peace and religious
tolerance.

108. The 2016 Nobel Literature Prize has been won by whom?
[A] Patrick Modiano
[B] William Faulkner
[C] Bob Dylan
[D] John Steinbeck

Answer:Bob Dylan
Explanation:
The 2016 Nobel Prize in literature has been awarded to US musician and poet Bob Dylan for
having created new poetic expressions within the great American song tradition. Dylan is the
rst American to receive the award since 1993. The prize of 8 million Swedish kronor will be
awarded to him. The literature prize is given for a lifetime of writing rather than for a single work.

109. Who has been appointed as new DG of Directorate General of Foreign Trade (DGFT)?
[A] Niliesh Mishra
[B] Ajay Kumar Bhalla
[C] Anup Wadhawan
[D] Jit Singh

Answer:Ajay Kumar Bhalla


Explanation:
Ajay Kumar Bhalla, a 1984 batch IAS ocer of Assam-Meghalaya cadre, has been appointed as
new Director General of Directorate General of Foreign Trade (DGFT). Presently, he is the
Additional Secretary in Commerce Ministry. He will succeed Anup Wadhawan, who has been
appointed as Additional Secretary in the Commerce Ministry. The DGFT organisation is an
attached oce of the Ministry of Commerce and Industry with headquarters at New Delhi. It is
entrusted with responsibilities for formulating and implementing the foreign trade policy with the
main objective of promoting Indias exports.

110. Bhumibol Adulyadej, who passed away recently, was the king of which country?
2016 GKToday | All Rights Reserved | www.gktoday.in

214

Current Aairs: October 2016 [Date-wise Compendium]


[A] Malaysia
[B] Thailand
[C] Indonesia
[D] Maldives

Answer:Thailand
Explanation:
King Bhumibol Adulyadej (88) of Thailand has recently passed away in Bangkok. He was the
worlds longest-reigning monarch and was seen as a stabilizing gure in a country hit by cycles
of political turmoil and multiple coups. Crown Prince Maha Vajiralongkorn will be the new
monarch.

111. Who has been elected as the new Secretary General of the United Nations?
[A] Anibal Silva
[B] Vitaly Antonio
[C] Antonio Guterres
[D] Matthew Rycroft

Answer:Antonio Guterres
Explanation:
Portugals ex-Prime Minister, Antonio Guterres has been ocially appointed as the 9th UN
secretary-general by the United Nations General Assembly (UNGA). For this, the 193 member
states of the UNGA adopted a resolution by acclamation to appoint Guterres. He will become the
worlds top diplomat on 1st January 2017 when Ban Ki-moons second ve-year term ends on
December 31, 2016.

112. The 2016 Women of India Festival has started in which of the following cities?
[A] New Delhi
[B] Jaipur
[C] Gandhinagar
[D] Allahabad

Answer:New Delhi
Explanation:
Maneka Gandhi, the Union Minister of Women and Child Development, has launched the 2016
Women of India Festival at Dilli Haat in New Delhi. The 10-day festival will oer the biggest
selection of organic products prepared by women. The festival promotes women entrepreneurs
from across India, which is organized and sponsored by the Union Ministry of Women and Child
Development. This years festival will have 230 stalls and about 372 participants. The list
includes producers from Delhi, NCR and 23 states and Union Territories

113. What is the theme of 2016 World Standards Day (WSD)?


2016 GKToday | All Rights Reserved | www.gktoday.in

215

Current Aairs: October 2016 [Date-wise Compendium]


[A] International Standards Creating confidence globally
[B] Standards level the playing field
[C] Standards Build Trust
[D] Standards the worlds common language

Answer:Standards Build Trust


Explanation:
The World Standards Day (WSD) is observed every year on October 14 to pay tribute to the
collaborative eorts of the thousands of experts worldwide that develop the voluntary technical
agreements that are published as international standards. The 2016 theme is Standards Build
Trust.

114. The National Seismic Programme (NSP) has been launched in which state of India?
[A] Arunachal Pradesh
[B] Sikkim
[C] Assam
[D] Odisha

Answer:Odisha
Explanation:
The National Seismic Programme (NSP) has been launched by Union Minister of State for
Petroleum and Natural Gas Dharmendra Pradhan in Odisha to trace hydrocarbon resources like
oil and natural gas in Mahanadi basin.The purpose of NSP is to undertake a fresh appraisal in all
sedimentary basins across India, especially where no or scanty data is available, for a better
understanding of the hydrocarbon potential.The programme was launched at Taranga village
under Soro block in Balasore district. Under this programme, Oil and Natural Gas Corporation
(ONGC) and Oil India Limited (OIL) have been entrusted to conduct 2D seismic Acquisition,
Processing and Interpretation (API) across India. The survey project will be completed by March
2019.

115. Shahlyla Ahmadzai Baluch, who passed away recently, was a football player of which country?
[A] Pakistan
[B] Afghanistan
[C] India
[D] Iran

Answer:Pakistan
Explanation:
Shahlyla Ahmadzai Baluch (21), one of the nest strikers for Pakistan womens football team, has
recently died in a car accident in Karachi. Ahmadzai represented Pakistan at the South Asian
Football Federation (SAFF) Women Football Championship 2014 in Islamabad, which was the last
international event that Pakistan womens team competed in. She also had the honour of being
the rst Pakistani woman footballer to get a hat-trick abroad when she played for a club in
2016 GKToday | All Rights Reserved | www.gktoday.in

216

Current Aairs: October 2016 [Date-wise Compendium]


Maldives in 2015.

116. Who of the following has been selected for the Sri Maharshi Valimiki Jayanti Award for
2016-17?
[A] D Borappa
[B] H Anjaneya
[C] Veeranna
[D] S G Siddaramaiah

Answer:Veeranna
Explanation:
Former Minister Veeranna has been selected for the Sri Maharshi Valimiki Jayanti Award for the
year 2016-17, which is instituted by the Karnataka government. The award carries a cash prize
of Rs. 5 lakh and a citation, which will be presented to Veeranna by Chief Minister Siddaramaiah
at Banquet Hall of the Vidhana Soudha on October 15, 2016. Veeranna hails from Koratagere in
Tumakuru district and served as the Minister in the Janata Party government headed by late
Chief Minister Ramakrishna Hegde. Due to his eorts, many communities such as Nayak,
Nayaka, Beda, Bedar and Valmiki have been included in the Scheduled Tribe community.

117. Who has been appointed as Whole-Time Member (WTM) of the Securities and Exchange
Board of India (SEBI)?
[A] Pradeep Kumar
[B] G Mahalingam
[C] Jivan Kher
[D] Prashant Saran

Answer:G Mahalingam
Explanation:
Gurumoorthy Mahalingam has been appointed as Whole-Time Member (WTM) of the Securities
and Exchange Board of India (SEBI) for a period of 5 years or till he attains the age of 65 years,
whichever is earlier. Apart from the Chairman, the SEBI can have 3 WTMs. He succeeded
Prashant Saran, who retired as WTM in 2016. Mahalingam is Executive Director and former
regional director of Reserve Bank of India (RBI).

118. Which former chief minister has been awarded by Ugandan government for his literary work?
[A] N. Rangaswamy
[B] Oommen Chandy
[C] Tarun Gogoi
[D] Ramesh Pokhriyal

Answer:Ramesh Pokhriyal
Explanation:
2016 GKToday | All Rights Reserved | www.gktoday.in

217

Current Aairs: October 2016 [Date-wise Compendium]

Dr. Ramesh Pokhriyal Nishank, the former Uttarakhand chief minister, has been conferred with
the prestigious award by the Ugandan Prime Minister Ruhakana Rugunda for promoting human
values through his literature. Currently, Nishank is the Member of Parliament (BJP) in 16th Lok
Sabha from Haridwar Constituency of Uttarakhand.

119. Which Indian personality has been awarded for Excellence in Technical Innovation by the
International Society of Automation (ISA)?
[A] Satish Pathak
[B] Pankaj Ahuja
[C] Naveen Jain
[D] Milind Sharma

Answer:Satish Pathak
Explanation:
Satish Pathak, a Pune-based innovator, has been awarded for Excellence in Technical
Innovation by the International Society of Automation (ISA). He got the award for his innovation
Digital Controlled Valve for Precise Liquid Dispensing by Gravity ow, which has the capacity of
saving for the industry and is a step change technology in industrial valves. ISA is a non-prot
technical society for engineers, technicians, business people, educators and students.

120. The first-ever BRICS-BIMSTEC Outreach Summit will be held in which state of India?
[A] Goa
[B] Uttar Pradesh
[C] Maharashtra
[D] Rajasthan

Answer:Goa
Explanation:
India is hosting the 8th BRICS Summit, followed by rst-ever BRICS-BIMSTEC Outreach Summit in
Goa on October 15, 2016. India is facilitating an outreach Summit with BIMSTEC leaders of
Bangladesh, Bhutan, Myanmar, Nepal, Sri Lanka and Thailand, which will strengthen Indias
bilateral relations with its partners.

121. Which committee has submitted its report on existence of Saraswati River to the Union
Government?
[A] M L Malaya committee
[B] Bhagya Shree committee
[C] N N Vohra committee
[D] K S Valdiya committee

Answer:K S Valdiya committee


Explanation:
2016 GKToday | All Rights Reserved | www.gktoday.in

218

Current Aairs: October 2016 [Date-wise Compendium]

The expert committee headed by Prof. KS Valdiya has submitted its report on existence of
Saraswati River to the Union government. As per the study, Saraswati River was not mythical
and it existed. The report is an assertion of the assumption that River Saraswati originated from
Adibadri in Himalaya to culminate in the Arabian Sea through the Runn of Kutch.

122. The first-ever BRICS U-17 football tournament has won by which country?
[A] India
[B] South Africa
[C] Brazil
[D] China

Answer:Brazil
Explanation:
Brazil has won the rst-ever BRICS U-17 football tournament by defeating South Africa in the
nal by 5-1 demolition of in the title clash in Margao, Goa.

123. Neeraj Goyat is associated with which sports?


[A] Wrestling
[B] Boxing
[C] Chess
[D] Cricket

Answer:Boxing
Explanation:
Indian Boxer Neeraj Goyat has recently won the 2016 WBC Asia welterweight title by defeating
Australian Ben Kite in New Delhi. It is his 2nd pro title. Earlier, he won the belt in 2015 by
defeating Filipino Nelson Gulpe. With this win, Neeraj has become the rst Indian boxer to defend
a pro title. The ght was organized by the Professional Boxing Organisation of India.

124. What is the theme of 2016 United Nations (UN) International Day of Rural Women?
[A] Climate is changing: Food and agriculture must too
[B] Realizing womens rights to land and other productive resources
[C] Prevention of all forms of violence against women and girls
[D] Promote women leadership in agriculture

Answer:Climate is changing: Food and agriculture must too


Explanation:
The United Nations (UN) International Day of Rural Women is observed every year on October
15 to recognize rural womens importance in increasing agricultural and rural development
worldwide. The 2016 theme is Climate is changing. Food and agriculture must too, which tries
to bring focus on farmers, shers and pastoralists who are standing on the frontlines of food
insecurity as temperatures rise, weather patterns become less predictable and climate-related

2016 GKToday | All Rights Reserved | www.gktoday.in

219

Current Aairs: October 2016 [Date-wise Compendium]


disasters become more frequent.

125. The World Student Day (WSD) is observed on which date?


[A] October 16
[B] October 17
[C] October 15
[D] October 14

Answer:October 15
Explanation:
The World Students Day is observed every year on October 15 to mark the importance of Indias
former President and great scientist Dr. APJ Abdul Kalam. It is a celebration of multiculturalism,
diversity and cooperation among students across the globe.

126. Shaurya Smarak war memorial has been launched in which city of India?
[A] Jaipur
[B] Bhopal
[C] Kanpur
[D] Mohali

Answer:Bhopal
Explanation:
Prime Minister Narendra Modi has recently inaugurated Shaurya Smarak a war memorial in
Bhopal, Madhya Pradesh, which is dedicated to the soldiers of India. The Shourya Smarak is built
on an area of about 8,000 square metre and the tales of their bravery are on display at this
memorial. It has been envisaged as a journey through the various experiences of life, war, death
and liberation of spirit (victory over death), all exhibited through an extremely non-archetypal
design.

127. Rashtriya Sanskriti Mahotsav-2016 has been started in which of the following cities?
[A] New Delhi
[B] Pune
[C] Udaipur
[D] Lucknow

Answer:New Delhi
Explanation:
The 2nd edition of Rashtriya Sanskriti Mahotsav (RSM)-2016 has started at the Indira Gandhi
National Centre for Arts (IGNCA), New Delhi on October 15th to 23rd, 2016. It will showcase the
rich cultural heritage of India in all its rich and varied dimensions viz. handicrafts, cuisine,
painting, sculpture, photography, documentation and performing arts-folk, tribal, classical and
contemporary- all in one place.

2016 GKToday | All Rights Reserved | www.gktoday.in

220

Current Aairs: October 2016 [Date-wise Compendium]


128. The 2016 India International Knit Fair will be held in which of the following states?
[A] Kerala
[B] Gujarat
[C] Assam
[D] Tamil Nadu

Answer:Tamil Nadu
Explanation:
The 43rd edition of India International Knit Fair 2016 will be held at India Knit Fair Complex in
Tirupur, Tamil Nadu on October 19-21. The 3-day expo will showcase Autumn/Winter and Mid
Summer collection 2017, with a complete display of latest fashion wear. The buyers from South
America, Israel, Australia, Dubai, Brazil, Spain, Italy, UAE, Japan, Kong Kong, UK, South Africa,
Slovakia and Singapore will visit the fair.

129. Which Indian-origin scientist has won the International Association Of Food Protection (IAFP)
Young Scientist scholarship award?
[A] Shalini Jaiswal
[B] Kirtiraj Kundlik Gaikwad
[C] Veer Chand Rathore
[D] Sunil Singhvi

Answer:Kirtiraj Kundlik Gaikwad


Explanation:
Kirtiraj Kundlik Gaikwad, an Indian-origin scientist, has won the International Association Of Food
Protection (IAFP) Young scientist scholarship award at St. Louise, USA for his research work on
development of novel oxygen absorbing package in the eld of active packaging for oxygen
sensitive food products. The award carries a $2000 and plaque.The purpose of IAFP is to
encourage young outstanding food safety scientist from all over the world.

130. The book Half of What I Say has been authored by whom?
[A] Anil Menon
[B] Suneetha Balakrishnan
[C] Anamika Kumari
[D] Kiran Doshi

Answer:Anil Menon
Explanation:
The book Half of What I Say has been authored by Anil Menon. It is a story about the place of
ction and truth in the midst of personal, political and social upheaval.

131. Who has won the 2016 Chinese Taipei Masters Mens singles badminton tournament?
[A] Sourabh Verma
2016 GKToday | All Rights Reserved | www.gktoday.in

221

Current Aairs: October 2016 [Date-wise Compendium]


[B] P C Thulasi
[C] Anand Yelegar
[D] Sameer Verma

Answer:Sourabh Verma
Explanation:
Indian shuttler Sourbah Verma has clinched the 2016 Mens singles Chinese Taipei Masters
Badminton Grand Prix tournament in by defeating Malaysias Daren Liew in the nal by 1210,
1210, 33 at Taipei, China.

132. What is the theme of 2016 World Food Day (WFD)?


[A] Social protection and Agriculture
[B] Feeding the world, caring for the earth
[C] Climate is changing. Food and agriculture must too
[D] Family Farming: Breaking the Cycle of Rural Poverty

Answer:Climate is changing. Food and agriculture must too


Explanation:
The World Food Day (WFD) is observed every year on October 16 to mark the foundation of Food
and Agriculture Organisation (FAO) of the United Nations in 1945. The global message for WFD
2016 is Climate is changing. Food and agriculture must too.

133. Kigeli V Ndahindurwa, who passed away recently, was the king of which country?
[A] Kenya
[B] Rwanda
[C] Egypt
[D] South Africa

Answer:Rwanda
Explanation:
Kigeli V Ndahindurwa (80), the last king of Rwanda, has recently passed away in United States.
He ruled the east African nation for less than a year in 1959 before being forced into exile after
clashing with the Belgian colonial power. In 1961, the monarchy was abolished. The king began a
life of exile in east Africa, including Kenya and Uganda, before leaving for America, where he
settled in the suburbs of Washington in 1992.

134. Shenzhou-11 a manned spacecraft has been successfully launched by which country?
[A] North Korea
[B] Japan
[C] Russia
[D] China
2016 GKToday | All Rights Reserved | www.gktoday.in

222

Current Aairs: October 2016 [Date-wise Compendium]


Answer:China
Explanation:
China has successfully launched its longest-ever manned space mission Shenzhou-11
spacecraft into space by a Long March-2F carrier rocket from the Jiuquan Satellite Launch Centre
near the Gobi Desert, China. The two astronauts will stay in space for 30 days to test complexs
ability to support human life. They will also conduct medical and scientic experiments. The
purpose of the mission is to dock with the Tiangong-2 space laboratory and gain experience from
a 30-day residence and to test its life-support systems.

135. The International Day for the Eradication of Poverty is observed on which date?
[A] October 15
[B] October 18
[C] October 19
[D] October 17

Answer:October 17
Explanation:
The International Day for the Eradication of Poverty is observed every year on 17 October for
eradication of poverty throughout the world. The 2016 theme is Moving from humiliation and
exclusion to participation: Ending poverty in all its forms.

136. Which bollywood actress has been bestowed with the 2016 ET Panache Trendsetter Award?
[A] Deepika Padukone
[B] Aishwarya Rai
[C] Sonam Kapoor
[D] Priyanka Chopra

Answer:Sonam Kapoor
Explanation:
Bollywood Actress Sonam Kapoor has been bestowed with the 2016 ET Panache Trendsetter
Award in Mumbai, Maharashtra. Beside her, Vijay Shekhar Sharma, Kavin Bharti Mittal, Bhavin
and Divyank Turakhia, Dipa Karmakar, Devendra Jhajharia, Ananya Birla, etc., has been
honoured with the award. The award recognizes the eorts of entrepreneurs, innovators,
newsmakers and athletes whose stories inspire millions and whose services transform lives. The
winners of each are harbingers of change, driven by professional passion, personal elan and a
commitment to quality.

137. The 2017 Indian Science Congress (ISC) will be held in which of the following states?
[A] Odisha
[B] Karnataka
[C] Kerala
[D] Andhra Pradesh
2016 GKToday | All Rights Reserved | www.gktoday.in

223

Current Aairs: October 2016 [Date-wise Compendium]


Answer:Andhra Pradesh
Explanation:
The 104th edition of Indian Science Congress (ISC) 2017 will be held at the Sri Venkateswara
University in Tirupati, Andhra Pradesh from January 3 to 7, 2017. In the 2017 ISC, 9 Nobel
Laureates from the United States, Japan, France, Israel and Bangladesh would attend the event
and share their experiences. In addition to this, a huge contingent of 200 scientists from foreign
nations, 10,000 scientists representing various national laboratories, faculty and research
scholars from Indian universities and several others will take part in the event to exchange their
views on a range of scientic issues, both on national and international level.

138. The India International Silk Fair (IISF) 2016 has been started in which city of India?
[A] New Delhi
[B] Pune
[C] Jaipur
[D] Lucknow

Answer:New Delhi
Explanation:
The 5th edition India International Silk Fair (IISF) 2016 has been inaugurated by the Union
Textiles Minister SmritiI rani at Pragati Maidan in New Delhi on October 15, 2016. It is the only
fair in the world professionally organized to facilitate International buyers to source exclusive
pure silk and silk blended fabrics, garments, made-ups, interior furnishing, fashion accessories,
shawls, scarves and oor coverings and will be showcasing the latest products of nearly 150
premier Export Houses of India.

139. Farm Tourism scheme has been started by which state government of India?
[A] Rajasthan
[B] Punjab
[C] Uttar Pradesh
[D] Haryana

Answer:Haryana
Explanation:
The Haryana government has recently started a Farm Tourism scheme to boost tourism sector
in the state. In it, registered farm owners can host tourists as part of holiday packages. This
scheme has been started to attract people to villages. Holiday packages will be oered to
tourists at registered farms near Delhi. The state government is also focusing on developing
ancient cultural heritage, historical and famous pilgrimage sites as tourism spots to aware
people about the history and culture of Haryana. It has also given the industry status to the
tourism sector.

140. Who has been conferred with the V Krishnamurthy Award for excellence by Centre for
Organisation Development (COD)?
2016 GKToday | All Rights Reserved | www.gktoday.in

224

Current Aairs: October 2016 [Date-wise Compendium]


[A] V N Srivastava
[B] R A Mashelkar
[C] Vinod Karnam
[D] Umeshwar Pandey

Answer:R A Mashelkar
Explanation:
R. A. Mashelkar, the former Director-General of CSIR, has been conferred with the V
Krishnamurthy Award for excellence by the Hyderabad-based Centre for Organisation
Development. The award was instituted in 2000 to recognize excellence in management.

141. How much fund will be provided by World Bank to Bangladesh to fight child malnutrition?
[A] $5 billion
[B] $1 billion
[C] $4 billion
[D] $2 billion

Answer:$1 billion
Explanation:
The World Bank will provide US $1 billion aid to Bangladesh over the next three years to ght
child malnutrition. The fund will support programmes to ensure sucient child nutrition and
include a conditional cash transfer scheme targeted at six lakh poor families in 43 sub-districts.

142. Which Indian sportsperson has been appointed as a member of the International Olympic
Committees (IOC) Athletes Commission?
[A] Devendra Jhajharia
[B] Saina Nehwal
[C] Abhinav Bindra
[D] Sakshi Malik

Answer:Saina Nehwal
Explanation:
Indian shuttler Saina Nehwal has been appointed as a member of the International Olympic
Committees (IOC) Athletes Commission. The Athletes Commission is chaired by Angela
Ruggiero and it comprises nine vice presidents and 10 other members. Sainas appointment to
the prestigious panel has come just when she is set to resume practice after recovering from a
knee injury.

143. The National SC/ST hub and zero effect on environment scheme for MSME sector has been
launched in which of the following cities?
[A] Rohtak
2016 GKToday | All Rights Reserved | www.gktoday.in

225

Current Aairs: October 2016 [Date-wise Compendium]


[B] Ludhiana
[C] Shimla
[D] Jabalpur

Answer:Ludhiana
Explanation:
Prime Minister Narendra Modi has launched the national SC/ST hub and zero eect on
environment scheme for MSME sector in Ludhiana, Punjab on October 18, 2016. This hub has
been set up under Prime Ministers Employment Generation scheme to provide support to
entrepreneurs from the SC/ST community. He will also present national MSME awards to 225
industrialists and will also distribute 150 spinning wheels to the women of weaker sections to
make them able to earn their livelihood.

144. The first National Tribal Carnival will be launch in which of the following cities?
[A] Agartala
[B] Guwahati
[C] New Delhi
[D] Indore

Answer:New Delhi
Explanation:
Prime Minister Narendra Modi will launch rst National Tribal Carnival in Delhi on October 25,
2016. The main purpose of this carnival is to promote a sense of inclusiveness amongst the
tribals. The carnival will showcase and promote various facets of tribal culture on a large scale.
Approx 1600 tribal artists and around 15000 tribal delegates from across the country are
expected to take part in the carnival.

145. Who has been appointed as the Ambassador of India to the United Nations Conference on
Disarmament (CD)?
[A] Ambuj Sharma
[B] Amandeep Singh Gill
[C] Vinod K Dasari
[D] T Venkataraman

Answer:Amandeep Singh Gill


Explanation:
Amandeep Singh Gill, an Indian Foreign Service (IFS) ocer of 1992 batch, has been appointed
as the Ambassador/PR of India to the United Nations Conference on Disarmament (CD) in
Geneva, Switzerland.

146. The first Made in India electric bus has been launched by which Indian company?
[A] Tata Motors
2016 GKToday | All Rights Reserved | www.gktoday.in

226

Current Aairs: October 2016 [Date-wise Compendium]


[B] Mahindra & Mahindra
[C] Ashok Leyland
[D] Fiat India Automobiles Limited

Answer:Ashok Leyland
Explanation:
Ashok Leyland Ltd has launched Indias rst made-in-India electric bus Circuit in Chennai, Tamil
Nadu. The Circuit series vehicles are all exclusively engineered on India-specic platforms that
can tackle varied topography, gradients and usage conditions. The bus is a zero-emission, nonpolluting vehicle, created specically for Indian road and passenger conditions. It is built on a
simple, mass-market platform that will enable the operator to cater to customers in city centres
with minimal operational and maintenance costs. These vehicles are integrated with a re
detection and suppression system (FDSS) and can travel 120 kilometers on a single charge.

147. Who has been selected for the 2015 Mathrubhumi Literary award?
[A] Pala Narayanan Nair
[B] Sukumar Azhikode
[C] C Radhakrishnan
[D] M T Vasudevan Nair

Answer:C Radhakrishnan
Explanation:
C Radhakrishnan, the well-known writer, has been selected for the Mathrubhumi Literary Award
in recognition of his contribution to the eld of literature.The award carries a cash prize of Rs. 2
lakh, a citation and a statuette.

148. Which bank has become the first Indian lender to execute banking transaction via blockchain
technology?
[A] State Bank of India
[B] HDFC bank
[C] ICICI bank
[D] Axis bank

Answer:ICICI bank
Explanation:
ICICI Bank has become the rst Indian lender to complete a banking transaction using blockchain
technology. This remittance transaction has been completed in partnership with Emirates NBD, a
lender from the West Asia. Blockchain is a digital ledger which keeps real-time record of each
and every transaction and is shared among a distributed network of computers, more like a
shared spreadsheet.

149. The book Jinnah Often Came To Our House has been authored by whom?
2016 GKToday | All Rights Reserved | www.gktoday.in

227

Current Aairs: October 2016 [Date-wise Compendium]


[A] Niranjan Kumar
[B] Chitra Divakaruni
[C] Kiran Doshi
[D] Amish tripathi

Answer:Kiran Doshi
Explanation:
The book Jinnah Often Came To Our House has been authored by Kiran Doshi and is shortlisted
for The Hindu Prize 2016. It provides a glimpse on the personal life of Muhammad Ali Jinnah, the
founder of Pakistan.

150. The National Summit on Fortification of Food has been held in which city?
[A] New Delhi
[B] Udaipur
[C] Bhopal
[D] Raipur

Answer:New Delhi
Explanation:
The 2-day National Summit on Fortication of Food has been held in New Delhi to address the
issue of micronutrient malnutrition. The summit was launched by Anupriya Patel, Minister of
State for Health and Family Welfare, and was conducted by the Food Safety and Standards
Authority of India (FSSAI).

151. The first-ever International Conference on Voter Education has started in which country?
[A] India
[B] United States
[C] United Kingdom
[D] Bhutan

Answer:India
Explanation:
The rst-ever International Conference on Voter Education has started in New Delhi from October
20, 2016 and is organised by The Election Commission of India (ECI). The topic of the Conference
is Voter Education for Inclusive, Informed and Ethical Participation. The 3-day Conference was
inaugurated by the Chief Election Commissioner Dr.Nasim Zaidi. The purpose of the conference
is to share best practices, policies and initiatives of voter education by election management
bodies. The 27 countries and 5 international organizations are participating in the event. The
Conference also aims to achieve a resolution to strengthen inclusive, informed and ethical
electoral participation among member countries with the help of voter education and outreach.

152. Mewa Ramgobin, who passed away recently, was well-known Gandhian activist of which
2016 GKToday | All Rights Reserved | www.gktoday.in

228

Current Aairs: October 2016 [Date-wise Compendium]


country?
[A] Bhutan
[B] South Africa
[C] Nepal
[D] Bangladesh

Answer:South Africa
Explanation:
Mewa Ramgobin (83), the South African struggle stalwart and former African National Congress
(ANC) Member of Parliament, has recently passed away in Cape Town, South Africa. He was the
husband of Mahatma Gandhis granddaughter Ela Gandhi, a human rights activist. Ramgobin was
the founder of Centre for Learning of Ubuntu and Chairperson of Phoenix Settlement Trust
founded by Mahatma Gandhi in 1904.

153. The book The Adivasi Will Not Dance has been authored by whom?
[A] EV Ramakrishnan
[B] Pradeep Sebastian
[C] Hansda Sowendra Shekhar
[D] Manu Chakravarthy

Answer:Hansda Sowendra Shekhar


Explanation:
The book The Adivasi Will Not Dance has been authored by Hansda Sowendra Shekhar, which
is shortlist for The Hindu Prize 2016. It is about a visit to Jharkhand by the President of India to
inaugurate a thermal power station that is coming up on land from which Santhals have been
evicted.

154. Which Water Jet Fast Attack Craft (WJFAC) Car Nicobar Class vessel has recently
commissioned into the Indian Navy?
[A] INS Tarmugli
[B] INS Kalpeni
[C] INS Kabra
[D] INS Tahiyu

Answer:INS Tahiyu
Explanation:
The 315-tonne INS Tahiyu has been commissioned into the Indian Navy and is allotted to the
Eastern Fleet of the Navy. INS Tahiyu is the 6th Water Jet Fast Attack Craft (WJFAC) Car Nicobar
Class vessel, which is built by Kolkata-based shipyard Garden Reach Shipbuilders and Engineers
(GRSE). It is an improved version of the earlier vessels and it can achieve a top operating speed
of 35 knots/ hour.The ship will be commanded by Commander Ajay Kashov and will have four

2016 GKToday | All Rights Reserved | www.gktoday.in

229

Current Aairs: October 2016 [Date-wise Compendium]


ocers and 41 crew members.

155. The 28th edition of Accountants General Conference has started in which city of India?
[A] Kolkata
[B] New Delhi
[C] Chennai
[D] Bengaluru

Answer:New Delhi
Explanation:
The 28th edition of Accountants General Conference has inaugurated by President Pranab
Mukherjee started in New Delhi. In the 2-day conference, accountants general from various
places of India will participate and will review the international best practices prevalent in audit
and accounting. The conference is organized by the Comptroller and Auditor General of India
(CAG).

156. The 2nd joint tactical exercise Sino-India Cooperation 2016 has been held in which city?
[A] Jaisalmer
[B] Ladakh
[C] Kutch
[D] Udaipur

Answer:Ladakh
Explanation:
The 2nd joint tactical exercise Sino-India Cooperation 2016 has been held between India and
China in the area of Border Personnel Meeting Hut at Chushul Garrison of Eastern Ladakh in
Jammu and Kashmir (J&K). During the day long exercise on Humanitarian Aid and Disaster Relief
(HADR) a ctitious situation of earthquake striking an Indian Border village was painted.
Afterwards, joint teams carried out rescue operations, evacuation and rendering of medical
assistance. The Indian team for the exercise was led by Brigadier R S Raman and that of the
Chinese was led by Sr Col Fan Jun.

157. Which city has topped the 2016 Global Power City Index (GPCI) ?
[A] Tokyo
[B] Rio de Janeiro
[C] London
[D] Mumbai

Answer:London
Explanation:
London has topped the 2016 Global Power City Index, as per 2016 GPCI report, which was
recently released by the Japan-based Mori Memorial Foundations Institute for Urban Strategies.
The report ranks 42 major cities of the world across six key measures of economy, research and

2016 GKToday | All Rights Reserved | www.gktoday.in

230

Current Aairs: October 2016 [Date-wise Compendium]


development, culture, accessibility, environment and livability. Mumbai, the only Indian city,
ranked 39th on the list of the worlds leading global cities for business, innovation and culture.

158. Which city has become the first Indian city to be officially declared as one of the venues for
hosting the 2017 FIFA U-17 World Cup?
[A] Mohali
[B] Kochi
[C] Chennai
[D] Kolkata

Answer:Kochi
Explanation:
Kochi has become the rst Indian city to be ocially declared as one of the venues for hosting
the 2017 FIFA U-17 World Cup. A 23-member high-level delegation consists of FIFA experts
ratied Kochi as venue after visiting Jawarharlal Nehru International Stadium. This will be the rst
time India will host an international football competition at worldwide level.

159. The book Driven: The Virat Kohli Story has been authored by whom?
[A] Sandeep Bamzai
[B] Rajkumar Sharma
[C] Vijay Lokapally
[D] Pradeep Sangwan

Answer:Vijay Lokapally
Explanation:
The book Driven: The Virat Kohli story has been written by well-known journalist Vijay
Lokapally, which is recently launch in New Delhi. It is on the life of the Indian Test captain Virat
Kohli.

160. Who has been awarded with the first International Prize in Statistics?
[A] Ron Wasserstein
[B] William Johnson
[C] David Cox
[D] Susan Ellenberg

Answer:David Cox
Explanation:
Sir David Cox has been awarded with the rst International Prize in Statistics, which will be
presented to him in July 2017 at the World Statistics Congress in Marrakech. The award carries
$75,000 and is considered the highest honor in its eld. It recognizes an individual or team for
major achievements using statistics to advance science, technology and human welfare. It also
helps to raise media and public awareness of the growing importance of statistics to modern life

2016 GKToday | All Rights Reserved | www.gktoday.in

231

Current Aairs: October 2016 [Date-wise Compendium]


and promotes understanding of the myriad and diverse ways statistics aects the lives of every
person alive today.

161. The first-ever global conference to strengthen Arbitration and Enforcement in India has started
in which city?
[A] Kochi
[B] Kolkata
[C] New Delhi
[D] Chennai

Answer:New Delhi
Explanation:
The rst-ever global conference to strengthen Arbitration and Enforcement in India has started in
New Delhi. The 3-day conference titled National Initiative on Strengthening Arbitration and
Enforcement is going on to change the face of dispute resolution in India. The conference has
inaugurated by President Pranab Mukherjee, which will provide impetus to commercial
arbitration in the country.

162. The Police Commemoration Day is celebrated on which day in India?


[A] October 23
[B] October 25
[C] October 21
[D] October 22

Answer:October 21
Explanation:
The Police Commemoration Day is celebrated every year on October 21 in all Police Lines
throughout India to commemorate the sacrices of ten policemen while defending the borders
with China in 1959. On 21st October 1959, Chinese Army personnel opened re and threw
grenades at the police party of 20 personnel. Ten of them died, seven were taken prisoners by
the Chinese and the remaining managed to escape. Since 1961, more than 33,000 police
personnel have sacriced their lives for security of the nation and service to the society.

163. The powerful Typhoon Haima has recently hit which of the following countries?
[A] India
[B] Malaysia
[C] Indonesia
[D] Philippines

Answer:Philippines
Explanation:
The very strong Typhoon Haima has recently hit the Philippines that cause widespread

2016 GKToday | All Rights Reserved | www.gktoday.in

232

Current Aairs: October 2016 [Date-wise Compendium]


destruction. The houses are destroyed as communities take shelter from sustained winds of
more than 225km/h. After Philippines, Haima made landfall over Haifeng County, Shanwei in the
Guangdong province of China.

164. Which union ministry has signed MoU with Indian Railways for smart railway stations?
[A] Ministry of Youth Affairs and Sports
[B] Ministry of Health and Family Welfare
[C] Ministry of Power
[D] Ministry of Urban Development

Answer:Ministry of Urban Development


Explanation:
The Ministry of Urban Development has signed MoU with the Ministry of Railways to develop
smart railway stations. As per the MoU, the railway stations and surrounding areas will be
redeveloped on smart city lines for better passenger amenities, easy access to stations, enabling
optimal utilization of land at railway stations. This means that the station and an adjoining area
of around 300 to 800 acres in each smart city and AMRUT city will be redeveloped. The validity of
the MoU is 5 years and can be extended with the consent of both the Ministries.

165. The Kaushalya SETU initiative has been launched by which state government for students?
[A] Rajasthan
[B] Uttar Pradesh
[C] Maharashtra
[D] Odisha

Answer:Maharashtra
Explanation:
The Maharashtra government has launched Kaushalya SETU (Self-Employment and Talent
Utilisation), a skill development programme for students. Under the Kaushalya Setu initiative,
students of Maharashtra State Board of Secondary & Higher Secondary Education, who fail to
clear Class X or SSC exam, would be accommodated in the skill development courses, so that
they do not lose a year.

166. The book The Sleepwalkers Dream has been authored by whom?
[A] Ambikagiri Raichoudhury
[B] Nalinibala Devi
[C] Dhrubajyoti Borah
[D] DP Basu

Answer:Dhrubajyoti Borah
Explanation:
The book The Sleepwalkers Dream has been authored by Dhrubajyoti Borah, a Guwahatibased Assamese writer and novelist. It narrates the journey of a group of militants towards an

2016 GKToday | All Rights Reserved | www.gktoday.in

233

Current Aairs: October 2016 [Date-wise Compendium]


elusive freedom and an uncertain future.

167. Who is the newly elected President of the International Gymnastics Federation (IFG)?
[A] Emilia Romagna
[B] Morinari Watanabe
[C] Georges Guelzec
[D] Bruno Grandi

Answer:Morinari Watanabe
Explanation:
Morinari Watanabe from Japan has been elected as the new President of the International
Gymnastics Federation (IFG) at the world governing bodys congress in Tokyo, Japan. He will
succeed Bruno Grandi, who is stepping down as FIG president in December 2016 after 20 years
in charge.With this, Watanabe has become the rst Asian to head IFG and is the rst Japanese
President of an Olympic international federation since Ichiro Ogimura was President of the
International Table Tennis Federation from 1987 to 1994. The headquarters of IFG is at
Lausanne, Switzerland.

168. Which Italian city has recently honoured the Dalai Lama with an honorary Citizenship award?
[A] Rome
[B] Florence
[C] Naples
[D] Milan

Answer:Milan
Explanation:
Lamberto Bertole has honoured the Dalai Lama Tenzin Gyatso, Tibets exiled spiritual leader with
the Milan Honorary Citizenship award at a ceremony held at the Arcimboldi Theatre in Milan,
Italy. Bertole is the Chairman of the Council of Milan.

169. Which country has won the World Railway Shooting Championship cup?
[A] India
[B] Germany
[C] France
[D] United States

Answer:India
Explanation:
Indian Railways have won the World Railway Shooting Championship Cup at Saint Mandrier,
France. This is the rst time that Railways have won this competition. Indian Railways team won
a total 10 medals 5 Gold, 3 Silver and 2 Bronze to bag the Overall Championship at the 15th
USIC- the International Shooting World Championship. The USIC is the International railway
sports association, whose purpose is to encourage the practice of amateur sport among railway

2016 GKToday | All Rights Reserved | www.gktoday.in

234

Current Aairs: October 2016 [Date-wise Compendium]


employees all over the world.

170. Which union minister has launched the Regional air Connectivity Scheme (RCS) to enable
more people to fly in smaller towns?
[A] Narendra Modi
[B] Raj Nath Singh
[C] Ashok Gajapathi Raju
[D] Arun Jaitley

Answer:Ashok Gajapathi Raju


Explanation:
Ashok Gajapathi Raju Pusapati, the Union Civil Aviation Minister, has launched the Regional air
Connectivity Scheme (RCS) in New Delhi that seeks to get more people to y in the smaller
towns. The RCS schemes is also called as Ude Desh ka Aam Nagrik (UDAN) (Let the common
man of the country y). Under the RCS scheme, airlines will have complete freedom to enter
into code sharing with larger airlines for connectivity and they will be exempted from various
airport charges. Airlines will get exclusive rights for 3 years to y on a particular regional route.
Airfares will be capped at 2,500 rupees for an hours ight for regional ights. The scheme is
expected to be rolled out by January 2017.

171. The Krishna Wildlife Sanctuary (KWS) is located in which state?


[A] Karnataka
[B] Andhra Pradesh
[C] Kerala
[D] Maharashtra

Answer:Andhra Pradesh
Explanation:
The Krishna Wildlife Sanctuary (KWS) is located in Andhra Pradesh. It is in news because
Lutrogale perspicillata (smooth-coated otter) has been sighted for the rst time in the mangrove
forest adjacent to the KWS in Krishna district. According to the International Union for
Conservation of Nature (IUCN), conservation status of the Smooth-coasted Otter is at vulnerable
list.

172. How much loan amount has been sanctioned by World Bank for construction of the Eastern
Dedicated Freight Corridor (EDFC)-III project of India?
[A] $700 million
[B] $550 million
[C] $444 million
[D] $650 million

Answer:$650 million
2016 GKToday | All Rights Reserved | www.gktoday.in

235

Current Aairs: October 2016 [Date-wise Compendium]


Explanation:
The Government of India (GoI) has signed $650 million loan agreement with the World Bank (WB)
for construction of the Eastern Dedicated Freight Corridor (EDFC)-III project. The EDFC-III will
build the 401 km Ludhiana-Khurja section which goes through Punjab, Haryana and Uttar
Pradesh. The purpose of the EDFC project is to augment railway freight carrying capacity along
the Railway Corridor between Ludhiana and Kolkata. The EDFC is a freight-only rail line that will
help faster and more ecient movement of raw materials and nished goods between the north
and eastern parts of India.

173. What is the theme of 2016 World Osteoporosis Day (WOD) ?


[A] Stop at One: Make Your First Break Your Last
[B] Love Your Bones: Perfect Your Future
[C] Real Men Build Their Strength from Within
[D] Serve Up Bone Health

Answer:Love Your Bones: Perfect Your Future


Explanation:
The World Osteoporosis Day (WOD) is observed every year on October 20 to raise global
awareness of the prevention, diagnosis and treatment of osteoporosis and metabolic bone
disease. The 2016 theme is Love Your Bones Protect your Future.

174. The book Andhere se ujale ki aur has been authored by which union minister of India?
[A] Ravi Shankar Prasad
[B] Uma Bharati
[C] Arun Jaitley
[D] Sushma Swaraj

Answer:Arun Jaitley
Explanation:
The book Andhere se ujale ki aur has been authored by Finance Minister Arun Jaitley, which
recently launched at the BJP headquarters in New Delhi. The book is a compilation of blogs and
articles by Jaitley that have appeared at various times.

175. Where is the headquarters of the Central Council of Homeopathy (CCH)?


[A] Pune
[B] Lucknow
[C] New Delhi
[D] Chennai

Answer:New Delhi
Explanation:
The Central Council of Homoeopathy (CCH) is a statutory apex body under the Ministry of Health

2016 GKToday | All Rights Reserved | www.gktoday.in

236

Current Aairs: October 2016 [Date-wise Compendium]


& Family Welfare, Department of AYUSH. It is in news because the Central Bureau of
Investigation (CBI) has recently arrested Dr. Ramjee Singh, the President of the CCH, and a
private person on charges of accepting Rs.20 lakh in bribe to give a favourable inspection report
for setting up of a homeopathy college. The headquarters of CCH is at New Delhi.

176. Almatti Dam is located in which of the following states?


[A] Andhra Pradesh
[B] Karnataka
[C] Kerala
[D] Maharashtra

Answer:Karnataka
Explanation:
The Almatti Dam is a hydroelectric project on the Krishna River at the Bagalkot district in North
Karnataka. It is in news because the dam has been chosen for the World Banks Award of
Excellence for best utilization of funds (Rs. 72 crore) for renovation to increase the strength of
the dam.The project was taken up under the Dam Rehabilitation and Improvement Project (DRIP)
for strengthening the reservoir, one of the largest reservoirs in Karnataka. The World Bank (WB)
had granted Rs.72 crore to the Krishna Bhagya Jal Nigam Limited to prevent seepage which
could damage the dam.

177. Which country has won the 2016 Kabaddi World Cup?
[A] Sri Lanka
[B] Iran
[C] India
[D] China

Answer:India
Explanation:
India has won the 2016 Kabaddi World Cup by defeating Iran in a nal by 38-29 in Ahmedabad,
Gujarat. This was Indias 3rd World Cup in the Standard Style version of Kabaddi.

178. The 2016 FIFA U-17 Womens World cup has been won by which country?
[A] Venezuela
[B] Japan
[C] Spain
[D] North Korea

Answer:North Korea
Explanation:
North Korea has won the 2016 FIFA U-17 Womens World cup by defeating Japan in the nal by
5-4 on penalty shootout in Jordan. With this, North Korea has become the rst team to win the

2016 GKToday | All Rights Reserved | www.gktoday.in

237

Current Aairs: October 2016 [Date-wise Compendium]


tournament twice.

179. Mehar Mittal, who passed away recently, was the legendary actor of which language?
[A] Punjabi
[B] Tamil
[C] Telugu
[D] Odia

Answer:Punjabi
Explanation:
Mehar Mittal (82), the legendary Punjabi actor, has recently passed away in Mount Abu,
Rajasthan. He was best known for playing comic roles in Punjwood lms and his notable works
were Sawa Lakh Se Ek Ladaun, Putt Jattan De, Babul Da Vehra, Bhulekha, Long Da
Lishkara, Peengan Pyar Deeyan and Jeeja Sali.

180. Indias first railway university will be set up in which city?


[A] Bengaluru
[B] Vadodara
[C] Varanasi
[D] Kochi

Answer:Vadodara
Explanation:
Indias rst railway university will be set up in Vadodara, Gujarat, which will help in carrying out
innovation and modernization for the Indian Railways. Initially, the present campus for National
Academy of Indian Railways (NAIR), Vadodara, will be used to start the railway university. After
land acquisition, a full-edged university will be started.

181. What is the theme of the 2016 United Nations Day?


[A] Freedom First
[B] Goals for Peace
[C] Strong UN: Better World
[D] Freedom & Peace for humanity

Answer:Freedom First
Explanation:
The United Nations Day is observed every year on October 24 to mark the anniversary of the
entry into force in 1945 of the UN Charter. With the ratication of this founding document by the
majority of its signatories, including the ve permanent members of the Security Council, the
United Nations ocially came into being. The 2016 theme is Freedom First. This years UN Day
will be used to highlight concrete actions people can take to help achieve the Sustainable
Development Goals.

2016 GKToday | All Rights Reserved | www.gktoday.in

238

Current Aairs: October 2016 [Date-wise Compendium]


182. Indias first startup magazine Cofounder has been launched by whom?
[A] Adhish Verma and Arunraj Rajendran
[B] Kirti Gaur and Mukund Dheer
[C] Deepak Singhvi and Shubh Roy
[D] Varhsa Sharma and Poorti Juneja

Answer:Adhish Verma and Arunraj Rajendran


Explanation:
Indias rst startup magazine Cofounder has been launched by Adhish Verma and Arunraj
Rajendran. The magazine will talk about real stories behind the early stage startups and
entrepreneurs. It would emerge as the new data storage and communications medium and
through its rst launch of magazine are essential to inform Indian CEOs and CIOs of the dierent
startups to aware the startup life, the struggles and the journey.

183. The Indo Tibetan Border Police (ITBP) raising day is celebrated on which day?
[A] October 24
[B] October 22
[C] October 23
[D] October 25

Answer:October 24
Explanation:
The Indo Tibetan Border Police (ITBP) has celebrated its 55th raising day on 24th October 2016.
It is one of the ve Central Armed Police Forces of India, raised on 24th October in 1962, under
the CRPF Act, in the wake of the Sino-Indian War of 1962The ITBP is deployed on border guarding
duties from Karakoram Pass in Ladakh to Jachep La in Arunachal Pradesh covering 3488
kilometer of India-China Border manning Border Outposts on altitudes raging from 9000 feet to
18700 feet in the Western, Middle and Eastern Sector of the Border.

184. Which of the following will be the official mascot of FIFA 2018 World Cup?
[A] Zakumi
[B] Willie
[C] Zabivaka
[D] Gauchito

Answer:Zabivaka
Explanation:
The 2018 FIFA World Cup will be held in Russia from 14 June to 15 July 2018. It is a quadrennial
international football tournament contested by the mens national teams of the member
associations of FIFA. The ocial mascot for the World Cup is Zabivaka. In Russian language
Zabivaka means the one who scores, a wolf who radiates fun, charm and condence. The wolf
will not only promote the event and entertain crowds at the stadiums, but also become an

2016 GKToday | All Rights Reserved | www.gktoday.in

239

Current Aairs: October 2016 [Date-wise Compendium]


ambassador for Russia and a worldwide celebrity.

185. The first National Tribal Carnival will be launched in which city?
[A] New Delhi
[B] Patna
[C] Guwahati
[D] Itanagar

Answer:New Delhi
Explanation:
The rst National Tribal Carnival will be launched in New Delhi to promote a sense of
inclusiveness amongst the tribals. The carnival will showcase and promote various facets of tribal
culture on a large scale. The underlying idea is to preserve and promote various facets of the
tribal life relating to culture, tradition, customs and their skills and to expose it to the general
public with a view to utilizing the potential for overall holistic development of the Scheduled
Tribes (STs).

186. Who has been crowned as the Mr. Asia 2016?


[A] Ryan Zhu
[B] Senthil Kumaran
[C] G. Balakrishna
[D] Manjeet Thakran

Answer:G. Balakrishna
Explanation:
G. Balakrishna from India has been crowned as the Mr. Asia 2016 at the recently held 5th PhilAsia bodybuilding championships in the Philippines. He is a resident of Ramagondanahalli near
Varthur in Bangalore, Karnataka.

187. What is the Indias rank in female literacy as per International Commission on Financing Global
Education Opportunity?
[A] 51st
[B] 45th
[C] 38th
[D] 23rd

Answer:38
Explanation:
India has been ranked 38th among the 51 developing countries in female literacy as per New
York-based International Commission on Financing Global Education Opportunity. The data from
new research on female literacy show that Indias school education system is under-performing
in terms of quality when compared to its neighbours viz. Pakistan, Bangladesh and Nepal.

2016 GKToday | All Rights Reserved | www.gktoday.in

240

Current Aairs: October 2016 [Date-wise Compendium]


188. The United Nations (UN) World Development Information Day is observed on which date?
[A] October 23
[B] October 25
[C] October 24
[D] October 26

Answer:October 24
Explanation:
The United Nations (UN) World Development Information Day is observed every year on October
24 to draw the attention of world public opinion to development problems and the need to
strengthen international cooperation to solve them

189. The book Dogs at the Perimeter has been authored by whom?
[A] Johanna Skibsrud
[B] Wajdi Mouawad
[C] Madeleine Thien
[D] Deborah Levy

Answer:Madeleine Thien
Explanation:
The book Dogs at the Perimeter has been authored by Madeleine Thien. Its story is about the
Cambodian genocide, which is as powerful as history, as magical as myth and a light shining on
one of the darkest chapters of modern history.

190. Which country is hosting the 2016 World Congress of International Association of Educators
for World Peace (IAEWP)?
[A] India
[B] United States
[C] Russia
[D] United Kingdom

Answer:India
Explanation:
C. Vidyasagar Rao, the Governor of Maharashtra, has inaugurated the 19th World Congress of
International Association of Educators for World Peace (IAEWP) on October 24 in Nashik,
Maharashtra with the theme Peace Education for Good Governance and Nonviolence. The 3day conference has been organized by International Association of Educators for World Peace
(IAEWP), an NGO of United Nations Economic and Social Council (ECOSOC), in collaboration with
city-based Gokhale Education Society (GES).

191. The World Polio Day is observed on which date?


[A] October 23
2016 GKToday | All Rights Reserved | www.gktoday.in

241

Current Aairs: October 2016 [Date-wise Compendium]


[B] October 24
[C] October 25
[D] October 26

Answer:October 24
Explanation:
The World Polio Day is observed every year by the World Health Organization (WHO) and its
allied agencies on October 24 to commemorate the ght against poliomyelitis. The day provides
an opportunity to celebrate progress towards a polio-free world and the tireless eorts of many
towards the goal.

192. Mitra Shakti 2016 the joint military exercise has been started between India and which
country?
[A] Afghanistan
[B] Sri Lanka
[C] Bangladesh
[D] Myanmar

Answer:Sri Lanka
Explanation:
The 4th edition of the India-Sri Lanka joint military exercise MITRA SHAKTI 2016 has started at
Sinha Regimental Centre in Ambepussa, Sri Lanka from October 24 and will be continued till
November 6, 2016. The main focus of this edition of the joint exercise is to enhance interoperability while carrying out Counter Insurgency (CI)/Counter Terrorism (CT) operations under
the United Nations Mandate.

193. The first workshop on use of International Household Survey Network (IHSN) toolkit has been
launched by which union minister?
[A] Kalraj Mishra
[B] Ramvilas Paswan
[C] Venkaiah Naidu
[D] D V Sadananda Gowda

Answer:D V Sadananda Gowda


Explanation:
The Minister for Statistics and Programme Implementation, D.V. Sadananda Gowda has launched
the International Household Survey Network (IHSN) Toolkit and also launched the new Website of
the Ministry in New Delhi. The IHSN is a standard bench mark which can be used as an active
management tool for better integration of administrative structures and statistical access for
policy making. It is about the latest trends & developments of data archives, storage and
dissemination technology tools available to user and is designed to address the technical issues
facing data producers.

2016 GKToday | All Rights Reserved | www.gktoday.in

242

Current Aairs: October 2016 [Date-wise Compendium]


194. Varun Singh Bhati, who won bronze at 2016 Rio Paralympic Games, is associated with which
sports?
[A] Shot Put
[B] High Jump
[C] Badminton
[D] Javelin Throw

Answer:High Jump
Explanation:
Varun Singh Bhati, a Para high jumper from India, has won bronze at 2016 Rio Paralympic
Games. Recently, he got cash reward Rs. 30 Lakhs by Vijay Goel, Union Minister of State
(Independent Charge) for Youth Aairs and Sports, at a function in New Delhi under the
Government of India (GoI)s scheme of special awards to medal winners in international sports
events.

195. The book Born to Fly: Fighter pilot MP Anil Kumar teaches us there is no battle mind cannot
win has been authored by whom?
[A] Sunny Rajput
[B] Nitin Sathe
[C] Manju Sharma
[D] P T Brinda

Answer:Nitin Sathe
Explanation:
The book Born to Fly: Fighter pilot MP Anil Kumar teaches us there is no battle mind cannot win
has been authored by Nitin Sathe, an air commander. It is about the life of ghter pilot MP Anil
Kumar, who became an icon of armed forces through his prolic writings despite having been
paralyzed after an accident. It was recently released by the Defense Minister Manohar Parrikar.

196. Which state has highest life-expectancy in India for all ages, as per recently data released by the
Registrar General of India (RGI)?
[A] Tamil Nadu
[B] Kerala
[C] Jammu and Kashmir
[D] Maharashtra

Answer:Jammu and Kashmir


Explanation:
Jammu and Kashmir (J & K) has the highest life expectancy in India for all ages, barring life
expectancy at birth, as per recently data released by the Registrar General of India (RGI). The
RGI publishes state level life expectancy-dened as the estimate of the average number of

2016 GKToday | All Rights Reserved | www.gktoday.in

243

Current Aairs: October 2016 [Date-wise Compendium]


additional years that a person of a given age can expect to live data for dierent ages: 0 (at
birth), 1, 5, 10, 20, 30, 40, 50, 60 and 70. Though, Kerala has the highest overall life expectancy
at birth, followed by Delhi and Jammu and Kashmir.

197. The National Solidarity Day is observed on which day in India?


[A] October 26
[B] October 22
[C] October 20
[D] October 24

Answer:October 20
Explanation:
The National Solidarity Day is observed every year on October 20 in India. It is the date on which
China attacked India in 1962.

198. The Urja Ganga gas pipeline project has been launched in which city of India?
[A] Patna
[B] Varanasi
[C] Guwahati
[D] Ranchi

Answer:Varanasi
Explanation:
Prime Minister Narendra Modi has recently laid the foundation stone of Urja Ganga gas pipeline
project of Rs. 51,000-crore in Varanasi, Uttar Pradesh. The purpose of the project is to provide
piped cooking gas to residents of Varanasi within two years and then to cater millions of people
in states like Bihar, Jharkhand, West Bengal and Odisha. The 800-km pipeline will be laid and
50,000 households and 20,000 vehicles will get PNG and CNG gas respectively.

199. Who has won the 2016 Formula 1 United States Grand Prix?
[A] Daniel Ricciardo
[B] Nico Rosberg
[C] Sebastian Vettel
[D] Lewis Hamilton

Answer:Lewis Hamilton
Explanation:
Lewis Hamilton, a British Formula One racing driver from England and racing for the Mercedes
AMG Petronas, has won the 2016 Formula 1 United States Grand Prix at the Circuit of the
Americas, Austin, Texas, United States.

200. Tom Hayden, who passed away recently, was anti-Vietnam war activist of which country?
[A] United States
2016 GKToday | All Rights Reserved | www.gktoday.in

244

Current Aairs: October 2016 [Date-wise Compendium]


[B] Russia
[C] Germany
[D] France

Answer:United States
Explanation:
Tom Hayden (76), an American social and political activist, has recently died in Santa Monica,
California. He was best known for his major role as an anti-Vietnam war, civil rights and radical
intellectual counterculture activist.

201. Who has won the 2016 Man Booker Prize?


[A] Sam Lipsyte
[B] Amanda Foreman
[C] Paul Beatty
[D] Marlon James

Answer:Paul Beatty
Explanation:
Paul Beatty has won the 2016 Man Booker Prize for his racial satire, The Sellout at a ceremony
in Londons Guildhall. With this, he becomes the rst American writer to win the Man Booker
Prize. The novel tells the story of a young black man who tries to reinstate slavery and racial
segregation in a suburb of Los Angeles. It is a satire on United States constitution, urban life, the
civil rights movement, the father-son relationship and the holy grail of racial equality. The Booker
Prize is a literary prize awarded each year for the best original novel, written in the English
language and published in the UK.

202. Which state government has tied up with Monetary Authority of Singapore (MAS) to promote
innovation financial services?
[A] Karnataka
[B] Kerala
[C] Odisha
[D] Andhra Pradesh

Answer:Andhra Pradesh
Explanation:
The Andhra Pradesh government has signed a FinTech Cooperation agreement with the
Monetary Authority of Singapore (MAS) to promote innovation nancial services. Under the
agreement, MAS and the state government will explore joint innovation projects on technologies
such as digital payments and blockchain and collaborate on the development of education
programmes on FinTech. They also agreed to discuss emerging FinTech trends and exchange
views on regulatory issues related to innovations in nancial services.

203. What is the Indias rank in the WEFs Global Gender Gap Index for 2016?
2016 GKToday | All Rights Reserved | www.gktoday.in

245

Current Aairs: October 2016 [Date-wise Compendium]


[A] 55th
[B] 87th
[C] 135th
[D] 105th

Answer:87th
Explanation:
India has been ranked 87th out of 144 countries on the World Economic Forums (WEF) Global
Gender Gap Index for 2016, improving from its 108 position in 2015. The list is topped by
Iceland, followed by Finland, Norway, Sweden and Rwanda. The Global Gender Gap report is an
annual benchmarking exercise that measures progress towards parity between men and women
in four areas: educational attainment, health and survival, economic opportunity and political
empowerment.

204. The book The Bangle Seller has been authored by whom?
[A] Maya Kalyanpur
[B] Anil Kumar
[C] M K Mani
[D] Brijesh Gulati

Answer:Maya Kalyanpur
Explanation:
The book The Bangle Seller has been authored by Maya Kalyanpur. It is a layered story that
captures an India in transition, in-between ancient tradition and development in a globalised
world. In simple words, it is a story of richly-woven tapestry of travel, romance, tradition and
culture of India.

205. Junko Tabei, who recently passed away, was the well-known mountaineer of which country?
[A] Japan
[B] China
[C] Nepal
[D] Malaysia

Answer:Japan
Explanation:
Junko Tabei (77), who was the rst woman to climb the worlds highest peak Mount Everest,
has recently passed away in Tokyo, Japan. She reached the summit of Mt.Everest in 1975. In
1992, she also became the rst woman to scale the Seven Summits, or the tallest mountains
on each of the seven continents.

206. Which technological giant has launched its first Cybersecurity Engagement Center (CSEC) in
India?
2016 GKToday | All Rights Reserved | www.gktoday.in

246

Current Aairs: October 2016 [Date-wise Compendium]


[A] Infosys
[B] Wipro
[C] Reliance
[D] Microsoft

Answer:Microsoft
Explanation:
Microsoft India has launched its rst full-scale Cybersecurity Engagement Center (CSEC) in Delhi,
which is rst-of-its-kind centre in India. This is Microsofts 7th Cybersecurity Centre in the world
and will function as a satellite to the companys Redmond Digital Crimes Unit. The CSEC will
bring together Microsoft capabilities to foster deeper Cybersecurity collaborations with public
and private sector organisations. The centre will also build a trusted and secure computing
environment, a critical enabler for Indias digital transformation.

207. Who has been named as the 2016 European Golden Boy Award?
[A] Raheem Sterling
[B] Anthony Martial
[C] Renato Sanches
[D] Marcus Rashford

Answer:Renato Sanches
Explanation:
Renato Sanches, a Portuguese professional footballer who plays as a midelder for German club
Bayern Munich and the Portugal national team, has won the 2016 European Golden Boy award.
The Golden Boy is an award that is given by sports journalists to a young football player playing
in Europe perceived to have been the most impressive during a calendar year

208. Biotech-KISAN scheme has been launched to empower small and women farmers. What does
KISAN stands for?
[A] Krishi Innovation Scientific Application Network
[B] Krishi Innovation Skill Application Network
[C] Krishi Innovation Science Application Network
[D] Krishi Innovation Sector Application Network

Answer:Krishi Innovation Science Application Network


Explanation:
Dr. Harsh Vardhan, the Union Science and Technology Minister, has launched a new initiatives
Biotech-KISAN (Krishi Innovation Science Application Network) scheme in New Delhi to empower
farmers, especially women. Under the scheme, fellowship will be given to women farmers for
training and education in the farm practice. The purpose of the initiative is to connect farmers,
scientist and science institution across the country. The scheme also aims to address individual
problem of the smallholding farmers by providing solution. The scheme will be implemented in

2016 GKToday | All Rights Reserved | www.gktoday.in

247

Current Aairs: October 2016 [Date-wise Compendium]


15 agro-climatic zones of India in phased manner with the objective of linking new technology to
the farm by understanding the problem of the local farmer.

209. Carlos Alberto, who died recently, was a legendary footballer from which country?
[A] Germany
[B] Brazil
[C] Italy
[D] France

Answer:Brazil
Explanation:
Carlos Alberto (72), former Brazilian World Cup-winning football captain, has recently passed
away in Rio de Janeiro, Brazil. He scored one of the greatest goals in FIFA World Cup history while
captaining Brazil to glory in the 1970 nal against Italy in Mexico. He was a classy defender who
played for Fluminense, Santos, Flamengo and the Cosmos and earned the nickname The
Captain for his leadership qualities.

210. What is the Indias rank in the World Banks Ease of doing business index for 2017?
[A] 130th
[B] 155th
[C] 117th
[D] 88th

Answer:130th
Explanation:
India has been ranked 130th out of 190 countries in the World Banks Ease of doing business
index for 2017. The list is topped by New Zealand, followed by Singapore, Denmark, Hong Kong,
South Korea, Norway, the UK, the US, Sweden and former Yugoslav Republic of Macedonia. The
latest edition of Doing Business 2017 report examines regulations that enhance or constrain
business activity of countries and ranks them according to the impacts of their regulatory
environment on business. The rankings are based on ten parameters starting a business,
dealing with construction permits, getting electricity, registering property, getting credit,
protecting minority investors, paying taxes, trading across borders, enforcing contracts and
resolving insolvency.

211. Which judicial committee on One Rank One Pension (OROP) has submitted its report to
Ministry of Defence ?
[A] H Devraj committee
[B] Ramanujam committee
[C] Prof. NR Madhava committee
[D] L Narasimha Reddy committee

Answer:L Narasimha Reddy committee


2016 GKToday | All Rights Reserved | www.gktoday.in

248

Current Aairs: October 2016 [Date-wise Compendium]


Explanation:
The one-man judicial committee on One Rank One Pension (OROP) has submitted its report to
Defence Minister Manohar Parrikar. The Centre had appointed the committee, under the
Chairmanship of retired Chief Justice of Patna High Court L Narasimha Reddy, to look into the
anomalies, if any, arising out of implementation of OROP.

212. Who has been appointed as the Director of Enforcement Directorate (ED)?
[A] Mahendra Verma
[B] Karnal Singh
[C] Bimal Gautam
[D] Bibek Debroy

Answer:Karnal Singh
Explanation:
Karnal Singh, a 1984-batch IPS ocer of the Union territories cadre, has been appointed as the
Director of Enforcement Directorate (ED). He will be in oce till August 31, 2017. Prior to this
appointment, he was holding additional charge as Director of ED for over a year. The post had
fallen vacant after the government had in August last year curtailed the tenure of Rajan S
Katoch, the previous Director of ED. The Directorate General of Economic Enforcement is an
economic intelligence and law enforcement agency responsible for enforcing economic laws and
ghting economic crime in India.

213. Who has been appointed as the new chairman of Global System Mobile Association (GSMA)?
[A] Azim Premji
[B] Sunil Mittal
[C] Dilip Sanghvi
[D] Mukesh Ambani

Answer:Sunil Mittal
Explanation:
Sunil Mittal, the founder and chairman of Bharti Enterprises, has been appointed as the chairman
of global telecom body Global System Mobile Association (GSMA) for a two-year term, starting
from January 2017. He will succeed current GSMA Chairman Jon Fredrik Baksaas. Mittal is the
rst Indian to be elected as the Chairman of GSMA , who will oversee the strategic direction of
GSMA, which represents nearly 800 of the worlds mobile operators and more than 300
companies in the broader mobile ecosystem.

214. What is the theme of the 2016 World Day for Audio Visual Heritage?
[A] Teach & Protect Your Story
[B] Archives at Risk: Protecting the Worlds Identities
[C] Making World Audio Visual Heritage
[D] Its your story dont lose it
2016 GKToday | All Rights Reserved | www.gktoday.in

249

Current Aairs: October 2016 [Date-wise Compendium]


Answer:Its your story dont lose it
Explanation:
The World Day for Audio Visual Heritage is observed every year on October 27 to raise general
awareness of the need for urgent measures to be taken and to acknowledge the importance of
audiovisual documents as an integral part of national identity. The 2016 theme is Its your story
dont lose it. The purpose of the day is to highlight the importance of audiovisual documents
such as lms, radio and television programmes.

215. Which committee has been constituted to monitor bird flu situation?
[A] G N Bajpai committee
[B] Munialappa committee
[C] KV Kamath committee
[D] Anant Mashelkar committee

Answer:Munialappa committee
Explanation:
The Union Agriculture Ministry has constituted Dr. Munialappa committee to monitor bird u
situation and help the state governments in taking steps to contain the spread of avian inuenza
(H5N8). Dr. Munialappa is the Joint Commissioner in the Department of Animal Husbandry, Dairy
and Fisheries (DADF). The committee will have representatives from Ministry of Health, Ministry
of Environment, Forests and Climate Change, Department of Agriculture Research and Extension
(DARE) and the Delhi government. Apart from this, the ministry has established a control room
with telephone numbers 011-23384190 and 09448324121.

216. Which of the following personalities have been honoured with the 2016 Sakharov Prize for
Freedom of Thought?
[A] Jafar Panahi and Nasrin Sotoudeh
[B] Raif Badawi and Malala Yousafzai
[C] Nadia Murad and Lamiya Aji Basha
[D] Hu Jia and Salih Mahmoud Osman

Answer:Nadia Murad and Lamiya Aji Basha


Explanation:
Nadia Murad Basee and Lamiya Aji Bashar have been honoured with the 2016 Sakharov Prize for
Freedom of Thought, the won the European Unions prestigious human rights award. Both are
survivors of sexual enslavement by Islamic State (IS) and have become spokespersons for
women aicted by the terrorist groups campaign of sexual violence. They are also public
advocates for the Yazidi community in Iraq, a religious minority that has been the subject of a
genocidal campaign by IS militants. The Sakharov award will be presented to them in a
ceremony that will be held in Strasbourg, France on December 14, 2016. The annual Sakharov
prize for freedom of thought, established in 1988, is named after the Soviet physicist and
outspoken dissident Andrei Sakharov and is awarded to individuals who have made an
exceptional contribution to the ght for human rights across the globe.

2016 GKToday | All Rights Reserved | www.gktoday.in

250

Current Aairs: October 2016 [Date-wise Compendium]


217. By which theme the first National Ayurveda Day is celebrated in India?
[A] Ayurveda: Cure and Protect Our Lives
[B] Live happily with Ayurveda
[C] Ayurveda for Prevention and Control of Diabetes
[D] Mission Madhumeha through Ayurveda

Answer:Ayurveda for Prevention and Control of Diabetes


Explanation:
The rst National Ayurveda Day is celebrated on October 28 with the theme Ayurveda for
Prevention and Control of Diabetes to create awareness on peoples health. The Ministry of
AYUSH has also launch Mission Madhumeha through Ayurveda, which will be a protocol for
treating diabetes through Ayurveda and will be implemented throughout India. The Government
of India (GoI) has declared that the National Ayurveda Day will be celebrated every year on the
occasion of Dhanwantari Jayanti, popularly known as Dhanteras.

218. Antarticas which sea has been declared as worlds largest marine protected area?
[A] Caspian Sea
[B] Black Sea
[C] Ross Sea
[D] Dead Sea

Answer:Ross sea
Explanation:
Antarticas Ross Sea has been declared as worlds largest Marine Protected Area (MPA) after the
landmark international agreement among 24 countries and the European Union in the
Commission for the Conservation of Antarctic Marine Living Resources (CCAMLR) meeting in
Hobart, Australia. The Ross Sea is one of the last intact marine ecosystems in the world, which
covers 1.6 million square kilometers. It is home to penguins, seals, Antarctic toothsh and
whales.

219. Raj Begum, who recently passed away, was the legendary singer of which state?
[A] Uttar Pradesh
[B] Telangana
[C] Andhra Pradesh
[D] Jammu & Kashmir

Answer:Jammu & Kashmir


Explanation:
Legendary Kashmiri singer and Padma Shri awardee, Raj Begum (89) has recently passed away
in Srinagar, Jammu & Kashmir. She was also called the Nightingale of Kashmir, whose
captivating voice has ruled generations of Kashmiri music lovers. She was one of the most
recognized female singers in the Kashmir Valley, who rendered thousands of songs from Radio

2016 GKToday | All Rights Reserved | www.gktoday.in

251

Current Aairs: October 2016 [Date-wise Compendium]


Kashmir in Srinagar and in live concerts at other places in India and abroad.

220. The book Family Life has been authored by which author?
[A] Shantanu Guha
[B] Akhil Sharma
[C] Amrish Tripathi
[D] Mrinal Jain

Answer:Akhil Sharma
Explanation:
The book Family Life has been authored by Akhil Sharma, an Indian born-American author. The
novel is the autobiography of the author, which mirrors the authors own experience of
emigrating from India to America as a child. The 2016 International Dublin Literary Award has
been awarded to him for his novel Family Life.

221. Kristalina Georgieva, who has been appointed chief executive of the World Bank (WB), is
belonged to which country?
[A] Israel
[B] Germany
[C] France
[D] Bulgaria

Answer:Bulgaria
Explanation:
Kristalina Georgieva, a former head of EU humanitarian aairs, has been appointed chief
executive of the World Bank.She will succeed Sri Mulyani Indrawati on January 2017. Georgieva
is a Bulgarian economist and administrator and currently serving as European Commission Vice
President for Budget and Human Resource.

222. Madhubabu Aain Sahayata Sibir (MASS) scheme has been launched by which state
government to provide legal assistance to deprived sections?
[A] Tripura
[B] Tamil Nadu
[C] Odisha
[D] Assam

Answer:Odisha
Explanation:
The Odisha government has launched new scheme Madhubabu Aain Sahayata Sibir (MASS) to
provide legal assistance to the deprived sections of the society at the grassroots level in all its
6,812 gram panchayats across the state. The scheme was named after great Odia lawyer Madhu
Babu, who is considered as the rst lawyer of Odisha. The legal assistance will be given in the

2016 GKToday | All Rights Reserved | www.gktoday.in

252

Current Aairs: October 2016 [Date-wise Compendium]


shape of free legal aid to ensure that opportunities for securing justice are not denied to any
citizen for economic and other reasons. The legal assistance lawyer, who would receive Rs 500
per sitting as remuneration, would help people especially the women, children, senior citizens,
SC, ST and socially and educationally backward to resolve long pending litigations.

223. What is the Indias position at the 2016 Global Youth Development Index (YDI)?
[A] 112th
[B] 133rd
[C] 98th
[D] 130th

Answer:133rd
Explanation:
India has been ranked 133rd out of 183 countries in the 2016 Global Youth Development Index
(YDI), which is compiled by the Commonwealth Secretariat on the countries prospects for young
people in employment, education, health, civic and political spheres. The list is topped by
Germany, followed by Denmark, Australia, Switzerland and United Kingdom(UK).

224. Indias first Design Yatra has started in which state?


[A] Manipur
[B] West Bengal
[C] Kerala
[D] Uttar Pradesh

Answer:Kerala
Explanation:
Indias rst Design Yatra has started in Kozhikode, Kerala on October 29 to spread awareness
and take designs to the masses. The yatra will travel to 29 cities across the country by February
2017, covering 15,000 kms in three cars. It is launched by the Institute of Indian Interior
Designers (IIID). The purpose of the yatra is to inspire people to design houses, schools, hospitals
and public places in such a way that they present local culture, consider convenience of people
and save the environment.

225. Which country has won the 2016 Asian Mens Hockey Champions Trophy?
[A] Pakistan
[B] Malaysia
[C] India
[D] Japan

Answer:India
Explanation:
India has won the gold medal at the 2016 Asian Mens Hockey Champions trophy by defeating
arch-rivals and defending champions Pakistan by 3-2 in the nal at Kuantan, Malaysia. This is the

2016 GKToday | All Rights Reserved | www.gktoday.in

253

Current Aairs: October 2016 [Date-wise Compendium]


second time India have won the title of this tournament.

226. Who is the newly elected Prime Minister of Spain?


[A] Javier Arenas
[B] Mariano Rajoy
[C] Alfredo Perez
[D] Maria Dolores

Answer:Mariano Rajoy
Explanation:
Mariano Rajoy Brey , the leader of conservative Popular Party (PP), is the newly re-elected Prime
Minister of Spain, after winning a parliamentary vote. Till date, he was the acting Prime minister
of Spain.

227. Which state has been declared Indias second Open Defecation Free (ODF) state?
[A] Kerala
[B] Assam
[C] Himachal Pradesh
[D] Odisha

Answer:Himachal Pradesh
Explanation:
Himachal Pradesh has been declared Indias second Open Defecation Free (ODF) state after
Sikkim.The state has successfully achieved 100% coverage and total rural sanitation coverage in
the state, with all 12 districts in the state achieving both these feats. With this newly acquired
status, Himachal Pradesh will entitle to receive World Bank funding under Rs 9,000 crore projects
to sustain sanitation campaign.

228. What is the theme of 2016 National Unity Day (NUD) of India?
[A] Growth and Unity of India
[B] Integration of India
[C] Bound to develop peaceful India
[D] Peace: Soul purpose of India

Answer:Integration of India
Explanation:
The National Unity Day (or Rashtriya Ekta Diwas) is observed every year in India on October 31
to mark the occasion of the birth anniversary of Independent Indias rst Deputy Prime Minister
Sardar Vallabhai Patel. The 2016 theme is Integration of India. The day would provide an
opportunity to re-arm the inherent strength and resilience of our nation to withstand the actual
and potential threats to the unity, integrity and security of our country.

229. Vikram Ingale is associated with which sports?


2016 GKToday | All Rights Reserved | www.gktoday.in

254

Current Aairs: October 2016 [Date-wise Compendium]


[A] Boxing
[B] Roller Skating
[C] Sprint
[D] Chess

Answer:Roller Skating
Explanation:
India has won its rst-ever medal in an international team roller skating racing event at the Asian
Championships in Lishui, China.The trio of Dhanush Babu, Vikram Ingale and Nikhilesh Tabhane
have won the bronze in the 3,000-metre team relay event.

230. Shashikala Kakodkar, who passed away recently, was the first woman chief minister of which
Indian state?
[A] Goa
[B] Maharashtra
[C] Karnataka
[D] Odisha

Answer:Goa
Explanation:
Shashikala Kakodkar (81), who passed away recently at Panaji, was the rst woman chief
minister of Goa. She was the daughter of the former Goa chief minister Dayanand Bandodkar.
Kakodkar was active in social and cultural elds and had headed the Bhartiya Bhasha Suraksha
Manch (BBSM), a front demanding mother tongue be made medium of instruction in elementary
education.

2016 GKToday | All Rights Reserved | www.gktoday.in

255

Вам также может понравиться